RNSG 1111 EXAM 3 (Units 5, 6, AND 7)

Pataasin ang iyong marka sa homework at exams ngayon gamit ang Quizwiz!

Sutures

thread like material to sew edges of around together for healing; can be done in layers; absorbable material used for inner layers;

Diaphragmatic breathing

useful for patients with pulmonary disease, postoperative patients, and women in labor to promote relaxation and provide pain control. The exercise improves efficiency of breathing by decreasing air trapping and reducing the WOB.

Afterload

resistance to left ventricular ejection

Diffusion

responsible for moving the respiratory gases from one area to another by concentration gradients

Falls:

result in minor to severe injuries such as bruises, hip fractures, or head trauma that result in reduced mobility and independence and increase the risk for premature death. - Patients who have underlying disease states are more susceptible to fall-related injuries.

Nasal Cannula

simple, comfortable device used for precise oxygen delivery. Attach it to a humidified oxygen source with a flow rate up to 6 L/min (24% to 40% oxygen). Flow rates equal to or greater than 4 L/min have a drying effect on the mucosa and thus need to be humidified.

Forced vital capacity

the maximum amount of air that can be removed from the lungs during forced expiration

Purulent wound exudate

thick yellow, green, tan, or brown

Identify actual or potential nursing diagnoses that apply to patients whose safety is threatened (7)

- Risk for Falls - Impaired home maintenance - Risk of injury - Deficient knowledge - Risk for poisoning - Risk for suffocation - Risk for trauma

Pulmonary circulation

moves blood to and from the alveolar capillary membranes gas exchange

Dehiscence

partial or total separation of wound layers; may be partial or total separation of wound layer; increased risk with abdominal surgeries

Identify the specific assessments to perform when considering possible threats to patient safety

- Activity and exercise - Medication history - History of falls - Home maintenance

Nursing interventions directed at eliminating environmental threats include:

- Basic Needs related to oxygen, nutrition and temperature - Prevent interruption of therapy - Prevent a confused or combative patient from removing life support equipment

Use of restraints must meet one of the following objectives:

- Reduce the risk of patients injury from falls - Prevent interruption of therapy - Prevent a confused or combative patient from removing life support equipment - Reduce risk of injury to others by the patient

A nurse is obtaining a health history from a newly admitted patient. Which patient statement about alcohol intake is based on a common physiological response? 1. "After I go drinking, I have to urinate during the night." 2. "When I drink, I get hungry in the middle of the night." 3. "Falling asleep is hard, but once asleep I sleep great." 4. "If I drink too much, I oversleep in the morning."

1. "After I go drinking, I have to urinate during the night." Alcoholic beverages are fluids that have a mild diuretic effect. Frequent nighttime awakening to empty a full bladder is called nocturia.

The nurse is reviewing the clinical record of a newly admitted older adult male patient. Which piece of data should cause the most concern? 1. WBC 30,000 mm^3 2. Temperature 103*F 3. Abdominal cramping 4. Blood pressure 110/86 mm Hg

1. WBC 30,000 mm^3

Question 8. A nurse is teaching a patient about ways to prevent infection. What best increases a patient's defense against microorganisms? 1. Covering a cough 2. Maintaining intact skin 3. Changing bed linen daily 4. Using an antiseptic mouthwash

2

When does a surgical wounds tend to show infection?

4th or 5th post op day

When is hot therapy contraindicated?

Areas of active bleeding, on acute localized inflammation, over a large area

What is the most powerful respiratory stimulant?

Carbon dioxide

Do you use a clean or sterile technique with oropharyngeal suctioning?

Clean

Which description best fits that of serous drainage from a wound? A) Fresh bleeding B) Thick and yellow C) Clear, watery plasma D) Beige to brown and foul smelling

Clear, watery plasma

Briefly explain how assessment of body alignment and posture is carried out. (Standing)

During standing, the head is erect and midline, body parts are symmetrical, spine is straight with normal curvatures, abdomen is comfortably tucked, knees are in a straight line between the hips and ankles and slightly flexed, and feet are flat on the floor.

Isotonic contractions

Exercises that cause muscle contraction and change in muscle length. (walking, swimming, biking)

Fowler Position

Head of bed (HOB) elevated 45 to 60 degrees and the knees are slightly elevated

When is cold therapy contraindicated?

If injury is edematous, presence of neuropathy, if patient is shivering, if patient has impaired circulation

Which of the following is a physiological effect of prolonged bed rest? 1. An increase in cardiac output 2. A decrease in lean body mass 3. A decrease in lung expansion 4. A decrease in urinary excretion of nitrogen

Immobility Causing decreased lung elastic recoiling and secretions accumulating in portions of the lung

Pressure Ulcer Stage 2

Involving the epidermis, or both epidermis & dermis; superficial abrasion, blister or shallow crater; drainage may or may not be present; partial thickness skin loss

When the nurse is performing surgical hand asepsis, where should the nurse keep his or her hands? 1 Below the elbows 2 Above the elbows 3 At a 45-degree angle 4 In a comfortable position

Keeping the hands above the elbows when performing a surgical scrub prevents contaminated water from coming into contact with the hands.

A chemical restraint is:

Medications used to manage a patient's behavior that are not a standard treatment or dosage for the patients condition

Braden Scale 13-14

Moderate risk

Coordinated body movement

Movement that is the result of weight, center of gravity, and balance.

Body balance

Occurs with a lower center of gravity and a wide, stable base of support

Sims Position

Patient places the weight on the anterior ileum, humerus and clavicle

Negative outcome of immobility to Adults

Physiological systems are at risk, loss of job

What part of the brain control the respiratory center?

Pons and medulla oblongata, which is part of the autonomic nervous system

Identify the descriptive characteristics of body alignment and mobility related to the following developmental stage. (Toddlers)

Posture is awkward because of slight swayback and protruding abdomen; toward the end of toddlerhood, posture appears less awkward, curves in the cervical and lumbar vertebrae are accentuated, and foot eversion disappears.

What are the three pressure-related factors that contribute to pressure ulcer development?

Pressure intensity, pressure duration, and tissue intolerance

Trochanter Roll

Prevents external rotation of the hips when the patient is in the supine position

Postural drainage position for adult both lower lobes - posterior lung segments

Prone in Trendelenburg's position

Postural drainage position for adult right lower lobe - posterior lung segment

Prone with right side of chest elevated in Trendelenburg's position

Postural drainage position for adult right middle lobe- posterior lung segment

Prone with thorax and abdomen elevated

Which of the following is an indication for a binder to be placed around a surgical patient with a new abdominal wound? A) Collection of wound drainage B) Reduction of abdominal swelling C) Reduction of stress on the abdominal incision D) Stimulation of peristalsis (return of bowel function) from direct pressure

Reduction of stress on the abdominal incision

A nurse changes a patient's dressing when it is soiled. What step is this nursing action breaking in the chain of infection? Infectious agent, Reservoir, Portal of exit, Mode of transmission, Portal of entry, Susceptible host

Reservoir

Abrasion

Scraping or rubbing away of epidermis; may result in localized bleeding and later weeping of serous fluid.

Negative outcome of immobility to Adolescents

Social isolation

Do you use a clean or sterile technique with nasopharyngeal suctioning?

Sterile

Explain how to calculate the patient's target heart rate (THR).

Subtract the patient's current age from 220 and obtain the target heart rate by taking 60% to 90% of the maximum.

Postural drainage position for adult both lower lobes - anterior lung segment

Supine in Trendelenburg's position

Postural drainage position for adult left upper lobe - anterior lung segment

Supine with head elevated

Activity intolerance

The amount of exercise or activity that a person is able to perform.

Body alignment

The individuals center of gravity is stable

A 47-year-old patient has arrived at the clinic after accidentally cutting his forearm with a pair of scissors. Which clinical manifestations would the nurse expect to indicate a local inflammation? Select all that apply. 1 Swelling 2 Redness 3 Pain 4 Anorexia 5 Vomiting

The local manifestations of inflammation include swelling, redness, and pain. These manifestations are caused by protective vascular reactions that help to combat inflammation. Anorexia and vomiting are systemic manifestations of inflammation.

On the Braden scale, does a higher or lower score indicate a high risk for developing a pressure ulcer?

The lower the total score places a patient at higher risk of developing a pressure ulcer

Which aspect of positioning a patient in the supported Fowler's position has a goal of decreasing flexion of vertebrae? 1 Place a small pillow under thigh 2 Place the head on a small pillow 3 Place a small pillow at the lower back 4 Place a pillow to support arms and hands

The nurse should place a small pillow at lower back to decrease flexion of vertebrae. Placing a small pillow under the thigh prevents hyperextension of the knee. Placing the head on a small pillow prevents flexion contractures of cervical vertebrae. Placing a pillow to support the arms and hands prevents shoulder dislocation.

What would a nurse use for a high-level disinfection? 1 Moist heat 2 Boiling water 3 Ethylene oxide gas 4 Hydrogen peroxide

The nurse uses chemical sterilants such as hydrogen peroxide, iodophors, phenolics, and quaternary ammonium compounds for high-level disinfection. Moist heat, boiling water, and ethylene oxide gas are used for sterilization.

Postural drainage position for adult right middle lobe - anterior lung segment

Three-fourths supine position with dependent lung in Trendelenburg's position

When is an application of a warm compress indicated? (Select all that apply.) A) To relieve edema B) For a patient who is shivering C) To improve blood flow to an injured part D) To protect bony prominences from pressure ulcers

To relieve edema To improve blood flow to an injured part

Walking helps to prevent contractures by:

Walking helps prevent contractures by increasing joint mobility.

Negative outcome of immobility to Older Adults

Weaker bones, increased risk of falls, increased physical dependence on others

Assistive Aids

When patients use assistive aids, it is important to routinely check the condition of rubber tips and the integrity of the aid. Patients are at risk for falls during transfer tasks and reaching while seated in a wheelchair.

When obtaining a wound culture to determine the presence of a wound infection, from where should the specimen be taken? A) Necrotic tissue B) Wound drainage C) Drainage on the dressing D) Wound after it has first been cleaned with normal saline

Wound after it has first been cleaned with normal saline

A nurse turns a patient's ankle so that the sole of the foot moves medially toward the midline. What should the nurse use when documenting exactly what was done during range-of-motion exercises? a. inversion b. adduction c. plantar flexion

a

Examples of health promotion activities that address mobility and immobility

a. Prevention of work-related injury b. Fall prevention measures c. Exercise d. Early detection of scoliosis

Identify the areas to evaluate to determine the effectiveness of the nursing interventions to enhance activity and exercise.

a. Pulse b. Blood pressure c. Strength d. Endurance e. Psychological well-being

Expected outcomes for the goal "patient skin remains intact"

a. Skin color and temperature return to normal baseline within 20 minutes of position change b. Changes position at least every 2 hours

Equipment-Related Accidents:

accidents result from malfunction, disrepair, or misuse of equipment, or from an electrical hazard, or rapid IV infusions.

Red wound

active healing; protect & cover, using gentle cleansing

vascular and cellular response

acute inflammation: rapid vasodilation that causes redness at the site and localized warmth, alowing phagocyteis to occur

Dressings for unstageable Pressure Ulcer

adherent film, gauze, enzymes, or none if eschar is dry and intact

Fistula

an abnormal passage between two organs or between an organ and the outside of the body; most form poor wound healing or a disease process; increased risk of infection

Inspiration

an active process, stimulated by chemical receptors in the aorta.

A registered nurse teaches a group of nursing students about critical thinking. Which statement if made by the nursing student indicates the need for further learning? a. "All factors should be considered during assessment." b. "Clinical judgments and decisions should be made before anticipating the information." c. "The nursing knowledge should be integrated with the knowledge from other disciplines." d. "It requires the synthesis of knowledge, experience, and information gathered from the patients."

b. "Clinical judgments and decisions should be made before anticipating the information." Clinical judgments should be made after analyzing the data and not before anticipating the information. All the factors (such as knowledge, experience, standards, and attitudes) should be considered during an assessment to make appropriate nursing diagnoses. Nursing knowledge should be integrated with knowledge from other disciplines in order to provide better care and hygiene. Effective critical thinking requires the synthesis of knowledge, experience, and information gathered from patients. p. 825

Sanguineous wound exudate

bright red blood; indicated active bleed

Surfactant

chemical produced in the lungs to maintain the surface tension of the alveoli and keep them from collapsing

Atelectasis

collapse of the alveoli that prevents normal exchange of oxygen and carbon dioxide.

TJC & CMS "Speak Up" campaign

encourages patients to take a role in preventing health care errors by becoming active, involved, and informed participants on the health care team.

Preload

end-diastolic pressure

Yellow wound

exudate & yellow fibrous debris; treat with moist-to-moist dressing

Hydrocolloid

forms a gel while fluid is absorbed; supports granulation; debrides necrotic wounds; left in place 3-5 days; use on shallow or moderate deep dermal ulcer

Pressure ulcer

impaired skin integrity related to unrelieved, prolonged pressure`

Patient-Centered Care:

important with nurses making patients and families their partners in recognizing fall risks and taking preventive action.

vector transmission

internal and external transmissions

droplet transmission

large particles that travel up to 3 feet and come in contact with the host

Culturing a wound

never culture from old drainage; clean with normal saline; apply pressure and moisten tip of cotton sway in culturette; return swab into tube containing a medium for organism growth; label, bad, and deliver to lab

Do you shake a dry-powered inhaler?

no

endogenous

occurs when part of the pateint's flora becomes altered and an overgrowth results

Serosanguineous wound exudate

pale pink & watery

Transient or normal hyperemia (blanching hyperemia)

press fingertip over affected area, skin blanches, then returns to red color; it is the body's attempt to overcome ischemia

Evisceration

protrusion of visceral organs through a wound opening; medical emergency;

Droplet Precautions

rubella, strep., MSA pneumonia, mumps

Partial thickness wound

shallow wounds involving loss of the epidermis (top layer) and possibly partial loss of the dermis.

Staples

small metal strips that hold the wound closed

Steri-strips

small pieces of fabric; tape that holds the edges of a wound together as it heals; useful for minor lacerations; less traumatic for children; falls off in about 10 days

Incision

smooth tissue cut

Huff cough

stimulates a natural cough reflex and is generally effective only for clearing central airways. While exhaling, the patient opens the glottis by saying the word huff. With practice he or she inhales more air and is able to progress to the cascade cough.

Compliance

the ability of the lungs to distend or expand in response to increased intraalveolar pressure

Tidal volume

the amount of air exhaled after normal inspiration.

Examples of heat therapy

warm compresses, warm soaks, sitz baths, commercial hot packs, hot water bottle; electric heating pad

Tendon

white, fibrous bands of tissue that connect muscle to bone at the origin or insertion of the muscle.

Question 2. A nurse is cleaning an emesis basin containing purulent material. What should the nurse do first? 1. Wash the basin with hot, soapy water. 2. Rinse the basin with cold running water. 3. Clean the basin with an antiseptic agent. 4. Spray the basin with a disinfectant and let it work for several minutes.

2

Which word is most closely associated with nursing care strategies to maintain functional alignment when patients are bed bound? 1. Endurance 2. Strength 3. Support 4. Balance

3

Which chemicals promote sleep? 1 Orexins 2 Hypocretins 3 L-tryptophan 4 Catecholamines

3 L-tryptophan

The nurse is assisting a patient to use a bedpan. What is the most important nursing intervention? 1.Dusting powder on the rim before placing the bedpan down under the patient 2.Positioning the rounded rim of the bedpan toward the front of the patient 3.Ensuring that the bedside rails are raised once the patient is on the bedpan 4. Encouraging the patient to help as much as possible when using the bedpan

3.Ensuring that the bedside rails are raised once the patient is on the bedpan

Nonrapid eye movement (NREM) sleep is divided into how many stages? Record your answer using a whole number. _________

4

27. A nurse is caring for a patient who has a vaginal radiation implant. What essential action should the nurse implement when caring for the patient? 1. Maintain the patient in the semi-Fowler position. 2. Talk while standing at the foot of the patient's bed. 3. Have the patient wear a detection badge while receiving care. 4. Wear a lead apron when providing direct nursing care to the patient. Hint

4 A lead apron provides a barrier between the nurse and the source of the radiation; it adheres to the principle of shielding.

A patient states that when turning on an electric radio a strong electrical shock was felt. What should the nurse do first? 1.Arrange for the maintenance department to examine the radio 2.Disconnect the radio from the source of energy 3.Check the skin for electrical burns 4. Take the patient's apical pulse

4. Take the patient's apical pulse

53. A nurse is teaching a group of children about the importance of firearm safety because of the severity of unintentional firearm injuries involving children. Place the steps in order that children should take when they see a gun. 1. Tell an adult. 2. Don't touch it. 3. Leave the area. 4. Stop the activity. 5. Identify the presence of a gun.

5 4 2 3 1

Risks in the Health Care Agency

- Medical errors - TJC and CMS "Speak Up" campaign - National Quality Forum mission - Environmental risks

The nurse is gathering a sleep history from a patient who is being evaluated for obstructive sleep apnea. Which common symptoms does the patient most likely report? Select all that apply. 1 Headache 2 Early wakening 3 Excessive daytime sleepiness 4 Difficulty falling asleep 5 Snoring

1 Headache 3 Excessive daytime sleepiness 5 Snoring

inflammatory exudate

accumulation of fluid and dead tissue cells, WBCs form at the site. Exudate may be serious, sanguineous, or purulent

Identify the strategies needed to provide safe nursing care. (3)

- Demonstrate effective use of technology ad standardized practices that support safety and quality - Demonstrate effective use of strategies to reduce the risk of harm to self or others - Use appropriate strategies to reduce reliance on memory

A 40-year-old patient is hospitalized for insomnia. The nurse reviews his medical records. The nurse finds that the patient has had insomnia for six months, and it started when he began working night shifts. After thoroughly questioning the patient, the nurse learns that he is stressed due to his workload. Which sleep deprivation symptoms will the nurse most likely find in the patient? Select all that apply. 1 Irritability 2 Agitation 3 Increased reflexes 4 Decreased sensitivity to pain 5 Decreased auditory and visual alertness

1 Irritability 2 Agitation 5 Decreased auditory and visual alertness

When using ice massage for pain relief, which procedures are correct? Select all that apply. 1 Apply ice using firm pressure over the skin. 2 Apply ice until numbness occurs and remove the ice for 5 to 10 minutes. 3 Apply ice until numbness occurs and discontinue application. 4 Apply ice for no longer than 10 minutes. 5 The ice is applied directly to the surface of the skin.

1, 2 Cold therapies are particularly effective for pain relief. Ice massage involves applying a frozen cup of ice firmly over the skin, which is covered with a lightweight cloth. When numbness occurs, remove the ice for usually 5 to 10 minutes.

The electrocardiogram of an elderly male patient who had chest pain shows signs of myocardial infarction. What are the likely sites for referred pain for a male patient with myocardial infection? Select all that apply. 1 Jaw 2 Groin 3 Left ear 4 Left arm 5 Left shoulder

1, 4, 5 When pathological changes in one part cause pain at a distant site on the body, then the pain is called referred pain. Pathological changes in the heart often cause referred pain in the jaw, left arm, and left shoulder, but they do not usually cause pain in the groin or left ear.

The scope of nursing practice is legally defined by: 1. State Nursing Practice Act 2. Professional nursing organization 3. Hospital policy and procedure manuals 4. Health care providers in the employing institution.

1. Determine the legal boundaries within each state.

A nurse is assigned to care for the following patients. Which of the patients is most at risk for developing skin problems and thus requiring thorough bathing and skin care?

1. A 44-year-old female who has had removal of a breast lesion and is having her menstrual period. 2. A 56-year-old male patient who is homeless and admitted to the emergency department with malnutrition and dehydration and who has an intravenous line. 3. A 60-year-old female who experienced a stroke with right-sided paralysis and has an orthopedic brace applied to the left leg. 4. A 70-year-old patient who has diabetes and dementia and has been incontinent of stool. 4

The nurse delegates needed hygiene care for an elderly stroke patient. Which intervention would be appropriate for the nursing assistive personnel to accomplish during the bath?

1. Checking distal pulses. 2. Providing range-of-motion (ROM) exercises to extremities. 3. Determining type of treatment for stage 1 pressure ulcer. 4. Changing the dressing over an intravenous site. 2

When a nurse delegates hygiene care for a male patient to a nursing assistive personnel, the NAP must use an electric razor to shave the patient with the following diagnosis:

1. Congestive heart failure. 2. Pneumonia. 3. Arthritis. 4. Thrombocytopenia. 4

The student nurse is teaching a family member the importance of foot care for his or her mother, who has diabetes. Which safety precautions are important for the family member to know to prevent infection? (Select all that apply).

1. Cut nails frequently. 2. Assess skin for redness, abrasions, and open areas daily. 3. Soak feet in water at least 10 minutes before nail care. 4. Apply lotion to feet daily. 5. Clean between toes after bathing. 2, 4, and 5

A patient prefers to remain in the low-Fowler position the majority of the time. What is the greatest potential problem associated with the low-Fowler position? 1. Pressure on the ischial tuberosities of the pelvis 2. Dorsiflexion contractures of the feet 3. External rotation of the hips 4. Adduction of the legs

1. In the low-Fowler position, the majority of the body's weight is borne by portions of the pelvis: bony protu- berances of the lower portion of the ischium (ischial tuberosities) and the triangular bone at the dorsal part of the pelvis (sacrum).

four stages of an infection

1. Incubation: entrance of pathogen and first symptoms 2. Prodromal: onset of nonspecific signs and symptoms to more specific symptoms 3. Illness: manifests signs and symptoms to type of infection 4. Convalescence: acute symptoms of infection disapear

The nurse raises a patient's arm over the head during range-of-motion exercises. What word should the nurse use when documenting exactly what was done during range-of-motion exercises? 1. Flexion 2. Supination 3. Opposition 4. Hyperextension

1. The shoulder, a ball-and-socket joint, flexes by raising the arm from a position by the side of the body forward and upward to a position beside the head.

The American Dental Association suggests that patients who are at risk for poor hygiene use the following interventions for oral care: (Select all that apply).

1. Use antimicrobial toothpaste. 2. Brush teeth 4 times a day. 3. Use 0.12% chlorhexidine gluconate (CHG) oral rinses. 4. Use a soft toothbrush for oral care. 5. Avoid cleaning the gums and tongue. 1, 3, and 4

What clinical manifestation indicates that a further nursing assessment is necessary to determine if the patient is having difficulty swallowing? 1.Debris in the buccal cavity 2.Abdominal cramping 3. Epigastic pain 4. Constipation

1.Debris in the buccal cavity

Which nursing intervention enhances an older adult's sensory perception and thereby helps prevent injury when walking from the bed to the bathroom? 1.Providing adequate lighting 2.Raising the pitch of the voice 3.Holding onto the patient's arm 4.Removing environmental hazards

1.Providing adequate lighting

Question 9. A nurse is assessing a patient's wound. Which characteristic of the wound's exudate indicates to the nurse that the wound may be infected? 1. Serous 2. Purulent 3. Sanguineous 4. Serosanguineous

2

The nurse is assessing a patient who had been administered morphine for pain relief. The nurse finds that the patient's respiratory rate is 5 breaths/minute. Which drug would be the most helpful in reversing this adverse effect? 1 Meperidine 2 Naloxone 3 Flumazenil 4 Metoclopramide

2

7. An emergency department nurse is caring for a patient in respiratory distress. The nurse anticipates that an oxygen-delivery system that provides the greatest percentage of oxygen will be ordered. Which type of oxygen mask should the nurse obtain? 1) Partial rebreather mask 2) Nonrebreather mask 3) Simple face mask 4) Venturi mask

2 A nonrebreather mask delivers from 80 to 100 percent concentrations of oxygen.

In a hospital, a use of restraint is ordered and renewed every two hours. What might be the age of the patient? 1 8 years 2 15 years 3 21 years 4 35 years

2 In hospital settings, each original restraint order and renewal is limited to 8 hours for adults, 2 hours for ages 9 to 17, and 1 hour for children under age 9. Therefore, a 15-year-old patient will require ordering and renewing of the restraint order every 2 hours. The 8-year-old child will require ordering and renewal every 1 hour. The 21-year-old and 35-year-old patients will require renewal every 8 hours.

A patient has been experiencing prolonged immobility because of a brain attack resulting in a coma. For which local response should the nurse monitor the patient? 1. Renal calculi 2. Contractures 3. Thrombophlebitis 4. Pathological fracture

2. A contracture is a localized response to immobility. When muscle fibers are not able to shorten or lengthen, eventually a permanent shortening of the muscles, tendons, and ligaments occurs.

When do contaminated wounds tend to show infection?

2nd or 3rd day after injury

A nurse is caring for a patient with a high fever secondary to septicemia. The primary health-care provider orders a cooling blanket (hypothermia blanket). Through which mechanism does the hypothermia blanket achieve heat loss? 1. Radiation 2. Convection 3. Conduction 4. Evaporation

3. Conduction

The patient reports episodes of sleepwalking to the nurse. Through understanding of the sleep cycle, the nurse recognizes that sleepwalking occurs during which sleep phase? 1. Rapid eye movement (REM) sleep 2. Stage 1 nonrapid eye movement (NREM) sleep 3. Stage 4 NREM sleep 4. Transition period from NREM to REM sleep

3. Stage 4 NREM sleep

The nurse is assessing a patient who is being admitted to the hospital. Which is the most important information collected by the nurse that indicates whether the patient is at risk for physical injury? 1.Weakness experienced during a prior admission 2.Medication that increases intestinal motility 3.Two recent falls that occured at home 4. The need for corrective eyeglasses

3.Two recent falls that occured at home

Technique to assess Musculoskeletal hazards of immobility and their abnormal findings

Inspection: decreased range of motion, Palpation: Joint contracture

Postural drainage position for adult left upper lobe - posterior lung segment

Side-lying with left side of chest elevated on pillows

Nursing intervention to reduce impact of immobility on the Respiratory System

a. Deep Breathe and cough every 1 to 2 hours b. Chest physiotherapy (CPT) c. Ensure intake of 1400 mL/day of fluid

The severity of a patient's illness depends on all of the following except: a. incubation period b. extent of infection c. susceptibility of the host d. pathogenicity of the microorganism

A. incubation period; The incubation period is the interval between the entrance of the pathogen into the body and appearance of first symptoms

15. A nurse is assessing a patient who started to have severe pain 3 days ago. When the nurse asks the patient to describe the pain, the patient states, "The pain feels like it is in my stomach. It is a burning pain, and it spreads out in a circle around the spot where it hurts the most." What type of pain does the nurse document that the patient is having at this time? a. Superficial pain b. Idiopathic pain c. Chronic pain d. Visceral pain

ANS: D Visceral pain comes from visceral organs, such as those from the gastrointestinal tract. Visceral pain is diffuse and radiates in several directions. Superficial pain has a short duration and is usually a sharp pain. Pain of an unknown cause is called idiopathic pain. Chronic pain lasts longer than 6 months.

Postural drainage position for child bilateral middle lobes - anterior lung segments

Sitting on nurse's lap, leaning against nurse

A nurse is caring for an adult patient who is scheduled for surgery. Which action is not associated with the nurse ensuring a valid, legal consent? 1. Observe the patient signing the consent form 2. Ensure that the patient is signing the consent form voluntarily 3. Assess if the patient has the capacity to make informed consent 4. Review with the patient the risks and benefits of the surfer before the consent is signed.

4

A patient is diagnosed with a stage IV pressure ulcer with eschar. Which medical treatment should the nurse anticipate the physician will order for this patient? 1. Heat lamp treatment three times a day 2. Application of a topical antibiotic 3. Cleansing irrigations twice daily 4. Débridement of the wound

4

Which safety precaution should be taken by the patient with muscle weakness while walking? 1 Using side rails 2 Using crutches 3 Using a belt restraint 4 Wearing rubber-soled shoes

4 Rubber-soled shoes are used by the patients with muscle weakness because they provide better grip on the floor. Side rails are placed on the sides of the bed to help patients in sitting and standing, but would not help the patients with ambulation. Crutches are assistive aids used by the patient who are unable to walk without support. Belt restraints are not used to support ambulation.

The nurse is caring for a patient with hyperthyroidism. What is the nurse likely to observe in the patient? 1 Fatigue 2 Chest pain 3 Sleepwalking 4 Difficulty falling asleep

4 Difficulty falling asleep

In a situation in which there is insufficient staff to implement competent care, a nurse should: 1.Organize a strike 2.Refuse the assignment 3.Inform the patients of the situation 4.Accept the assignment but make a protest in writing to the administration

4. Accept the assignment but make a protest in writing to the administration

The nurse turns the palm of a patient's hand downward when performing range-of-motion exercises. What word should the nurse use when documenting exactly what was done? 1. External rotation 2. Circumduction 3. Lateral flexion 4. Pronation

4. Pronation of the hand occurs by rotating the hand and arm so that the palm of the hand is facing down toward the floor.

A nurse is transferring a patient from the bed to a wheelchair using a mechanical lift. Which is a basic nursing intervention associated with this procedure? 1. Lock the base lever in the open position when moving the mechanical lift 2. Raise the mechanical lift so that the patient is six inches off the mattress 3. Keep the wheels of the mechanical lift locked throughout the procedure 4. Ensure the patient's feet are protected when on the mechanical lift

4. The legs dangle from the sling and therefore may drag across the linens or hit other objects if not protected.

A nurse educator is teaching a group of newly hired nursing assistants. Which patient should they be taught is at the greatest risk for injury? 1.School-aged children 2.Comatose teenager 3.Postmenopausal woman 4.Confused middle-aged man

4.Confused middle-aged man

What should be the maximum 24-hour dose of acetaminophen for an adult patient whose liver and kidney function tests are normal? Record your answer using a whole number, and please note that no comma is needed. ___ mg

4000 Acetaminophen is one of the safest analgesics available. However, its mode of action is unknown. The maximum 24-hour dose given to an adult with no kidney or liver diseases is 4000 mg.

A 6-year-old boy is admitted to the pediatric unit with chills and a fever of 104°F (40°C). What physiological process explains why the child is at risk for developing dyspnea? A) Fever increases metabolic demands, requiring increased oxygen need. B) Blood glucose stores are depleted, and the cells do not have energy to use oxygen. C) Carbon dioxide production increases as result of hyperventilation. D) Carbon dioxide production decreases as a result of hypoventilation.

A

A patient was admitted after a motor vehicle accident with multiple fractured ribs. Respiratory assessment includes signs/symptoms of secondary pneumothorax, which includes which of the following? A) Sharp pleuritic pain that worsens on inspiration B) Crackles over lung bases of affected lung C) Tracheal deviation toward the affected lung D) Increased diaphragmatic excursion on side of rib fractures

A

The nurse is caring for a patient who exhibits labored breathing and uses accessory muscles. The patient has crackles in both lung bases and diminished breath sounds. Which would be priority assessments for the nurse to perform? (Select all that apply.) A) SpO2 levels B) Amount of sputum production C) Change in respiratory rate and pattern D) Pain in lower calf area

A. SpO2 levels B. Amount of sputum production C. Change in respiratory rate and pattern

Mixed wound color

treat black, then yellow, and lastly red

Puncture

typically causes by an object piercing the skin

A patient is diagnosed with a stage IV pressure ulcer with eschar. Which medical treatment should the nurse anticipate the physician will order for this patient? 1. Heat lamp treatment three times a day 2. Application of a topical antibiotic 3. Cleansing irrigations twice daily 4. Débridement of the wound

4. Thick, leather-like, necrotic devitalized tissue (eschar) must be removed surgically or enzymatically before wound healing can occur.

A nurse is assessing a patient experiencing acute pain. Which characteristic is more common with acute pain than with chronic pain? 1. Self-focusing 2. Sleep disturbances 3. Guarding behaviors 4. Variations in vital signs

4. Variations in vital signs Acute pain stimulates the sympathetic nervous system, which responds by increasing pulse, respirations, and blood pressure. Chronic pain stimulates the parasympathetic nervous system, which results in lowered pulse and blood pressure.

Risk factors for pressure ulcer development

Impaired sensory perception, impaired mobility, alteration in level of consciousness, shear, friction, and moisture

Hand Rolls

Maintain the thumb in slight adduction and in opposition to the fingers

A nurse places a patient with a sacral pressure ulcer in the left Sims position. How should the nurse position the patient's right arm? 1. On a pillow 2. Behind the back 3. With the palm up 4. In internal rotation

1

A 47-year-old man complains of sleep deprivation. What findings should the nurse expect on assessment? Select all that apply. 1 Ptosis 2 Nystagmus 3 Decreasd reflexes 4 Fine-motor clumsiness 5 Gross-motor clumsiness

1 Ptosis 3 Decreasd reflexes 4 Fine-motor clumsiness

A nurse is assisting a patient to eat. Which are common nursing interventions that support patient safety when the nurse is feeding the patient? Select all that apply. 1) Elevating the head of the bed 2) Serving small amounts of food at a time 3) Avoiding talking while the patient is eating 4) Providing ample time to chew and swallow 5) Keeping the upper side rails in the raised position 6) Ensuring that dentures are in place and fit comfortably

1 2 4 6

A patient is diagnosed with chronic fatigue syndrome. Which is most important for the nurse to explore in relation to the patient's status? 1. Ability to provide self-care 2. Physical mobility 3. Social isolation 4. Gas exchange

1. Ability to provide self-care Chronic fatigue syndrome is a condition characterized by the onset of disabling fatigue.

A school nurse is teaching a class of adolescents about the function of the integumentary system. Which fact about how the skin protects the body against infection is important to include in this discussion? 1. Cells of the skin are constantly being replaced, thereby eliminating external pathogens 2. Epithelial cells are loosely compacted on skin, providing a barrier against pathogens 3. Moisture on the skin surface prevents colonization of pathogens 4. Alkalinity of the skin limits the growth of pathogens

1. Cells of the skin are constantly being replaced, thereby eliminating external pathogens

A resident brings several electronic devices to a nursing home. One of the devices has a two-pronged plug. What rationale should the nurse provide when explaining why an electrical device must have a three pronged plug? 1. Controls stray electrical currents 2. Promotes efficient use of electricity 3. Shuts off the appliance if there is an electrical surge 4. Divides the electricity among the appliances in the room.

1. Controls stray electrical currents

An 88-year-old patient comes to the medical clinic regularly. During a recent visit the nurse noticed that the patient had lost 10 lbs in 6 weeks without being on a special diet. The patient tells the nurse that he has had trouble chewing his food. Which of the following factors are normal aging changes that can affect an older adult's oral health? (Select all that apply).

1. Dentures do not always fit properly. 2. Most older adults have an increase in saliva secretions. 3. With aging the periodontal membrane becomes tighter and painful. 4. Many older adults are edentulous, and remaining teeth are often decayed. 5. The teeth of elderly patients are more sensitive to hot and cold. 1 and 4

A patient with a malignant brain tumor requires oral care. The patient's level of consciousness has declined, with the patient only being able to respond to voice commands. Place the following steps in the correct order for administration of oral care.

1. If patient is uncooperative or having difficulty keeping mouth open, insert an oral airway. 2. Raise bed, lower side rail, and position patient close to side of bed with head of bed raised up to 30 degrees. 3. Using a brush moistened with chlorhexidine paste, clean chewing and inner tooth surfaces first. 4. For patients without teeth, use a toothette moistened in chlorhexidine rinse to clean oral cavity. 5. Remove partial plate or dentures if present. 6. Gently brush tongue but avoid stimulating gag reflex. 2, 5, 1, 3, 6, and 4

A nurse turns a patient's ankle so that the sole of the foot moves medially toward the midline. What word should the nurse use when documenting exactly what was done during range-of-motion exercises? 1. Inversion 2. Adduction 3. Plantar flexion 4. Internal rotation

1. Inversion, a gliding movement of the foot, occurs by turning the sole of the foot medially toward the midline of the body.

The practitioner orders a vest restraint for a patient. What should the nurse do first when applying this restraint? 1. Perform an inspection of the patient's skin where the restraint is to be placed. 2.ensure that the back of the vest is positioned on the patient's back 3. Permit four fingers to slide between the patient and the restraint 4.Secure the restraint to the bed frame using a slipknot

1. Perform an inspection of the patient's skin where the restraint is to be placed.

A nurse places a patient in the orthopneic position. What is the primary reason for the use of this position? 1. Facilitate respirations 2. Support hip extension 3. Prevent pressure ulcers 4. Promote urinary elimination

1. Sitting in the high-Fowler position and leaning forward allows the abdominal organs to drop by gravity, which promotes contraction of the diaphragm. The arms resting on an over-bed table increases thoracic excursion.

order of steps for performing a sterile procedure

1. assemble all equipemnt 2. don caps, masks and eyewear 3. open sterile packages 4. open sterile items on a flat surface 5. open a sterile item while holding it 6. prepare sterile field 7. pour sterile solutions 8. surgical scrub 9. apply sterile gloves. 10. don a sterile gown.

Question 1. A practitioner orders a wound to be packed with a wet-to-damp gauze dressing. What should the nurse explain to the patient is the primary reason for this type of dressing? 1. Minimizes the loss of protein 2. Facilitates the healing process 3. Increases resistance to infection 4. Prevents the entry of microorganisms

2

Which psychological symptoms are associated with a sleep deprived patient? Select all that apply. 1 Ptosis 2 Agitation 3 Irritability 4 Disorientation 5 Cardiac arrhythmias

2 Agitation 3 Irritability 4 Disorientation

The nurse is learning about nursing malpractice. Which statements should the nurse consider as good nursing practice? Select all that apply. 1 Nursing notes can be illegible as long as the nurse can read them. 2 Nursing notes should be maintained properly. 3 Nursing records should be properly preserved. 4 Institutions should maintain complete nursing records. 5 Only basic duties should be recorded in nursing records

2, 3, 4 Nursing notes are important, because they contain evidence needed to understand the care received by a patient. The notes should be legible to all people, not just the nurse. Similarly, complete nursing records should be properly preserved, because they may be required to show what care was provided. Basic nursing duties as well as advanced nursing care should be recorded in the nursing records.

A nurse is giving a back rub. Which stroke is most effective in inducing relaxation at the end of the procedure? 1. Percussion 2. Effleurage 3. Kneading 4. Circular

2. Effleurage Effleurage involves long, smooth strokes sliding over the skin. When performed slowly with light pressure at the end of a back rub it has a relaxing, sedative effect.

Which is the most important nursing intervention that supports a patient's ability to sleep in the hospital setting? 1. Providing an extra blanket 2. Limiting unnecessary noise on the unit 3. Shutting off lights in the patient's room 4. Pulling curtains around the patient's bed at night

2. Limiting unnecessary noise on the unit Noise is a serious deterrent to sleep in a hospital. The nurse should limit environmental noise (e.g., distributing fluids, providing treatments, rolling drug and linen carts) and staff communication noise.

A patient has a cast from the hand to above the elbow because of a fractured ulna and radius. After the cast is removed, the nurse teaches the patient active range-of-motion exercises. Which patient action indicates that further teaching is necessary? 1. Moves the elbow to the point of resistance 2. Keeps the elbow flexed after the procedure 3. Assesses the elbow's response after the procedure 4. Puts the elbow through its full range at least three times

2. This is undesirable because it contributes to a flexion contracture. Functional alignment is preferred because it minimizes stress and strain on muscles tendons, ligaments, and joints.

The nurse notices that a patient has received oxycodone/acetaminophen (5/325), two tablets PO every 3 hours for the past 3 days. What concerns the nurse the most? 1 The patient's level of pain 2 The potential for addiction 3 The amount of daily acetaminophen 4 The risk for gastrointestinal bleeding

3 The major adverse effect of acetaminophen is hepatotoxicity. The maximum 24-hour dose is 4 g. It is often combined with opioids (e.g., oxycodone) because it reduces the dose of opioid needed to achieve successful pain control.

Which statement made by an older adult est demonstrates understanding of taking sleep medication? 1. "I'll take the sleep medicine for 4 or 5 weeks until my sleep problems disappear." 2. "Sleep medicines won't cause any sleep problems once I stop taking them." 3. "I'll talk to my health care provider before I use an over-the-counter sleep medication." 4. "I'll contact my health care provider if I feel extreme sleepy in the mornings."

3. "I'll talk to my health care provider before I use an over-the-counter sleep medication."

The nurse is taking a sleep history from a patient. Which statement made by the patient needs further follow-up? 1. "I feel refreshed when I wake up in the morning." 2. "I use soft music at night to help me relax." 3. "It takes me about 45 to 60 minutes to fall asleep." 4. "I take pain medication for my leg pain about 30 minutes before I go to bed."

3. "It takes me about 45 to 60 minutes to fall asleep."

During which time frame do people tend to be the sleepiest? 1. 12 noon and 2 p.m. 2. 6 a.m. and 8 a.m. 3. 2 a.m. and 4 a.m. 4. 6 p.m. and 8 p.m.

3. 2 a.m. and 4 a.m. Research demonstrates that most people experience sleep-vulnerable periods between 2 a.m. and 6 a.m. and between 2 p.m. and 5 p.m.

A nurse is performing passive range-of-motion exercises for a patient who is in the supine position. Which motion occurs when the nurse bends the patient's ankle so that the toes are pointed toward the ceiling? 1. Supination 2. Adduction 3. Dorsal flexion 4. Plantar extension

3. Dorsal flexion (dorsiflexion) of the joint of the ankle occurs when the toes of the foot point upward and backward toward the anterior portion of the lower leg.

A patient with a history of thrombophlebitis should not have pressure exerted on the popliteal space. In what position should the nurse avoid placing this patient? 1. Prone 2. Supine 3. Contour 4. Trendelenburg

3. In the contour position the head of the bed and the knee gatch are slightly elevated. The elevated knee gatch puts pressure on the popliteal spaces.

A nurse is caring for patients receiving a variety of interventions for pain management. Which pain relief method has the shortest duration of action? 1. Patient-controlled analgesia 2. Intramuscular sedatives 3. Intravenous narcotics 4. Regional anesthesia

3. Intravenous narcotics Intravenous analgesics act within 1 to 2 minutes but drug inactivation (biotransformation) also is fast, so there is a short duration of action.

A nurse concludes that a patient has the potential for impaired mobility. Which assessment reflects a risk factor that may have precipitated this conclusion? 1. Exertional fatigue 2. Sedentary lifestyle 3. Limited range of motion 4. Increased respiratory rate

3. Limited range of motion is associated with contracture formation and impaired mobility.

A patient states, "The pain moves from my chest down my left arm." Which characteristic of pain is associated with this statement? 1. Pattern 2. Duration 3. Location 4. Constancy

3. Location This is referred pain, which is pain felt in a part of the body that is at a distance from the tissues causing the pain. Referred pain is related to location of pain.

A nurse is concerned about a patient's ability to withstand exposure to pathogens, which blood component should the nurse monitor? 1. Platelets 2. Hemoglobin 3. Neutrophils 4. Erythrocytes

3. Neutrophils

The fatigue is so overwhelming and consuming that it interferes with the activities of daily living. Which is most important for nurses to understand when caring for patients in pain? 1. Patients who are in pain will request pain medication. 2. Patients usually are able to describe the characteristics of their pain. 3. Patients need to know that the nurse believes what they say about their pain. 4. Patients will demonstrate vital signs that are congruent with the intensity of their pain.

3. Patients need to know that the nurse believes what they say about their pain. Pain is a personal experience, and the nurse must validate its presence and severity as perceived by the patient. This conveys acceptance and respect and promotes the development of trust.

55. A health team member is using a type C fire extinguisher to put out a fire in a health-care facility. What kind of fire is the health team member attempting to extinguish because a type C fire extinguisher is the only extinguisher that should be used in this situation? 1. Burning material in a garbage can 2. Smoke from a rag in a maintenance closet 3. Smoldering sparks from a patient's mattress 4. Flames emanating from a toaster in a pantry

4 A toaster is an electrical appliance; fire involving live electrical wires or equipment is extinguished with a type C or type ABC fire extinguisher.

The physician tells the nurse to administer a second drug to a patient already on oxycodone. Which drug would be safe for this patient? 1 Fentanyl 2 Morphine 3 Codeine 4 Acetaminophen

4 Acetaminophen is safe to use in combination with opioids like oxycodone for pain relief. A health care provider may do so to lower the opioid dose. Fentanyl, morphine, and codeine are also opioids, so combining them with another opioid would have potentially dangerous synergistic effects.

Four major areas for assessment of patient mobility:

a. Range of motion b. Gait c. Exercise and activity tolerance d. Body alignment

Identify the three components to assess mobility.

a. Range of motion b. Gait c. Exercise

Procedure-Related Accidents:

accidents occur during therapy - includes medication administration errors, IV therapy errors, improper application of external devices, and improper performance of procedures. - The potential for infection is reduced when surgical asepsis is used for sterile dressing changes or any invasive procedure such as insertion of a urinary catheter.

The nurse caring for a male patient observes the nursing assistive person (NAP) performing perineal care. Which observed action indicates a need for further teaching for the NAP? a. The NAP used clean gloves. b. The NAP did not retract the foreskin before cleansing. c. The NAP used the clean portion of a washcloth for each cleansing wipe. d. The NAP used a circular motion to cleanse from urinary meatus outward.

b. The NAP did not retract the foreskin before cleansing. Secretions collect beneath the foreskin and can promote bacterial growth if not removed. Using clean gloves, clean portions of the washcloth for each cleansing wipe, and using a circular motion to cleanse from the urinary meatus outward are all appropriate measures while performing perineal care. p. 859

vehicles transmission

contaminated items

Penrose drain

soft, flat rubber tubing; a pin or clip is placed through the drain to prevent it from slipping; the drain is pulled or advanced as drainage decreases to permit healing deep within the drainsite.

Periwound

the condition of the surrounding tissue of the ulcer

Oxygen saturation

the percentage of hemoglobin that is bound with oxygen

Side Rails:

used to prevent a patient, such as one who is sedated, from falling out of bed are not considered a restraint. - Increase patient mobility and/or stability - Most commonly used as restraint - Can cause falls or death

The nurse is teaching a group of nursing assistive personnel about the types of fire extinguishers. Which type of fire can be extinguished using a type A fire extinguisher? Select all that apply. 1 Wood 2 Cloth 3 Gasoline 4 Paper 5 Paint

1, 2, 4 A type A fire extinguisher is used to extinguish fire caused by ordinary combustible materials such as wood, cloth, or paper. Fire caused by gasoline and paint is extinguished by type B fire extinguishers.

What is the correct order of steps for safely using a fire extinguisher in the home? 1. Squeezing the handle 2. Pulling the pin to unlock the handle 3. Aiming low at the base of the fire 4. Sweeping the unit from side to side

2, 3, 1, 4 The nurse should instruct the patient to memorize the PASS technique for safely using a fire extinguisher: pulling the pin to unlock the handle, aiming low at the base of the fire, squeezing the handle, and sweeping the unit from side to side.

The nurse is caring for a patient who has decreased mobility. Which intervention is a simple and cost-effective method for reducing the risks of stasis of pulmonary secretions and decreased chest wall expansion? A) Antibiotics B) Frequent change of position C) Oxygen humidification D) Chest physiotherapy

B

A nurse is evaluating an ambulating patient's balance. What factor about the patient is most important for the nurse assess? 1. Posture 2. Strength 3. Energy level 4. Respiratory rate

1

A nurse is caring for patients with a variety of wounds. Which wounds will likely heal by primary intention? Select all that apply. 1. Cut in the skin from a kitchen knife. 2. Excoriated perianal area 3. Abrasion of the skin 4. Surgical incision 5. Pressure ulcer

1. Cut in the skin from a kitchen knife 4. Surgical incision

Which should the nurse o to interrupt the transmission link in the chain of infection? 1. Wash the hands before providing care to a patient 2. Position a commode next to a patient's bed. 3. Provide education about a balanced diet 4. Change a dressing when it is soiled

1. Wash the hands before providing care to a patient.

Which age group is at the highest risk for accidental poisoning at home? 1 Adults 2 Toddlers 3 Older adults 4 Adolescents

2 oddlers, preschoolers, and young school-age children are at greater risk for accidental poisoning at home, as they tend to put objects in the mouth. Adults, older adults, and adolescents are generally not prone to accidental poisoning at home, because they are knowledgeable of poisonous substances.

A patient has a wound that is healing by secondary intention. Which solution to cleanse the wound and dressing should the nurse expect will be ordered to support wound healing? 1. Normal saline and a gauze dressing. 2. Normal saline and a wet-to-damp dressing 3. Povidone-iodine and a dry sterile dressing 4. Half peroxide and half normal saline and a wet-to-dry dressing.

2. Normal saline and a wet-to-dry dressing

A nurse identifies that a patient has an inflammatory response. Which localized patient response support this conclusion? Select all that apply. 1. Fever 2. Swelling 3. Erythema 4. Bradypnea 5. Tachycardia

2. Swelling 3. Erthema

A patient with a history of thrombophlebitis should not have pressure exerted on the popliteal space. In what position should the nurse avoid placing this patient? 1. Prone 2. Supine 3. Contour 4. Trendelenburg

3

While assessing a patient for joint mobility, the nurse notices that the patient is unable to dorsiflex the foot. Which condition does the nurse suspect in the patient? 1 Scoliosis 2 Torticollis 3 Joint contracture 4 Disuse osteoporosis

A patient with the type of joint contracture known as foot drop is permanently fixed in plantar flexion and is unable to dorsiflex the foot. Disuse osteoporosis refers to atrophy and decreased density of the bone tissue. Torticollis involves inclining the head to the affected side with the sternocleidomastoid muscle is contracted. Scoliosis refers to a lateral S- or C-shaped spinal column with vertebral rotation, and unequal heights of the hips and shoulders.

Which of the following is not an element in the development or chain of infection? a. Means of transmission b. Infectious agent or pathogen c. Formation of immunoglobulin d. reservoir for pathogen growth

C. Formation of immunoglobulin; infection occurs in a cycle that depends on the presence of certain elements

Explain immobility related respiratory change: Atelectasis

Collapse of Aveoli

Steps in CPR

Compression Airway Breathing

A patient experiences menopausal symptoms such as hot flashes, mood swings, and irritability. Which sleep disorder is likely to be found in the patient? 1 Nocturia 2 Insomnia 3 Obstructive apnea 4 Central sleep apnea

Correct2 Insomnia

Identify the complications of immobility in relation to the metabolic functioning of the body

Decreases the metabolic rate; alters the metabolism of CHO, fats, and proteins; causes fluid and electrolyte and calcium imbalances; and causes GI disturbances

Which is a common debilitating contracture? 1 Disuse 2 Atrophy 3 Footdrop 4 Shortening of the muscle

Footdrop is a common and debilitating contracture in which the foot is permanently fixed in plantar flexion. Disuse, atrophy, and shortening of muscle fibers are the causes of joint contractures.

Friction

Force that occurs in a direction to oppose movement

Describe how Damage to the nervous system affect mobility:

Impaired body alignment, balance and mobility

Increased cervical flexion is observed when the patient is placed in which position? 1 Sims' position 2 Prone position 3 Side-lying position 4 Supported Fowler's position

Increased cervical flexion may occur if a patient is placed in the supported Fowler's position, because the pillow at the head is too thick and the head thrusts forward. Patients positioned in the Sims' position may experience lack of foot support. Patients positioned in the prone position may suffer from neck hyperextension. Patients positioned in the side-lying position may suffer from lateral flexion of the neck.

Technique to assess Respiratory hazards of immobility and their abnormal findings

Inspection: Asymmetrical chest wall movement, Auscultation: Crackles

Technique to assess Skin hazards of immobility and their abnormal findings

Inspection: Break in skin integrity, Palpation: Skin

Technique to assess Elimination hazards of immobility and their abnormal findings

Inspection: Decreased urine output, Palpation: Distended abdomen, Auscultation: Decreased bowel sounds

Nursing assistive personnel (NAP) are applying antiembolic elastic stockings to the patient. Which instructions should the NAP give to the patient? Select all that apply. 1 Massage the legs if they ache. 2 Wear garters regularly. 3 Elevate the legs while sitting. 4 Make a habit of sitting cross-legged. 5 Avoid wrinkles in stockings.

It is necessary to elevate legs while sitting and before applying stockings to improve venous return. Antiembolic stockings that are free of wrinkles will fit the legs more properly. Massaging the legs may further deteriorate the condition or mobilize a thrombi, so massage should be avoided. Sitting cross-legged and wearing garters promote venous stasis and should be avoided.

An older adult who was in a car accident and fractured the femur has been immobilized for 5 days. Which nursing diagnosis is related to patient safety when the nurse assists this patient out of bed for the first time? 1 Chronic pain 2 Impaired skin condition 3 Risk for ineffective cerebral tissue bloodflow 4 Risk for inability to tolerate activity

Patients on bed rest are at risk for inability to tolerate activity, which increases patients' risk for falling. The patient is in acute pain, not chronic pain. The patient could have some skin breakdown, but this is not relevant to getting the patient out of bed. The patient's cerebral tissue bloodflow is not an issue in this situation.

Proprioception

The awareness of the position of the body and its parts.

Work of breathing

The effort required to expand and contract the lungs

Side-lying Position

The patient rests on the side with the body weight on the dependent hip and shoulder

Electrochemical Activity

The transmission of an impulse from the nervous system to the musculoskeletal system that requires a neurotransmitter to transfer signals

Laceration

Torn, jagged wound. Can cause severe bleeding.

Directed coughing

a deliberate maneuver that is effective when spontaneous coughing is not adequate

Complication of immobility in relation to the musculoskeletal system:

a. Loss of endurance, strength, and muscle, mass and decreased stability and balance b. Impaired calcium metabolism c. impaired joint mobility d. Osteoporosis e. Joint contractures f. Footdrop

What is the significant cause of bleeding in the oral cavity? a. Trauma b. Gingivitis c. Xerostomia d. Dental caries

a. Trauma Trauma is the significant cause of bleeding in the oral cavity. Gingivitis is caused by inflammation of the gums. Xerostomia is a condition of decreased saliva secretion in the mouth. Dental caries may be due to tooth decay produced by the interaction of food with bacteria. p. 823

Pursed lip breathing

involves deep inspiration and prolonged expiration through pursed lips to prevent alveolar collapse.

Chest percussion

involves rhythmically clapping on the chest wall over the area being drained to force secretions into larger airways for expectoration. Cannot perform on patients with broken ribs, osterporiuos, and bleeding disorders.

tissue repair

involves the defensive, reconstructive, and maturative stages

Airborne precautions

measles, chickenpox, varicella zoster, TB

Black wound

necrotic tissue; debride it

Granulation tissue

newly formed tissue with small capillaries and connective tissue; red, moist tissue appearing in the wound bed which indicates healing

Dressings for Pressure Ulcer Stage 1

none, transparent dressing, hydrocolloid

Expiration

passive process that depends on the elastic recoil properties of the lungs, requiring little or no muscle work

Why are the elderly prone to skin breakdown?

reduced skin elasticity; multiple medical conditions & polypharmacy; dermal-epidermal junction becomes flattened; reduced inflammatory response; loss of subcutaneous tissue; altered nutrition; reduced arterial & venous flow; reduced oil production by sebaceous glands

Perfusion

relates to the ability of the cardiovascular system to pump oxygenated blood to the tissues and return deoxygenated blood to the lungs

Ventilation

the process of moving gases into and out of the lungs

Surgical debridement

the removal of devitalized tissue by using a scalpel, scissors, or other sharp instrument. It is the quickest method of debridement. It is usually indicated when the patient has signs of cellulitis or sepsis.

A patient receiving chemotherapy experiences stomatitis. The nurse advises the patient to use:

1. Community mouthwash. 2. Alcohol-based mouth rinse. 3. Normal saline rinse. 4. Firm toothbrush. 3

Examples of cold therapy

aquathermia pad, moist cold compresses, cold pack, ice bad or collar, commercial cold packs

Patient-inherent accidents

are classified as self-induced. (seizures, burns, inflicted cuts)

Isotonic Contractions

contractions that generate force by changing the length of the muscle and can be concentric contractions or eccentric contractions. A concentric contraction causes muscles to shorten, thereby generating force.

Full thickness wound

extending into the dermis (involving both layers of tissue) heal by scar formation because deeper structures do not regenerate

Hemovac

round drain with springs inside that must be compressed to create a suction

osteomalacia

softening of the bones, typically through a deficiency of vitamin D or calcium.

A patient has a wound infection. Which local human responses should the nurse expect to identify? Select all that apply. 1. Neutropenia 2.Malaise 3. Edema 4. Fever 5. Pain

3. Edema 4. Pain

Normal pulse oximetry

95%-100%

Contusion

bruise

A nurse is caring for a group of hospitalized patients. Which should the nurse do first to prevent patient infections? 1. Provide small bedside bags to dispose of used tissues 2. Encourage staff to avoid coughing near patients 3. Administer antibiotics as prescribed 4. Identify patients at risk

$. Identify patients at risk

When brushing a patient's hair, the. Urge identifies white oval particles attached to the hair behind the ears. For which should the nurse assess the patient? 1. Pediculosis 2. Hirsutism 3. Dandruff 4. Scabies

1. Pediculosis

A patient is diagnosed with narcolepsy. Which class of drugs does the nurse anticipate will be in the patient's treatment plan to prevent sleep attacks? 1 Hypnotics 2 Stimulants 3 Anticonvulsants 4 Benzodiazepines

2 Stimulants

A patient with anemia who is 24 weeks pregnant complains to the nurse of recurrent, rhythmical movements of her feet and legs before the onset of sleep. What could be the reason for her condition? Select all that apply. 1 Renal failure 2 Gender 3 Pregnancy 4 Her gestational age 5 Lower level of iron

2 Gender 3 Pregnancy 5 Lower level of iron

Profuse smoke is coming out of the heating unit in a patient's room. What should the nurse do first? 1.Open the window 2.Activate the fire alarm 3.Move the patient out of the room 4.Close the door to the patient's room

3.Move the patient out of the room

Which group of patients is most at risk for severe injuries related to falls? 1 Adolescents 2 Older adults 3 Toddlers 4 Young children

2 Older adults Some older adults walk more slowly and are less coordinated. They also take smaller steps, keeping their feet closer together, which decreases the base of support. Thus, body balance is unstable, and they are at greater risk for falls and injuries.

Which is a congenital defect? 1 Arthritis 2 Scoliosis 3 Osteoporosis 4 Osteomalacia

2 Scoliosis Scoliosis is a structural curvature of the spine associated with vertebral rotation; it is a congenital defect. Arthritis is an inflammatory joint disease that causes systemic signs of inflammation and destruction of the synovial membrane and articular cartilage. Osteoporosis is an aging disorder that results in the reduction of bone density or mass. Osteomalacia is an uncommon metabolic disease characterized by inadequate and delayed mineralization, resulting in compact and spongy bones.

Question 19. A nurse is assessing a patient for a systemic response to an inflammatory process. For what responses should the nurse monitor the patient? Select all that apply. 1. Pain 2. Fever 3. Edema 4. Erythema 5. Leukocytosis

2, 5

Which nursing actions protect patients as susceptible hosts in the chain of infection? Select all that apply. 1. Wearing personal protective equipment 2. Administering childhood immunizations 3.Recapping a used needle before discarding 4. Instituting prescribed immunoglobulin therapy 5. Disposing of soiled gloves in a waste container

2. Administering childhood immunizations 4. Instituting prescribed immunoglobulin therapy

The nurse is attending to a patient diagnosed with restless leg syndrome (RLS). What are the characteristics of this syndrome? Select all that apply. 1 It is usually associated with nocturia. 2 It occurs immediately after waking up. 3 Patients have a continuous itching sensation. 4 It is more common in those having thrombocytopenia. 5 There is recurrent rhythmical movement of the feet and legs

3 Patients have a continuous itching sensation. 5 There is recurrent rhythmical movement of the feet and legs

A nurse in a community center is conversing with a group of older adults who voiced fears about falling. What is the most common consequence associated with older adults' fear of falling that the nurse should discuss with them? 1. Impaired skin integrity 2. Occurrence of panic attacks 3. Self-imposed social isolation 4. Decreased physical conditioning

4

Which term is used to explain the relationship of one body part to another along a horizontal or vertical line? 1 Friction 2 Body balance 3 Body alignment 4 Coordinated body movement

3 Body alignment Body alignment refers to the relationship of one body part to another along a horizontal or vertical line. Friction is a force that opposes movement. Body balance is a technique in which a relatively low center of gravity is balanced over a wide, stable base of support and a vertical line falls from the center of gravity through the base of support. Coordinated body movement is a result of weight, center of gravity, and balance.

A nurse plans to use a trochanter roll when repositioning a patient. Where should the nurse place the trochanter roll? 1. Under the small of the back 2. Behind the knees when supine 3. Alongside the ilium to mid-thigh 4. In the palm of the hand with the fingers flexed

3. A trochanter roll is a rolled wedge, pillow, or sandbag placed by the lateral aspect of the leg between the iliac crest and knees to prevent external hip rotation.

Which sleep-related side effect is seen with the use of antidepressants and anticonvulsants? 1 Nocturia 2 Daytime drowsiness 3 Decreased total sleep time 4 Suppressed rapid eye movement (REM) sleep

4 Suppressed rapid eye movement (REM) sleep

What event describes an impulse that is transmitted from the nervous system to the musculoskeletal system? 1 Isotonic contraction 2 Voluntary movement 3 Isometric contraction 4 Electrochemical activity

4 Electrochemical activity The transmission of an impulse from the nervous system to the musculoskeletal system is an electrochemical activity that requires a neurotransmitter to transfer signals. Isotonic contractions are related to muscles and not related to the nervous system. The transmission of impulses from the nervous system to the musculoskeletal system are automatic, not voluntary, movements. Isometric contractions are related to muscles and not related to the nervous system.

Explain immobility related cardiovascular change: Orthostatic Hypotension

A drop in blood pressure greater than 20 mm Hg in systolic pressure or 10 mm Hg in diastolic pressure

The nurse sets up a nonbarrier sterile field on the patient's over-bed table. In which of the following instances is the field contaminated? a. Sterile saline solution is spilled on the field b. The nurse, who has a cold, wears a double mask c. Sterile objects are kept within a 1-inch border of the field d. The nurse keeps the top of the table above his or her waist

A. Sterile saline solution is spilled on the field; If moisture leaks through a sterile package's protective covering, organisms can travel to the sterile object

Instrumental activities of Daily living (IADL)

Activities beyond ADLs that are necessary to be independent in society

Trapeze Bar

Allows the patient to pull with the upper extremities to raise the trunk off the bed, assist in transfer, or perform exercises

How would you assist patients with hemiplegia or hemiparesis

Always stand on the patient's affected side and support the patient by using a gait belt

Braden Scale: 15-18

At risk

Technique to assess Cardiovascular hazards of immobility and their abnormal findings

Auscultation: Orthostatic hypotension, Palpation: Peripheral Pulses

What is cartilage? 1 Connection between bones 2 White fibrous bands of tissues that connect muscles to bones 3 Nonvascular supporting connective tissue located chiefly in joints 4 Shiny white flexible bands of fibrous tissues that bind joints together

Cartilage is nonvascular supporting connective tissue located chiefly in joints and the thorax, trachea, nose, and ear. Joints are the connections between bones. Tendons are the white, glistening fibrous bands of tissues that connect muscles to bones, and are strong, flexible, and inelastic. Ligaments are shiny white flexible bands of fibrous tissues that bind joints together, connect bones and cartilage, and aid joint flexibility and support.

Disuse atrophy:

Cells and the tissue reduce in size and function in response to prolonged inactivity resulting from bed-rest, trauma, casting, or local nerve damage

Which intervention is appropriate to include on a care plan for improving sleep in the older adult? 1 Decrease fluids 2 to 4 hours before sleep. 2 Exercise in the evening to increase fatigue. 3 Allow the patient to sleep as late as possible. 4 Take a nap during the day to make up for lost sleep.

Correct1 Decrease fluids 2 to 4 hours before sleep

Which microorganism causes gas gangrene? 1 Escherichia coli 2 Neisseria gonorrheae 3 Staphylococcus aureus 4 Clostridium perfringens

Clostridium perfringens causes gas gangrene. Escherichia coli causes gastroenteritis and urinary tract infection. Neisseria gonorrhoeae causes gonorrhea and pelvic inflammatory disease. Staphylococcus aureus causes wound infection and pneumonia.

The nurse has a sleep disorder due to working rotating shifts. Which physiological symptoms are observed in the nurse? Select all that apply. 1 Blurred vision 2 Increased reflexes 3 Decreased judgment 4 Slowed response time 5 Increased visual alertness

Correct 1 Blurred vision Correct 3 Decreased judgment Correct 4 Slowed response time

While preparing a sterile field, a nurse opens the outermost flap by stretching his or her arm away from the sterile field. What is the reason for this action? 1 To ensure sterility of the package 2 To prevent a break in the technique 3 To avoid contamination of the sterile field 4 To free the dominant hand for unwrapping

During the preparation of a sterile field, the nurse's arm should be stretched away from the sterile field to avoid contamination of the field. Placing the pack of sterile drapes on the work surface ensures the sterility of the package. Assembling the necessary equipment before starting the procedure prevents a break in the technique. Opening the sterile item by holding the outside wrapper in the nondominant hand helps to free the dominant hand to unwrap the outer wrapper.

Which vaccinations are recommended to reduce the risk of infectious diseases in older adults? Select all that apply. 1 Flu vaccination 2 DTaP vaccination 3 Rubella vaccination 4 Varicella vaccination 5 Pneumonia vaccination

Flu and pneumonia vaccinations are recommended for older adults to reduce the risk of infectious diseases. DTaP vaccinations are effective for preventing whooping cough in children. Children are vaccinated for rubella infections. Varicella vaccination is used to prevent chicken pox in children.

How to use a metered-dose inhaler:

Follow 6 rights of medical administration 2 identifiers Remove mouth cover and give a test spray if new or if it hasn't been used in a few days Position patient in upright position Shake inhaler 5-6 times Tilt head slightly backward, depress the canister while patient is inhaling slowly and deep through the mouth for 3- 5 seconds Hold breathe for 10 seconds. Exhale through pursed-lips

The nurse is assessing a group of patients in a health screening program. A patient complains of itching and irritation under the right arm and the nurse suspects a localized infection. What assessments should be done on this patient? Select all that apply. 1 Examine for paleness of skin. 2 Palpate the area for tenderness. 3 Inquire about pain and tightness. 4 Inspect the area for redness and swelling. 5 Inquire about gastrointestinal disturbances.

Gentle palpation of the infected area may reveal some degree of local tenderness due to inflammation. Inquiring about pain and tightness is important, because they may be caused by edema. Infected areas generally appear red and swollen due to inflammation. Paleness of skin is not a manifestation of infection. Gastrointestinal disturbances are not related to localized infection and may sometimes be found in systemic infections.

Which is the most effective way to control the transmission of infection in health care facilities? 1 Vaccinations 2 Isolation precautions 3 Hand hygiene practices 4 Use of clean equipment

Hand hygiene practices are the most effective way to break the chain of infection and control the transmission of infection. Vaccinations are effective measures to prevent the occurrence of infection in an individual. Even if a patient is isolated, the nurse caring for the patient has a risk of infection. Therefore, the nurse should perform hand hygiene before and after providing patient care. The use of clean equipment without hand hygiene may spread infection.

A community nurse is conducting an awareness program for sex workers and community members with substance abuse problems. What should the nurse tell the attendees about prevention of the spread of the hepatitis C virus? 1 A symptomatic patient cannot transmit hepatitis C. 2 Hepatitis C can be transmitted through the fecal-oral route. 3 Only symptomatic patients can transmit the virus. 4 Both symptomatic and asymptomatic patients can transmit the virus.

Hepatitis C is a communicable disease. A person with or without symptoms can transmit the virus. It is present in blood and body fluids. It can be spread through sexual contact but not through the fecal-oral route.

Braden Scale 10-12

High risk

Which statement is true regarding respiratory changes due to immobility? 1 Hyperventilation may occur due to immobility. 2 Increased cough productivity may be seen due to immobility. 3 Entire lung lobe or a whole lung collapse may be due to immobility. 4 Distribution of mucus in the bronchi may decrease due to immobility.

Immobility sometimes may cause respiratory changes like atelectasis and hypostatic pneumonia. The atelectasis secretions may block a bronchiole or bronchus, which can sometimes collapse an entire lung lobe or a whole lung. The blockage of a bronchiole and collapse of the lung can lead to hypoventilation. Immobility can lead to a proportional decline in the patient's ability to cough productively. Distribution of mucus increases in the bronchi due to immobility.

In which order does the chain of infection cycle occur chronologically? 1. Infectious agent 2. Reservoir 3. Portal to exit 4. Mode of transmission 5. Portal to entry 6. Host

Infection occurs in a cycle that depends on the presence infectious agents, reservoirs, portal to exit, mode of transmission, portal to entry, and host. First, infectious agents choose a reservoir to multiply. After multiplying, they exit through sites such as the skin, urinary tract, and reproductive tract. These agents find different modes of transmission to enter the host.

3 processes to partial thickness wound repair

Inflammatory response, epithelial proliferation & migration, & re-establishment of epidermal layer

Pressure Ulcer Stage 1

Intact kin with non-blanchable redness of a localized area, usually over a bony prominence; skin is not broken but is red or discolored; partial thickness skin loss; skin can be warm or cool

isometric exercise

Isometric exercise or isometrics are a type of strength training in which the joint angle and muscle length do not change during contraction (compared to concentric or eccentric contractions, called dynamic/isotonic movements).

Postural drainage position for child bilateral lobes - anterior lung segments

Lying supine on nurse's lap, back supported with pillow

Which foot problem does the nurse anticipate if the patient reports using oval corn pads at home to treat foot corns? a. Burns b. Infection c. Abrasions d. Ulceration

Oval corn pads exert pressure on the toes, which decreases circulation to the surrounding tissues. This decrease in circulation may ultimately cause ulcerations. Burns may be a result of using over-the-counter liquid preparations to remove corns. Cutting corns or calluses with a razor blade or scissors may cause infection. Using oval corn pads does not cause abrasions. p. 863

Chart/Exhibit 1 P1- weakness in both legs- 4 point crutch gait P2- amputation- 4 point crutch gait P3- Arthritis of knees- 2 point crutch gait P4- fracture- swing through gait In a health care setting, the nurse is caring for four patients with musculoskeletal disorders. Which patient's crutch movements are similar to the arm motions of normal walking? 1 Patient 1 2 Patient 2 3 Patient 3 4 Patient 4

Patient 3 (two point crutch gait) The patient moves the crutch at the same time as the opposite leg while using a two-point crutch gait. Therefore, the arm movements are similar to arm motions during normal walking. Therefore, patient 3 with arthritis in the knees using a two-point crutch gait has movements similar to the arm motions of normal walking. Patient 1 with a four-point crutch gait requires weight bearing on both legs. Therefore, it does not resemble the arm motions used during normal walking. Patient 2 with a three-point crutch gait requires the patient to bear all of the weight on one foot. The amputated leg does not touch the ground. Therefore, the crutch movements are not similar to arm motions during normal walking. Patient 4 using a swing-through gait places the crutches one stride in front and swings the body through them.

Which risk factor causes secondary infections? 1 Trauma 2 Heredity 3 Nutrition 4 Chronic disease

Physical trauma may cause fractures and internal bleeding, which may lead to secondary infections. Heredity causes diseases, such as sickle cell disease, diabetes resulting in anemia, and delayed healing. Poor nutrition causes obesity and anorexia, resulting in an impaired immune response. Chronic disease causes chronic obstructive pulmonary disease, heart failure, diabetes resulting in pneumonia, skin breakdown, and venous stasis ulcers.

The nurse reviews discharge instructions with a patient who has osteoporosis. Which statement by the patient indicates that the patient understands the instructions? 1 "I will avoid intake of leafy green vegetables." 2 "I will avoid exercises, because they may cause bone fracture." 3 "I will reduce consumption of food containing calcium." 4 "I will stop smoking as soon as possible."

Patients with osteoporosis should make lifestyle changes to prevent the disease from becoming worse. Smoking poses a major risk for osteoporosis, but this risk can be drastically reduced if the patient stops smoking. Intake of leafy green vegetables is helpful for the patient who has osteoporosis and should not be avoided. Exercise is helpful in keeping the bones strong. The patient should increase intake of calcium to maintain bone health.

After assessing a patient with immobility, the nurse observes that the patient has acute respiratory distress syndrome. Which positioning of the patient would be appropriate to improve oxygenation? 1 Prone position 2 Supine position 3 30-degree lateral position 4 Semi-Fowler's position

Positioning the patient in the prone position would be beneficial in certain patients with pulmonary conditions such as acute respiratory distress syndrome, because it helps improve oxygenation. The supine position would be beneficial for a postoperative patient to prevent footdrop. Patients with pressure ulcers would benefit from being placed in a 30-degree lateral position. Patients with a head injury would benefit from the semi-Fowler's position.

The nurse is caring for a patient with acute respiratory distress syndrome. While positioning the patient, the nurse observes hyperextension of the lumbar spine. Which patient positioning would likely have caused this condition? 1 Sims' position 2 Prone position 3 Side-lying position 4 Supported Flower's position

Prone positioning is most suitable for patients with acute respiratory distress syndrome and acute lung injury. The potential trouble points with patients in the prone position include hyperextension of the lumbar spine and neck hyperextension. The trouble points of the Sims' position and the side-lying position are lateral flexion of the neck and lack of foot support. The trouble points of the supported Fowler's position are increased cervical flexion and pressure on the posterior aspects of the knee.

Explain the mnemonic RACE to set priorities in case of fire:

R - Rescue and remove all patients in immediate danger A - Activate the alarm C - Confine the fire by closing doors and windows and turning off oxygen and electrical equipment E - Extinguish the fire using an extinguisher

Postural drainage position for adult left lower lobe lung segment

Right side-lying in Trendelenburg's position

What does the Braden Scale evaluate? A) Skin integrity at bony prominences, including any wounds B) Risk factors that place the patient at risk for skin breakdown C) The amount of repositioning that the patient can tolerate D) The factors that place the patient at risk for poor healing

Risk factors that place the patient at risk for skin breakdown

Postural drainage position for child bilateral apical lung segment

Sitting on nurse's lap, leaning slightly forward flexed over pillow

Postural drainage position for adult apical lung segment

Sitting on the side of bed

Which type of pressure ulcer is noted to have intact skin and may include changes in one or more of the following: skin temperature (warmth or coolness), tissue consistency (firm or soft), and/or pain? A) Stage I B) Stage II C) Stage III D) Stage IV

Stage I

To which patients do standard precautions apply? 1 All patients receiving care 2 Patients with blood-borne infections 3 Patients with infected, draining wounds 4 Patients believed to have an infectious disease

Standard precautions were implemented to provide safety for caregivers and patients regardless of infectious status. The answer choices concerning the use of standard precautions only for patients with blood-borne infections; those patients with infected, draining wounds; or patients believed to have infectious diseases are incorrect because they limit the scope of standard precautions that are used with certain populations.

What is the portal of exit of the influenza virus? 1 Blood 2 Respiratory tract 3 Reproductive tract 4 Skin and mucous membrane

The influenza virus is released from the body via the respiratory tract when an infected person sneezes or coughs. Organisms that cause communicable disease such as Hepatitis B and HIV exit from wounds and bloody stool. Organisms such as Neisseria gonorrheae and HIV exit through the reproductive tract during sexual contact. Any break in the skin and mucous membranes allows pathogens to exit the body; the influenza virus does not exit through the skin.

A registered nurse teaches a patient about measures to control the exit and entry of microorganisms. Which statement made by the patient needs correction? 1 "I will brush my teeth regularly." 2 "I will apply lotion to my skin appropriately." 3 "I will apply water-insoluble ointment to my lips." 4 "I will clean my perineal area by wiping from the urinary meatus toward the rectum."

The patient should maintain the integrity of his or her skin and mucous membranes to reduce the risk of microorganism infections. The patient should apply water-soluble ointment to the lips to keep them lubricated and maintain skin integrity. The patient should brush his or her teeth regularly to prevent the drying of mucous membranes. The patient should apply lotion to the skin appropriately to keep the skin lubricated. The patient should clean the perineal area from the urinary meatus toward the rectum to prevent the entry of infectious microorganisms into the urinary tract.

Which of the following is an appropriate intervention to maintain the respiratory system of the immobilized patient? 1. Turn the patient every 4 hours 2. Maintain a maximum fluid intake of 1500 mL/day 3. Apply an abdominal binder continuously while the patient is in bed 4. Encourage the patient to deep breathe and cough every 1 to 2 hours

This technique produces a forceful, productive cough without excessive fatigue

A patient is suspected of having malaria. Which mode of transmission spreads malaria? 1 Vector 2 Vehicle 3 Airborne 4 Direct contact

Vector transmission, such as infection by a mosquito, is responsible for malaria. Vehicles such as water, solution, and blood do not transmit malaria. Respiratory infections are possible through the airborne transmission of microorganisms. Malaria is not transmitted by direct contact with infected persons.

Describe how Muscle abnormalities affect mobility:

Weakness and wasting of muscles. which increase disability and deformity; bruises, contusions, fractures

What is an infective disease that can be transmitted directly from one person to another considered? 1 A susceptible host 2 A communicable disease 3 A port of entry to a host 4 A port of exit from the reservoir

When an infectious disease can be transmitted directly from one person to another, it is termed a communicable disease. No vector is necessary for transmission. A susceptible host is someone who is more prone to develop an infectious disease process. The port of entry is where microorganisms enter the body and the portal of exit is where microorganisms exit a host such as blood or the skin.

A nurse is transferring a patient from a bed to a wheelchair. What should the nurse do to quickly assess the patient's tolerance to the change in position? a. obtain a blood pressure b. monitor for bradycardia c. determine if the patient feels dizzy d. allow the patient to adjust to the change in position

c

Serous wound exudate

clear, watery plasma

Example of an intervention for an infant and toddler:

- Have infants sleep on their back side. Teach parents the mnemonic "back to sleep" - Infants should be immunized - Do not fill cribs with pillows, bumper pads, large stuffed toys, or comforters. Use snug-fitting sheets - Do not attach pacifiers to string or ribbon and place around child's neck - Follow all instructions for preparing storing formula - Use large, soft toys without small parts - Do not leave mesh sides of playpens lowered (Spaces between crib slats to be less 6 cm apart) - Never leave crib sides down or babies unattended on changing tables or in infant seats, swings, or high chairs

A nurse is teaching a patient who is sleep deprived about factors that can promote sleep. What statement if made by the nurse should be included in the teaching? 1 "Exercise 2 hours before bedtime to promote sleep." 2 "Consume chocolates in the evening to promote sleep." 3 "Have a large and heavy meal at night to promote sleep." 4 "Work until you are excessively fatigued to promote sleep."

1 "Exercise 2 hours before bedtime to promote sleep."

Which structure connects muscle to bone? 1 Tendon 2 Cartilage 3 Ligament 4 Syndesmosis

1 Tendon Tendons are white, fibrous bands of tissue that connect muscle to bone at the origin or insertion of the muscle. Cartilage acts as a shock absorber between articulating bones, whereas ligaments are the structures that connect bones and cartilage. Syndesmosis is a fibrous joint between the tibia and fibula.

Which drugs may provide relief from bone pain? Select all that apply. 1 Calcitonin 2 Gabapentin 3 Nortriptyline 4 Bisphosphonates 5 Infusional lidocaine

1, 4 Calcitonin and biphosphates are effective in relieving bone pain. Gabapentin, nortriptyline, and infusional lidocaine are typically used to treat neuropathic pain, not bone pain.

A nurse is evaluating an ambulating patient's balance. What factor about the patient is most important for the nurse assess? 1. Posture 2. Strength 3. Energy level 4. Respiratory rate

1. Assessing posture will identify whether the patient's center of gravity is in the midline from the middle of the forehead to a midpoint between the feet and, therefore, balanced within the patient's base of support.

A nurse educator is evaluating whether a new staff nurse understands the relationship between a fever and an infection. Which statement by the new staff nurse indicates an understanding of this relationship? 1. Phagocytosis cells release pyrogens that stimulate the hypothalamus 2. Leukocyte migration precipitates the inflammatory response 3. Erythema increases the flow of blood throughout the body 4. Pain activates the sympathetic nervous system

1. Phagocytosis delays release pyrogens that stimulate the hypothalamus

A patient has hemiplegia as a result of a brain attack (cerebrovascular accident). Which complication of immobility is of most concern to the nurse? 1. Dehydration 2. Contractures 3. Incontinence 4. Hypertension

2

The nurse is attending to a postsurgical patient who underwent a nephrectomy. What observations would tell the nurse the patient is in severe pain? Select all that apply. 1 The patient is motionless. 2 The patient has a reduced attention span. 3 The patient is constantly asking for pain relief medication. 4 The patient has clenched teeth and is biting his or her lips. 5 The patient is talking incessantly for a long time.

2, 3, 4 A patient in acute pain may not be able to concentrate on anything. The patient may have a reduced attention span and may focus only on pain relief. The nurse may observe the patient clenching teeth or biting his or her lips to tolerate or suppress the pain. These patients are usually physically restless due to pain and they do not interact or talk incessantly.

The nurse incorporates which priority nursing intervention into a plan of care to promote sleep for a hospitalized patient? 1. Have patient follow hospital routines. 2. Avoid waking patient for nonessential tasks. 3. Give prescribed sleeping medications at dinner. 4. Turn television on low to late-night programming.

2. Avoid waking patient for nonessential tasks.

A nurse is teaching a patient various techniques to promote sleep. Which internal stimulus that most commonly interferes with sleep should the nurse include in the teaching? 1. Ringing in the ears 2. Bladder fullness 3. Hunger 4. Thirst

2. Bladder fullness Bladder fullness causes pressure in the pelvic area that interrupts sleep. Awakening to void during the night is a common occurrence, particularly in older adult men.

A nurse identifies that a patient's pressure ulcer has just partial-thickness skin loss involving the epidermis and dermis. What stage pressure ulcer should the nurse document based on this assessment? 1. Stage I 2. Stage II 3. Stage III 4. Stage IV

2. In a stage II pressure ulcer the partial-thickness skin loss presents clinically as an abrasion, blister, or shallow crater.

The physician writes an order to apply a wrist restraint to a patient who has been pulling out a surgical wound drain. Place the steps for applying the restraint in the correct order. 1. Explain what you plan to do. 2. Wrap a limb restraint around wrist or ankle with the soft part toward the skin and secure. 3. Determine that restraint alternatives fail to ensure patient's safety. 4. Identify the patient using proper identifier. 5. Pad the patient's wrist.

3, 1, 2, 4, 5 Because the patient is pulling out a surgical wound drain, the first step is determining that restraint alternatives fail to ensure patient's safety. Next, identify the patient using the proper identifier and then explain what you plan to do. Next, pad the patient's wrist and finally, wrap a limb restraint around the wrist or ankle with the soft part toward the skin and secure.

The developmental stage that carries the highest risk of an injury from a fall is: 1. Preschool 2. Adulthood 3. School Age 4. Older Adulthood

4

A student nurse who is employed as a nursing assistant may perform any function that: 1.Have been learned in school 2.Are expected of a nurse at that level 3.Are identified in the position's job description 4.Require technical rather than professional skill

3. Need to perform only those tasks that appear in the job description for a nurse's aide or assistant

A nurse is assessing a patient in pain. Which word might the nurse use when documenting the pattern of a patient's pain? 1. Tenderness 2. Moderate 3. Episodic 4. Phantom

3. Episodic The word episode refers to an incident, occurrence, or time period; therefore, the word episode refers to a pattern of pain and is concerned with time of onset, duration, recurrence, and remissions.

A nurse is caring for a group of patients experiencing various medical conditions. The patient with which condition is at the greatest risk for a wound infection? 1. Surgical creation of a colostomy 2. First-degree burn on the back. 3. Puncture of the foot by a nail 4. Paper cut on the finger

3. Puncture of the foot by a mail

A nurse is repositioning a patient to the left lateral position. What action should the nurse implement when positioning this patient? 1. Rest the right leg on top of the left leg 2. Maintain knee flexion at ninety degrees 3. Place the ankles in plantar flexion 4. Left shoulder protracted

4. In the left lateral (side-lying) position, the left arm is positioned in front of the body with the shoulder pulled forward (protracted). This reduces the pressure on the joint in the shoulder and the acromial process.

A patient consistently tries to pull out a urinary retention catheter. As a last resort to maintain integrity of the catheter and patient safety, the nurse obtains an order for a restraint. Which type of restraint is most appropriate in this situation? 1. Mummy restraint 2.Elbow restraint 3.Jacket restraint 4. Mitt restraint

4. Mitt restraint

Contracture

A deformity that is the result of a stiffness or constriction in your muscles, joints, tendons, ligaments, or skin that restricts normal movement. It develops when your normally pliable connective tissues become less flexible. This means your range of motion will be limited.

Which of the following describes a hydrocolloid dressing? A) A seaweed derivative that is highly absorptive B) Premoistened gauze placed over a granulating wound C) A debriding enzyme that is used to remove necrotic tissue D) A dressing that forms a gel that interacts with the wound surface

A dressing that forms a gel that interacts with the wound surface

When a patient on respiratory isolation must be transported to answer part of the hospital, the nurse: a. Places a mask on the patient before leaving the room b. Obtains a health care provider's order to prohibit the patient from being transported c. Instructs the patient to cover his or her mouth and nose with a tissue when coughing or sneezing d. Advises other health team members to wear masks and gowns when coming in contact with the patient

A. Placing a mask on the patient before leaving the room; Patients who are transported outside of their rooms need to wear surigical masks to protect other patients and personnel

Postural drainage position for adult right upper lobe - anterior lung segment

Supine with head elevated

Which nursing intervention is appropriate for preventing atelectasis in the postoperative patient? A) Postural drainage B) Chest percussion C) Incentive spirometer D) Suctioning

C

A patient reports difficulty falling asleep. Which questions should the nurse ask to assess the patient's insomnia? Select all that apply. 1 "How frequently do you eat out?" 2 "How easily do you fall asleep?" 3 "What causes you to awaken early?" 4 "What do you think of when you try to fall asleep?" 5 "Do you stay with your parents?"

Correct 2 "How easily do you fall asleep?" Correct 3 "What causes you to awaken early?" Correct 4 "What do you think of when you try to fall asleep?"

The nurse works in a sleep clinic. The nurse understands that sleep medicines should be administered with caution in some patients. In which patient can benzodiazepines be given safely? 1 A 70-year-old patient with renal failure 2 A nursing mother 3 A 30-year-old patient 4 A pregnant patient

Correct3 A 30-year-old patient

A patient has been diagnosed with severe iron deficiency anemia. During physical assessment for which of the following symptoms would the nurse assess to determine the patient's oxygen status? A) Increased breathlessness but increased activity tolerance B) Decreased breathlessness and decreased activity tolerance C) Increased activity tolerance and decreased breathlessness D) Decreased activity tolerance and increased breathlessness

D

Which of the following best describes an iatrogenic infection? a. It results from a diagnostic or therapeutic procedure b. It results from an extended infection of the urinary tract c. It involves an incubation period of 3 to 4 weeks before it can be detected d. It occurs when patients are infected with their own organisms as a result of immunodeficiency

D. It occurs when patients are infected with their own organisms as a result of immunodeficiency; An iatrogenic infection occurs when part of the patient's flora becomes altered and an overgrowth results

The registered nurse is teaching a nursing student about the pathological influence on mobility. Which statement made by the nursing student indicates a need for further learning? 1 "Damage to the cerebellum causes problems with balance." 2 "Right-sided cerebral hemorrhage causes left-sided hemiplegia." 3 "Direct trauma to the central nervous system results in bruises and contusions." 4 "Complete transection of spinal cord causes bilateral loss of voluntary motor control below the level of the trauma."

Direct trauma to the musculoskeletal system results in bruises, sprains, and contusions. Damage to the cerebellum causes problems with balance and motor impairment. A person with right-sided cerebral hemorrhage with necrosis has destruction of the right motor strip that results in left-sided hemiplegia. A complete transection of spinal cord results in bilateral loss of voluntary motor control below the level of trauma because motor fibers are cut.

What major infections are caused by Escherichia coli? Select all that apply. 1 Hepatitis A 2 Pneumonia 3 Gastroenteritis 4 Food poisoning 5 Urinary tract infections

Gastroenteritis and urinary tract infections are major infections caused by Escherichia coli. The hepatitis A virus causes Hepatitis A. Pneumonia and food poisoning are major infections caused by Staphylococcus aureus.

Which metabolic changes are observed in immobile patients? 1 Increased appetite 2 Increased peristalsis 3 Increased metabolic rate 4 Increased nitrogen levels

In an immobile patient, his or her body often excretes more nitrogen that in ingests, resulting in negative nitrogen balance. Immobility disrupts normal metabolic functioning, and causes gastrointestinal disturbances such as decreased appetite and slowing of peristalsis. The metabolic rate decreases in patients with immobility.

Prone Position

Lies face or chest down

Assessment of Gait

Particular manner or style of walking: mechanics involve coordination of skeletal, neurological, and muscular systems

Assessment of Exercise and activity tolerance

Physical activity for conditioning the body, improving health, and maintaining fitness

Which disease requires contact precautions? 1 Scabies 2 Measles 3 Diphtheria 4 Pharyngitis

Scabies spreads through skin contact and the nurse should take contact precautions. Measles require airborne precautions. Diphtheria and pharyngitis require droplet precautions.

Postural drainage position for adult right upper lobe - posterior lung segment

Side-lying with right side of chest elevated on pillows

The nurse is reviewing the data of patients who have undergone surgery. Which patient would be at the highest risk of orthostatic hypotension based on the given data? 1 Patient A 2 Patient B 3 Patient C 4 Patient D

The longer the duration of a patient's immobility, the higher the risk is for orthostatic hypotension. Therefore, the patient who underwent hip replacement and required bed rest for 90 days would be at the highest risk of orthostatic hypotension.

Assessment of Range of motion

The maximum amount of movement available at a joint in one of the three planes of the body: sagittal, frontal, or transverse: exercises are active and passive

A patient who is infected with herpes simplex complains of itching and tingling. There are no visible lesions found on examination. To which stage of herpes simplex infection does this patient belong? 1 Illness stage 2 Incubation stage 3 Prodromal stage 4 Convalescence stage

The prodromal stage is defined as the interval from onset of nonspecific signs and symptoms to more specific symptoms related to the type of infection or disease. In this stage the microorganisms grow and multiply. Itching and tingling sensations are nonspecific symptoms of herpes simplex that occur before the lesions appear. In the illness stage, the patient actually develops the signs and symptoms specific to the type of infection. The incubation period is an interval between entrance of the pathogen into body and appearance of the first symptoms. In the convalescence stage, acute symptoms of infection will disappear.

A nurse is teaching a class to nursing assistants about how to care for patients who are immobile. What should the nurse include about why immobilized people develop contractures? 1. Muscles that flex, adduct, and internally rotate are stronger than weaker opposing muscles 2. Muscular contractures occur because of excessive muscle flaccidity 3. Muscle mass and strength decline at a progressive rate weekly 4. Muscle catabolism exceeds muscle anabolism

The state of balance between muscles that serve to contract in opposite directions is impaired with immobility. The fibers of the stronger muscles contract for longer periods than do those of the weaker, opposing muscles. This results in a change in the loose connective tissue to a more dense connective tissue and to fibrotic changes that limit range of motion.

The nurse is teaching the procedure for ear irrigation on a 4-year-old patient to a nursing student. Which statement made by the nursing student demonstrates a need for further teaching? a. "I should pull the pinna down and back." b "I should wash the ear canal with a Water Pik set on the number 2 setting." c. "I should direct the fluid toward the superior aspect of the ear canal." d. "I should assist the patient in lying on his side with the affected ear up."

a. "I should pull the pinna down and back." While irrigating the ear of patients above 3 years of age, the pinna should be pulled up, not down, and back. The ear canal should be washed using a bulb-irrigating syringe or a Water Pik set on the number 2 setting. The irrigating fluid should be directed toward the superior aspect of the ear canal. The patient should lie on his or her side with the affected ear up. p. 845

Nursing intervention to reduce impact of immobility on the Metabolic System

a. A high protein, high-caloric diet b. Vitamin B and C supplements

Which statement is correct about types of baths? a. A partial bed bath is indicated in a patient with pain. b. A sponge bath is indicated in a patient sitting in a chair. c. A complete bed bath is given to patients who are very ill. d. A complete bed bath offers a reduction in bathing time. e. A tub bath is given to remove infected tissue from wounds.

a. A partial bed bath is indicated in a patient with pain. b. A sponge bath is indicated in a patient sitting in a chair. c. A complete bed bath is given to patients who are very ill. A partial bed bath includes bathing selected areas and is indicated in a patient with pain. A sponge bath involves bathing a patient using a bath basin or sink while the patient sitting in a chair. A complete bed bath is given to a patient who is bedridden and ill. A bag bath offers an ease of use, reduction of the bathing time, and promotes patient comfort. A therapeutic bath is given to a patient to for smoothing effects and in for the promotion of healing. p. 854

Nursing intervention to reduce impact of immobility on the Psychosocial System

a. Anticipate change in the patient's status and provide routine and informal socialization b. Stimuli to maintain patient's orientation

A patient reports dry and flaky skin. During an assessment, the nurse notices a loss of protective oils from the skin. What are possible reasons for the patient's condition? a. Bathing frequently b. Using alkaline soaps c. Perspiring excessively d. Using hot water frequently e. Using excessive deodorants

a. Bathing frequently d. Using hot water frequently If the patient bathes and uses hot water frequently, the skin may become dry and flaky and it may lose protective oils. Using alkaline soaps will neutralize the protective acid condition of the skin. Excessive perspiration can harbor microorganisms. The use of excessive deodorants may cause chemical irritation. p. 822

The nurse is monitoring an older adult patient who has undergone coronary artery bypass graft (CABG) for blocked blood vessels of the heart. Which dermatological findings in the patient indicate healthy skin? a. Pink color b. Symmetry c. Supple skin d. Warm skin e. Hair follicle density

a. Pink color c. Supple skin d. Warm skin Changes in color, thickness, temperature, and hydration of the skin are various dermatological presentations that help the nurse to differentiate between healthy and unhealthy skin. A pink color indicates healthy skin. Suppleness of the skin indicates that the skin is well hydrated, and warm skin indicates adequate circulation. Symmetry and hair follicle density do not indicate significant pathological change in postoperative patients. Study Tip: You have a great resource in your classmates. We all have different learning styles, strengths, and perspectives on the material. Participating in a study group can be a valuable addition to your nursing-school experience. p. 822

Nursing intervention to reduce impact of immobility on the Integumentary system

a. Positioning and skin care b. Use therapeutic devices to relieve pressure

Complications of Immobility in relation to the urinary system

a. Urinary stasis: Renal pelvis fills before urine enters the ureters b. Renal Calculi: Calcium stones that lodge in the renal pelvis

Nursing intervention to reduce impact of immobility on the Elimination System

a. Well hydrated b. Prevent urinary stasis and calculi and infections

Indications for home oxygen therapy

an arterial partial pressure (PaO2) of 55 mm Hg or less or an arterial oxygen saturation (SaO2) of 88% or less on room air at rest, on exertion, or with exercise.

The nurse is assessing a patient who complains of toothache. The nurse finds that the patient holds the toothbrush bristles at a 45-degree angle when brushing the gum line. The patient also brushes the teeth from the crown to the gum line. On examination, the mucous membranes of the patient's mouth are dry. What should the nurse advise the patient about oral care? a. Use lemon-glycerin preparations. b. Brush the teeth from the gum line to the crown. c. Hold the toothbrush bristles at a 90-degree angle to the gum line. d. Use an alcohol-based mouthwash rather than flossing.

b. Brush the teeth from the gum line to the crown. For optimal results the patient should follow the correct procedure for brushing the teeth. Brushing should be done from the gum line to the crown to prevent erosion of the gums. Lemon-glycerin preparations should not be encouraged, because they are harmful to the teeth. Glycerin dries and shrinks the mucous membranes and gums. Lemon exhausts the salivary reflex and erodes tooth enamel. For effective removal of plaque the toothbrush bristles should be held at a 45-degree angle to the gum line when brushing the gum line. An alcohol-based commercial mouthwash can dry the membranes and does not replace flossing. p. 839

Which guidelines should be followed while performing a bed bath to prevent contamination of a patient's clean areas? a. Starting with the arm farthest away from the nurse b. Washing from the cleanest areas to the dirtiest areas c. Uncovering the patient's entire body d. Folding the washcloth into a mitt around the nurse's hand e. Using a proximal to distal motion to wash a patient's extremities

b. Washing from the cleanest areas to the dirtiest areas d. Folding the washcloth into a mitt around the nurse's hand Rationale The nurse should wash from the cleanest areas to the dirtiest areas to prevent the transfer of bacteria from a dirty area to a clean area. The nurse should fold the washcloth into a mitt around the hand to prevent splashing water from contaminating a patient's clean areas. The nurse should start with the arm farthest away from the nurse and then the nearest arm to help prevent back strain and to allow for better access to the patient. The nurse should uncover only the area of the body that is to be washed to ensure the patient's privacy. The nurse should use a distal to proximal motion for washing to promote venous return. Test-Taking Tip: Be alert for grammatical inconsistencies. If the response is intended to complete the stem (an incomplete sentence) but makes no grammatical sense to you, it might be a distractor rather than the correct response. Question writers typically try to eliminate these inconsistencies. pp. 853, 856

The nurse is preparing the bed of a patient. Which of the nurse's actions are appropriate in this situation? a. Placing the soiled linen on the floor b. Placing soiled linen directly in the hamper c. Keeping the linen away from the uniform d. Leaning over the mattress while placing the linen e. Placing the clean linen in a dirty-linen container if it touches the floor

c. Keeping the linen away from the uniform e. Placing the clean linen in a dirty-linen container if it touches the floor Rationale While preparing the bed, the nurse should keep the linen away from the uniform. The nurse should place clean linen in the dirty-linen container if it touches the floor. The nurse should place soiled linen in special linen bags to avoid transmitting infection. The nurse should place the linen in special linen bags before placing it in a hamper. Leaning over the mattress while placing the linen may cause back injuries. p. 848

Primary intention

clean surgical incision with little tissue loss; skin edges are approximated and closed; decreased risk of infection

exogenous

come from microorganisms outside the individual that do not exist in normal floras

Venturi Mask

delivers higher oxygen concentrations of 24% to 60% with oxygen flow rates of 4 to 12 L/min, depending on the flow-control meter selected.

Residual volume

he amount of air left in the alveoli after a full expiration

Dressings for Pressure Ulcer Stage 3

hydrocolloid, hydrogel, calcium alginate, gauze

Dressings for Pressure Ulcer Stage 4

hydrogel, calcium alginate, gauze

National Quality Forum Mission

improving the quality of health care in America by: - Building consensus on national priorities and goals for performance improvement and working in partnership to achieve them; - Endorsing national consensus standards for measuring and publicly reporting on performance; and - Promoting the attainment of national goals through education and outreach programs.

Hematoma

localized collection of blood under the skin; red-bluish bruise; firm

Secondary Intention

loss of tissue (burn, pressure, ulcer, severe laceration); wound isn't closed until all evidence of infection and all debris has been removed; increased risk of infection

direct transmission

person-to-person or physical source and susceptible host

indirect transmission

personal contact of a susceptible host with a contaminated inanimate object

Dermabond

skin glue; tissue adhesive; used with small superficial lacerations; not useful over joints; for straight, non-jagged edges

Airway resistance

the increase in pressure that occurs as the diameter of the airways decreases from mouth/nose to alveoli.

Hydrogel

used for partial or full-thickness; deep wounds with some exudate; necrotic; radiation damaged skin; useful in painful wounds, soothing and does not adhere to wounds

Simple face mask

used for short-term oxygen therapy. It fits loosely and delivers oxygen concentrations from 35% to 50% FIO2. This is contraindicated for patients with carbon dioxide retention because retention can be worsened. Flow rates should be 5 L or more to avoid rebreathing exhaled carbon dioxide retained in the mask.

After how many minutes into the sleep cycle does a sleeper usually reach rapid eye movement (REM) sleep? _________ minutes

90

Which human response to illness alerts a nurse that a patient has the greatest risk for aspiration during meals? 1.Bulimia 2.Lethargy 3.Anorexia 4.Stomatitis

2.Lethargy

Example of an intervention for an older adult:

- Provide information about neighborhood resources to help an older adult maintain an independent lifestyle. - Educate older adults about safe driving tips

Example of an intervention for an adult:

- Referring them to resources such as classes to help them quit smoking and for stress management, including employee assistance programs - Encourage adults to exercise regular, maintain a healthy diet, practice relaxation techniques, and get adequate sleep.

Example of an intervention for a school-age child:

- Teach children the safe use of equipment for play or work - Teach children proper bicycle safety, including use of helmet and rules of the road - Teach children not to operate electrical equipment while unsupervised

Example of an intervention for a preschooler:

- Teach children to swim at an early age but always provide supervision near water - Teach children how to cross streets and walk in parking lots. Instruct them to never tun out after ball or toy - Teach children not to talk to, go with, or accept any item from a stranger

Environmental Risks

- Chemicals found in some medications (e.g., chemotherapy), anesthetic gases, cleaning solutions, and disinfectants are potentially toxic if ingested, absorbed into the skin, or inhaled. - Material safety data sheets (MSDSs) are required resources available in any health care agency.

Example of an intervention for an adolescent:

- Encourage enrollment in driver education class - Provide information about the effects os using alcohol and drugs - Refer adolescents to community and school-sponsored activities - Teach them safe use of the internet

Specific Risks to the Patient's Safety within the Health Care Environment:

- Falls - Patient-inherent accidents - Procedure-related accidents - Equipment-related accidents

Fall Prevention:

- Keep the bedside environment free from clutter. - Explain how to use the call light. - Yellow wristbands mean risk for fall.

When you are assigned to a patient who has a reduced level of consciousness and requires mouth care, which physical assessment techniques should you perform before the procedure? (Select all that apply).

1. Oxygen saturation 2. Heart rate 3. Respirations 4. Gag reflex 5. Response to painful stimulus 3 and 4

A nurse is teaching a class to nursing assistants about how to care for patients who are immobile. What should the nurse include about why immobilized people develop contractures? 1. Muscles that flex, adduct, and internally rotate are stronger than weaker opposing muscles 2. Muscular contractures occur because of excessive muscle flaccidity 3. Muscle mass and strength decline at a progressive rate weekly 4. Muscle catabolism exceeds muscle anabolism

1

A nurse places a patient in the orthopneic position. What is the primary reason for the use of this position? 1. Facilitate respirations 2. Support hip extension 3. Prevent pressure ulcers 4. Promote urinary elimination

1

Question 10. A patient sustains soft tissue injuries from a motor vehicle collision. Which intervention may be helpful in limiting the stress of both edema and bleeding into tissue? 1. Applying a cold compress 2. Exerting direct pressure 3. Performing effleurage 4. Providing massage

1

Question 11. What should the nurse do to interrupt the transmission link in the chain of infection? 1. Wash the hands before providing care to a patient 2. Position a commode next to a patient's bed 3. Provide education about a balanced diet 4. Change a dressing when it is soiled

1

The school nurse is teaching health-promoting behaviors that improve sleep to a group of high school students. Which points should be included in the education? (Select all that apply.) 1. Do not study in your bed. 2. Go to sleep each night whenever you feel tired. 3. Turn off your cell phone at bedtime. 4. Avoid drinking coffee or soda before bedtime. 5. Turn on the television to help you fall asleep.

1. Do not study in your bed. 3. Turn off your cell phone at bedtime. 4. Avoid drinking coffee or soda before bedtime.

A nurse is caring for a patient who is experiencing pain. For which common psychological response to pain should the nurse assess the patient? 1. Experiencing fear related to loss of independence 2. Withdrawing from social interactions with others 3. Asking for pain medication to relieve the pain 4. Verbalizing the presence of nausea

1. Experiencing fear related to loss of independence Psychological or affective responses to pain relate to feelings and emotional distress. Fear of being dependent on others and loss of self-control are psychological responses to pain.

Which most common cause of sleep deprivation in the hospital should the nurse consider when planning care? 1. Fragmented sleep 2. Early awakening 3. Restless legs 4. Sleep apnea

1. Fragmented sleep Sleep deprivation occurs with frequent interruptions of sleep because the sleeper returns to stage I rather than to the stage that was interrupted. There is a greater loss of stage III and IV non-rapid-eye-movement (NREM) sleep, which is essential for restorative sleep.

A nurse places a patient with a sacral pressure ulcer in the left Sims position. How should the nurse position the patient's right arm? 1. On a pillow 2. Behind the back 3. With the palm up 4. In internal rotation

1. In the left Sims position the patient's right arm and leg are supported on pillows to prevent internal rotation of the shoulder and hip.

Which patient information collected b the nurse reflects a systemic response to a wound infection? Select all that apply 1. Increased body temperature 2. Increased heart rate 3. Leukocytosis 4. Exudate 5. Edema 6. Pain

1. Increased body temperature 2. Increase heart rate 3. Leukocytosis

A nurse uses long firm, strokes distal to proximal while bathing a patient's legs because:

1. It promotes venous circulations. 2. It covers a larger area of the leg. 3. It completes care in a timely fashion. 4. It prevents blood clots in legs. 1

A 12-year-old boy is experiencing nocturnal enuresis. Which strategies should the nurse explore with the boy and his parents? Select all that apply. 1. Limiting fluid intake after dinner 2. Voiding immediately before going to bed 3. Eliminating caffeinated beverages from the diet 4. Thinking about waking up dry when going to bed at night 5. Having the boy change his own bed linens when he wets the bed

1. Limiting fluid intake after dinner Limiting fluid intake after dinner reduces the amount of urine production while asleep. 2. Voiding immediately before going to bed Voiding empties the bladder and makes room for urine produced during the night. 3. Eliminating caffeinated beverages from the diet Caffeine irritates the mucous membranes of the urinary system and stimulates the need to void. 4. Thinking about waking up dry when going to bed at night Positive imagery supports self-esteem and may become a self-fulfilling prophesy.

When assessing patients who have difficulty sleeping, the nurse assesses for which common physiological responses to insomnia? Select all that apply. 1. Vertigo 2. Fatigue 3. Irritability 4. Headache 5. Frustration

1. Vertigo Shortened non-rapid-eye-movement (NREM) sleep can result in vertigo, which is a physiological response to sleep deprivation. 2. Fatigue Interrupted NREM sleep can result in fatigue, which is a physiological response to sleep deprivation. 4. Headache Shortened NREM sleep can result in headache, which is a physiological response to sleep deprivation.

A patient sits for excessive lengths of time in a wheelchair. Which sites should the nurse assess for skin breakdown in this patient? 1. Ischial tuberosities 2. Bilateral scapulae 3. Trochanters 4. Malleoli

1. When in the sitting position, the hips and knees are flexed at 90 degrees and the body's weight is borne by the pelvis, particularly the ischial tuberosities, which are bony protuberances of the lower portion of the ischium. Using a wheelchair results in prolonged sitting unless interventions are implemented to promote local circulation.

The nurse is preparing a patient for a physical examination. What is most important for the nurse to do in this situation? 1.Identify the positions that may be contraindicated for the patient during the examination 2.Explore the patient's attitude toward health-care providers 3.Inquire about the other professionals caring for the patient 4.Ask when the patient last had a physical examination

1.Identify the positions that may be contraindicated for the patient during the examination

The nurse must apply a hospital gown that does not have snaps on the shoulders to a patient receiving an intravenous infusion in the forearm.What should the nurse do? 1.Insert IV bag and tubing through the sleeve from inside of the gown first 2.Disconnect the IV at the insertion site, apply the gown, and then reconnect the IV 3. Close the clamp on the IV tubing no more than 15 seconds while putting on the gown 4.Don the gown on the arm without the IV, drape the gown over the other shoulder and adjust the closure behind the neck

1.Insert IV bag and tubing through the sleeve from inside of the gown first

The nurse is orienting a newly admitted patient to the hospital. It is most important for the nurse to teach the patient how to: 1.Notify the nurse when help is needed 2.Get out of the bed to use the bathroom 3.Raise and lower the head and foot of the bed 4.Use the telephone system to call family members

1.Notify the nurse when help is needed

Explain immobility related cardiovascular change: Thrombus

Accumulation of platelets, fibrin, clotting factors, and cellular elements of the blood attached to the interior wall of a vein or artery that occludes the lumen of the vessel

The nurse is caring for a patient who has a respiratory infection. The nurse understands that an infection occurs in a cycle and involves several elements. What are the elements in the chain of infection? Select all that apply. 1 An infectious agent 2 A vaccine schedule Correct3 The source of pathogen growth 4 A clean surrounding 5 A susceptible host

An infectious agent is the main pathogen or infection-causing organism that spreads through the chain of infection . The source for pathogen growth is the reservoir where the pathogens can multiply, survive, and wait until they are transferred to a susceptible host. A susceptible host is the element in the chain of infection that receives the pathogens and is in a favorable condition for their growth and transmission. A vaccine schedule is the plan of immunization and is not an element in the chain of infection. A clean surrounding is an ideal situation to prevent the spread of pathogens and is not an element in the chain of infection.

A primary health-care provider prescribes oxycodone oral solution 15 mg every 6 hours. The drug is supplied in a 500-mL bottle that indicates 5 mg/5 mL. How much oral solution should the nurse administer? Record your answer using a whole number.

Answer: 15 mL

The nurse is instructed to clean artery forceps contaminated with blood. Arrange the steps of cleaning in the appropriate order. 1. Rinse the artery forceps with cold water. 2. Wash the artery forceps with soap and water. 3. Rinse the artery forceps with warm water. 4. Dry the artery forceps.

Any object contaminated with organic material like blood should be rinsed with cold running water. If hot water is used in the beginning, the protein in the organic material would coagulate and stick to the object. This may make cleaning difficult. After rinsing with cold water, the object should be washed with soap and water. Next, the object should be rinsed in warm water, and finally dried for reuse.

Which equipment is used to sterilize surgical instruments? 1 Autoclave 2 Boiling water 3 Chemical sterilants 4 Ethylene oxide (ETO) gas

Autoclaves use moist heat to kill pathogens and spores on surgical instruments to prevent infections. Boiling water is used to clean urinary catheters, suction tubes, and drainage collection devices. Chemical sterilants are used to disinfect heat-sensitive instruments and equipment such as endoscopes and respiratory therapy equipment. Ethylene oxide (ETO) gas is used for medical materials.

A patient is admitted to the emergency department with suspected carbon monoxide poisoning. Even though the patient's color is ruddy, not cyanotic, the nurse understands that the patient is at a risk for decreased oxygen-carrying capacity of blood because carbon monoxide does which of the following: A) Stimulates hyperventilation, causing respiratory alkalosis B) Forms a strong bond with hemoglobin, creating a functional anemia. C) Stimulates hypoventilation, causing respiratory acidosis D) Causes alveoli to overinflate, leading to atelectasis

B

A nursing student performs surgical hand asepsis after assisting a registered nurse in a surgical procedure. Which action made by the nursing student needs correction? 1 Keeping the hands below the waist level 2 Turning off the faucet using the knees 3 Using a continuous motion to rinse from the fingertips to the elbows 4 Using a rotary motion to move the towel from the fingers to the elbows

Because the area below the waist level is considered unsterile, the nurse should keep his or her hands above the waist. The faucet should be closed by using the knees to prevent contamination of the hands. The nurse should use a continuous motion to rinse from the fingertips to the elbows, allowing water to run off at the elbows. The nurse should use a rotary motion to move the towel from the fingers to the elbows during drying to dry the skin from the hands to the elbows.

When repositioning an immobile patient, the nurse notices redness over a bony prominence. What is indicated when a reddened area blanches on fingertip touch? A) A local skin infection requiring antibiotics B) Sensitive skin that requires special bed linen C) A stage III pressure ulcer needing the appropriate dressing D) Blanching hyperemia, indicating the attempt by the body to overcome the ischemic episode.

Blanching hyperemia, indicating the attempt by the body to overcome the ischemic episode.

Which type of medical equipment is cleaned outside the health care facilities? 1 Endoscopes 2 Drainage collection devices 3 Respiratory therapy equipment 4 Heat-tolerant surgical instruments

Boiling is done outside health care facilities for items such as drainage collection devices. Endoscopes and respiratory therapy equipment are cleaned by chemical disinfectants in health care settings. Heat-tolerant surgical instruments are cleaned by autoclaves in the health care facilities.

The nurse places the patient in prone position. Which nursing action reduces the flexion or hyperextension of the cervical vertebrae? 1 Bringing the dependent shoulder blade forward 2 Supporting the lower legs with pillows to elevate the toes 3 Turning the patient's head to one side and supporting it with a small pillow 4 Placing a small pillow under the patient's abdomen below the level of diaphragm

By turning the patient's head to one side and supporting it with a small pillow in prone position, flexion or hyperextension of the cervical vertebrae can be prevented. Bringing the dependent shoulder blade forward in the side-lying position helps prevent the patient's weight from resting directly on the shoulder joint. Supporting the lower legs with pillows to elevate the toes may prevent foot drop. Placing a small pillow under the patient's abdomen below the level of diaphragm in prone position helpsdecrease the hyperextensionof lumbar vertebrae and strain on lower back.

A patient is admitted with severe lobar pneumonia. Which of the following assessment findings would indicate that the patient needs airway suctioning? A) Coughing up thick sputum only occasionally B) Coughing up thin, watery sputum easily after nebulization C) Decreased independent ability to cough D) Lung sounds clear only after coughing

C

While assessing a patient, the nurse finds complete elongation of the deltoid muscle even in a normal position. How does the nurse interpret this observation? 1 Normal finding 2 Presence of orthopnea 3 Presence of contracture 4 Limited movement of the arms

Complete elongation of the deltoid muscle is a normal finding that is used to assess the mobility of the shoulders. Orthopnea may cause a patient to lean on the table in front of his or her chair in an attempt to breathe more easily. A limited range of motion may indicate the presence of contracture. Limited movement of the shoulder may result in decreased movement of the arms.

Resistive isometric exercises

Contraction of muscles while pushing against a stationary object or resisting the movement of the object (push-ups)

A patient has narcolepsy. The nurse finds that the patient's drowsiness has increased. What advice does the nurse offer the patient to decrease drowsiness? Select all that apply. 1 "Avoid eating heavy meals." 2 "Avoid long-distance driving." 3 "Avoid the consumption of alcohol." 4 "Avoid eating a protein-rich diet." 5 "Avoid chewing gum."

Correct 1 "Avoid eating heavy meals." Correct 2 "Avoid long-distance driving." Correct 3 "Avoid the consumption of alcohol."

A mother is unable to put her newborn baby to sleep. What measures should the nurse teach the mother to promote sleep in the newborn? Select all that apply. 1 Put the crib near an open window. 2 Rock the baby gently. 3 Talk to the baby loudly. 4 Sing a lullaby to the baby. 5 Hold the baby snugly in a blanket.

Correct 2 Rock the baby gently. Correct 4 Sing a lullaby to the baby. Correct 5 Hold the baby snugly in a blanket.

What is the nurse least likely to ask parents while performing a sleep assessment of an infant? 1 To describe the infant's eating pattern 2 To describe the sleeping environment of the infant 3 To keep a sleep log of the infant's sleep behavior only at night 4 To evaluate the typical behavior patterns that foster or impair sleep

Correct3 To keep a sleep log of the infant's sleep behavior only at night

A patient is admitted with the diagnosis of severe left-sided heart failure. The nurse expects to auscultate which adventitious lung sounds? A) Sonorous wheezes in the left lower lung B) Rhonchi midsternum C) Crackles only in apex of lungs D) Inspiratory crackles in lung bases

D

What is the removal of devitalized tissue from a wound called? A) Debridement B) Pressure reduction C) Negative pressure wound therapy D) Sanitization

Debridement

What are the risk factors for developing pathological fractures in patients with immobility? Select all that apply. 1 Decreased metabolism 2 Decreased urinary output 3 Decreased tissue catabolism 4 Decreased calcium regulation 5 Decreased urine concentration

Decreased metabolism and calcium regulation are the major risk factors for developing pathological fractures in patients with immobility. Decreased urinary output is a urinary elimination change seen in patients with immobility due to decreased intake of fluids. Decreased tissue catabolism is tissue breakdown due to muscle weakness and decreased muscle mass in patients with immobility. Decreased urinary concentration is a urinary elimination change seen in patients with immobility due to decreased fluid intake and output.

Body mechanics

Describes the coordinated efforts of the musculoskeletal and nervous system

Unstageable/Unclassified Ulcer

Either stage 3 or 4 pressure ulcer; depth of injury unknown; wound bed had sloughing or eschar; if skin is intact & dry with an eschar without signs/symptoms of infection, eschar will not be removed

The registered nurse is teaching a nursing student about psychosocial effects on patients with immobility. Which statements made by the nursing student indicate a need for further learning? Select all that apply. Correct1 "Every patient responds to immobility in a same way." 2 "Patients with restricted mobility may have depression." 3 "Impaired mobility can cause social isolation and loneliness." 4 "Immobilization leads to emotional and behavioral responses." 5 "Withdrawn patients often want to participate in their own care."

Every patient responds to immobility in a different way. Withdrawn patients often do not want to participate in their own care. Patients with restricted mobility may experience depression. Impaired mobility can cause social isolation and loneliness. Immobilization often leads to emotional and behavioral responses, sensory alterations, and changes in coping.

Isometric contractions

Exercises that involve tightening or tensing muscles without moving body parts (quadriceps set exercises)

The nurse is caring for a patient who has been immobile after a spine surgery. Which urinary changes are likely to occur in the first 72 postoperative hours? Select all that apply. Correct1 Urinary output decreases. 2 Risk for developing chronic renal failure increases. 3 Urine is more diluted. 4 Urinary stasis occurs. 5 Risk for developing urinary tract infection increases.

Fluid intake is often diminished during immobility, causing a decrease in urine output and an increase in the concentration (not dilution) of the urine. The urine produced by the kidneys needs gravitational force to enter the ureters. During immobility, the urine in the kidney fills up the pelvis. This condition is called urinary stasis. It increases the risk of urinary infection, because microorganisms can grow well in stagnated urine. Chronic renal failure will not happen in the first 72 hours after surgery, but untreated renal calculi and urinary tract infections may gradually lead to chronic renal failure.

The nurse assesses a patient's condition and suspects that the patient has footdrop. Which assessment finding supports the nurse's suspicion? 1 Internal rotation of entire foot 2 Both the legs bent outward at knee 3 The foot is permanently fixed in plantar flexion 4 Legs curved inward, so knees come together as person walks

Footdrop is the leg contracture in which the foot is permanently fixed in plantar flexion, and the patient is unable to lift the toes off the ground. Internal rotation of the entire foot is a postural abnormality called pigeon toes. Bowleg is a condition in which the patient has both the legs bent outward at the knee. Knock-knee is the postural abnormality in which legs curve inward so the knees come together as the person walks.

Which of the following is a potential hazard that you should assess when the patient is in the prone position? 1. Planter flexion 2. Increased cervical flexion 3. Internal Rotation of the shoulder 4. Unprotected pressure points at the sacrum and heels

Footdrop. Allowing the foot to be dorsiflexed at the ankles prevents this

Shear

Force exerted against the skin while the skin remains stationary and the bony structures move.

Pressure Ulcer Stage 4

Full thickness tissue loss with exposed bone, tendon, or muscle; extensive destruction; tissue necrosis; slough and eschar may be present; often includes undermining and tunneling

While performing hand hygiene, the nurse avoids wearing rings. What is the rationale behind this action? 1 To ensure complete antimicrobial action 2 To prevent a Staphylococcus aureus infection 3 To prevent an increase in the number of bacteria residing on the hands 4 To provide enough time for the antimicrobial solution to be effective

Gram-negative bacilli such as Enterobacter and Staphylococcus aureus are more common under rings; therefore, the nurse should not wear rings to avoid infections. The nurse rubs the hands together by covering all the surfaces of the hands and fingers with antiseptic to ensure complete antimicrobial action. The nurse's fingernails should be less than a quarter-inch long to decrease the number of bacteria residing on hands. The nurse rubs his or her hands together with an antiseptic for several seconds and allows his or her hands to dry before applying gloves to provide enough time for the antimicrobial solution to be effective.

In the hospital setting, what is the most likely means of transmitting infection between patients? 1 Exposure to another patient's cough 2 Sharing equipment among patients 3 Disposing of soiled linen in a shared linen bag 4 Contact with a healthcare worker's hands

Hands become contaminated through contact with the patient and the environment and serve as an effective vector of transmission. Exposure to another patient's cough and the sharing of equipment between patients can also lead to cross infection between patients, but proper hand hygiene by all healthcare workers is the most effective way to break the chain of infection.

Which is the most effective way to break the chain of infection? 1 Hand hygiene 2 Wearing gloves 3 Placing patients in isolation 4 Providing private rooms for patients

Hands become contaminated through contact with the patient's environment. Clean hands interrupt the transmission of microorganisms. Wearing gloves, placing patients in isolation and providing private rooms also can help break the chain of infection, but hand hygiene is the most effective method.

Identify the descriptive characteristics of body alignment and mobility related to the following developmental stage. (Young to middle adults)

Healthy adults also have the necessary musculoskeletal development and coordination to carry out ADLs.

All of the following measures are used to assess for deep vein thrombosis, except: 1. Checking for positive Homan's signs 2. Asking the patient about the presence of calf pain 3. Observing the dorsal aspect of the lower extremities for redness, warmth, and tenderness 4. Measuring the circumference of each leg daily, placing the tape measure at the midpoint of the knee

Homan's signs is no longer a reliable indicator in assessing for DVT

Which condition is associated with increased risk of footdrop? 1 Kyphosis 2 Hemiplegia 3 Osteoporosis 4 Disuse syndrome

In foot drop, the foot is permanently fixed in plantar flexion, resulting limited mobility. Patients with hemiplegia are at increased risk of developing footdrop. Kyphosis refers to increased convexity in curvature of the thoracic spine. Osteoporosis may result from decreased bone density. Disuse syndrome refers to impaired physical mobility.

Explain immobility related respiratory change: Hydrostatic pneumonia

Inflammation of the lung from stasis or pooling of secretions

A patient with influenza is admitted to a hospital. Which infection-control precautions should the nurse take to prevent spread of the virus? Select all that apply. 1 Wearing gloves while reviewing the medical report 2 Wearing a surgical mask within 3 feet of the patient 3 Wearing a sterile gown while entering the patient's room 4 Maintaining proper hand hygiene during the assessment 5 Placing the patient in an airborne infection isolation room

Influenza is an example of an infection that is transmitted by large droplets. Therefore droplet precautions are required, which include wearing a surgical mask within 3 feet of the patient and maintaining hand hygiene during the assessment. While reviewing the patient's medical report, the nurse does not need to wear gloves. The patient is placed in an airborne infection isolation room when an airborne infection, such as tuberculosis, is present or suspected. Contact precautions require a gown and gloves, as in the case when caring for a patient with Clostridium dificile (C. dif) or drug resistant microorganism strains (MRSA, ORSA, VRE).

Technique to assess Metabolic hazards of immobility and their abnormal findings

Inspection: Muscle atrophy, Anthropometric measurements: Decreased amounts of subcutaneous fat, Palpation: Edema

Non rebreather mask

It has a one-way valves that prevent exhaled air from returning to the reservoir bag. The flow rate should be a minimum of 10 L/min and deliver FIO2 of 60% to 80%. Frequently inspect the reservoir bag to make sure that it is inflated. If it is deflated, the patient is breathing large amounts of exhaled carbon dioxide. High-flow oxygen systems should be humidified.

While assessing a patient with impaired mobility, the nurse prioritizes which type of related complication? 1 Social isolation 2 Respiratory 3 Integumentary 4 Musculoskeletal

Lack of movement and exercise places patients at risk for respiratory complications. The metabolic changes seen in patients with impaired mobility are altered endocrine metabolism and calcium resorption. An ulcer is characterized by inflammation and usually forms over a bony prominence. It is seen due to integumentary changes in patients with impaired mobility. The musculoskeletal changes seen in patients with impaired mobility are temporary impairment and permanent disability. According to Maslow's hierarchy of needs, physiological complications take priority over social isolation. Respiratory complications take top priority to ensure the ABCs: Airway, Breathing, Circulation.

Which microorganism exits through a man's urethral meatus during sexual contact? 1 Ebolavirus 2 Clostridium difficile 3 Neisseria gonorrhea 4 Legionella pneumophila

Neisseria gonorrhea exits through a man's urethral meatus or a woman's vaginal canal during sexual contact. Ebolavirus is transmitted through blood or body fluids. Clostridium difficile causes antibiotic-induced diarrhea. Legionella pneumophila grows only at certain temperatures.

While assessing a child, the nurse finds that the child's legs are bent outward at the knee. Which instruction to the parents is most beneficial for the child? 1 "You need to limit phosphorus intake for your child." 2 "You should provide a vitamin D-rich diet for your child." 3 "You should limit mobility in your child for a few days." 4 "You should purchase a Denis Browne splint for your child."

Outward bending of the legs at the knee indicates bowlegs. It is generally associated with rickets, which occurs due to deficiency of vitamin D. Therefore, the child should be provided with foods that are rich in vitamin D. Phosphorus reduces the risk of rickets in the children. Therefore, parents should not limit phosphorus in a child's diet. Limiting mobility in a child with rickets can impair mobility permanently. A Denis Browne splint is used to reduce the risk of clubfoot; however, it is not useful as a treatment for rickets.

A patient who had undergone a hysterectomy 10 days ago came for a follow-up visit. The patient notices purulent drainage at the incision site. The nurse suspects wound infection and performs assessment for confirmation. Which clinical findings would the nurse evaluate? Select all that apply. 1 Pain 2 Redness 3 Paleness 4 Tenderness 5 Cold sensation

Pain is an important finding in wound infection because it is due to inflammation. Redness is commonly seen in wounds that are infected. Tenderness is observed in a wound infection due to localized swelling and inflammation. The infected wound appears red; it does not appear pale. The skin around the infected wound is warm to the touch.

Which assessment finding may indicate orthostatic hypotension associated with prolonged immobility? 1 Pallor 2 Pain upon breathing 3 Decreased heart rate 4 Increased cardiac output

Pallor may indicate the presence of orthostatic hypotension. The further findings to confirm this condition includes dizziness, light-headedness, nausea, and tachycardia. Pain upon breathing may indicate pneumonia but is not associated with orthostatic hypotension. Increase heart rate is seen in orthostatic hypotension, not decreased heart rate. Decreased cardiac output is seen in orthostatic hypotension, not increased cardiac output.

Which condition is observed by placing the patient in the Sims' position? 1 Excessive lateral flexion 2 Increased cervical flexion 3 Lateral flexion of the neck 4 Plantar flexion of the ankles

Patients positioned in the Sims' position may suffer from lateral flexion of the neck. Excessive lateral flexion of the spine may occur if a patient is positioned in a side-lying position. Patients positioned in the prone position may experience plantar flexion of the ankles. Increased cervical flexion is observed in patients positioned in the supported Fowler's position.

Your ungloved hands come into contact with the drainage from your patient's wound. What is the correct method to clean your hands? 1 Wash them with soap and water. 2 Use an alcohol-based hand cleaner. 3 Rinse them and use the alcohol-based hand cleaner. 4 Wipe them with a paper towel.

Physically removing wound drainage is most effectively accomplished by washing with soap and water. According to the Centers for Disease Control and Prevention (CDC), when hands are visibly soiled due to contact with bodily fluids, only cleaning with soap and water and using friction to clean the hands helps prevent contamination.

While positioning a patient in the supported supine position, the nurse places a pillow under the upper shoulders. What is the rationale behind this intervention? 1 Maintain correct alignment 2 Reduce external rotation of hip 3 Provide support for lumbar spine 4 Reduce internal rotation of shoulder

Placing of pillows under the shoulders while positioning a patient in the supine position maintains correct alignment and prevents flexion contractures of cervical vertebrae. Placing trochanter rolls to the lateral surface of the patient's thighs may reduce external rotation of the hip. Placing a small rolled towel under the lumbar area of the back provides support for the lumbar spine. Placing pillows under pronated forearms reduces internal rotation of the shoulder and prevents extension of the elbows.

A patient who had a hysterectomy 10 days ago has come for a follow-up visit. The patient is experiencing pain and itching at the incision site. The nurse suspects a wound infection and performs an assessment for confirmation. When assessing this patient, what actions should the nurse perform to reduce the spread of infection? Select all that apply. 1 Call for a senior nurse. 2 Wait for the laboratory results. 3 Perform hand-hygiene practices. 4 Use gloves when assessing the wound.5 Use appropriate, personal protective equipment.

Proper hand-hygiene practices are important to control the spread of infection to other sites or other patients. The nurse should use gloves when assessing the wound to prevent cross contamination of the wound and her hand. The nurse should use appropriate, personal protective equipment (PPE) when assessing the wound to prevent the microorganisms from spreading. Calling a senior nurse may be considered only if there is additional assistance required. Waiting for the laboratory results is not required to perform an assessment.

The nurse who is working in a postoperative unit realizes that there is chipped nail polish on her fingers. Another colleague who has artificial nails tells the nurse that it is not a concern. Which of them poses a greater risk for contracting an infection to the patients? 1 There is no risk with either situation. 2 The nurse with artificial nails has a higher risk. 3 The nurse with chipped nail polish has a higher risk. 4 Both nurses have an equal risk of causing infection.

Research has shown that health care providers with chipped nail polish or with artificial nails have greater numbers of microorganisms, and therefore pose a greater risk to the patients. The Centers for Disease Control and Prevention's (CDC) hand hygiene guidelines recommend that artificial nails should not be worn by health care providers when working with high-risk patients.

Supine Position

Rest on their backs; all body parts are in relation to each other

Explain seizure precautions:

Seizure precautions are nursing interventions to protect patients from traumatic injury, positioning for and providing privacy and support after the event

Which treatment is provided for patients with lordosis? 1 Knee braces 2 Denis Browne splint 3 Spine-stretching exercises 4 Bracing with ankle-foot orthotic

Spine stretching exercises is a treatment provided for patients with lordosis. Knee braces are provided for patients with knock-knee. A Denis Browne splint is provided for patients with clubfoot. Bracing with ankle-foot orthotic is provided for patients with footdrop.

Which type of specimen is collected by using a sterile tongue blade? 1 Stool specimen 2 Urine specimen 3 Blood specimen 4 Wound specimen

Stool specimens are collected with sterile tongue blades. Urine specimens are collected with needleless safety syringes. Blood specimens are collected with 20-mL needle-safe syringes. Wound specimens are collected with sterile cotton-tipped swabs or syringes and collection tubes.

Which nursing intervention would increase the risk for joint dislocation in a patient with hemiplegia who is immobile for an extended period? 1 Supporting the patient by holding the arm 2 Supporting the patient with assistive devices 3 Lowering the patient to the floor if he or she faints 4 Instructing the patient to use a cane while walking

Supporting the patient by holding the arm may increase the risk of joint dislocation if the patient falls. Assistive devices, such as gait belts, reduce the risk of falling by maintaining the center of gravity in the midline. Lowering the patient to the floor if he or she faints helps reduce the risk for falls. Instructing the patient to use a cane while walking also helps reduce the risk of falls, but it would not be beneficial for patients with hemiplegia.

Which action should the nurse avoid while opening a sterile item on a flat surface? 1 Keeping the inner contents sterile before use 2 Grasping 3.5 cm of the border to maneuver the field on the table surface 3 Holding the item with one hand while pulling the wrapper away with the other hand 4 Using 1 inch of the inner surface of the package border as a sterile field to add sterile items

The nurse should grasp only 2.5 cm (1 inch) of the border to maneuver the field on a table surface while opening a sterile item on a flat surface. The inner contents should be kept sterile before use to prevent infection. The nurse should hold the item in one hand while pulling the wrapper away with the other hand. The nurse should use nearly 1 inch of the inner surface of the package border around the edges as a sterile field to add sterile items.

Which patient is at greatest risk for developing multiple adverse effects of immobility? 1 1-year-old child with a hernia repair 2 80-year-old woman who has suffered a hemorrhagic cerebrovascular accident (CVA) 3 51-year-old woman following a thyroidectomy 4 38-year-old woman undergoing a hysterectomy

The older the patient and the greater the period of immobility, which can be significant following a hemorrhagic stroke, the greater is the number of systems that can be affected by the immobility.

The nurse is assessing a patient who complains of joint pain. Which types of range of motion assessed by the nurse involve sagittal plane movement? Select all that apply. 1 Eversion 2 Pronation 3 Extension 4 Abduction 5 Dorsiflexion

The range of motion movements in the sagittal plane include extension and dorsiflexion. Eversion and inversion occur in the frontal plane. Pronation occurs in the transverse plane. Abduction is a frontal plane movement.

Which instrument used by the nurse requires surface disinfection? 1 Endoscope 2 Cardiac catheter 3 Urinary catheter 4 Blood pressure cuff

There are two types of disinfection: disinfection of surfaces and high-level disinfection. Noncritical items such as blood pressure cuffs require a surface disinfection. Semi-critical items such as endoscopes require high-level disinfection. Critical items such as cardiac and urinary catheters require sterilization.

Transparent Film dressing

Traps wounds moisture over the wound; ideal for small superficial wounds; can remain in place for up to 5-7 days; allows visual assessment of wound bed without removing dressing

Irrigation of wound

Use a 35 mL syringe with a 19 gauge needle that will deliver saline at 8 psi using sterile technique. Wear mask, googles, gow, and gloves.

Which nursing intervention should the nurse use to minimize friction on a bedridden patient? A) Keeping the skin clean and dry at all times B) Repositioning the patient every 2 hours C) Using an egg-crate pad to distribute the weight evenly. D) Using a lift sheet to move the patient up in bed

Using a lift sheet to move the patient up in bed

Which skin care measures are used to manage a patient who is experiencing fecal and urinary incontinence? A) Keeping the buttocks exposed to air at all times B) Using a large absorbent diaper, changing when saturated C) Using an incontinence cleaner, followed by application of a moisture-barrier ointment D) Frequent cleaning, applying an ointment, and covering the areas with a thick absorbent towel

Using an incontinence cleaner, followed by application of a moisture-barrier ointment

contact precautions

VRE and MRSA

Braden Scale 9 or below

Very high risk

A patient with left-sided weakness asks the nurse, "Why are you walking on my left side? I can hold on to you better with my right hand." What would be the best therapeutic response? 1 "Walking on your left side lets me use my right hand to hold on to your arm. In case you start to fall, I can still hold you." 2 "Would you like me to walk on your right side so you feel more secure?" 3 "Either side is appropriate, but I prefer the left side. If you like, I can have another nurse walk with you who will hold you on the right side." 4 "By walking on your left side I can support you and help keep you from injury if you should start to fall. By holding your waist I would protect your shoulder if you should start to fall or faint.

Walking on the affected (weak side) side and holding the patient around the waist or using a gait belt gives the nurse better control if the patient starts to fall. If the nurse were holding the patient's arm as the patient was falling, this might dislocate the shoulder.

Healthcare professionals have an increased risk of musculoskeletal injuries, because their occupation involves lifting and transferring patients. How can the nurse reduce the risk of musculoskeletal injuries when lifting any person or object? 1 Keep the object away from the body. 2 Bend at the hips and not at the knees. 3 Relax the abdominal muscles. 4 Keep trunk erect and bend the knees.

When lifting an object or person, the trunk should be kept erect and the knees should be bent so that multiple muscle groups are used in a coordinated manner, which prevents strain on single muscles. When lifting, the object should be as close to the body as possible. This helps place the object in the same plane as the lifter and close to the center of gravity for balance. The body should be bent at the knees to maintain the center of gravity and promote use of the stronger leg muscles to do the lifting. The abdominal muscles should be tightened, and the pelvis should be tucked, to provide balance and help protect the back.

The nurse assists a surgical technician in preparing a sterile field. Which action made by the nurse indicates a need for correction? 1 Allowing the flap to lie flat on the table's surface 2 Grasping the outer edge of the tip of the outermost flap 3 Standing close to the sterile field while opening the last flap 4 Opening the outermost flap of the sterile kit away from the body

While preparing a sterile field, the nurse should open the last flap while standing away to field as to avoid contamination. The flap should be allowed to lie flat on the table surface. The outer edge of the tip of the outermost flap should be grasped because the outer surface of the package is considered unsterile. The outermost flap of the sterile kit should be kept away from the body to prevent contamination.

Briefly explain how assessment of body alignment and posture is carried out. (Sitting)

While sitting, the head is erect and the neck and vertebral column are in straight alignment, body weight is distributed on the buttocks and thighs, thighs are parallel and in a horizontal plane, and feet are supported on the floor.

Name the three important dimensions to consistently measure to determine wound healing.

Width, length, & depth

Chest physiotherapy

a group of therapies for mobilizing pulmonary secretions. These therapies include postural drainage, chest percussion, and vibration.

Wound vac

applies a localized negative pressure to draw the edges of a wound together

Tertiary Intention

delayed primary closure; ensure that there is no infection before; could be a surgical wound that got infected and it needs to stay open to get the infection out, once the infection is removed then the surgeon will close it up to heal by primary intention

Sloughing

is the shedding of dead tissue

Quad Cough

technique is for patients without abdominal muscle control such as those with spinal cord injuries. While the patient breathes out with a maximal expiratory effort, the patient or nurse pushes inward and upward on the abdominal muscles toward the diaphragm, causing the cough.

Autolytic debridement

uses synthetic dressings over a wound to allow the eschar to be self-digested by the action of enzymes that are present in wound fluids. You accomplish this by using dressings that support moisture at the wound surface. If the wound base is dry, use a dressing that adds moisture; if there is excessive exudate, use a dressing that absorbs the excessive moisture while maintaining moisture at the wound bed. Some examples of these dressings are transparent film and hydrocolloid dressings.

Medical Errors:

when something that was planned as part of medical care doesn't work out or when the wrong plan was used.

Question 13. A nurse is caring for an obese patient with an abdominal surgical incision that has a separation of wound edges with a large amount of exudate. The primary health-care provider orders a wet to moist dressing every 6 hours. What is most important for the nurse to do when implementing this procedure? 1. Secure the dressing using Montgomery straps. 2. Cleanse the wound with half-normal saline and peroxide. 3. Ensure the gauze packing is saturated and dripping with saline. 4. Dry the wound by patting with gauze from the skin toward the wound.

1

Question 15. During the initial assessment of a newly admitted patient, the nurse observes several small red excoriated lines on the patient's wrists. What is the nurse's best intervention at this time? 1. Place the patient on contact precautions. 2. Place the patient in a negative-pressure room. 3. The patient should be oriented to the assigned bedroom. 4. The patient's condition should be communicated to the primary health-care provider.

1

Question 19. A nurse is assessing a patient who had numerous stitches several days ago for a traumatic injury to the base of the right index finger. Which assessment of the site indicates that the inflammatory response has progressed to an infectious process? 1. Yellow discharge 2. Swelling around the site 3. Inability to flex the finger 4. Feeling of heat when touched

1

Question 20. A nurse is caring for patients with a variety of wounds. Which wound will most likely heal by primary intention? 1. Cut in the skin from a kitchen knife 2. Excoriated perianal area 3. Abrasion of the skin 4. Pressure ulcer

1

Question 3. A patient has been on prolonged bedrest. Which nursing intervention will best prevent pressure ulcers in this patient? 1. Placing an air mattress on the bed 2. Applying a moisture barrier to the sacral area 3. Massaging bony prominences every eight hours 4. Raising the head of the bed to the low-Fowler position

1

Question 4. A nurse is caring for a patient with an infection. For which most common response to infection should the nurse assess the patient? 1. Fever 2. Anorexia 3. Headache 4. Dehydration

1

Question 5. A patient is on contact precautions because of a wound infection. Which action by the nurse is essential when maintaining contact precautions? 1. Ensuring that the nurse's uniform does not come in contact with contaminated surfaces 2. Placing a surgical mask on the patient when being transported outside the room 3. Wearing a surgical mask when entering the patient's room 4. Keeping the door to the patient's room closed at all times

1

Question 9. Which patient information collected by the nurse reflects a systemic response to a wound infection? 1. Hyperthermia 2. Exudate 3. Edema 4. Pain

1

The nurse raises a patient's arm over the head during range-of-motion exercises. What word should the nurse use when documenting exactly what was done during range-of-motion exercises? 1. Flexion 2. Supination 3. Opposition 4. Hyperextension

1

Which is the earliest nursing assessment that indicates permanent damage to tissues because of compression of soft tissue between a bony prominence and a mattress? 1. Nonblanchable erythema 2. Circumoral cyanosis 3. Tissue necrosis 4. Skin abrasion

1

The nurse is caring for a 6-year-old child with hypovolemic shock. The child is a Jehovah's Witnesses. The child needs an immediate blood transfusion, but the child's parents refuse to provide consent. Which would be the most appropriate action? 1 Administer blood to the child. 2 Administer intravenous fluids. 3 Provide oral hydration and nutrition. 4 Seek court order for blood transfusion

1 Jehovah's Witnesses is a cultural and religious group that refuses blood transfusions based on their religious beliefs. According to the Durable Power of Attorney for Health Care (DPAHC) document, the court will grant an order allowing hospitals and healthcare providers to treat children of Christian Scientists or Jehovah's Witnesses who have denied consent for treatment of their minor children. Therefore, to save the life of the child it is most appropriate for the health care provider or nurse to administer blood to the child. Refusing a blood transfusion in the case of hypovolemic shock would most likely result in the death of the child. Administering only intravenous fluids may not help, because the child is in hypovolemic shock. Oral hydration and nutrition will not be sufficient to resuscitate a child who has had severe blood loss. The court will allow the healthcare providers to resuscitate the child; thus the medical personnel need not seek or wait for court orders.

The nurse wraps and ties a cloth to prevent bleeding from the site of injury for a patient who is a victim of a traffic accident. The patient is taken to the hospital and presents with signs of infection. Which action will be taken against the nurse? 1 The nurse will not face any action. 2 The nurse will be given a warning for gross negligence. 3 The nurse will face liable charges for not getting an informed consent. 4 The nurse will be sued for not obtaining orders from the primary healthcare provider before intervening.

1 The nurse has wrapped a cloth to control bleeding in the patient, thus providing appropriate care at the accident site. The nurse cannot be blamed for the patient's infection, because the infection could be due to many reasons. The nurse has not shown gross negligence in this case. The nurse does not need informed consent in the case of a life-saving emergency, nor does the nurse need an order from the healthcare provider before intervening in this case.

The nurse assesses a child in a psychiatric facility. The nurse finds that the child has suicidal thoughts. How should the nurse instruct the child's parents? 1 "You should punish your child for saying such things." 2 "Your child needs to be admitted to the facility immediately." 3 "Don't worry; your child will become normal with medication." 4 "You should carefully observe every movement of your child makes at home."

1 According to the Mental Health Parity Act, patients having suicidal tendencies are to be admitted to the mental health units for supervision. The nurse should instruct the parents that their child should be admitted to the facility, because the child has suicidal tendencies. Punishing the child would not avoid suicidal behavior and may aggravate the tendencies. Having suicidal ideation is life threatening and requires close monitoring of the patient; medications would be of no use in this case. Letting the child go home despite knowing that the child has suicidal tendencies would violate the Mental Health Parity Act.

The nurse is assessing a postsurgical patient who is in acute pain. The patient is not willing to change position for x-rays. The nurse tells the patient that a sedative injection will be administered if the patient does not cooperate for the procedure. Which tort is indicated? 1 Assault 2 Battery 3 Invasion of privacy 4 False imprisonment

1 Assault places an apprehensive patient within harmful or offensive contact without consent. In this case, the nurse is threatening to give sedative injections if the patient does not cooperate with the procedure. This is an example of an assault on the patient. Battery is intentional touching without consent. Invasion of privacy refers to the unwanted intrusion into the private affairs of the patient. False imprisonment is an intentional tort in which a patient is restrained without a legal warrant.

40. A nurse is caring for a 60-year-old adult who is a resident in a rehabilitation center recovering from a right-sided brain attack (cerebrovascular accident, stroke). The patient has orders for out of bed ambulating with assistance as tolerated. Which intervention is most important? 1. Assessing balance 2. Using a bed alarm 3. Encouraging the use of a walker 4. Teaching to rise slowly from a lying to sitting position Hint

1 Assessment is the first step of the nursing process. The nurse must first assess the patient for the presence of problems with strength and balance before moving a patient out of bed.

20. A nurse is caring for an older adult who had a brain attack (cerebrovascular accident) and has a residual problem with chewing and swallowing. What is the greatest risk considering the patient's physical status? 1. Aspiration 2. Constipation 3. Fluid volume deficit 4. Inadequate nutrition Hint

1 Difficulty chewing results in food particles that are dangerously large at the time of swallowing and difficulty swallowing may result in secretions, fluids, or solids entering the tracheobronchial tree rather than the esophagus. The patient is at risk for aspiration.

57. A nurse is teaching a health promotion class for a group of recently retired older adults. Which information should be included to help reduce the risk of the major cause of physical injury in older adults? 1. Remove throw rugs from the home. 2. Look both ways before crossing a street. 3. Do warm-up exercises before engaging in sports. 4. Read the label of an over-the-counter drug before taking it.

1 Falls are the major cause of injury in this age-group. Removing throw/scatter rugs minimizes the risk of a fall.

A patient is abusive and rude with the student nurse. The student nurse documents that the patient is uncooperative and shows symptoms of alcohol withdrawal. As a result the patient will be transferred to a different floor. Which is the best classification of this nurse's error? 1 Libel 2 Slander 3 Malpractice 4 Invasion of privacy

1 Libel is documentation of false entries or defamation of character. The nurse is offended by the patient's behavior, so the nurse documents signs of alcohol withdrawal, even though this is not indicated by rude behavior alone. Slander is oral defamation of character. The nurse is documenting the report, but not verbalizing it, so this is not considered slander. Malpractice is negligence of a professional role. This nursing action does not indicate negligence. Invasion of privacy typically involves releasing a patient's private information without the patient's consent. The nurse has not violated the patient's privacy in this instance.

5. A school nurse is teaching a group of high school students about safety precautions. What should the nurse include in the teaching session that addresses the leading cause of accidental deaths in adolescents? 1. Use a seat belt when riding in a car. 2. Engage in sports that are within your physical abilities. 3. Wear a life preserver when engaging in water-related activities. 4. Resist peer pressure to participate in experimenting with illegal drugs. Hint

1 Motor vehicle accidents are the leading cause of death in the adolescent age group. Many adolescents believe that they are invincible and do not wear seat belts when riding in a car. Seat belts do limit morbidity and mortality associated with motor vehicle accidents.

Hospital administrators have warned the healthcare team about invasion of a patient's privacy. Which could have been the reason for this warning? 1 The nurse read text messages on the patient's cell phone. 2 The health care provider asked the nurse to catheterize the patient. 3 The nurse published a report on the patient's condition without his or her consent. 4 The healthcare team provided cardiopulmonary resuscitation (CPR) without the family's consent.

1 Privacy refers to the patient's right of keeping personal information from being disclosed. Reading text messages on the patient's cell phone is an invasion of the patient's privacy. The healthcare provider asking the nurse to catheterize a patient is an example of collaborative care by the healthcare team. Publishing a report on the patient's condition without consent is a breach of confidentiality. Confidentiality protects the patient's information once it has been disclosed in the healthcare setting. According to the health care law in the United States, cardiopulmonary resuscitation (CPR) should be provided to the patient when required unless the patient has given a Do Not Resuscitate (DNR) order. Consent need not be obtained from the family when providing CPR.

12. A nurse enters a patient's room and observes smoke coming out of an air conditioning duct. The nurse transfers the patient into a wheel chair, moves the patient into the corridor, and closes the door to the patient's room while exiting the room. What should the nurse do next? 1. Report the fire. 2. Attempt to put out the fire. 3. Close the doors on the unit. 4. Evacuate all the patients to another unit. Hint

1 The nurse should report the fire as soon as the safety of the patient in immediate danger is ensured. This guarantees that additional staff members and the fire department are notified to assist in this emergency. Remember the mnemonic RACE: Rescue, Alarm, Confine, and Extinguish.

8. A nurse in the emergency department hears a patient and a family member arguing with each other in a room at the end of the unit. What should the nurse do first? 1. Get another staff member and go to the room together. 2. Ask what is going on and then set limits. 3. Go to the room and attempt to intervene. 4. Have a security guard sent to the room. Hint

1 The situation needs to be assessed, but a health team member should never enter a volatile situation alone. When entering the room the health team members should stand between the patient/visitor and the door.

A 70-year-old diabetic patient is advised to do aerobic exercises. Which activity should the patient perform? 1 Walking 2 Yoga 3 Active range of motion 4 Resistance training

1 Walking Walking is a type of aerobic exercise that would help to metabolize the excess sugar in the blood. Yoga is not a type of aerobic exercise. Active range of motion is one type of stretching and flexibility exercise. Resistance training is meant to increase the muscle strength and endurance.

36. A nurse is providing morning care to a hospitalized patient. Which statement by the nurse is most important before leaving the room after providing care? 1. "Push your call bell if you need anything after I leave." 2. "Would you like the door to your room closed?" 3. "Your lunch will be served at around noon." 4. "Is your room temperature comfortable?" Hint

1 This is the most important statement because it addresses maintaining the patient's safety and security. This statement tells the patient how to alert the nurse in an emergency or call for any reason that the patient desires assistance.

8. Which is the most common nursing intervention to help prevent falls in a hospital? 1) Placing the bed in the lowest position to the floor 2) Using a commode for toileting the patient 3) Transferring patients with two caregivers 4) Locking the wheels of a wheelchair

1 A greater risk for injury to a patient occurs when the mattress of the bed is further from the floor. It is safer if the bed is in the lowest position and the patient's feet are flat on the floor when getting out of bed.

According to Edelman and Mandle's strategies of health promotion, which is classified as a passive strategy? 1 Clean water laws 2 Exercise programs 3 Wearing seat belts 4 Nutrition programs

1 A safe environment and a healthy lifestyle are necessary for health promotion. Clean water laws are considered passive strategies of health promotion. Passive strategies include public health and government legislative interventions. Exercise programs, wearing seat belts, and nutrition programs are considered active strategies of health promotion. An active strategy involves lifestyle changes.

The nurse is assessing a group of patients in the medical surgical unit and ties colored wristbands (as per the American Hospital Association guidelines) to the patients based on the assessment. Which group of patients should receive red wristbands? 1 The patients who have allergies 2 The patients who are at risk of falling 3 The patients who have a psychiatric illness 4 The patients who have a do-not-resuscitate (DNR) order

1 According to the recommendations of the American Hospital Association (AHA), standardized wristband colors should be used for patients. Red bands should be given to patients with allergies. Yellow bands should be given to patients who are at risk of falls. There is no specific color for identifying patients with psychiatric illness according to the AHA. Purple bands are given to patients who have a do-not-resuscitate (DNR) order.

The nurse is teaching a vulnerable population of patients about pollution. Which instruction should the nurse provide to help patients limit exposure to air pollution? 1 "Avoid smoking." 2 "Keep your home noise free." 3 "Avoid using bottled water." 4 "Discard the bioactive waste properly."

1 At home, school, or in the workplace, the most common cause of air pollution is smoking. Excessive noise is also a form of pollution called as noise pollution that affects the health. Bottled or boiled water should be used for drinking and cooking when there is water contamination. Improper disposal of radioactive and bioactive waste causes land pollution rather than air pollution.

A primary health care provider prescribes 10 mg of codeine every 4 hours to a patient who has chronic pain from cancer. However, after taking a second dose of the prescribed drug, the nurse notices that the patient is very drowsy and nauseous. Which dose alteration may provide effective pain relief while improving the drowsiness and nausea? 1 5 mg codeine every 4 hours 2 10 mg codeine every 8 hours 3 20 mg codeine every 8 hours 4 5 mg codeine every 12 hours

1 Because codeine is short-acting, relief is likely only attainable with administration every 4 hours. If 5 mg is not enough, or the side effects remain, the patient may require a different opioid. Even at every 8 hours, 10-mg and 20-mg doses are too much for the patient to tolerate without adverse side effects. A 5-mg dose every 12 hours is probably too long of an interval for this patient to obtain relief from the pain.

Which task is delegated to nursing assistive personnel (NAP)? 1 Checking on a restraint 2 Assessing a patient's behavior 3 Applying restraints appropriately 4 Orientating the patient to the environment

1 Checking on a restraint can be delegated to nursing assistive personnel (NAP). Assessing a patient's behavior, orientating the patient to the environment, and determining the need and appropriate use of the restraints should be performed by the nurse and are not delegated to NAP.

A patient with bronchial carcinoma reports constipation for the past 2 months. The patient has been on meperidine and ibuprofen for pain relief for the past 6 months. The patient has also been taking metformin and captopril for the past 10 years. What could be the most probable reason for constipation in the patient? 1 Side effects of the opioid 2 Side effects of the captopril 3 Interaction of metformin and captopril 4 Metastasis of cancer to other organs

1 Constipation is a common side effect of opioids that are used for pain relief. Captopril is an ACE-inhibitor drug that is used to treat hypertension. Cough is the common side effect of captopril. Metformin is an oral hypoglycemic drug. Interaction between metformin and captopril does not cause constipation. It is unlikely that metastasis of cancer caused the constipation.

While caring for a patient with epilepsy, the nurse recognizes the possibility of tonic-clonic seizure. What is the priority intervention by the nurse? 1 Inserting a bite-block 2 Helping the patient to stand 3 Conducting a head to toe evaluation 4 Notifying the primary health care provider

1 During a seizure, the patient may clench the teeth, which could damage the mucous membranes of the oral cavity. The teeth can also become broken. Therefore, the nurse should immediately insert a bite-block in advance. The nurse should not position the patient to stand. Instead, the nurse should assist the patient to bed. A head to toe evaluation would be appropriate after the seizure attack. The nurse may notify the primary health care provider after inserting the bite-block.

A health care provider writes the order for an opioid-naïve patient who returned from the operating room following a total hip replacement. The order states, "Fentanyl patch 100 mcg, change every 3 days." Based on this order, the nurse plans to implement which actions? 1 The nurse calls the health care provider and questions the order. 2 The nurse applies the patch on the third postoperative day. 3 The nurse applies the patch as soon as the patient reports pain. 4 The nurse places the patch as close to the hip dressing as possible.

1 Fentanyl is 100 times more potent than morphine and not recommended for acute postoperative pain.

The primary health care provider prescribes intravenous (IV) opioid medication for flank pain associated with a kidney stone in the ureter. On the follow-up visit, the patient reports thigh pain to the nurse. What does the nurse infer from patient's report? 1 The patient is experiencing referred pain. 2 The patient is experiencing neuropathic pain. 3 The patient has acute pain progressing to chronic pain. 4 The patient has pain perception due to previous opioid medication.

1 Flank pain is associated with kidney stones in the ureter. The spread of pain to uninjured tissue is referred pain. Here, the pain spreads to the uninjured thigh tissue. Neuropathic pain refers to pain caused by nerve damage rather than by tissue injury or damage. When pain is short term and associated with an acute event such as a kidney stone, it is acute pain, not chronic pain. IV opioid administration would decrease the perception of pain intensity of the kidney stone but would be unrelated to the new complaint of thigh pain.

Which nursing advice should be given to parents to promote safety for their infant? 1 "You should take your infant for immunizations." 2 "You should attach pacifiers to strings or ribbons." 3 "You should place infants on their stomachs to sleep." 4 "You should place large stuffed toys or comforters around the infant when the infant is sleeping."

1 Immunization is advised for a child, because it reduces the risk of sudden infant death syndrome (SIDS). Pacifiers should not be attached to strings or ribbons, because they may cause choking. The best position for children when sleeping is on their back or side, because these positions reduce the risk of sudden infant death syndrome. Large stuffed toys or comforters should not be placed around the child when they are sleeping, because they may cause suffocation or entrapment.

In a health care setting, the restraint order for a patient is renewed every hour. What is the likely age of the patient? 1 6 years 2 14 years 3 23 years 4 65 years

1 In a hospital setting, each original restraint order and renewal is limited to 8hours for adults, 2hours for children ages 9 to 17, and one hour for children under the age of 9. Therefore, a restraint order for a 6-year-old child will require renewal every hour. A restraint order for a 14-year-old patient will require renewal every 2 hours. Restraint orders for the 23-year-old and 65-year-old patients will require renewal every 8 hours.

A couple approaches the nurse to seek guidance regarding taking their 10-year-old child on a long ride in a car. During the discussion, the nurse learns that the car has front-seat passenger air bags. What advice should the nurse provide to this couple? 1 Advise that the child ride in the back seat. 2 Suggest that the child ride in the front seat. 3 Suggest keeping the child free from any restraints. 4 Advise of the need for an appropriate car seat for this child.

1 It is safe for a 10-year-old child to ride in the back seat. In case of accidents or a car crash, the child would sustain fewer injuries if seated in the back seat. In case of a sudden stop or a car crash, the child would be susceptible to suffering severe head injuries if unrestrained. Use of seat belts for the child is advised. An appropriate car seat is usually required for children less than 8 years of age or 80 pounds in weight.

A patient has accidently consumed kerosene. Which nursing intervention would further complicate the patient's condition? 1 Inducing vomiting 2 Maintaining the airway 3 Administering oxygen 4 Measuring oxygen saturation

1 Kerosene is a poisonous substance. Inducing vomiting in a patient who has consumed kerosene is dangerous, because it can cause aspiration. Maintaining the airway is the primary measure to reduce the risk of aspiration and is thus important in cases of kerosene poisoning. Oxygen administration is helpful in kerosene poisoning. Measuring oxygen saturation is an important intervention for this patient, because it helps to identify the need for intubation and other assistive measures.

35. A nurse is preparing a program regarding health promotion for individuals who are middle-aged adults. Which information is most important to include regarding the prevention of injury? 1. Avoiding alcohol when planning to drive 2. Learning to use meditation to reduce stress 3. Doing warm-up exercises before engaging in sports 4. Maintaining an exercise program to increase strength and endurance Hint

1 Motor vehicle accidents, often linked to alcohol use, are the leading cause of death and injury to middle-aged adults. This is the reason why this topic is most important to include in a discussion of injury prevention.

A 62-year-old woman is being discharged home with her husband after surgery for a hip fracture from a fall at home. What should the nurse consider when providing discharge teaching about home safety to this patient and her husband? 1 A safe environment promotes patient activity. 2 Assessment focuses on environmental factors only. 3 Teaching home safety is difficult to do in the hospital setting. 4 Most accidents with the older adult are caused by lifestyle factors.

1 Older adults are frequently fearful of falling and thus often limit activity. A safe environment, which decreases the risk of a fall, promotes patient activity.

The nurse has given one unit of transmucosal fentanyl to an opioid-tolerant patient with breakthrough pain. The patient is still not feeling pain relief. How many more units of the drug can the nurse administer before notifying the primary health care provider? Record your answer using a whole number. ___ unit(s)

1 One transmucosal fentanyl unit is given to patients with breakthrough pain. It is swabbed over the buccal mucosa and gums to be dissolved in the mouth. It should not be chewed. The nurse has given one unit of fentanyl already; if the pain persists, the nurse can administer one more unit of fentanyl. A patient can be given a total of 2 units of transmucosal fentanyl per episode of breakthrough pain. If the patient's pain is not relieved, then the nurse should notify the primary health care provider.

A patient complains of nausea after receiving the first dose of morphine for pain. What should the nurse do? 1 Treat nausea with an anti-nausea medication and continue to use morphine 2 Request an order for a nonsteroidal anti-inflammatory drug (NSAID) instead of morphine. 3 Encourage the patient to wait as long as possible for the next dose. 4 Withhold the next dose of morphine until reevaluated by the health care provider.

1 Opioids can cause nausea and vomiting because of the action on the brainstem centers. This side effect decreases with repeated use, but until then, treatment for nausea should be instituted. Decreasing the dose may be ineffective for pain relief. Asking the patient to wait for pain relief is unethical. Withholding the dose may increase the pain.

A registered nurse is teaching a nursing student about using nonsteroidal antiinflammatory drugs (NSAIDs) for pain management. Which of the nursing student's statements indicates a need for further teaching? 1 "NSAIDs work by depressing the central nervous system." 2 "NSAIDs act by inhibiting the synthesis of prostaglandins." 3 "Patients allergic to aspirin are more likely to be allergic to other NSAIDs." 4 "Use of NSAIDS in older adults may result in increased risk of adverse events."

1 Opioids, not nonsteroidal antiinflammatory drugs (NSAIDs), depress the central nervous system. The other statements indicate effective teaching: NSAIDs inhibit prostaglandin synthesis, which inhibits cellular responses to inflammation; this helps relieve pain. An allergy to aspirin may be indicative of an allergy to other NSAIDs, and NSAIDs may put older adults at an increased risk for gastrointestinal bleeding.

A patient has had arthritic pain for 8 years and has surgery to remove a buildup of septic fluid. Postoperative, the patient received morphine through a patient-controlled analgesia (PCA) device for the management of pain. What is the advantage of PCA that the nurse should teach the patient? 1 PCA allows self-administration of analgesics. 2 PCA is associated with a risk of overdose. 3 PCA does not allow administration of opioids. 4 PCA allows intramuscular administration of medications.

1 PCA allows the patient to self-administer analgesic medication whenever needed. There is no risk of overdosage due to the programming. Opioids can be safely administered using PCA. It allows intravenous or subcutaneous administration of medications.

Which instructions are crucial for the nurse to give to both family members and the patient who is about to be started on patient-controlled analgesia (PCA) of morphine? 1 Only the patient should push the button. 2 Do not use the PCA until the pain is severe. 3 The PCA prevents constipation. 4 Notify the nurse when the button is pushed.

1 Patient preparation and teaching are critical to the safe and effective use of PCA devices. Patients need to understand PCA and be physically able to locate and press the button to deliver the dose. Be sure to instruct family members not to push the button for the patient.

A postoperative patient is currently asleep. Which statement is correct? 1 The sedative administered may have helped him sleep, but assessment of pain is still needed. 2 The intravenous (IV) pain medication is effectively relieving his pain. 3 Pain assessment is not necessary. 4 The patient can be switched to the same amount of medication by the oral route.

1 Sedatives, antianxiety agents, and muscle relaxants have no analgesic effect; however, they can cause drowsiness impaired coordination, judgment, and mental alertness and contribute to respiratory depression. It is important to avoid attributing these adverse effects solely to the opioid. You need to conduct a thorough reassessment.

What is the leading cause of injuries involving home medical oxygen? 1 Smoking 2 Lead poisoning 3 Exposure to severe cold for long periods 4 Exposure to high concentrations of carbon monoxide

1 Smoking is the leading cause of burns, reported fires, deaths, and injuries involving home medical oxygen. Exposure to severe cold for prolonged periods of time causes frostbite and accidental hypothermia. Exposure to excessive levels of lead poisoning may lead to learning and behavioral problems in children, as well as brain and kidney damage. Exposure to high concentrations of carbon monoxide may cause death within 1 to 3 minutes.

The nursing instructor asks the student nurse to differentiate between A and C peripheral nerve fibers. Which statement made by the student nurse indicates effective learning? 1 "The A fibers are myelinated and the C fibers are unmyelinated." 2 "The A fibers are smaller in diameter and the C fibers are larger in diameter." 3 "The A fibers transmit signals slowly and the C fibers transmit signals rapidly." 4 "The A fibers cause diffuse sensation and the C fibers cause localized sensations."

1 The A fibers are myelinated, whereas the B fibers are unmyelinated. The A fibers are larger in diameter, whereas the B fibers are smaller in diameter. Because A fibers are larger in diameter, they transmit signals to the central nervous system (CNS) more rapidly than the smaller C fibers. The sensations caused by the stimulation of the A fibers are localized, whereas the sensations caused by the stimulation of the B fibers are diffuse.

Which is a serious reportable event included in the National Quality Forum List? 1 Immediate postoperative death 2 Hypothermia due to severe cold conditions 3 A fall associated with administration of diuretics 4 Injury resulting from physical assault that occurred at a patient's home

1 The National Quality Forum has compiled a list of serious reportable events that are the major focus of health care providers for patient safety initiatives. Events such as intraoperative or immediate postoperative death should be reported immediately because they are serious reportable events. Hypothermia due to severe conditions is not included on the list. Likewise, a fall associated with administration of diuretics is not listed as a serious reportable event. Injury due to physical assault in the health care facility, not at a patient's home, is a serious reportable event included on the National Quality Forum List.

When performing pain assessment, the nurse shows a series of photographs to a child and asks the child to point to the face that shows how he or she feels. Which pain-rating scale is the nurse using for pain assessment? 1 Oucher scale 2 Numeric rating scale (NRS) 3 Visual analogue scale (VAS) 4 Verbal descriptor scale

1 The Oucher scale consists of six cartoon faces of a child ranging from a smiling face to less happy faces, to a final sad, tearful face. The child is asked to point to the face that best matches his or her pain. With the NRS, the nurse asks the child to choose a number to rate the level of pain. The VAS has the patient assess the pain on a 10-centimeter line, ranging from no pain to severe pain. With the verbal descriptor scale, the nurse asks the child to describe his or her feelings about the intensity of pain.

What is recommended on the World Health Organization (WHO) analgesic ladder while caring for a patient with cancer pain? 1 Transitioning use of adjuvants with nonsteroidal anti-inflammatory drugs (NSAIDs) to opioids 2 Using acetaminophen for refractory pain 3 Limiting the use of opioids because of the likelihood of side effects 4 Avoiding total sedation regardless of how severe the pain is

1 The WHO analgesic ladder transitions from the use of nonopioids (NSAIDs) with or without adjuvants to opioids with or without adjuvants. Acetaminophen is recommended for lesser levels of pain. Side effects related to the use of opioids may be unavoidable but are treatable. Treatment for severe pain may result in some level of sedation.

At 3 AM the emergency department nurse hears that a tornado hit the east side of town. Which action should the nurse take first? 1 Prepare for an influx of patients. 2 Contact the American Red Cross. 3 Determine how to restore essential services. 4 Evacuate patients per the disaster plan.

1 The emergency department nurse first needs to prepare for the potential influx of patients. Staff should be aware of the disaster plan. Patients may need to be evacuated but not initially. The American Red Cross is not contacted initially. Determination of how to restore essential services is part of the disaster plan and is determined before an actual event.

A patient is on a lidocaine patch for neuropathic pain. How should the lidocaine be given to the patient to achieve adequate pain control and avoid lidocaine toxicity? 1 12-hours-on, 12 hours-off schedule 2 12-hours on, 6 hours-off schedule 3 48-hours -on, 12 hours-off schedule 4 24-hours-on, 12 hours-off schedule

1 The lidocaine patch is a topical analgesic and is used for cutaneous neuropathic pain control in adults. Three patches of the appropriate size are placed over and around the pain site. To avoid lidocaine toxicity, the 12-hours-on, 12-hours-off schedule is used. All the other schedules such as the 12-hours-on, 6-hours-off schedule; 48-hours-on, 12-hours-off schedule; and the 24-hours-on, 12-hours-off schedule may cause toxicity.

During the subjective data collection for pain assessment, the nurse asks the patient, "Can you tell me what your discomfort feels like?" What is the reason for this question? 1 The nurse wants the patient to identify the quality of pain. 2 The nurse wants the patient to identify the severity of pain. 3 The nurse wants the patient to identify the duration of pain. 4 The nurse wants the patient to indentify the intensity of pain.

1 The nurse asks questions such as, "Can you tell me what your discomfort feels like?" to assess the quality of pain. To identify the severity of pain, the nurse can ask, "On a scale of 0 to 10, how bad is your pain now?" To identify the onset and duration of pain the nurse can ask, "When did your pain start?" To identify the intensity of pain the nurse can ask, "How much pain do you have now?"

The nurse explains patient-controlled analgesia to a patient. If the patient has understood this information, what would be the patient's most appropriate statement? 1 The device reduces the risk of an overdose of medication. 2 The caregivers can operate the device if the patient is unable to do so. 3 The patient will be lying in a prone position during the procedure. 4 The patient will decide about the loading dose of the analgesic drug.

1 The nurse should teach about the use of patient-controlled analgesia (PCA) to a patient before any procedure. It is important to tell the patient that PCA reduces any risk of overdose. It should be emphasized to the patient that the patient-controlled analgesia device (PCA device) should not be operated by the caregivers. The caregivers are not able to perceive the patient's pain and thus cannot decide the amount of drug required. The patient should be placed in a comfortable position in which the IV line is accessible. The prone position is not likely to be a comfortable position for the patient. The patient does not decide the loading dose of the drug; the loading dose is prescribed before use.

4. A nurse is caring for a patient who is unable to cough effectively. Which is an appropriate expected outcome when providing nursing care for this patient? 1) Presence of bronchovesicular breath sounds 2) Respirations remain at about ten per minute 3) Self-administers oxygen when necessary 4) Uses pursed-lip breathing

1 The presence of bronchovesicular breath sounds is an appropriate expected outcome for a patient experiencing a threat to respiratory status related to an inability to cough effectively. Effective coughing clears the bronchi of secretions, which results in bronchovesicular breath sounds.

46. Which nursing intervention is most essential to provide for patient safety, regardless of the patient's individual health issue? 1. Keep the bed in the lowest position. 2. Ensure the call bell is within reach. 3. Raise the 4 side rails when in bed. 4. Check the patient every 2 hours. Hint

1 There is always a potential to fall even when in bed; keeping the bed in the lowest position allows the feet to touch the floor before standing and if a fall occurs, the potential for injury probably will be less than from a higher height.

9. A patient in the emergency department tells the nurse, "I am allergic to penicillin." What should the nurse do next? 1. Ask the patient about what kind of responses occurred. 2. Ask the patient about the presence of other types of allergies. 3. List the allergy on an allergy alert band on the patient's wrist. 4. List the allergy in the patient's medication administration record. Hint

1 This information relates to the specific allergy identified by the patient. The specific reaction to each allergy should also be determined as each allergy is identified.

The nurse is caring for a patient in the home and is checking for hazards. Which assessment made by the nurse is priority? 1 Assessing the adequacy of light 2 Assessing the presence of safety devices 3 Assessing the locks to doors and windows 4 Assessing the kitchen and bathroom for safety

1 When the nurse is caring for the patient at home, the priority action is to assess the adequacy of light. Sufficient lighting in the room ensures that everything is visible and reduces the risk for injuries. The presence of safety devices, the presence of locks, and the kitchen and bathroom should be assessed after the adequacy of light is assessed.

. A patient questions the nurse about narcolepsy. Which response by the nurse is accurate? 1 "Narcolepsy is a dysfunction of mechanisms that regulate sleep and wake states." 2 "Narcolepsy is a disorder characterized by the lack of airflow through the nose and mouth." 3 "Narcolepsy is a symptom that patients experience when they have chronic difficulty falling asleep." 4 "Narcolepsy involves sudden muscle weakness during intense emotions such as anger, sadness, or laughter."

1 "Narcolepsy is a dysfunction of mechanisms that regulate sleep and wake states."

A nurse is preparing a health promotion and illness prevention class for a group of parents of young school-aged children. Which information should the nurse include in the program to help prevent sensory disturbances in children in this age-group? SELECT ALL THAT APPLY. 1. Have children vaccinated against rubella, mumps, and measles. 2. Discourage sports that use sticks, such as hockey and lacrosse. 3. Seek regular eye examinations to screen for eye problems. 4. Allow use of pointed scissors only when sitting at a table. 5. Encourage the use of sunglasses with UV protection.

1 3 5 Answer 1 is correct because rubella, mumps, and measles can cause hearing loss. Children should be immunized against these diseases. Answer 3 is correct because regular eye examinations help identify problems early so that they can be treated. Answer 5 is correct because sunglasses with UV protection help avoid damage from ultraviolet rays. In addition, the nurse should encourage parents to teach their children to never look directly into the sun. Answer 2 is incorrect because children can engage in sports that involve sticks if they are taught the proper use of sports equipment and wear protective gear (e.g., helmets, eye shields). Answer 4 is incorrect because young school-aged children always should use round, not sharp, tipped scissors.

A sleeper's random eye movement (REM) sleep is around 30% of the sleep time. The sleeper belongs to which age group? 1 Infants 2 Neonates 3 Young adults 4 Preschoolers

1 Infants

For what does an electromyogram (EMG) monitor in a patient diagnosed with a sleep disorder? 1 Muscle tone 2 Eye movements 3 Electrical activity of the heart 4 Electrical activity in the cerebral corte

1 Muscle tone

A patient has been diagnosed with narcolepsy. What would the nurse teach the family caregivers about narcolepsy? Select all that apply. 1 "Sleep paralysis is usually present in patients who have narcolepsy." 2 "Symptoms of narcolepsy first appear in the adolescent stage." 3 "Patients who have narcolepsy are prescribed sleep-promoting medications." 4 "Narcolepsy is a dysfunction of mechanisms that regulate sleep and wake states." 5 "Excessive nighttime sleepiness is a common complaint associated with narcolepsy."

1 "Sleep paralysis is usually present in patients who have narcolepsy." 2 "Symptoms of narcolepsy first appear in the adolescent stage." 4 "Narcolepsy is a dysfunction of mechanisms that regulate sleep and wake states."

A patient reports difficulty falling asleep. Further assessment shows that the patient consumes alcohol. What are the effects of alcohol on sleep? Select all that apply. 1 Alcohol promotes sleep. 2 Alcohol prevents the patient from falling asleep. 3 Alcohol causes the patient to remain awake. 4 Alcohol awakens the patient early and causes difficulty in returning to sleep. 5 Alcohol reduces rapid eye movement (REM) sleep.

1 Alcohol promotes sleep. 4 Alcohol awakens the patient early and causes difficulty in returning to sleep. 5 Alcohol reduces rapid eye movement (REM) sleep.

A patient complains that he falls asleep uncontrollably at inappropriate times and has vivid dreams during such times. Which advice should the nurse give to the patient to avoid factors that increase drowsiness? Select all that apply. 1 Avoid alcohol 2 Avoid smoking 3 Avoid relaxing activities 4 Avoid heavy meals 5 Avoid long-distance driving

1 Avoid alcohol 4 Avoid heavy meals 5 Avoid long-distance driving

A head nurse is teaching nursing students about the different phases of sleep. What information should the nurse include? Select all that apply. 1 Children experience more stage 4 nonrapid eye movement (NREM) sleep. 2 Dreams occur during rapid eye movement (REM) sleep only. 3 Personality has an influence on dreams. 4 Dreams occur during both NREM and REM sleep. 5 There is a decreased epinephrine release during REM sleep

1 Children experience more stage 4 nonrapid eye movement (NREM) sleep. 3 Personality has an influence on dreams. 4 Dreams occur during both NREM and REM sleep.

Many kinds of noise are generated during the delivery of health care in a variety of settings. What may be the effects of these noises on the patient? Select all that apply. 1 Delayed healing 2 Increased agitation 3 Decreased heart rate 4 Lowered blood pressure 5 Impaired immune function

1 Delayed healing 2 Increased agitation 5 Impaired immune function

The school nurse is teaching a group of high school students about health-promoting behaviors that improve sleep. Which points should be included in the education? Select all that apply. 1 Do not study in your bed. 2 Go to sleep each night whenever you feel tired. 3 Turn off your cell phone at bedtime. 4 Avoid drinking coffee or soda before bedtime. 5 Turn on the television to help you fall asleep.

1 Do not study in your bed. 3 Turn off your cell phone at bedtime. 4 Avoid drinking coffee or soda before bedtime.

After assessing a patient, the nurse identifies that the patient is predisposed to central sleep apnea. What assessment findings would suggest this condition? Select all that apply. 1 Encephalitis 2 Nasal polyps 3 Enlarged tonsils 4 Brainstem injury 5 Deviated septum

1 Encephalitis 4 Brainstem injury

An obese patient is diagnosed with obstructive sleep apnea. Which conservative approaches are included in the patient's treatment plan? Select all that apply. 1 Limit alcohol consumption. 2 Motivate the patient to lose weight. 3 Recommend that the patient take sedatives. 4 Recommend that the patient take herbal medicines. 5 Avoid the consumption of stimulants before going to bed.

1 Limit alcohol consumption. 2 Motivate the patient to lose weight. 5 Avoid the consumption of stimulants before going to bed.

The nurse is assessing a patient who has sleep apnea. Which symptoms are likely to be found in the patient? Select all that apply. 1 Loud snoring 2 Muscle weakness 3 Decreased sex drive 4 Difficulty concentrating 5 Excessive urination during the night

1 Loud snoring 3 Decreased sex drive 4 Difficulty concentrating

The nurse is explaining the physiology of sleep to the student nurses. What information should the nurse include? Select all that apply. 1 Prostaglandin D2 controls sleep. 2 The release of serotonin from the bulbar synchronizing region prevents sleep. 3 Major sleep center is located in the hypothalamus. 4 The ascending reticular activating system (RAS) maintains wakefulness. 5 Hypocretins promote nonrapid eye movement sleep.

1 Prostaglandin D2 controls sleep. 3 Major sleep center is located in the hypothalamus. 4 The ascending reticular activating system (RAS) maintains wakefulness.

A patient has a leg fracture. Which statement made by the patient about crutch safety requires further teaching by the nurse? 1 "I should lean on the crutches to support my body weight." 2 "I should regularly inspect the structural intactness of the crutches." 3 "I should dry the crutch tips using paper towels if they become wet." 4 "I should immediately replace any worn crutch tips to prevent slipping."

1 "I should lean on the crutches to support my body weight." The nurse teaches the patient about crutch safety in order to prevent further injuries. The patient should not lean on the crutches to support body weight because that may damage the crutches and also increase the risk of falling. Therefore, the patient's first statement requires correction by the nurse. The patient should regularly inspect the structure of the crutches because cracked crutches will increase the risk of falls. The patient should dry the crutch tips using a paper or cloth towel to decrease the risk of slipping. The patient should replace worn crutch tips with new ones to prevent slipping.

A graduate nurse is being mentored by a more experienced nurse. They are discussing the ways nurses need to remain active professionally. Which statements below indicate the new graduate understands ways to remain involved professionally? Select all that apply. 1 "I am thinking about joining the health committee at my church." 2 "I need to read newspapers, watch news broadcasts, and search the Internet for information related to health." 3 "I will join nursing committees at the hospital after I have several years of experience and better understand the issues affecting nursing." 4 "Nurses do not have very much voice in legislation in Washington, DC, because of the shortage of nurses." 5 "Nurses should take direction from physicians in the hospital setting."

1 2 Nurses need to be actively involved in their community and be aware of current issues in health care. Staying abreast of current news and public opinion through the media is essential. Nurses need to join nursing committees at all levels of their career, not just when they have experience. Nurses have a powerful voice in the legislature.

42. A nurse is caring for a patient who is hearing impaired and legally blind. What should the nurse do to prevent a sensory deficit? SELECT ALL THAT APPLY. 1. Encourage the patient to wear prescribed hearing aids. 2. Provide a telephone, radio, and talking books. 3. Encourage purchase of a talking watch. 4. Provide a foot bath and back massage. 5. Speak in a high tone of voice.

1 2 3 4

56. A school nurse is teaching a third-grade class about safety precautions. What should the nurse include in the teaching session? SELECT ALL THAT APPLY. 1. Run in the opposite direction if approached by a stranger to go someplace. 2. Take a bicycle safety course when you get a two-wheeled bicycle. 3. Get instructions before using equipment for the first time. 4. Use a regular car seat belt when you reach 50 pounds. 5. Stop, drop, and roll if your clothes are on fire.

1 2 3 5

An 8-year-old girl is brought to the emergency room with severe abdominal pain. The nurse has to get consent before medical treatment can begin. The parents of the child are divorced and the mother has custody. The patient is accompanied by her mother, father, and sister, the latter aged 19 years old. What information should be provided before the patient's guardian gives consent? Select all that apply. 1 A complete explanation of the procedure or treatment 2 Names and qualifications of the people performing the treatment 3 The exact number of days required for complete cure and treatment 4 A description of possible adverse effects or side effects of the treatment 5 An explanation that once the guardian signs the consent, treatment must be given

1 2 4 The patient or the patient's guardian should give consent only after receiving information about the procedure or treatment. The information should also include the names of the people who will be treating the patient and the possible side effects of the treatment or procedure. The nurse should also inform the guardian that she can later refuse treatment even if she initially signed consent. It is not practical for the nurse to anticipate the exact number of days required for a cure, and such information is not provided before the guardian gives consent.

Which statements best describe a consent form? Select all that apply. 1 It may be signed by an emancipated minor. 2 It protects the healthcare facility but not the healthcare provider. 3 It signifies that the patient understands all aspects of the procedure. 4 It signifies that the patient and family have been told about the procedure. 5 It must be signed by the patient or responsible party at the healthcare facility, and consent may not be obtained by phone or fax.

1 3 An emancipated minor may sign a consent form. The consent form signifies that the patient understands all aspects of the procedure. The document protects the surgeon and the healthcare facility in that it indicates that the patient knows and understands all aspects of the procedure. Only in the cases of underage children or unconscious or mentally incompetent people must a family member be aware of the procedure. The consent may be obtained by fax or phone with appropriate witnesses.

25. Which nursing action supports a patient's basic need for safety and security? SELECT ALL THAT APPLY. 1. Positioning a commode next to a patient's bed at night 2. Teaching a patient how to transfer from the bed to a chair 3. Placing a patient's dentures in a labeled denture cup in the bedside table 4. Assessing a patient's abdomen for clinical indicators of urinary retention 5. Ensuring that a patient with neutropenic precautions is provided a mask when being transported for an x-ray

1 3 2 5

A patient is admitted to the ambulatory surgery unit for an elective procedure. When performing a physical assessment the nurse identifies that the patient has Pediculus capitis (head lice). Place the nurses interventions in the order in which they should be implemented. 1. Establish contact isolation 2. Comb the hair with a fine-toothed comb. 3. Notify the provider of the patient's condition. 4. Obtain a prescription for a pediculicidal shampoo. 5. Wash the patient's hair with a pediculicidal shampoo.

1 3 4 5 2

34. A school nurse is planning a class about safety precautions for a group of parents who have children between the ages of 7 and 10 years. What information should the nurse include in the teaching plan? SELECT ALL THAT APPLY. 1. "Supervise stretching exercises before your child engages in physical activities." 2. "Check your child's toys to ensure that they are larger than a clenched fist." 3. "Select activities that are appropriate for your child's developmental level." 4. "Provide a four-point safety seat for your child when in the car." 5. "Ensure that your child wears a seat belt when in the car." Hint

1 3 5

Which actions, if performed by a registered nurse, would result in both criminal and administrative sanctions against the nurse? Select all that apply. 1 Taking or selling controlled substances 2 Refusing to provide healthcare information to a patient's child 3 Reporting suspected abuse and neglect of children 4 Applying physical restraints without a written physician's order 5 Administering the wrong medication to the patient

1 4 The inappropriate use of controlled substances is prohibited by every Nurse Practice Act. A physical restraint can be applied only on the written order of a healthcare provider based on Joint Commission and Medicare guidelines. Refusing to provide healthcare information to a patient's child, reporting suspected abuse and neglect of children, and administering the wrong medication to a patient would not result in both criminal and administrative sanctions against the nurse.

38. A nurse is caring for a patient who is in the intensive care unit. Which action should the nurse implement to prevent sensory overload? SELECT ALL THAT APPLY. 1. Organize care into blocks of time. 2. Provide objects that are pleasant to touch. 3. Maintain frequent meaningful interaction. 4. Medicate for pain immediately when it occurs. 5. Explain the meaning of sounds in the environment. Hint

1 4 5

A nurse is caring for a patient who has an order for a vacuum-assisted closure device using black foam to facilitate wound healing. The nurse verifies the order, explains to the patient what is to be done and why, gathers equipment, washes the hands, sets a sterile field, and dons sterile gloves. Place the following steps in the order in which they should be implemented. 1. Trim the black foam to the size of the wound cavity. 2.Pinch and cut a 2-cm round hole in the center of the transparent film. 3. Connect the suction device tubing to the collection canister tubing and pump 4. Place the foam in the wound cavity without overlapping onto the surrounding skin 5. Place the suction device over the hole in the film and apply gentle pressure to secure in place 6. Apply the transparent film 1 or 2 inches beyond wound edges without stretching or wrinkling the film

1 4 6 2 5 3

A nurse plans to remove a patient;s wound dressing. The nurse identifies the patient, explains what is going to be done and why, washes the hands, collects equipment, provides for the patient's privacy, and places the patient in an appropriate and comfortable position. Place the following steps in the order in which they should be implemented when removing the soiled dressing. 1. Don clean gloves 2. Pull tape away from the skin gently 3. Assess the volume, color, and order of exudate 4. Place the soiled dressing and gloves in a bio-hazardous waste receptacle. 5. Remove the dressing b lifting the edge of the dressing upward and toward the center of the wound 6. Loosen the edges of the tape around the dressing starting from the outside and moving toward the center of the dressing

1 6 2 5 3 4

What developmental and environmental factors significantly influence the activity and exercise regimen to be followed by a patient? Select all that apply. 1 Age 2 Ethnic origin 3 Work culture 4 Marital status 5 Number of children

1 Age 2 Ethnic origin 3 Work culture A patient's age, cultural and ethnic origin, and work culture may have developmental and environmental effects on a patient's activity and exercise habits. The number of children a patient has and the patient's marital status are not developmenal and environmental factors that might influence a patient's exercise regimen.

The nurse plans to provide education to the parents of school-age children. Which option does the nurse include to emphasize the results of children being less physically active outside of school? 1 An increase in obesity 2 An increase in heart disease 3 Higher computer literacy 4 Improved school attendance and grades

1 An increase in obesity It is increasingly clear that many children today are less active, resulting in an increase in childhood obesity. Strategies for physical activity incorporated early into a child's daily routine may provide a foundation for a lifetime commitment to exercise and physical fitness. Research shows that exercise plays a role in secondary prevention or recurrence of heart disease for adult patients already diagnosed with coronary heart disease.

A patient is admitted to the hospital with osteoporosis and lower back pain. The patient loses balance when trying to stand and walk. The patient has a nursing diagnosis of body imbalance. What instructions does the nurse give the patient? Select all that apply. 1 Instruct the patient to widen the base of support by separating the feet. 2 Instruct the patient to bring the knees closer together to maintain a broad base. 3 Instruct the patient to lower the center of gravity closer to the base of support. 4 Instruct the patient to keep the center of gravity away from the base of support. 5 Instruct the patient to maintain a vertical line from the center of gravity through the base of support.

1 Instruct the patient to widen the base of support by separating the feet. 3 Instruct the patient to lower the center of gravity closer to the base of support. 5 Instruct the patient to maintain a vertical line from the center of gravity through the base of support. To maintain body balance, the patient must attain a posture that requires the least muscular work and places the least strain on muscles, ligaments, and bones. To do this, the patient must first separate the feet to a comfortable distance to widen the base of support. Then the patient must try to increase balance by bringing the center of gravity closer to the base of support. The body posture is adjusted such that the vertical line from the center of gravity falls through the base of support to attain body balance. The knees should not be kept closer, because this could decrease the width of the base of support and impair balance. Increasing the distance between the center of gravity and the base of support would also impair the balance of the patient. Knees should be kept wide. Keeping the center of gravity away from the base of support will result in a loss of balance while standing or walking.

The nurse is caring for a pregnant patient. The nurse understands that the body posture and alignment in a pregnant patient may change. Where does the center of gravity of a pregnant woman shift? 1 To the anterior 2 To the posterior 3 To the left lateral side 4 To the right lateral side

1 To the anterior In pregnant women, changes in body posture and alignment occur due to weight gain and the growing fetus. The center of gravity of the body shifts to the anterior. Therefore, a pregnant woman leans backward and may report back pain. The center of gravity does not shift to the posterior or to any lateral sides; this might cause abnormal curvature of the spine.

The registered nurse and a nursing student are discussing opioid pain management therapy and naloxone (Narcan). Which of the nursing student's statements indicate a need for further teaching? Select all that apply. 1 "The infusion rate of an intravenous push of naloxone should be 0.5 mL for 1 minute." 2 "0.4 mg of naloxone should be diluted with 15 mL saline." 3 "Opioid-naïve patients should be closely monitored for sedation." 4 "Administering naloxone faster than the recommended rate may cause severe pain." 5 "If an adult patient experiences respiratory depression, naloxone should be administered."

1, 2 Naloxone (Narcan) is used to reverse the effects of opioids, especially in cases of overdose. While administering naloxone, the intravenous (IV) push should be at a rate of 0.5 mL every 2 minutes, not for 1 minute, until the respiratory rate is greater than eight breaths/min. Generally, 0.4 mg of naloxone is diluted with 9 mL, not 15 mL, saline. The remaining statements are correct. Opioid-naïve patients should be closely monitored for sedation, which occurs before respiratory depression. If naloxone is administered too quickly, the patient may experience severe pain and other serious complications. If an adult patient who is on pain management therapy with opioid analgesics experiences respiratory depression, naloxone should be administered.

A patient is confused and has become violent after recovering from a seizure episode. How should the nurse reorient this patient and provide further care? Select all that apply. 1 Answer the patient's questions. 2 Explain what happened. 3 Leave the patient alone. 4 Offer coffee or tea. 5 Expect the patient to reorient as time passes.

1, 2 Once the patient gains consciousness, the patient may be disoriented. The nurse should reorient and reassure the patient by acknowledging what happened and answering any questions. Every attempt should be made to stay with the patient throughout recovery. In the recovery phase, the patient will be drowsy and should be allowed to sleep. The patient should not be offered coffee or tea, because they stimulate the central nervous system and may not allow the patient to sleep. Even though patients become reoriented on their own, the nurse should make an attempt to reorient the patient so that the patient gains a better understanding of the situation.

Which questions should the nurse ask a patient to determine environmental comfort in a patient's home? Select all that apply. 1 "Are you using space heaters?" 2 "Are you using an air conditioning system?" 3 "Are your fire extinguishers routinely serviced?" 4 "Do you keep spoiled vegetables in the refrigerator?" 5 "Do you check the expiration date of milk products before eating them?"

1, 2, 3 Assessment of the environmental comfort of a patient's home includes a review of the usage and maintenance of space heaters, air conditioning systems, and fire extinguishers. Assessment for the risk of food infection or poisoning includes assessing a patient's knowledge about food preparation and storage practices such as whether he or she keeps food in the refrigerator until it spoils and checks the expiration dates of food products.

The nurse is teaching a group of parents about accidental poisoning. Which are effects of lead poisoning in a child? Select all that apply. 1 Adverse effects on child's growth 2 Learning and behavioral problems 3 Brain and kidney damage 4 Gastrointestinal infection 5 Increased susceptibility to fractures

1, 2, 3 Lead may be found in soil and water systems. It may cause poisoning if inhaled or ingested. Lead poisoning may affect the child's growth and cause learning and behavioral problems. In extreme cases, it may lead to kidney and brain damage. Lead poisoning is not known to cause gastrointestinal infections, because it is not a pathogen. It is also not associated with increased susceptibility to fractures.

The nurse has to administer opioids to a female patient after a surgical procedure. Which conditions may require special consideration before administration of opioids? Select all that apply. 1 Breastfeeding 2 Dialysis 3 Respiratory disease 4 History of orthopedic surgery 5 Chronic headache

1, 2, 3 Special considerations such as a breastfeeding mother, a patient on dialysis, and any respiratory conditions should be assessed carefully before administering opioids. Opioids may pass into the breast milk. It is excreted through the kidneys, and a patient on dialysis may require adjustment of the dose. Opioids tend to depress the respiratory system. Therefore, a preexisting respiratory disease may become aggravated. A history of orthopedic surgery and chronic headaches do not affect opioid administration.

The nurse is teaching a patient the use of patient-controlled analgesia (PCA). Which interventions should the nurse perform? Select all that apply.

1, 2, 3 The nurse should teach the patient about PCA and evaluate the patient's understanding by asking the patient to repeat what the nurse has taught. The patient should control the administration of the medication based on the pain. The device is programmed to prevent overdose. The family members should not operate the PCA device for the patient because the dose depends on the patient's perception of pain. The patient should be taught the use of the device before the procedures in order to be ready to administer the analgesia after awakening from sedation.

Which interventions are followed by the nurse to protect older adults in acute care settings? Select all that apply. 1 Providing bed rails for safety 2 Keeping the environment free from clutter 3 Explaining and demonstrating how to use the call light 4 Responding quickly to call lights and bed/chair alarms 5 Placing the call device close to the patient before a nurse-patient interaction

1, 2, 3, 4 The nurse should provide bed rails to prevent falls in older adults, keep the environment free from clutter, explain and demonstrate the use of call lights, and respond quickly to call lights and bed/chair alarms. The call devices are given to patients at the end of the nurse-patient interaction.

The nurse is caring for a patient who sustained a femur fracture following a fall. Which common physical hazards can increase the risk of falling? Select all that apply. 1 Inadequate lighting 2 Barriers in the normal walking path 3 Sudden decrease in blood glucose levels 4 Lack of safety devices in homes 5 Sudden rise in blood pressure

1, 2, 4 Common physical hazards that increase the risk of falls are inadequate lighting, barriers in the normal walking path, and lack of safety devices in homes. Although a sudden decrease in blood glucose levels and a sudden increase in blood pressure may increase the risk of fall, these are not physical hazards.

Which patients are at high risk of hypothermia? Select all that apply. 1 Patients who have angina 2 Patients who are homeless 3 Patients who have kidney disorders 4 Patients who have taken a drug overdose 5 Patients who are exposed to carbon monoxide

1, 2, 4 Patients with cardiovascular disease such as angina, people who are homeless, and people who have ingested drugs or alcohol in excess are at high risk for hyperthermia. Patients with kidney disorders are not at elevated risk of hypothermia. Patients who are exposed to high concentrations of carbon monoxide may have a risk of death.

The nurse has used restraints for a disoriented patient. Which reasons would justify the use of restraints? Select all that apply. 1 To help reduce the risk of patient injury from falls 2 To prevent the patient from removing IV infusions 3 To help to control the patient 4 To help to reduce the risk of injury to others by the patient 5 To minimize the need for supervision of the patient

1, 2, 4 Restraints are a means to maintain patient safety. Nurses use restraints to protect patients who are confused, disoriented, repeatedly fall, or try to remove medical devices such as intravenous (IV) infusions or oxygen equipment. A disoriented patient can harm others and should be restrained. A restraint is not used to control the patient or to discontinue care.

The nurse finds that a patient has sustained seizures lasting longer than 5 minutes. Which strategies should be included in the care plan to prevent hypoxia in this patient? Select all that apply. 1 Suctioning the airway 2 Maintaining a patent airway 3 Placing in prone position 4 Providing oxygen via nasal cannula 5 Inserting an oral airway

1, 2, 4 Seizures beyond 5 minutes may deprive the patient of oxygen. Suctioning should be performed, bevcause it is needed to keep the airway patent. The airway and oxygenation should be maintained either via nasal cannula or a face mask. Placing the patient in prone position would obstruct the airway and thus should be avoided. An oral airway should be attempted only if access is easy and the breathing is impaired.

A patient sustained a cerebrovascular accident. The patient reports associated weakness on the left side of the body. On further assessment, the nurse learns that the patient has a visual disturbance and uncoordinated gait. How should the nurse ensure the patient's safety? Select all that apply. 1 Eliminate the risk of falls. 2 Consult with a physical therapist. 3 Encourage the use of coping skills. 4 Recommend an assessment of the eyes. 5 Establish a therapeutic relationship.

1, 2, 4 The patient is at a risk of falls due to visual impairment and uncoordinated gait. The nurse should suggest modification of the home environment to prevent risk of falls. A physical therapist should be consulted to help increase muscle strength, balance, and endurance. A visual assessment by an ophthalmologist is important for correction of any visual disturbances. Encouraging the use of coping skills and establishing a therapeutic relationship are helpful to the patient with anxiety-related disorders.

Question 18. A nurse is caring for a patient with pressure ulcers over the right and left ischial tuberosities. In what position should the nurse place the patient to relieve pressure in these areas? Select all that apply. 1. Sims' 2. Prone 3. Orthopneic 4. Right lateral 5. High-Fowler

1, 2, 4 (Rationale Option 1: An ischial tuberosity is a bony prominence located at the base of each ischium bone at the base of the pelvic girdle. No weight is placed on these boney prominences when in the Sims' position. Option 2: An ischial tuberosity is a bony prominence located at the base of each ischium bone at the base of the pelvic girdle. No weight is placed on these boney prominences when in the prone position. Option 3: The orthopneic position should be avoided. An ischial tuberosity is a bony prominence located at the base of each ischium bone at the base of the pelvic girdle. Body weight rests on these boney prominences when the body is in the sitting position. Option 4: An ischial tuberosity is a bony prominence located at the base of each ischium bone at the base of the pelvic girdle. No weight is placed on these boney prominences when in the right lateral position. Option 5: The high-Fowler position should be avoided. An ischial tuberosity is located at the base of each ischium bone at the base of the pelvic girdle. Body weight rests on these bones when the body is in the high-Fowler position.)

The nurse is explaining the Americans with Disabilities Act (ADA) to a patient with human immunodeficiency virus (HIV). Which information should the nurse include? Select all that apply. 1 People with HIV who are asymptomatic also come under the category of disabled people. 2 People with HIV have the right to decide whether to disclose their infection. 3 Healthcare workers have the choice to not treat patients who are HIV positive. 4 Healthcare professionals who are HIV positive can also choose to decide whether to disclose their infection. 5 The motive of ADA is to provide equal opportunities for people with disabilities

1, 2, 4, 5 According to the Americans with Disabilities Act (ADA), asymptomatic human immunodeficiency virus (HIV) is considered a disability. This act gives the HIV-infected individuals the opportunity to decide whether to disclose their disability. Healthcare providers may choose not to disclose the fact that they have HIV. This act aims at removing any discrimination and providing equal opportunities for people with disabilities. Healthcare workers cannot discriminate against HIV-positive patients.

A patient sustained minor burns in a fire at home. After stabilizing the patient, the nurse asks the patient to obtain a fire extinguisher at home. Which instructions should the nurse provide to the patient? Select all that apply. 1 It should be inaccessible to the children. 2 It should be placed on each level near an exit. 3 It should be kept at a low level in the room. 4 It should be placed in clear view. 5 It should be kept away from stoves and heating appliances.

1, 2, 4, 5 Fire extinguishers should be placed where they are inaccessible to children. Children may play with this equipment, which may lead to accidents. To enhance fire safety, a fire extinguisher should be placed on each level of the home and near the exit doors so that it is reachable when required. The extinguishers should be easily visible so that they can be accessed when required. They should be placed away from stoves and heating appliances to prevent explosion of the equipment. They should not be placed at a low level in a room. If one is placed at a low level, it would be accessible to children, which can lead to accidents

The nurse is calculating the fall risk score of an 85-year-old patient using a standard fall assessment tool. Which factors contribute to the patient's overall fall risk? Select all that apply. 1 Age 2 Fall history 3 Gender 4 Weight 5 Medications

1, 2, 5 A fall assessment tool must consider several aspects of a patient's condition and history in order to determine the patient's overall fall risk. The patient's age, fall history, and current medications are considered because older patients are more likely to fall, as are patients with a history of falling, or patients who are on medications that make them dizzy or groggy or that impair their motor skills. Gender and weight are not factors considered in standard fall assessment tools because patients of any gender or weight can be at risk for falling.

A patient has had arthritic pain for 8 years. Which questions should the nurse ask to assess the patient's pain? Select all that apply. 1 "Which factors relieve your pain?" 2 "How would you describe the pain?" 3 "Are you having any trouble passing stools?" 4 "Are you allergic to any food item or medication?" 5 "On a scale of 0 to10, how high would you rate the pain?"

1, 2, 5 To assess the pain completely and accurately, the nurse needs to assess its onset, palliative factors, quality, radiation, severity, and time factors related to the pain. Asking about palliative factors helps to determine the factors that influence the pain. A description of the pain helps to understand the nature and location of the pain. Asking a patient to rate the pain on a pain scale helps to assess the intensity of the pain. Asking questions regarding elimination and allergies does not help in pain assessment.

Question 5. A nurse is changing a dressing for a stage IV pressure ulcer that has an area of necrotic tissue on the edge of half the wound. The health-care provider's prescription states: Irrigate wound twice a day with normal saline, apply Collagenase, and pack wound with moist gauze. Identify the principles that apply when the nurse uses medical and surgical asepsis when changing the dressing. Select all that apply. 1. Use clean gloves to remove the old dressing. 2. Apply Collagenase over the entire surface of the wound bed. 3. Use the inside of the sterile glove package as the sterile field. 4. Fill the wound and cover ¼ inch of the skin surrounding the wound with wet gauze. 5. Position the irrigating solution on the side of the sterile field farthest from the patient.

1, 3

A nurse is performing Timed Get up and Go (TUG) for a patient. What is the correct order of steps in in which the assessment will be performed? 1. Give verbal instructions to stand up and walk 10 feet as quickly and safely as possible. 2. Begin counting. 3. Have the patient rise from a straight back chair without using arms for support. 4. Check time elapsed. 5. Look for unsteadiness in the patient's gait. 6. Have the patient return to the chair and sit down without using arms for support.

1, 3, 2, 5, 4, 6 The first step while performing TUG is giving verbal instructions to stand up and walk 10 feet as quickly and safely as possible. The next step is having the patient rise from a straight back chair without using arms for support. This is followed by beginning to count. The next step is looking for unsteadiness in the patient's gait. The nurse then has the patient return to the chair and sit down without using the arms for support. The last step is to check the time elapsed.

The nurse works in a nursing home. Of which are common causes of death in the elderly population that the nurse should be aware? Select all that apply. 1 Falls 2 Poisoning 3 Hypothermia 4 Heat stroke 5 Motor vehicle accident

1, 3, 4 Falls are a common cause of accidental death in elderly patients due to poor vision, gait and balance problems, and the effects of various medications. Hypothermia and heat stroke are also common causes of death in the elderly, becuse they are more vulnerable to temperature changes. Poisoning is more common in toddlers and small children, because they have a habit of ingesting chemicals such as cleaning solutions and medicines found in the house. Motor vehicle accidents are a more common cause of death in younger adults than in the elderly.

The nurse is planning effective pain management for a patient. What patient barriers prevent pain management? Select all that apply. 1 Lack of money 2 Fear of legal repercussions 3 Difficulty in filling prescriptions 4 Extensive documentation requirements 5 Belief that patients need to learn to live with pain

1, 3, 4 Lack of money prevents access to appropriate resources for pain medications. Difficulty in filling prescriptions can prevent the patient from using pain medications. A requirement of extensive documentation makes the process tedious, interfering with the prescriber's directions for effective pain management. The fear of legal repercussions and a belief that patients need to learn to live with pain are barriers erected by health care providers.

A patient is in the first postoperative day following a nephrectomy. The patient is receiving morphine through a patient-controlled analgesia (PCA) device for management of pain. The nurse decides to use the ABCDE approach while assessing and managing pain for this patient. What are the correct components of the ABCDE approach? Select all that apply.

1, 3, 4 The ABCDE approach helps in accurately assessing pain and its management. A stands for "Ask regularly about the pain." B stands for "Believe the patient and family in the report of pain." C stands for "Choose pain control options appropriate for the patient." D stands for "Deliver interventions in an orderly and coordinated fashion." E stands for "Empower patients and their families."

The nurse accompanies a patient for an x-ray. How does the nurse reduce exposure to radiation? Select all that apply. 1 Using lead aprons while entering the x-ray unit 2 Wearing a normal apron while entering the x-ray unit 3 Limiting the time spent near the source of the radiation 4 Maintaining the proper distance from the source of the radiation 5 Wearing a black apron while entering the x-ray unit

1, 3, 4 To reduce the risk of exposure, the nurse should wear a lead apron when entering radiation rooms, limit the time spent near the source of the radiation to limit the duration of radiation, and maintain a safe distance from the source to reduce the dose of radiation. Normal aprons and black aprons do not protect from radiation exposure and hence should not be worn when entering the radiation rooms.

The nurse is caring for a patient on pain management therapy. Which types of therapy cause a release of endorphins that can block the transmission of painful stimuli? Select all that apply. 1 Massage 2 Opioid analgesics 3 Cold application 4 Nonsteroidal antiinflammatory drugs (NSAIDs) 5 Transcutaneous electrical nerve stimulation (TENS)

1, 3, 5 Cutaneous stimulation releases endorphins, which block the transmission of painful stimuli. Massage, cold application, or transcutaneous electrical nerve stimulation (TENS) all stimulate the skin, which may be helpful in reducing pain perception via endorphin release. Opioid analgesics reduce pain by binding with opiate receptors to modify pain perception, not by releasing endorphins. Nonsteroidal antiinflammatory drugs (NSAIDs) reduce pain by inhibiting prostaglandin synthesis, which inhibits the body's cellular response to inflammation.

The nurse is teaching a group of caregivers about the concept of pain in older adults. What should the nurse include in the teachings? Select all that apply. 1 Older patients underreport pain. 2 Sleeping indicates pain relief. 3 Opioids are safe to use in older patients. 4 Older adults tend to perceive more pain. 5 Older adults with cognitive impairment do not experience less pain.

1, 3, 5 Older patients underreport pain with the fear of losing their independence, and do not want to alarm loved ones. Opioids are safe to use with proper monitoring of the patient to note any side effects. There is no evidence that cognitively impaired older adults experience less pain compared to individuals with intact cognitive function. Sleeping does not indicate pain relief. It indicates exhaustion, and in fact, pain may prevent the patient from having a good sleep. Age does not dull the sensitivity to pain. Older adults perceive pain as much as young adults.

The nurse is caring for a patient who is on opioid therapy. For which findings is the nurse carefully observing the patient? Select all that apply. 1 Decreased pulse rate 2 Increased respiratory rate 3 Decreased blood pressure 4 Pupil dilatation 5 Peripheral edema

1, 3, 5 Potential adverse effects of opioids include bradycardia (decreased pulse rate), hypotension (decreased blood pressure), and peripheral edema due to the accumulation of fluids. Decreased, not increased, respiratory rate may occur with opioid administration. Pupil constriction may occur with the use of opioids, but pupil dilatation is an effect of opioid withdrawal.

The nurse is learning about the effects of pain on the sympathetic system. What are the manifestations of sympathetic stimulation in response to the pain? Select all that apply. 1 Increased heart rate 2 Rapid, irregular breathing 3 Increased glucose level 4 Decreased blood pressure 5 Decreased gastrointestinal motility

1, 3, 5 The stimulation of the sympathetic branch of the autonomous nervous system causes an increased heart rate, an increased glucose level, and decreased gastrointestinal motility. Stimulation of the parasympathetic branch results in rapid, irregular breathing and decreased blood pressure.

A patient is in the first postoperative day following a nephrectomy. The patient is receiving morphine through a patient-controlled analgesia (PCA) device for management of pain. The patient is apprehensive about being given opioid drugs and is afraid of becoming addicted to the drug. The patient is also afraid of chronic side effects. What explanation should the nurse give the patient? Select all that apply. 1 Opioids can be used safely in cases of moderate to severe pain. 2 Opioids can be given only after surgery or for postsurgical pain. 3 Slow titration prevents potentially dangerous opioid-induced side effects. 4 The drug is administered carefully, because its action cannot be reversed. 5 In case of any adverse effects, opioid antagonist drugs can be given to reverse the effects.

1, 3, 5 There are many misconceptions about the use of opioid drugs. Opioids can be safely given to people for management of moderate to severe pain. Opioids are given in slow titration to prevent the appearance or development of any side effects. In rare cases, there may be respiratory depression as an adverse effect of opioid drugs. In such cases, an opioid antagonist drug can be administered to the patient to reverse the effects of opioids. It is not mandatory to give opioids only after surgery. They can be administered to relieve pain of any origin. The action of opioids can be reversed with the proper antagonist drug.

A patient has recovered from seizures in a postoperative ward. How should the nurse ensure continued safety of the patient? Select all that apply. 1 Raise the side rails of the bed. 2 Place the patient in supine position. 3 Keep the call light and intercom near the patient. 4 Avoid the use of pillows. 5 Place the bed in the lowest position.

1, 3, 5 To ensure continued safety of a patient following seizures, the side rails of the bed should be raised to prevent a fall. Placing the call light and intercom within reach of the patient would help in receiving assistance, if needed. The bed should be lowered to the lowest position to prevent a fall and risk of injury. The patient should be positioned lying on the side, not in the supine position. Lying on the side prevents the tongue from falling back and obstructing the airway. A pillow should be used to support the head once the seizure is over.

The nurse is caring for an older adult in the home. The nurse is concerned about the risk of injury. Following an assessment, the nurse finds that the patient has visual impairment. Which actions should the nurse perform to reduce the risk of injury for this patient? Select all that apply. 1 Keep the home well lit. 2 Keep the rooms ventilated. 3 Keep clean eyeglasses at hand. 4 Teach range-of-motion exercises. 5 Perform a home hazard assessment.

1, 3, 5 Keeping the home well-lit will help the patient see objects without trouble. Clean and protected eyeglasses at hand will help the patient see clearly. During the home hazard assessment, the nurse should walk through the home with the patient and discuss how the patient normally conducts daily activities and whether the environment poses problems. Keeping the rooms ventilated may not help the patient in reducing risk of injury. Range-of-motion exercises may be performed by patients with altered mobility.

The nurse is learning about negligence in unintentional torts. Which actions would the nurse consider as common acts of negligence? Select all that apply. 1 Failure to follow orders 2 Failure to perform malpractice 3 Failure to document monitoring 4 Failure to follow policies and guidelines 5 Failure to explain the risks of a surgery to a patient

1, 3, and 4 Failure to follow orders is an act of negligence because it is the duty of the nurse to follow all given orders. Documentation of monitoring is one of the best practices to prevent legal issues and is important to communicate with other healthcare team members. Policies and guidelines are created in accordance with laws and regulations, so they should be followed. Malpractice is professional negligence and should be avoided. Explaining the risks of a medical procedure to a patient is not the nurse's responsibility.

The nurse works in a postsurgical ward. Which statements by the nurse indicate common misconceptions about pain? Select all that apply. 1 Psychogenic pain is not real. 2 Chronic pain is not psychological. 3 Patients who cannot speak can feel pain. 4 Administering analgesics regularly leads to drug addiction. 5 Patients who abuse substances overreact to discomfort.

1, 4, 5 Psychogenic pain is real and requires intervention. Regular administration of analgesics does not lead to drug addiction. However, some analgesics such as morphine should not be overused. The patients who abuse substances do not overreact to discomfort; their discomfort may be real. Chronic pain is not psychological; it may be real and can have an impact on daily activities. Patients who do not speak can still feel pain and need intervention.

Which statements about opioid analgesics for pain management are correct? Select all that apply. 1 Opioid analgesics act on higher centers of the brain. 2 Use of opioid analgesics will increase libido in male patients. 3 Opioid analgesics are prescribed for relieving mild forms of pain. 4 The short-acting forms of opioids provide pain relief for approximately 4 hours. 5 Prolonged use of opioid analgesics will increase patient tolerance to depression of the central nervous system

1, 4 Opioid analgesics act on higher centers of the brain and spinal cord by binding with opiate receptors. The short-acting forms of opioid analgesics provide pain relief for approximately 4 hours. Opioid analgesics will decrease the testosterone levels in male patients, decreasing, not increasing, libido. Opioid analgesics are prescribed to relieve moderate to severe levels of pain; other drugs are more appropriate for mild pain. Prolonged use of opioid analgesics will increase patient tolerance to most opioid side effects except central nervous system depression.

Which questions if asked by the nurse are useful for evaluating a patient's perception of safety? Select all that apply. 1 "Are you still afraid of falling?" 2 "What factors led to your fall?" 3 "What questions do you have about your safety?" 4 "Do you feel safer as a result of the changes in home?" 5 "Do you need help locating community resources to help make your home safer?

1, 4 Questions such as "Are you still afraid of falling?" and "Do you feel safer as a result of the changes?" help the nurse understand the patient's perceptions related to safety. When patient outcomes are not met, the nurse should ask questions such as "What factors led to your fall?" "What questions do you have about your safety?" and "Do you need help locating community resources to help make your home safer?"

The human body has a mechanism to reduce pain perception by inhibitory neurotransmitters. What are the inhibitory neurotransmitters of pain in the brain? Select all that apply. 1 Serotonin 2 Histamine 3 Substance P 4 Norepinephrine 5 Gamma aminobutyric acid

1, 4, 5 During the process of pain modulation, endogenous opioids, serotonin, norepinephrine, and gamma aminobutyric acid (GABA) are some of the inhibitory neurotransmitters released to inhibit the pain impulse. This happens in the fourth and final phase of the nociceptive process. Histamine and substance P have no role in pain modulation. Histamine is released by mast cells and plays a major role in the inflammatory process. Substance P transmits pain impulses from the periphery to higher brain centers.

Which statements are associated with state nurse practice acts? (Select all that apply) 1. Nurse practice acts identify what a nurse can and cannot do 2. A purpose of state nurse practice acts is to regulate the practice of nursing 3. The federal government is the regulating agency responsible for state nurse practice acts. 4. Student nurses are not obligated to meet the same standards of care as are licensed nurses. 5. If a student nurse is involved in a disciplinary action by the state board of nursing, the student may be prohibited from taking the national Council Licensure Examination.

1,2,5

20. Which identifies accurate nursing documentation notations? Select all that apply. 1.The client slept through the night. 2.Abdominal wound dressing is dry and intact without drainage. 3.The client seemed angry when awakened for vital sign measurement. 4.The client appears to become anxious when it is time for respiratory treatments. 5.The client's left lower medial leg wound is 3 cm in length without redness, drainage, or edema.

1,2,5 Factual documentation contains descriptive, objective information about what the nurse sees, hears, feels, or smells. The use of inferences without supporting factual data is not acceptable because it can be misunderstood. The use of vague terms, such as seemed or appears is not acceptable because these words suggest that the nurse is stating an opinion.

Which actions meet obligations regarding the Health Insurance Portability and Accountability Ac (HIPPA) of 1996? Select all that apply 1. Allowing patients the right to review and copy their clinical record 2.Providing accès to a patients clinical record to all members of the nursing team. 3. Holding patient rounds in a hallway out of the hearing range of the patient being discussed 4. Explaining to a person inquiring about the condition of a patient that information about the patient is confidential 5. Giving medical information about an inconspicuous patient to a patient daughter who is the person indicated in the patient's proxy directive.

1,4,5

A patient is admitted to a home health-care program for palliative care. When completing the admission interview and nursing assessment the patient empathically states, "I want no intervention to extend my life other than keeping me comfortable." When talking with family members they want the patient to continue chemo. What should the nurse do? 1. Protect the patients right to self- determination 2. initiate a referral to psychiatric nurse practitioner. 3. Ask the patient to reconsider chemotherapy for the sake of the family. 4. Refer family members to the primary health-care provider to resolve the issue.

1.

15. The nurse has just assisted a client back to bed after a fall. The nurse and health care provider have assessed the client and have determined that the client is not injured. After completing the incident report, the nurse should implement which action next? 1.Reassess the client. 2.Conduct a staff meeting to describe the fall. 3.Document in the nurse's notes that an incident report was completed. 4.Contact the nursing supervisor to update information regarding the fall.

1. After a client's fall, the nurse must frequently reassess the client because potential complications do not always appear immediately after the fall. The client's fall should be treated as private information and shared on a "need to know" basis. Communication regarding the event should involve only the individuals participating in the client's care. An incident report is a problem-solving document; however, its completion is not documented in the nurse's notes. If the nursing supervisor has been made aware of the incident, the supervisor will contact the nurse if status update is necessary.

24. The nurse calls the heath care provider (HCP) regarding a new medication prescription because the dosage prescribed is higher than the recommended dosage. The nurse is unable to locate the HCP, and the medication is due to be administered. Which action should the nurse take? 1.Contact the nursing supervisor. 2.Administer the dose prescribed. 3.Hold the medication until the HCP can be contacted. 4.Administer the recommended dose until the HCP can be located.

1. If the HCP writes a prescription that requires clarification, the nurse's responsibility is to contact the HCP. If there is no resolution regarding the prescription because the HCP cannot be located or because the prescription remains as it was written after talking with the HCP, the nurse should contact the nurse manager or nursing supervisor for further clarification as to what the next step should be. Under no circumstances should the nurse proceed to carry out the prescription until obtaining clarification.

A patient is being admitted to the hospital and the nurse is performing a complete assessment. Which is the most therapeutic question the nurse can ask about the quality of the patient's sleep? 1. "How would you describe your sleep?" 2. "Do you consider your sleep to be restless or restful?" 3. "Is the number of hours you sleep at night good for you?" 4. "Does your bed partner complain about your sleep behaviors?"

1. "How would you describe your sleep?" This open-ended question requires patients to explore the topic of sleep as it relates specifically to their own experiences.

Which statement made by the parent of a school-age child requires follow-up by a nurse? 1. "I encourage evening exercise about an hour before bedtime." 2. "I offer my daughter a glass of warm milk before bedtime." 3. "I will make sure that the room is dark and quiet at bedtime." 4. "We use quiet activities such as reading a book before bedtime."

1. "I encourage evening exercise about an hour before bedtime."

Which patient statements indicate that further teaching by the nurse is necessary regarding how to ensure protection from food contamination? Select all that apply. 1. "I should stuff a turkey an hour before putting it in the oven." 2. "I love juicy rare hamburgers with onion and tomato." 3. "I prefer chicken salad sandwiches with mayonnaise." 4. "I know to spit out food that does not taste good." 5. "I should defrost frozen food in the refrigerator."

1. "I should stuff a turkey an hour before putting it in the oven." 2. "I love juicy rare hamburgers with onion and tomato."

While planning morning care, which of the following patients would have the highest priority to receive his or her bath first?

1. A patient who just returned to the nursing unit from a diagnostic test. 2. A patient who prefers a bath in the evening when his wife visits and can help him. 3. A patient who is experiencing frequent incontinent diarrheal stools and urine. 4. A patient who has been awake all night because of pain 8/10. 3

Arrange the steps of a nurse following guidelines to initiate an exercise program in a patient in sequential order. 1. Monitor progress 2. Assess fitness level 3. Design the fitness program 4. Assemble equipment 5. Get started using the exercise routine

1. Assess fitness level 2. Design the fitness program 3. Assemble equipment 4. Get started using the exercise routine 5. Monitor progress To initiate an exercise program, the nurse first needs to assess the patient's fitness level by seeking approval from the health care provider. The fitness program is then designed, taking fitness goals into consideration. All suitable equipment is then assembled and an exercise routine is started gradually. The progress is then monitored at 6 weeks and then every 3 to 6 months.

A patient with a body mass index (BMI) of 36 has a sedentary job. She has been advised to reduce her weight through exercise. To begin an exercise program, she needs to go through five steps. Arrange the steps in the appropriate order. 1. Assemble equipment. 2. Get started. 3. Design the fitness program. 4. Monitor progress. 5. Assess fitness level.

1. Assess fitness level. 2. Design the fitness program. 3. Assemble equipment. 4. Get started. 5. Monitor progress A health care provider assesses the patient's fitness level, which is used as a basis for the fitness program; then the fitness program needs to be designed. The exercise equipment should be assembled accordingly. Next, the patient should begin the program. Progress is monitored regularly to determine the effect of the exercise. Fitness is assessed at 6 weeks and then every 3 to 6 months.

A nurse is helping a patient who is experiencing mild pain to get ready for bed. Which nursing action is most effective to help limit pain? 1. Assisting with relaxing imagery 2. Obtaining a prescription for an opioid 3. Encouraging the patient to take a warm shower 4. Recommending that the patient be more active during the day

1. Assisting with relaxing imagery Imagery, the internal experience of memories, dreams, fantasies, or visions, uses positive images to distract, which reduces stress, limits mild pain, and promotes relaxation and sleep.

The nurse calls the health care provider (HCP) regarding a new medication prescription because the dosage prescribed is higher than the recommended dosage. The nurse is unable to locate the HCP, and the medication is due to be administered. Which action should the nurse take? 1. Contact the nursing supervisor. 2. Administer the dose prescribed. 3. Hold the medication until the HCP can be contacted. 4.Administer the recommended dose until the HCP can be located.

1. Contact the nursing supervisor. Rationale: If the HCP writes a prescription that requires clarification, the nurse's responsibility is to contact the HCP. If there is no resolution regarding the prescription because the HCP cannot be located or because the prescription remains as written after talking with the HCP, the nurse should contact the nurse manager or nursing supervisor for further clarification as to what the next step should be. Under no circumstances should the nurse proceed to carry out the prescription until obtaining clarification. Test-Taking Strategy: Eliminate options 2 and 4 first because they are COMPARABLE OR ALIKE and are unsafe actions. Holding the medication can result in client injury. The nurse needs to take action. The correct action clearly identifies the required action in this situation.

A patient is having difficulty sleeping and may be experiencing shortened non-rapid- eye movement (NREM) sleep. Which patient assessments support this conclusion? Select all that apply. 1. Decreased pain tolerance 2. Inability to concentrate 3. Excessive sleepiness 4. Irritability 5. Confusion

1. Decreased pain tolerance An increased sensitivity to pain is associated with disturbed non-rapid- eye-movement (NREM) sleep. During NREM sleep the body is engaged in restoring physiological properties of the body. 3. Excessive sleepiness During NREM sleep the parasympathetic nervous system dominates and the vital signs and metabolic rate are low; also, growth hormone is consistently secreted, which provides for anabolism. Shortened NREM sleep decreases these restorative processes, resulting in fatigue, lethargy, and excessive sleepiness.

Which sleep-hygiene actions at bedtime can the nurse delegate to the nursing assistant? (Select all that apply.) 1. Giving the patient a backrub 2. Turning on quiet music 3. Dimming the lights in the patients room 4. Giving the patient a cup of coffee 5. Monitoring for the effect of the sleeping medication that was given

1. Giving the patient a backrub 2. Turning on quiet music 3. Dimming the lights in the patients room

A nurse is assessing a patient experiencing chronic pain. Which characteristics are more common with chronic pain than with acute pain? Select all that apply. 1. Gradual onset 2. Long duration 3. Anticipated end 4. Psychologically depleting 5. Responds to conventional interventions

1. Gradual onset Chronic pain has a gradual progressive onset because it usually is related to a long-term problem (e.g., diabetic neuropathy). Acute pain has a rapid onset because it usually is related to abrupt trauma to the body (e.g., surgical incision, damage from an automobile collision). 2. Long duration Chronic pain is categorized as pain longer than 6 months' duration. Acute pain is categorized as pain shorter than 6 months' duration. 4. Psychologically depleting Chronic pain is psychologically depleting because it drains both physical and emotional resources; this is related to the unrelenting nature of the pain and that it usually continues for life.

Which is the earliest nursing assessment that indicates permanent damage to tissues because of compression of soft tissue between a bony prominence and a mattress? 1. Nonblanchable erythema 2. Circumoral cyanosis 3. Tissue necrosis 4. Skin abrasion

1. Nonblanchable erythema refers to redness of intact skin that persists when finger pressure is applied. This is the classic sign of a stage I pressure ulcer.

The nurse observes an adult Middle Eastern patient attempting to bathe himself with only his left hand. The nurse recognizes that this behavior likely relates to:

1. Obsessive compulsive behavior. 2. Personal preferences. 3. The patient's cultural norm. 4. Controlling behaviors. 3

A patient is experiencing anxiety. Which aspect of sleep should the nurse expect primarily will be affected as a result of the anxiety? 1. Onset 2. Depth 3. Stage II 4. Duration

1. Onset Anxiety increases norepinephrine blood levels through stimulation of the sympathetic nervous system, which results in prolonged sleep onset.

Which nursing actions protect patients from infection are the portal of entry portion of the chain of infection? Select all that apply. 1. Positioning an indwelling urine collection bag below the level of the patient's pelvis 2. Using sterile technique when administering an intramuscular injection 3. Enclosing a urine specimen in a bio-hazardous transport bag. 4. Wearing clean gloves when handling a patient's excretions 5. Washing the hands after removal of soiled gloves 6. Maintaining a dressing over a surgical incision

1. Positioning an indwelling urine collection bag below the level of the patient pelvis. 2. Using sterile technique when administering an intramuscular injection. 6. Maintaining a dressing over a surgical incision.

What is the proper position to use for an unresponsive patient during oral care to prevent aspiration? (Select all that apply).

1. Prone position 2. Sims' position 3. Semi-Fowler's position with head to side 4. Trendelenburg position 5. Supine position 2 and 3

The nurse has just assisted a client back to bed after a fall. The nurse and health care provider have assessed the client and have determined that the client is not injured. After completing the incident report, the nurse should implement which action NEXT? 1. Reassess the client. 2. Conduct a staff meeting to describe the fall. 3. Document in the nurse's notes that an incident report was completed. 4. Contact the nursing supervisor to update information regarding the fall.

1. Reassess the client. Rationale: After a client's fall, the nurse must frequently reassess the client because potential complications do not always appear immediately after the fall. The client's fall should be treated as private information and shared on a "need to know" basis. Communication regarding the event should involve only the individuals participating in the client's care. An incident report is a problem-solving document; however, its completion is not documented in the nurse's notes. If the nursing supervisor has been made aware of the incident, the supervisor will contact the nurse if status update is necessary. Test-Taking Strategy: Focus on the data in the question and the strategic word NEXT. Using the STEPS OF THE NURSING PROCESS will direct you to the correct option. Remember that assessment is the first step.

A nurse must collect the following specimens. Which specimens do not require the use of surgical aseptic technique? Select all that apply. 1. Stool for occult blood 2. Stool for ova and parasites 3. Oropharyngeal mucous for a culture 4. Urine from a retention catheter for a urinalysis 5. Exudate from a wound for culture and sensitivity

1. Stool for occult blood 2. Stool for ova and parasites

The nurse is developing a plan for a patient who was diagnosed with narcolepsy. Which interventions should the nurse include on the plan? (Select all that apply.) 1. Take brief, 20-minute naps no more than twice a day. 2. Drink a glass of wine with dinner. 3. Eat the large meal at lunch rather than dinner. 4. Establish a regular exercise program. 5. Teach the patient about the side effects of modafinil (Provigil).

1. Take brief, 20-minute naps no more than twice a day. 4. Establish a regular exercise program. 5. Teach the patient about the side effects of modafinil (Provigil).

Which primary defenses protect the body from infection? Select all that apply. 1. Tears in the eyes 2. Healthy intact skin 3. Cilia of respiratory passages 4. Alkalinity of gastric secretions 5. Bile in the gastrointestinal system 6. Moist environment of the epidermis

1. Tears in the eyes 2. Healthy intact skin 3. Cilia of respiratory passages

A nurse is listening to a student provide instruction to a patient who is having difficulty with activities needed to care for soft contact lenses. Which of the following statements by the nursing student might require some correction by the nurse?

1. Use tap water to clean soft lenses. 2. Follow recommendations of lens manufacturer when inserting the lenses. 3. Keep lenses moist or wet when not worn. 4. Use fresh solution daily when storing and disinfecting lenses. 1

The nurse is attending to an older adult patient who has sustained a fall and has broken a femur. The nurse explains to the patient that as the body ages, the bones become weak due to osteoporosis and become more prone to fracture. Which statements are true about osteoporosis? Select all that apply. 1. The cause may be hormonal imbalances or insufficient intake of nutrients. 2 There is a structural curvature of the spine associated with vertebral rotation. 3. Osteoporosis is a disorder of aging and results in the reduction of bone density or mass. 4 There is inadequate and delayed mineralization, resulting in compact and spongy bone. 5 The bone remains biochemically normal but has difficulty maintaining integrity and support.

1. The cause may be hormonal imbalances or insufficient intake of nutrients. 3. Osteoporosis is a disorder of aging and results in the reduction of bone density or mass. 5 The bone remains biochemically normal but has difficulty maintaining integrity and support. In osteoporosis, the bones remain biochemically normal but have a reduction in density or mass. The cause of osteoporosis is uncertain, and theories vary from hormonal imbalances to insufficient intake of nutrients. Osteoporosis is common in aging adults. Osteomalacia, not osteoporosis, is an uncommon metabolic disease characterized by inadequate and delayed mineralization, resulting in compact and spongy bones. Scoliosis is a structural curvature of the spine associated with vertebral rotation.

Which identifies accurate nursing documentation notations? SELECT ALL THAT APPLY. 1. The client slept through the night. 2. Abdominal wound dressing is dry and intact without drainage. 3. The client seemed angry when awakened for vital sign measurement. 4. The client appears to become anxious when it is time for respiratory treatments. 5. The client's left lower medial leg wound is 3 cm in length without redness, drainage, or edema.

1. The client slept through the night. 2. Abdominal wound dressing is dry and intact without drainage. 5. The client's left lower medial leg wound is 3 cm in length without redness, drainage, or edema. Rationale: Factual documentation contains descriptive, objective information about what the nurse sees, hears, feels, or smells. The use of inferences without supporting factual data is not acceptable because it can be misunderstood. The use of vague terms, such as SEEMED or APPEARS is not acceptable because these words suggest that the nurse is stating an opinion. Test-Taking Strategy: Focus on the SUBJECT, accurate documentation notations. Eliminate options 3 and 4 because they are COMPARABLE OR ALIKE and include vague terms (seemed, appears).

Which concept should the nurse consider when assessing a patient's pain? 1. The expression of pain is not always congruent with the pain experienced. 2. Pain medication can significantly increase a patient's pain tolerance. 3. The majority of cultures value the concept of suffering in silence. 4. Most people experience approximately the same pain tolerance.

1. The expression of pain is not always congruent with the pain experienced. An obvious response to pain is not always apparent because psychosociocultural factors may dictate behavior. Fear of the treatment for pain, lack of validation, acceptance of pain as punishment for previous behavior, and the need to be strong, courageous, or uncomplaining are factors that influence behavioral responses to pain.

Integrity of the oral mucosa depends on salivary secretion. Which of the following factors impairs salivary secretion? (Select all that apply).

1. Use of cough drops 2. Immunosuppression 3. Radiation therapy 4. Dehydration 5. Presence of oral airway 3 and 4

Question 3. A patient has a small infected pressure (decubitus) ulcer and is on contact precautions. What should the nurse wear in addition to gloves when providing a bed bath? 1. Face shield 2. Clean gown 3. Respiratory mask 4. Protective eyeglasses

2

A nurse identifies that a patient's pressure ulcer has just partial-thickness skin loss involving the epidermis and dermis. What stage pressure ulcer should the nurse document based on this assessment? 1. Stage I 2. Stage II 3. Stage III 4. Stage IV

2

A nurse is caring for a patient with impaired mobility. Which position contributes most to the formation of a hip flexion contracture? 1. Low-Fowler 2. Orthopneic 3. Supine 4. Sims'

2

A nurse is caring for a variety of patients, each experiencing one of the following problems. Which health problem places a patient at the greatest risk for complications associated with immobility? 1. Incontinence 2. Quadriplegia 3. Hemiparesis 4. Confusion

2

A nurse is implementing care based on moral and ethical principles. What should the nurse do FIRST to accomplish a personal professional code of ethics consistent with American Nurses Association Code of Ethics for Nurses? 1.Deliever care that preserves and protects patient autonomy and self-determination 2.Explore personal value and beliefs because they affect nursing decisions 3. Implementing culturally competent and sensitive nursing care to patients 4. respect the inherent worth and uniqueness of each individual patient

2

A nurse is making an occupied bed. What should the nurse do to prevent plantar flexion? 1. Tuck in the top linens on just the sides of the bed 2. Place a toe pleat in the top linens over the feet 3. Let the top linens hang off the end of the bed 4. Use trochanter rolls to position the feet

2

A patient has a cast from the hand to above the elbow because of a fractured ulna and radius. After the cast is removed, the nurse teaches the patient active range-of-motion exercises. Which patient action indicates that further teaching is necessary? 1. Moves the elbow to the point of resistance 2. Keeps the elbow flexed after the procedure 3. Assesses the elbow's response after the procedure 4. Puts the elbow through its full range at least three times

2

An emaciated patient is at risk for developing a pressure ulcer. In which position should the nurse avoid placing the patient? 1. Low-Fowler 2. Side-lying 3. Supine 4. Prone

2

How long before bedtime should a patient take melatonin? Record your answer using a whole number. _______________________ hour(s)

2

Question 1. A visiting nurse is caring for a bed-bound patient with a pressure ulcer. Which nursing intervention associated with providing pressure ulcer care in the home is often different from providing pressure ulcer care in the acute-care setting? 1. Measuring a wound weekly versus daily 2. Employing medical asepsis versus sterile technique 3. Changing dressings every day versus three times a day 4. Using a bulb syringe versus a piston syringe when irrigating

2

Question 13. A nurse is monitoring the results of a culture and sensitivity report from a sample from a wound. What does the sensitivity part of the report indicate? 1. All of the microorganisms present 2. Antibiotics that should be effective 3. Virulence of the organisms in the exudate 4. Extent of the patient's response to the pathogens

2

Question 7. A nurse responds to a call light of a patient who had a colon resection 2 days ago for the removal of a cancerous tumor. The patient states, "It feels very wet and funny under my dressing." Which additional information collected by the nurse supports the conclusion that the patient may be experiencing dehiscence? 1. The patient reported throbbing pain at the site of the incision. 2. The patient verbalized, "I just felt a popping sensation at my incision." 3. The patient informed the nurse of voiding excessively during the night. 4. The patient stated, "I feel bloated and will be much better if I have a bowel movement."

2

The nurse notes that an advance directive is on a patient's medical record. Which statement best describes an advance directive guideline? 1 A living will allows an appointed person to make healthcare decisions when the patient is in an incapacitated state. 2 A living will is invoked only when the patient has a terminal condition or is in a persistent vegetative state. 3 The patient cannot make changes in the advance directive once admitted to the hospital. 4 A durable power of attorney for health care is invoked only when the patient has a terminal condition or is in a persistent vegetative state.

2 A living will does not assign another individual to make decisions for the patient. A durable power of attorney for healthcare is active when the patient is incapacitated or cognitively impaired. A cognitively intact patient may change an advance directive at any time.

32. A nurse is teaching a group of parents of school-aged children about motor vehicle accidents and child safety. Which information is most important for the nurse to include in this program? 1. Front passenger seats can be occupied by children when they weigh more than 110 lb. 2. Ensure that safety seats are appropriate for the height and weight of the child. 3. Children 14 years of age should ride in the back seat of a vehicle. 4. Children 4 to 10 years of age should use a booster seat. Hint

2 Age, height, and weight appropriate automobile safety seats are essential to protect infants and children when in a motor vehicle accident.

18. A primary health-care provider orders warm compresses to be applied to the site of an intravenous catheter that had become red and inflamed. What should the nurse explain to the patient is the desired outcome of this therapy? 1. "The area will feel less tense, which will decrease the risk of bleeding." 2. "Circulation to the area will increase, which will promote healing." 3. "Circulation to the area will decrease, which will limit edema." 4. "The area will feel numb, which will decrease discomfort." Hint

2 Heat causes vasodilation, which increases circulation and capillary permeability in the area; this brings oxygen, nutrients, and WBCs to the area and removes toxic wastes.

The surgical nurse is collecting the necessary documents to obtain a patient's informed consent before a surgery. In which instance should the nurse refrain from obtaining informed consent? 1 The patient voices no concerns regarding the surgery. 2 The patient was administered morphine. 3 The patient's family members are not present. 4 The surgery is associated with risk for severe complications.

2 Informed consent is usually obtained when the patient is alert, is able to understand and process information, and can make decisions. Morphine may cause drowsiness, preventing the patient from making informed decisions. Even if the patient voices no concerns or has no questions, the consent form still needs to be presented and explained to the patient. Presence of family members is not necessary for giving informed consent unless the patient is unable to do so for any reason. Every surgery is associated with risks, which are explained in the associated documentation, so this is not a valid reason for not obtaining an informed consent.

22. A nurse is ambulating a patient in the hall. During the walk, the patient states, "I am dizzy and I feel very weak." What action by the nurse is most appropriate at this time? 1. Return the patient to bed quickly. 2. Lower the patient to the floor gently. 3. Walk toward the patient's room slowly. 4. Call another nurse to assist with the patient. Hint

2 Lowering the patient to the floor gently allows the nurse to prevent the patient's head from hitting the floor. It controls the patient's movement toward the floor, preventing injury to both the patient and nurse.

19. A patient is confused and trying to pull out an indwelling urinary catheter. The nurse tried various interventions to protect the patient from pulling out the catheter to no avail. The nurse notifies the primary health-care provider and asks for an order for a restraint. What type of restraint should the nurse anticipate will be ordered? 1. Belt 2. Mitt 3. Vest 4. Wrist

2 Mitt restraints are the least restrictive of the restraints presented in the options and are most appropriate for this patient. A mitt restraint permits the patient to move the upper extremities but will not permit the patient to grasp a tube.

11. A primary health-care provider orders mitt restraints for a patient. Which should the nurse determine is the therapeutic purpose of mitt restraints? 1) Keep the patient from falling out of bed 2) Prevent the patient from pulling out a feeding tube 3) Maintain the patient's upper extremities in functional alignment 4) Remind the patient to call for assistance when getting out of bed

2 Mitt restraints cover the hands, which prevent the fingers from grasping and pulling out catheters.

21. A nurse is completing a patient assessment for the purpose of determining factors that place the patient at risk for falls. Which factor should cause the most concern when completing this assessment? 1. Uses a walker 2. Has a history of falls 3. Takes a diuretic twice a day 4. Has a urinary retention catheter Hint

2 Most assessment tools indicate that a history of a recent fall is a significant factor regarding the risk for a future fall. A tool such as the Morse Fall Scale identifies 6 factors to examine: history of falls, secondary diagnosis, use of an ambulatory aid, intravenous infusion, gait and transfer ability, and mental status. Two factors on this scale (history of a fall within the last 3 months and uses furniture as an ambulatory aid) independently identify a patient as a level 2 risk potential out of 3 levels of concern (1-preventing, 2-modified risk, 3-strict fall risk).

The patient rates his pain as a 6 on a scale of 0 to 10, with 0 being no pain and 10 being the worst pain. The patient's wife says that he can't be in that much pain, because he has been sleeping for 30 minutes. Which is the most accurate resource for assessing the pain? 1 The patient's wife is the best resource for determining the level of pain, because she has been with him continually for the entire day. 2 The patient's report of pain is the best method for assessing the pain. 3 The patient's health care provider has the best knowledge of the level of pain that the patient that should be experiencing. 4 The nurse is the most experienced at assessing pain.

2 A patient's self-report of pain is the single most reliable indicator of the existence and intensity of pain.

A student nurse who has not been trained to administer medications is assisting a registered nurse while caring for various patients in the general ward. Which intervention by the student nurse may result in a malpractice lawsuit? 1 Collecting the vital signs of a patient 2 Administering a subcutaneous injection 3 Checking the body mass index of a patient 4 Assisting the registered nurse with an enteral nutrition feeding

2 Student nurses should not perform tasks if they are unprepared because their actions can cause harm to patients. Administering a subcutaneous injection without proper preparation may lead to patient harm and expose the student nurse to a malpractice orientation phase. The student nurse can collect vital signs, because this does not cause harm to the patient and is within the scope of practice for a student nurse. Checking the patient's body mass index is within the scope of practice for a student nurse. The student nurse can assist the registered nurse while providing enteral nutrition. Because this is done under the supervision of a licensed professional, the nurse will not face a malpractice lawsuit.

31. A nurse is teaching new parents about care of their newborn. What should the nurse teach them to do to prevent the leading cause of accidental death in infants? 1. Give your infant toys that are bigger than an adult's clenched fist. 2. Place your infant in a four-point infant seat when in an automobile. 3. Support your infant when placed on an elevated surface. 4. Position your infant on the back when sleeping. Hint

2 The leading cause of death in infants is motor vehicle accidents. A four-point infant car seat provides the necessary support to help limit injuries during a motor vehicle accident. The seat should be placed in the back seat of the car with the infant facing the rear of the car.

43. A nurse has an order from a primary health-care provider to apply wrist restraints to maintain a patient's safety. What should the nurse do first when applying this type of restraint? 1. Remove the wrist restraints to provide skin care every 3 hours. 2. Ensure that the wrists are well padded when applying wrist restraints. 3. Permit 3 finger-widths to slide between the patient's wrists and the restraints. 4. Tie the tales of the wrist restraints to the frame of the bed using a square knot. Hint

2 This helps to prevent injury to the wrists and is the first thing that the nurse should do of the options presented.

14. A nurse is caring for a patient who is blind. What should the nurse do to facilitate patient safety? 1. Encourage the patient to have a bed bath rather than a shower. 2. Walk about a foot ahead and have the patient grasp your arm. 3. Provide a small light in the room and bathroom at night. 4. Provide cold rather than hot drinks.

2 This is the proper way to assist a blind patient with ambulation. By walking slightly ahead and with the patient holding the nurse's arm, the nurse is able to guide the patient around obstacles.

A surgeon obtains patient consent for an appendectomy. While operating, the surgeon also removes the patient's gall bladder. This mistake could be classified as which type of tort? 1 Assault 2 Battery 3 False imprisonment 4 Defamation of character

2 Torts are intentional or unintentional mistakes that are infringements of civil law. Removing the gall bladder without consent may be considered battery because the surgeon's actions go beyond the scope of the patient's original consent, which was only for the appendectomy. Assault involves the intentional threat of bodily harm, but there is no indication the surgeon meant to harm this patient by removing the gall bladder. Forcing the patient to stay in the hospital even if the patient is unwilling would indicate false imprisonment. Making false, damaging statements about a patient would indicate defamation of character.

A patient is experiencing left-sided weakness due to a cerebrovascular accident. The nurse observes that the patient fails to notice people approaching from the left side. Which nursing intervention will be most beneficial for the patient in this situation? 1 Establishing a therapeutic relationship with the patient 2 Reminding the patient to scan the entire environment when walking 3 Consulting with a physical therapist to help the patient with strengthening exercises 4 Encouraging the family to eat with the patient, and reminding the patient to eat food on left side of the plate

2 A patient who fails to notice people approaching from the left side may have unilateral neglect related to brain injury. The appropriate intervention in this case is to remind the patient to scan the entire environment when walking. Establishing a therapeutic relationship with the patient is helpful if the patient exhibits signs of anxiety related to falls or health status. Consulting with a physical therapist to help the patient with strengthening exercises will be beneficial for a patient who is at risk of falls. Encouraging the family to eat with the patient and reminding the patient to eat food on the left side of the plate is beneficial for patients who have unilateral neglect related to brain injury.

The primary health care provider (PHP) administers epidural anesthesia to a patient with chronic cancer pain. The PHP instructs the nurse to monitor the patient every 15 minutes. Which intervention does the nurse implement to prevent complications? 1 Inspect the catheter for breaks. 2 Administer antiemetics as ordered. 3 Change the infusion tubing every 24 hours. 4 Assess for bladder and bowel distention.

2 A patient with chronic cancer pain is usually administered epidural anesthesia for pain management. This patient has to be monitored for side effects every 15 minutes. Nausea and vomiting are common side effects associated with epidural anesthesia. To prevent such undesirable complications, the nurse administers antiemetics as ordered. To maintain catheter function, the nurse inspects the catheter for breaks. The nurse changes the infusion tubing every 24 hours to prevent infection. To maintain urinary and bowel function, the nurse assesses for bladder and bowel distension.

A postoperative patient reports pain at the site of surgery. On examination, the nurse finds that the incision is healing well and there are no signs of infection. The nurse instructs a student nurse to give a placebo drug to the patient. Which action would be the most appropriate action for the student nurse to take? 1 Follow the instructions given by the nurse. 2 Question the action of the placebo prescription. 3 Administer another analgesic drug. 4 Administer the placebo and inform the primary health care provider immediately.

2 A placebo for pain does not have any analgesic properties. If a placebo is ordered, it must be questioned. The student should not blindly follow the instructions without knowing the purpose of administering the placebo. The student cannot directly administer another analgesic drug without an appropriate order to do so.

A nurse is caring for a patient who has an order for a vest restraint. Which action should the nurse implement? 1. Provide skin care every 3 hours. 2. Tie the restraint to the bed frame. 3. Check the restraint every 2 hours. 4. Release the restraint every 4 hours. Hint

2 A restraint should be tied to the bed frame. If tied to the bed rail the patient may be injured or the restraint may become too tight and impair respirations when the rail is lowered. If the rail is raised, the restraint will become too loose.

A patient is brought to the emergency room in an unconscious state. On taking the history the nurse finds that the patient had accidentally consumed agricultural pesticides. Which intervention will prevent the potential of aspiration in this patient? 1 Avoiding the use of nasogastric tubes in the patient 2 Positioning the patient with head turned 3 Positioning the patient with head straight 4 Making arrangements for suctioning of the secretions

2 An unconscious patient is susceptible to aspirating the stomach contents into the bronchial tree. Positioning the patient with head turned prevents aspiration. Nasogastric tubes also prevent aspiration of secretions, because they do not allow the stomach contents to aspirate into the bronchial tree. If the patient's head is held straight, there is a higher chance of aspiration. Suctioning has no preventive role in aspiration, but it helps to remove active secretions of the lungs.

During a preoperative assessment, a patient reports history of a heart attack and use of anticoagulant medications. If epidural anesthesia is administered to the patient for the surgery, for which possible complication should the nurse look? 1 Synergistic effects 2 Hematoma 3 Allergic reaction 4 Respiratory depression

2 Because anticoagulants reduce the action of the blood's platelets, hematoma is a possible complication when epidural anesthesia is administered to a patient on anticoagulants. Anticoagulants and anesthesia have different actions, so a synergistic effect is not a concern. Anticoagulants do not predispose a patient to an allergic reaction to epidural anesthesia. Respiratory depression is an adverse effect of opioids, but anticoagulants do not make it more likely.

51. A nurse observes a nursing assistant lift a heavy object from the floor. Which action indicates that the nursing assistant is using inappropriate body mechanics? 1. Keeping the back as straight as possible 2. Holding the object away from the body 3. Placing the feet wide apart 4. Bending the knees Hint

2 Holding the object away from the body is improper body mechanics. It does not keep the weight close to the body's center of gravity, which minimizes stress on the back's vertebrae, muscles, and tendons.

Which nursing activity is performed during the assessment of a patient? 1 Selecting nursing interventions to promote safety 2 Identifying patient perceptions safety needs and risks 3 Consulting with occupational and physical therapists for assistive devices 4 Selecting interventions that will improve the safety of the patient's home environment

2 Identifying the patient's perceptions of safety needs and risks is involved in the critical thinking model for safety assessment. The critical thinking model for safety planning involves selecting nursing interventions to promote safety, consulting with occupational and physical therapists for assistive devices, and selecting interventions that will improve the safety of the patient's home environment.

According to the National Quality Forum, which event is included under patient-protection events? 1 Abduction of a patient 2 Infant discharge to the wrong person 3 Disability associated with a medication error 4 Surgery performed on the wrong body part

2 Infant discharge to the wrong person is included under patient-protection events. Events like the abduction of a patient are considered to be criminal events. Disability associated with a medication error is included under care-management events. Surgeries performed on the wrong body part are included under surgical events.

The registered nurse is teaching the parent of a toddler about interventions to promote safety. Which statement made by the parent indicates the need for further teaching? 1 "I will place window guards on all the windows. " 2 "I will avoid installing safety locks on floor-level cabinets. " 3 "I will install keyless locks on door above my child's reach. " 4 "I will avoid using infant seats and swings when the child becomes too active. "

2 Installing safety locks on floor-level cabinets helps to baby-proof the home and prevent toddlers exploring their world with their hands and mouth. Therefore, it decreases the risk of choking and poisoning. Placing window guards on all the windows would be beneficial to prevent children from falling out of windows. Installing keyless locks on the door out of the child's reach prevents the child from leaving home and wandering off. It is very important to avoid the use of infant seats and swings when the child becomes too active, because the child may fall out of or trip over these accessories and may suffer injury.

The nurse is caring for a patient who has severe pain due to muscle cramps. How does the nurse interpret this pain? 1 Visceral pain 2 Somatic pain 3 Referred pain 4 Cutaneous pain

2 Muscle cramps indicate that the patient has initiation of pain from musculoskeletal tissues. Therefore, the patient has somatic pain. If the pain arises from internal organs such as the gastrointestinal (GI) tract or pancreas, it indicates visceral pain. If the patient has pain at a particular site and injury at a different site, it indicates referred pain. If the patient has pain due to damage to the skin's surface, it indicates cutaneous pain.

During emotional pain assessment, the patient reports numbness and tingling sensations interspersed with shooting or electric-like pain. What does the nurse infer from the patient's report? 1 The patient is experiencing idiopathic pain. 2 The patient is experiencing neuropathic pain. 3 The patient is experiencing nociceptive visceral pain. 4 The patient is experiencing nociceptive somatic pain.

2 Neuropathic pain is characterized by burning, shooting, or electric-like pain accompanied by a tingling sensation. Idiopathic pain is chronic pain in the absence of an identifiable physical or psychological cause. When idiopathic pain is present, it is generally more than what would be expected for the organic pathological condition. Nociceptive pain originating from visceral sites is described as aching or cramping, or as aching or throbbing when it originates from somatic sites.

Which statement is true regarding nonpharmacological pain interventions? 1 Nonpharmacological interventions should only be used alone. 2 Nonpharmacological interventions are useful for patients who cannot tolerate pain medications. 3 Nonpharmacological interventions have a clear set of guidelines regarding intensity and duration. 4 Nonpharmacological interventions should be used in place of pharmacological therapies for acute pain.

2 Nonpharmacological pain relief can be useful for patients who cannot tolerate pain medications. The remaining statements, however, are false. Nonpharmacological interventions may be used alone, but they can also be used in combination with pharmacological therapies. Depending on the nonpharmacological therapy, research is still in progress to determine clear guidelines for intensity and duration. For acute pain, nonpharmacological therapy should never replace pharmacological therapy.

A nurse instructs a patient to color code the hot water faucets and dials. What might be the possible age group of the patient? 1 Young adult 2 Older adult 3 Adolescent 4 Preschooler

2 Older adults are instructed to color code the hot water faucets and dials to prevent burns and scalds. The color coding makes it easier for an older adult to know which is hot and which is cold. Young adults and adolescents usually do not confuse hot and cold water, so this suggestion may not be helpful for them. Preschoolers usually need a parent's help taking baths and would not use hot-water faucets and dials.

Which class of pain management drugs may interfere with bowel or bladder function? 1 Anticonvulsants 2 Opioid analgesics 3 Nonopioid analgesics 4 Nonsteroidal antiinflammatory drugs

2 Opioid analgesics are effective when used for pain management, but a common side effect is disruption of bowel or bladder function. Anticonvulsants are more commonly associated with side effects like dizziness, fatigue, and confusion than with disrupted bowel and bladder function. Nonopioid analgesics and nonsteroidal antiinflammatory drugs more commonly result in gastric bleeding, hypertension, and nausea than in disruption of bowel and bladder function.

Which signs or symptoms in an opioid-naïve patient is of greatest concern to the nurse when assessing the patient 1 hour after administering an opioid? 1 Oxygen saturation of 95% 2 Difficulty arousing the patient 3 Respiratory rate of 10 breaths/minute 4 Pain intensity rating of 5 on a scale of 0 to 10

2 Opioid-naïve patients may develop a rare adverse effect of respiratory depression. Sedation always occurs before respiratory depression. The change in the level of consciousness supersedes oxygen saturation of 95% and moderate pain.

The nurse concludes that a patient has radiating pain. Which assessment findings support the nurse's conclusion? 1 The patient has pain from a small cut or laceration. 2 The patient has pain in the back accompanied by pain in the leg. 3 The patient has a crushing sensation with pain in the chest. 4 The patient has a burning sensation with severe stomach pain.

2 Pain extending from the initial site of injury to another body part is radiating pain. Therefore, because the patient has pain in the back accompanied by pain in the leg, it indicates radiating pain. Pain resulting from stimulation of the skin is cutaneous pain. A patient with pain from a small cut or laceration has cutaneous pain. If the patient has pain at one site but injury at a different site, it indicates referred pain. A patient experiencing a crushing sensation with pain in chest and a burning sensation with severe stomach pain indicates referred pain.

A patient is being discharged home on an around-the-clock (ATC) opioid for chronic back pain. Because of this order, the nurse anticipates an order for which class of medication? 1 Stool softener 2 Stimulant laxative 3 H2 receptor blocker 4 Proton pump inhibitor

2 Patients usually become tolerant to the side effects of opioids, with the exception of constipation. Routinely administering stimulant laxatives, not simple stool softeners, will prevent and treat constipation in these patients.

Which condition places a patient at the greatest risk for developing an infection? 1. Implantation of a prosthetic device 2. Burns over more than 20% of the body 3. Presence of an indwelling urinary catheter. 4. More that 2 puncture sites from laparoscopic surgery

2. Burns over more than 20% of body

While treating a patient, the primary health care provider encourages the patient to watch funny videos. This is an example of which pain management technique? 1 Relaxation 2 Distraction 3 Acupressure 4 Music therapy

2 Patients who are bored or in isolation may think more frequently about their pain, thus perceiving it more acutely. Watching videos may direct the patient's attention to something other than pain, reducing awareness of it. This is an example of using distraction to manage pain. Relaxation techniques include meditation, yoga, guided imagery, and progressive relaxation exercises. Acupressure is applying pressure to specific points on the body in order to influence nerve pathways to decrease pain perception. Like distraction, music therapy works by taking the patient's attention away from the pain, but this is done with music, not videos.

The nurse would expect all of the following physiological effects of exercise on the body systems except: 1. Change in metabolic rate 2. Decreased cardiac output 3. Increased respiratory rate and depth 4. Increased muscle tone, size, and strength

2 Rationale: Exercise increases cardiac output.

The nurse teaches the parent of an infant about interventions that promote the child's safety. Which statement by the parent does the nurse find effective for the prevention of choking? 1 "I will immunize my child as recommended." 2 "I will use large soft toys that have no small parts." 3 "I will avoid leaving the mesh sides of playpens lowered." 4 "I will make sure that my child sleeps on the back or side."

2 Small parts of toys may become dislodged. Therefore, the use of large, soft toys without small parts, such as buttons, would prevent choking and aspiration. Immunizing the infant would reduce the risk of sudden infant death syndrome (SIDS). When the mesh sides of a playpen are left lowered, the child's head may become lodged resulting in asphyxiation. The parent should ensure that the child sleeps on the back or side, because this prevents SIDS.

Which patient should be provided with a yellow wristband according to American Hospital Association (AHA)? 1 Patient with allergies 2 Patient with risk of falls 3 Patient with hypothermia 4 Patient who does not require resuscitation

2 The AHA has issued an advisory recommending that hospitals standardize wristband colors. Yellow is issued for patients with fall risks, and red is used for patients with allergies. There is no specific wristband indicated for patients with hypothermia. A purple wristband is given to patients who do not require resuscitation.

A patient with a history of a stroke that left her confused and unable to communicate returns from interventional radiology following placement of a gastrostomy tube. The health care provider's order reads, "Hydrocodone/acetaminophen 1 tab, per tube, q4 hours, prn." Which action by the nurse is most appropriate? 1 No action is required by the nurse because the order is appropriate. 2 Request to have the ordered changed to ATC (around the clock) for the first 48 hours. 3 Ask for a change of medication to meperidine 50 mg IVP, q3 hours, prn. 4 Begin the hydrocodone/acetaminophen when the patient shows nonverbal symptoms of pain.

2 The American Pain Society (2003) states that if you anticipate pain for most of the day, you should consider ATC administration. Insertion of a gastrostomy tube is painful. This patient will most likely experience pain for at least the next 48 hours.

The registered nurse (RN) is teaching a group of patients from a vulnerable population regarding measures to prevent fires at home. Which statement by a member of the group indicates the need for further teaching? 1 "We should avoid smoking at home." 2 "We should use old model space heaters." 3 "We should use the cooking equipment effectively." 4 "We should place carbon monoxide detectors in the house."

2 The U.S Consumer Product Safety Commission estimates that more than 25,000 residential fires every year are associated with the use of space heaters. Therefore, families should be advised to use only newer-model space heaters that have all of the current safety features. The leading cause of fire-related death is careless smoking, especially when smoking in bed. Therefore, smoking should be completely avoided. The improper use of cooking equipment and appliances are the main source of in-home fires and fire injuries. Therefore, families should be taught about effective use of cooking equipment. Smoke detectors and carbon monoxide detectors should be placed throughout the home.

6. A resident in a nursing home wanders during the night and scares other residents by going into their rooms. What is the most effective intervention that the nurse should implement to address this situation? 1. Explain to others that the resident means no harm. 2. Position a bed alarm on the wandering resident's bed. 3. Close the doors of the other residents' rooms at night. 4. Place bolsters against four side rails of the wandering resident's bed. Hint

2 The alarm will ring when the resident exits the bed. When the alarm rings, the nurse can immediately intervene to walk with the resident or engage the resident in a distracting activity. This is the only action among the options offered that effectively protects the other residents.

An opioid-naïve patient is on naloxone for respiratory depression caused by methadone overdose. The nurse is instructed to reassess the patient every 15 minutes for 2 hours following drug administration. What is the reason behind the schedule of reassessment of the patient? 1 The half-life of naloxone is greater than that of methadone. 2 Duration of the action of naloxone is less than that of methadone. 3 Naloxone acts as an agonist to methadone after 2 hours of administration. 4 Naloxone can cause methadone withdrawal symptoms in an opioid-naïve patient.

2 The duration of action or half-life of naloxone is less than that of methadone. Therefore, recurrence of respiratory depression by the relatively long action of methadone can be prevented by reassessing the patient every 15 minutes for 2 hours after naloxone administration. Methadone has a greater half-life than naloxone. Therefore, the effect of methadone is more prolonged than that of naloxone. Naloxone is an opioid-antagonist drug. Naloxone does not act as an agonist to morphine after 2 hours. Opioid-naïve patients are patients who have not taken opioid medications for at least a week. Naloxone causes morphine withdrawal symptoms only in patients who are physically dependent on morphine, not the patients who are opioid naïve.

A patient with rheumatoid arthritis reports acute joint pain in the hand. Which intervention is inappropriate for providing pain relief? 1 Encouraging the patient to listen to music or watch television 2 Collaborating with an occupational therapist to provide assistive devices for grooming 3 Administering ordered analgesics around the clock for 24 to 48 hours 4 Applying cool compresses to the patient's joints with the prescriber's approval

2 The nurse may collaborate with an occupational therapist to provide assistive devices to the patient for grooming, but this is not done to relieve pain; rather, this is an intervention to help the patient dress and prepare for the day if the joint pain is making this difficult. The remaining interventions are appropriate for pain relief. Music and television can help relieve pain by taking the patient's attention away from it. Analgesic administration is a pharmacological therapy method to provide pain relief. Cool compresses may also help soothe the pain caused by rheumatoid arthritis.

A patient who is on aspirin therapy for pain relief reports that there has been no change in the pain even after taking the drug. On assessment, the nurse finds that the patient had a history of a bleeding gastric ulcer and obstructive sleep apnea. What immediate action should the nurse take? 1 Add an opioid analgesic. 2 Stop the aspirin administration. 3 Increase the dose of aspirin. 4 Stop the aspirin and give ibuprofen.

2 The nurse should be aware of some of the common contraindications of analgesics. Nonsteroidal anti-inflammatory drugs (NSAIDs) should not be given to a patient with a history of gastrointestinal bleeding or renal insufficiency. Therefore, administration of aspirin should be stopped for this patient. Opioids should not be given to a patient with a history of obstructive sleep apnea, because they cause respiratory depression. Increasing the dose of aspirin would further worsen the gastrointestinal bleeding. Ibuprofen is also an NSAID and, therefore, should be avoided in this patient.

A patient was diagnosed with left-sided neglect after suffering a cerebrovascular accident. Which nursing intervention would be most effective to ensure the patient's safety? 1 Teaching the patient to use a walker 2 Reminding the patient to scan the environment while walking 3 Encouraging the patient to see an ophthalmologist for visual assessment 4 Teaching the patient to perform strengthening exercises on the left side of the body

2 The nurse should remind the patient to scan the environment when walking in the event of left-sided neglect after suffering a cerebrovascular accident, because the patient may fail to notice people or things approaching from the left. A patient with cerebrovascular accident-caused left-sided weakness should be educated regarding the use of a walker. Visiting an ophthalmologist is effective for those patients who have problems in seeing objects at a distance. The nurse should teach the patient with left sided weakness to perform strengthening exercises on the left side of the body.

Which statement about transcutaneous electrical nerve stimulation (TENS) is incorrect? 1 TENS is helpful in reducing pain perception. 2 TENS is effective for chronic and postsurgical pain control. 3 A TENS unit consists of a battery-powered transmitter, lead wires, and electrodes. 4 TENS requires a health care provider's order that identifies the site(s) for electrode placement.

2 Transcutaneous electrical nerve stimulation (TENS) is effective for acute, emergent, and postsurgical and procedural pain control but not for chronic pain. The remaining statements are correct. TENS is helpful in reducing pain perception. A TENS unit consists of a battery-powered transmitter, lead wires, and electrodes. TENS requires a health care provider's order that identifies the site(s) for electrode placement.

29. A nurse is caring for a patient with wrist restraints. Which action is most important when caring for this patient? 1. Release the restraints every 3 hours and provide skin care. 2. Offer fluids and assist the patient to toilet each time the restraints are released. 3. Check on the patient every hour and ensure that circulation is not impaired by the restraints. 4. Ensure that every 48 hours the primary health-care provider assesses the need for the restraints. Hint

2 When a restraint is removed every 2 hours, the nurse should assess for the presence of edema, capillary refill, sensation, function, skin integrity, and erythema. In addition, the patient should be offered fluids and the opportunity to go to the bathroom. Finally, skin care and range-of-motion exercises should be provided and the patient's position changed before restrains are reapplied.

15. A nurse working on the night shift in a long-term care facility identifies a small fire in a magazine rack in the corner of a community lounge. Because all of the residents are in their rooms, the nurse activates the fire alarm and then instructs staff members to close all doors on the unit. The nurse then obtains a type A extinguisher to fight the fire. Place the following steps in the order in which they should be implemented. 1. Pull the pin on the extinguisher. 2. Drag the extinguisher on the floor to the area of the fire. 3. Aim the nozzle of the extinguisher at the base of the flames. 4. Squeeze the handles together to discharge water from the extinguisher. 5. Sweep the nozzle of the extinguisher back and forth at the base of the flames.

2 1 3 4

While taking a patient's medical history, a nurse records that the patient has asthma. Which medications for pain management might the physician avoid prescribing? Select all that apply. 1 Tramadol 2 Naproxen 3 Ibuprofen 4 Oxycodone 5 Hydromorphone

2, 3 Some patients who have asthma or an allergy to aspirin are also allergic to other nonsteroidal antiinflammatory drugs (NSAIDs). Naproxen and ibuprofen are NSAIDs, so the physician may avoid prescribing these medicines to the patient for pain management. Tramadol, oxycodone, and hydromorphone are opioid analgesics, which may be less risky for allergic reaction in this patient.

A patient's stool specimen is positive for Clostridium difficile. Which isolation precautions should the nurse institute for this patient? 1. Droplet 2. Contact 3. Reverse 4. Airbone

2. Contact

The nurse is teaching the mother of a school-age child about interventions to promote safety. Which statement made by the mother indicates the need for further teaching? 1 "I should teach child proper bicycle safety." 2 "I should teach my child about the effects of using alcohol and drugs." 3 "I should teach my child the safe use of equipment for play and work." 4 "I should teach my child to operate electrical equipment under supervision."

2 The mother does not need to teach a school-age child about the effects of using alcohol and drugs at this age. This intervention would be more appropriate for adolescents, because they are more prone to risk-taking behaviors. The mother should teach the child about proper bicycle safety to reduce the risk of falls. The mother should teach the child the safe use of equipment for play and work to avoid injury. The mother should teach the child to operate electrical equipment only under supervision.

A patient reports difficulty falling asleep. The nurse asks questions to assess the symptoms of insomnia and analyzes the different stages of the patient's sleep cycle. What are the characteristics of stages 2 and 3 of nonrapid eye movement (NREM) sleep? Select all that apply. 1 Sleepwalking may occur. 2 Body functions become slow. 3 Arousal remains relatively easy. 4 Muscles are completely relaxed. 5 Vital signs are significantly lower than during waking hours.

2 Body functions become slow. 3 Arousal remains relatively easy. 4 Muscles are completely relaxed.

A patient is overweight and is a chronic smoker with type II diabetes. The patient is suffering from obstructive sleep apnea that is affecting his marital relationship. What nursing actions should the nurse perform to help the patient manage sleep apnea? Select all that apply. 1 Refer the patient to a marriage counselor. 2 Provide the patient with emotional support. 3 Instruct the patient to avoid coffee before bedtime. 4 Encourage the patient to lose weight and quit smoking. 5 Advise the patient to create an environment conducive to sleep before bedtime.

2 Provide the patient with emotional support. 4 Encourage the patient to lose weight and quit smoking.

The nurse is assessing a patient who complains of sleep deprivation. Which physiological symptoms should the nurse expect to find in the patient? Select all that apply. 1 Increased response time 2 Ptosis or blurred vision 3 Cardiac arrhythmias 4 Diminished reflexes 5 Decreased hearing

2 Ptosis or blurred vision 3 Cardiac arrhythmias 4 Diminished reflexes 5 Decreased hearing

Which statement made by the patient indicates the need for further teaching about the exercises to be done before initiating crutch walking? 1 "I should discontinue the exercise if I am not comfortable." 2 "I should contract my muscles for more than 30 seconds before relaxing them." 3 "I should start the exercise slowly and then increase repetitions as my physical condition improves." 4 "I should isometrically exercise muscle groups that are used for walking four times per day until I am ambulatory."

2 "I should contract my muscles for more than 30 seconds before relaxing them." The exercise program for a patient with crutch walking requires isometric exercises of the biceps and triceps. In this exercise, the muscles should be contracted for no more than 10 seconds before relaxing them. Therefore, when the patient states that muscles should be contracted for more than 30 seconds, it indicates the need for further teaching. Isometric exercises should be discontinued if the patient experiences discomfort, fatigue, or pain. Isometric exercises should always be started slowly and then the repetitions should be increased as the physical condition is improved. The muscle groups, quadriceps, and gluteal muscles should be exercised during the isometric exercise.

A nursing instructor asks a nursing student about the principles of safe patient transferring and positioning. Which statement by the student indicates inadequate learning? Select all that apply. 1 "The lower the center of gravity, the greater the stability of the nurse." 2 "The narrower the base of the support, the greater the stability of the nurse." 3 "If I balance activity between my arms and legs this will reduce the risk of back injury." 4 "I should face the direction opposite to movement to prevent abnormal twisting of the spine." 5 "The equilibrium of an object is maintained as long as the line of gravity passes through its base of support.

2 "The narrower the base of the support, the greater the stability of the nurse." 4 "I should face the direction opposite to movement to prevent abnormal twisting of the spine." The wider the base of the support, the greater the stability of the nurse. Facing the direction of the movement prevents abnormal twisting of the spine. The lower the center of gravity, the greater the stability of the nurse. Dividing balanced activity between the arms and legs reduces the risk of a back injury. The equilibrium of an object is maintained as long as the line of gravity passes through its base of support. Topics

The nurse has failed to obtain informed consent before performing a procedure on a patient. Which type of torts result from this nursing action? Select all that apply. 1 Assault 2 Battery 3 Negligence 4 False imprisonment 5 Libel

2 3 Negligence is performing an action that falls below a standard of care. Failure to obtain informed consent is an act of negligence. Assault refers to an action that places a person within harmful or offensive contact without consent. Battery is any intentional touching without consent. Because the nurse has failed to obtain informed consent, doing any intervention on the patient would be considered as battery. False imprisonment is the unjustified restraint of a person without legal warrant. Failure to obtain informed consent would not result in assault or false imprisonment. Libel is written defamation of character.

When witnessing a patient giving informed consent prior to undergoing surgery, which actions should the nurse perform? Select all that apply. 1 If the patient refuses to sign the consent, leave the situation as it is. 2 Confirm that the patient has understood the information about the surgery. 3 Inform the healthcare provider if the patient refuses to undergo the surgery. 4 Sign the consent form as a witness, once the patient voluntarily gives consent. 5 Ask a nursing student to witness the informed consent if the nurse is busy.

2 3 4 The patient needs to understand the surgical procedure and voluntarily give consent, so the nurse should enquire about the patient's understanding and answer any questions. If the patient refuses to undergo the surgery, the nurse should inform the healthcare provider, so any harmful consequences of refusal can be explained to the patient. The nurse's signature witnessing the consent means that the patient voluntarily gave consent, that the signature is authentic, and that the patient appears to be competent to give consent. If the patient refuses to sign the consent in spite of repeated explanations, this rejection should be documented, signed, and witnessed. Due to the legal nature of the document, a nursing student should not be asked to witness informed consent forms.

The nurse attends to a patient who sustained injuries when crossing the street. The patient was hit by a car that failed to stop at a stop sign. The patient was rushed to the emergency department and then to surgery to repair injuries. After surgery, the patient was transferred to the medical-surgical unit for postsurgical management. The healthcare provider informs the nurse that patient confidentiality should be strictly maintained. What should the nurse interpret from this? Select all that apply. 1 The nurse should respect that the patient has the right to keep personal information from being disclosed. 2 The nurse should avoid discussing the patient's medical reports in public areas with other healthcare professionals. 3 The nurse should not disclose information about a procedure to the patient. 4 The nurse must protect any private information about the patient, once it has been disclosed in healthcare settings. 5 Message boards where daily nursing care information is posted in the patient's room cannot contain information revealing the patient's medical condition.

2 3 4 Confidentiality means that nurses and all healthcare providers must avoid discussing patients in public hallways and provide reasonable levels of privacy in communicating with and about patients in any manner. Confidentiality protects private patient information once it has been disclosed in healthcare settings. Privacy is the right of the patient to keep personal information from being disclosed. Patient have the right to get every possible detail before undergoing a particular procedure. Message boards used in patient's hospital rooms to post daily nursing care information can no longer contain information revealing the patient's medical condition.

30. A nurse is planning a class about safety precautions for a group of parents who have toddlers. What information should the nurse include in the teaching plan? SELECT ALL THAT APPLY. 1. Position them on the back when sleeping. 2. Attend toddlers when on elevated surfaces. 3. Use a rear-facing car seat until 4 years of age. 4. Keep plastic inserts in unused electrical outlets. 5. Turn pot handles toward the back of stove when cooking. Hint

2 4 5

50. What nursing intervention can give a patient a sense of control regarding personal safety? SELECT ALL THAT APPLY. 1. Inform the patient why an identification band should be worn. 2. Instruct the patient how to lock the wheels on a wheel chair. 3. Keep the patient's bed in the lowest position. 4. Teach the patient how to use the call bell. 5. Orient the patient to the environment.

2 4 5

What is the priority nursing intervention to prevent patient problems associated with latex allergies? 1.Use nonlatex gloves 2.Identify persons at risk 3.Keep a latex-safe supply cart available 4. Administer an antihistamine prophylactically

2.Identify persons at risk

During preparation for practicing a mobility assessment, a student nurse encounters the term proprioception. Which phrase best explains the term proprioception? 1 Orientation to time, place, and person 2 Awareness of the position of the body and its parts 3 Perception of pressure over the palmar and plantar surfaces 4 Perception of abnormal thermal sensation on the skin

2 Awareness of the position of the body and its parts Proprioception is defined as the awareness of the position of the body and its parts. Proprioceptors present in the nerve endings of muscles, joints, and tendons monitor proprioception. It is required by the body to maintain proper posture. There is no special term used for orientation. When pressure is applied over the palmar and plantar surfaces, a touch sensation is perceived. Study Tip: Proprioception can be thought of as position sense.

A patient is admitted to the hospital with injury to the knee joint following a fall. The nurse notices an increased mobility of the joint while assessing the range of motion (ROM). What could be the reason for the increased mobility of the knee joint? 1 Arthritis of the joint 2 Ligament tears in the joint 3 Contractures of the joint 4 Fluid collection in the join

2 Ligament tears in the joint While assessing the ROM of the patient, if increased mobility is noticed, it indicates that there is a possibility of ligament tears. Arthritis is an inflammation of the joint resulting in decreased mobility and stiffness. Contractures and fluid collection in the joint may decrease joint mobility and cause stiffness.

What group of muscles contract to accomplish the same movement? 1 Skeletal muscles 2 Synergistic muscles 3 Antigravity muscles 4 Antagonistic muscles

2 Synergistic muscles Synergistic muscles contract to accomplish the same movement. Skeletal muscles are attached to the skeleton by tendons. Antigravity muscles continuously oppose the effect of gravity on the body and permit a person to maintain an upright posture. Antagonistic muscles relax when active mover muscles contract.

What is the name for the white, glistening, and fibrous bands of tissue that connect muscle to bone? 1 Joints 2 Tendons 3 Cartilage 4 Ligaments

2 Tendons Tendons are white, glistening, fibrous bands of tissue that connect muscles to bones. A joint is the connection between bones. Cartilage is nonvascular; it supports connective tissue that sustains weight and serves as a shock absorber between articulating bones. Ligaments are white, shiny, flexible bands of fibrous tissue that bind joints and connect bones with cartilage.

The nurse is transferring a patient from the hospital bed to a stretcher. The patient responds to reflexes and is physically sound enough to assist in the transfer. Which nursing actions should be a part of the transfer activity? Select all that apply. 1 The nurse fastens a gait belt around the patient's waist. 2 The nurse divides balanced activity between the arms and legs. 3 The nurse faces the opposite direction of the patient's movement. 4 The nurse tells the patient to keep the body in proper alignment. 5 The nurse stands with feet wide enough apart to improve the base of stability.

2 The nurse divides balanced activity between the arms and legs. 4 The nurse tells the patient to keep the body in proper alignment. 5 The nurse stands with feet wide enough apart to improve the base of stability. The nurse divides the balanced activity between the arms and legs while transferring or positioning a patient who is able to assist. It allows proper distribution of pressure on larger muscles such as arms and legs in order to reduce the risk of back injury. The nurse maintains the body in proper alignment, which lowers the center of gravity and provides stability. The nurse stands with feet apart, which provides a wide base for support. When the patient is able to assist, mechanical items such as a gait belt are not necessary for holding. The nurse should face the direction of the movement during transfer in order to prevent abnormal twisting. Abnormal twisting may cause vertebral injuries in the nurse.

Which environmental issue is a hindrance to activity and exercise? 1 Hormonal changes and increased osteoclastic activity with increasing age 2 Work sites reluctant in motivating employees for physical fitness regimens 3 A patient's decisions to change his or her behavior to include a daily exercise routine 4 A patient's knowledge, values, and beliefs about exercise in relation to health

2 Work sites reluctant in motivating employees for physical fitness regimens Activity and exercise promotion (or lack thereof) at work sites is an environmental factor that affects a patient's ability to exercise. Hormonal changes and increased osteoclastic activity with increasing age are developmental factors that affects activity and exercise. A patient's decision to change his or her behavior to include a daily exercise routine and the patient's knowledge, values, and beliefs about exercise in relation to health are behavioral factors that influence activity and exercise.

While caring for a patient with left-sided weakness, the nurse suspects anxiety related to fear of falling. Which assessment findings would further confirm the nurse's suspicion? Select all that apply. 1 Difficulty seeing distant objects 2 Difficulty focusing during conversations 3 Difficulty understanding medication instructions 4 Limited ability to perform fine and gross motor skills on the left side 5 Moving the head to the right side in response to loud noises on the left side

2, 3 If a patient has difficulty focusing or difficulty understanding teaching during a conversation, it may indicate anxiety related to fear of falling or health status. When the patient exhibits a limited ability to perform fine and gross motor skills on the left side, it indicates impaired physical mobility. If the patient has difficulty seeing objects at a distance, the patient is at risk of falls. Unresponsiveness of the patient to loud noises on the left side indicates unilateral neglect related to brain injury.

The nurse advises a patient with neuropathic pain to undergo guided imagery therapy to alleviate pain. Which pharmacological treatment interventions would be beneficial to the patient for pain management? Select all that apply. 1 Corticosteroids 2 Anticonvulsants 3 Antidepressants 4 Muscle relaxants 5 Bisphosphonates

2, 3 The nonpharmacological interventions that are usually recommended for pain relief in a patient with neuropathic pain include relaxation and guided imagery. This allows patients to alter affective-motivational and cognitive pain perception. The pharmacological pain management therapies that would be beneficial to a patient with neuropathic pain include anticonvulsants such as gabapentin and antidepressants such as nortriptyline. Gabapentin acts on the supraspinal region to stimulate noradrenaline-mediated descending inhibition to reduce neuropathic pain. Nortriptyline alleviates neuropathic pain by altering neurotransmitter levels. Corticosteroids relieve pain associated with inflammation and bone metastasis. Muscle relaxants have no analgesic effect. Bisphosphonates are prescribed for bone pain.

The nurse is explaining and demonstrating the correct use of fire extinguishers at home to a group of adolescents. Which order of steps listed by an adolescent indicates effective learning? 1. Squeeze the handle. 2. Pull the pin to unlock the handle. 3. Aim low at the base of the fire. 4. Sweep the unit from side to side.

2, 3, 1, 4 When the nurse is explaining and demonstrating the use of fire extinguisher at home, the nurse should first pull the pin to unlock the handle. The next step is to aim low at the base of the fire. Then, the nurse then squeezes the handle and sweeps the unit from side to side.

The nurse is caring for an older adult in the home and is concerned about the risk of injury in this patient. Which activities should the nurse perform to assess risk of injury in this patient? Select all that apply. 1 Inspect the patient's food. 2 Perform a home hazard appraisal. 3 Inquire about the patient's visual acuity. 4 Observe the patient's posture and balance. 5 Assess the patient's gastrointestinal system.

2, 3, 4 A home hazard appraisal may reveal issues such as poor lighting, small items, or excessive furniture in the house that can increase the risk of injury. Inquiring about the patient's visual acuity helps assess the risk of falling or tripping over objects at home. Assessing the patient's posture and balance reveals any balance, coordination, or movement-related issues. Inspecting the patient's food does not help in assessment of risk. Assessing patient's gastrointestinal system does not affect the risk of injury.

The nurse is assessing a patient with acute pain. Which statements are true about acute pain? Select all that apply. 1 Acute pain is not protective. 2 Acute pain has an identifiable cause. 3 Acute pain has limited tissue damage. 4 Acute pain results in prolonged hospitalization. 5 Patients with acute pain seek numerous health care providers.

2, 3, 4 Acute pain has an identifiable cause, limited tissue damage, and an emotional response. Acute pain results in prolonged hospitalization as it seriously threatens a patient's recovery, so the health team members treat it aggressively. Acute pain is protective, unlike chronic pain, which is not protective. A patient with chronic pain may seek numerous health care providers because of its unknown cause.

A group of nursing students is learning about nociceptive and neuropathic pain. What are examples of neuropathic pain? Select all that apply. 1 Aching muscles 2 Diabetic neuropathy 3 Trigeminal neuralgia 4 Nerve root compression 5 Throbbing pain at knee joint

2, 3, 4 Neuropathic pain arises when there is abnormal processing of sensory input by the peripheral or central nervous system. Pain felt along the distribution of many peripheral nerves as in diabetic neuropathy is a neuropathic pain. Pain felt partly along the distribution of a damaged nerve such as in nerve root compression is also an example of neuropathic pain. Pain associated with trigeminal neuralgia is also a neuropathic pain. Aching muscles and a throbbing pain at the knee joint are examples of nociceptive pain.

The nurse works in the radiology department of a hospital. Which guidelines should the nurse follow with respect to patient safety? Select all that apply. 1 Counsel patients that radiation is not harmful. 2 Ask patients to wear lead aprons when in the radiation zone. 3 Limit patients' time spent near the source of radiation. 4 Make the distance from the source as great as possible. 5 Allow patients to freely move in the radiation zone as desired.

2, 3, 4 Radiation is a health hazard, so hospitals have strict guidelines concerning care of patients receiving radiation. The patient should wear a lead apron to protect against the radiation. The patient should spend a minimum amount of time in the radiation zone and should maintain safe distance from the source of radiation. The patient should be counseled that radiation could be harmful, and the patient should not move freely in the radiation zone.

The nurse is caring for a patient who is at risk of falls due to improper gait. Which measures should the nurse take to ensure patient safety? Select all that apply. 1 Apply restraints. 2 Ensure that the patient wears rubber-soled slippers. 3 Move the patient on crutches or walkers after ensuring the patient's integrity. 4 Remove excess furniture from the path. 5 Advise the family members to accompany the patient when the nurse is not present.

2, 3, 4 Use of rubber-soled slippers helps in preventing slips and decreases the risk of falls. The patient should be encouraged to use assistive aids such as crutches and walkers to provide support. Excess furniture can be in the way during ambulation and should be removed. Restraints should not be used, because they can make the patient restless and also increase the risk of immobility-related complications. Family members should be instructed regarding the patient's issues, but it is the nurse's responsibility to take care of the patient.

A patient has hemiplegia as a result of a brain attack (cerebrovascular accident). Which complication of immobility is of most concern to the nurse? 1. Dehydration 2. Contractures 3. Incontinence 4. Hypertension

2. Contractures result from permanent shortening of muscles, tendons, and ligaments. Routine range-of-motion exercises and maintaining the body in functional alignment can prevent contractures.

The nurse is caring for an older adult in the home and is concerned about infection control in this patient. What should the nurse inquire about when assessing this patient's risk of food poisoning? Select all that apply. 1 Daily water intake 2 Hand-washing practices 3 Knowledge about food-storage practices 4 Monthly consumption of fruits and vegetables 5 Practice of checking expiration dates on milk products

2, 3, 5 Good hand-washing practices include washing hands before and after handling food items. Proper knowledge about food-storage practices ensures that the food consumed is safe and free of contamination. Regular practice of checking expiration dates on milk products ensures that the products are safe for consumption. Knowledge of daily water intake does not help in assessing the risk of food poisoning. Knowledge of monthly consumption of fruits and vegetables is not related to the risk of food poisoning.

Which risk factors are seen in young and middle-age adults? Select all that apply. 1 Risk of falls 2 Use of firearms 3 Substance abuse 4 Incidence of suicide 5 Inadequate nutrition

2, 3, 5 The risk factors that are seen in young and middle-age adults are use of firearms, substance abuse, and inadequate nutrition. Risk of falls is significant for older adults. Incidence of suicide is a common risk factor seen in adolescents.

The nurse is assessing a hospitalized patient with acute pain. Which questions should the nurse ask the patient for an appropriate assessment? Select all that apply. 1 "How bad is your pain now?" 2 "What makes your pain worse?" 3 "Describe your pain." 4 "What is the worst pain you have had in past 24 hours?" 5 "Show me where you are hurt. Does it stay there or does it spread?"

2, 3, 5 When assessing a patient with acute pain, the questions should be specific. The questions should aim to determine the intensity, location, and quality of pain. Ask provocative questions such as, "What makes the pain worse?" Ask about the region of the pain and the radiation of pain. Asking how bad the pain is may not yield specific details. Instead, the patient should be asked to rate the pain on a scale of 0 to 10. Other details can be asked once the patient is comfortable.

A patient starts having seizures in a postoperative unit. Which measures would ensure normal breathing and reduced risk of musculoskeletal injury? Select all that apply. 1 Tighten the waist belt. 2 Loosen the collar. 3 Restrain the patient. 4 Hold the limbs loosely. 5 Place something soft under the head.

2, 4 During seizures patients may have altered breathing and may be at risk of sustaining musculoskeletal injury. Loosening the patient's collar facilitates breathing movements by reducing the effort required for chest expansion. If the patient is standing or sitting, guide the patient to the floor and protect the head by cradling it in the nurse's lap or placing a pad under the head. Limbs should be held loosely if the patient is flailing. Tightening the waist belt restricts abdominal expansion, further aggravating respirations in the patient. The patient should not be restrained during seizures, because this increases the chance of musculoskeletal injury.

A nurse is caring for a patient with impaired mobility. Which position contributes most to the formation of a hip flexion contracture? 1. Low-Fowler 2. Orthopneic 3. Supine 4. Sims'

2. While in the high-Fowler position the patient is then positioned leaning forward with arms resting on an over-bed table. In this orthopneic position, the hips are extensively flexed creating an angle less than 90 degrees.

A patient is experiencing unilateral neglect related to a brain injury. How can the nurse help this patient to restore normal capabilities? Select all that apply. 1 Promote the use of effective coping skills. 2 Remind the patient to scan the home environment. 3 Establish therapeutic communication. 4 Encourage family members to eat along with the patient. 5 Teach the patient to touch the affected side of the body with the unaffected hand.

2, 4, 5 A patient who has had a cerebrovascular accident may have unilateral neglect, which can increase the patient's risk for falling. The patient should be reminded to scan the home environment while walking to prevent the risk of falling. Encourage family members to eat with the patient so they can remind the patient to try to use the affected side of the mouth to eat the food. Teaching the patient to touch the affected side of the body with the unaffected hand helps the patient to become aware of the affected side. Coping skills and therapeutic communication may help the patient to cope better but are more useful for an anxious patient.

The nurse is teaching a group of nursing students about concepts of pain in infants. Which information should the nurse include in the teaching? Select all that apply. 1 Infants cannot express pain. 2 Absorption of drugs is faster than expected. 3 Infants are less sensitive to pain than adults are. 4 Preterm neonates have greater sensitivity to pain than older children do. 5 Assessment of pain involves behavioral cues and physiological indicators.

2, 4, 5 Absorption of drugs in infants is faster than expected. The drugs that are excreted by the kidneys should be administered in a lower dosage. Preterm neonates have greater sensitivity than term neonates or older children. Using behavioral cues such as facial expression and physiological indicators such as changes in vital signs provide proper assessment of pain. Infants cannot verbalize pain but respond with behavioral changes. Term neonates have the same sensitivity to pain as older children.

The nurse works in a psychiatric unit and understands that the use of restraints may be useful for ensuring patients' safety. Which patients would need a temporary restraint? Select all that apply. 1 Alert patients 2 Confused patients 3 Accommodating patients 4 Patients who repeatedly fall 5 Patients who try to remove medical devices

2, 4, 5 Confused patients are prone to falls and injuries, if not restrained. Patients who repeatedly fall are at an increased risk of becoming injured and should be restrained. Patients who try to remove medical devices should be restrained to prevent them from removing medical devices. Alert patients are generally cooperative and able to communicate and therefore do not require restraints. Accommodating patients are cooperative and do not require restraints.

The registered nurse is teaching a nursing student about applying transcutaneous electrical nerve stimulation (TENS) to a patient. Which of the nursing student's statements indicate a need for further teaching? Select all that apply. 1 "I should set the frequency to no more than 50 Hz." 2 "I should use TENS on patients who have chronic cancer pain." 3 "I should place TENS electrodes directly over or near the site of pain." 4 "I should apply hair or skin preparations before placing TENS electrodes." 5 "I should remove TENS electrodes if the patient feels a buzzing or tingling sensation."

2, 4, 5 Transcutaneous electrical nerve stimulation (TENS) is effective in treating acute, emergent, and postsurgical and procedural pain control, but not chronic conditions, like cancer pain. The nurse should not apply any hair or skin preparations before attaching the TENS electrodes. Buzzing or tingling sensations are normal, and do not require the nurse to remove electrodes. The other statements indicate understanding. The range of frequency of TENS is 10 Hz to 50 Hz. The TENS electrodes should be placed directly over or near the site of pain.

Which adjuvant drugs are preferred for treating neuropathic pain? Select all that apply. 1 Corticosteroids 2 Anticonvulsants 3 Opioid analgesics 4 Nonopioid analgesics 5 Tricyclic antidepressants

2, 5 Anticonvulsants and tricyclic antidepressants can be effective for treating chronic pain, especially neuropathic pain. Corticosteroids are typically used to relieve pain from inflammation and bone metastasis. Opioid and nonopioid analgesics are not adjuvant drugs.

A primary health care provider recommends ibuprofen to a patient in pain. Which statements about this medication are correct? Select all that apply. 1 It depresses the central nervous system in order to relieve pain. 2 It acts by inhibiting the synthesis of prostaglandins. 3 It is highly recommended for older adults experiencing pain. 4 It is the most effective prescription drug available for pain relief. 5 One of its serious side effects is gastrointestinal bleeding.

2, 5 Nonsteroidal antiinflammatory drugs (NSAIDs), such as ibuprofen, act by inhibiting prostaglandin synthesis, thereby inhibiting cellular response to inflammation and thus reducing pain. Gastrointestinal bleeding is a major adverse effect of NSAIDs. The remaining statements are incorrect. Opiates, not NSAIDs, depress the central nervous system to relieve pain. Because of the risk for gastrointestinal bleeding, ibuprofen and other NSAIDs are not frequently the first choice for treating pain in older adults. Ibuprofen is not a prescription drug; it is widely available over the counter.

A toaster is on fire in the pantry of a hospital unit. What should the nurse do first? 1.Unplug the toaster 2.Activate the fire alarm 3.Put out the fire with an extinguisher 4.Evacuate the patients from the room next to the kitchen

2.Activate the fire alarm

A home care nurse is visiting an older adult who experienced a brain attack 2 years ago and now has a sacral pressure ulcer. The nurse identifies that the patient and spouse both appear unkempt and the spouse appears tired and irritated. The patient is very quiet and avoids eye contact. Which nursing action is most appropriate? 1. Explore with the patient and spouse their concern. 2. Assess the patient for signs of physical abuse and neglect 3. Discuss with the patient and spouse additional resources to help with the patients care. 4. Assess that patients pressure ulcer and report findings to the primary health-care provider.

2.

The nurse is caring for a patient who is in early stages of cardiac failure. The patient tells the healthcare provider and the nurse that he wishes to end his life without any suffering as soon as possible. Which would be the most appropriate action? 1 Assist the patient in suicide as per his wish. 2 Explain to the patient that his life can be extended. 3 Ask the patient's family to make decisions regarding the patient's death. 4 Ask the patient to wait for court orders regarding the decision.

2. The patient does not have end-stage cardiac failure; thus medical interventions would be of help for the patient to extend his life. Assisted suicide violates the Code of Ethics for nurses, is illegal in most states, and is not appropriate in a patient with a non-terminal disease. Because the patient could be managed by means of drugs and other medical interventions, there is no need for the patient's family to make decisions concerning the patient's death.

Nursing staff members are sitting in the lounge taking their morning break. An unlicensed assistive personnel (UAP) tells the group that she thinks that the unit secretary has acquired immunodeficiency syndrome (AIDS) and proceeds to tell the nursing staff that the secretary probably contracted the disease from her husband, who is supposedly a drug addict. Which legal tort has the UAP violated? 1.Libel 2.Slander 3.Assault 4.Negligence

2. Defamation is a false communication or a careless disregard for the truth that causes damage to someone's reputation, either in writing (libel) or verbally (slander). An assault occurs when a person puts another person in fear of a harmful or an offensive contact. Negligence involves the actions of professionals that fall below the standard of care for a specific professional group.

A 72-year-old patient asks the nurse about using an over-the-counter antihistamine as a sleeping pill to help her get to sleep. What is the nurse's best response? 1. "Antihistamines are better than prescription medications because these can cause a lot of problems.: 2. "Antihistamines should not be used because they can cause confusion and increase your risk of falls." 3. "Antihistamines are effective sleep aids because they do not have many side effects." 4. "Over-the-counter medications when combined with sleep hygiene measures are a good plan for sleep.

2. "Antihistamines should not be used because they can cause confusion and increase your risk of falls."

The nurse is contacting the health care provider about a patient's sleep problem. Place the steps of SBAR (situation, background, assessment, recommendation) in the correct order. 1. Mrs. Dodd, 46 years old, was admitted 3 days ago following a motor vehicle accident. She is in balanced skeletal traction for a fractured left femur. She is having difficulty falling asleep. 2. "Dr, Smithson, this is Pam, the nurse caring for Mrs. Dodd. I'm calling because Mrs. Dodd is having difficulty sleeping." 3. "I am calling to ask if you would order a hypnotic such as zolpidem (Ambien) to use on a prn basis." 4. Mrs. Dodd is taking her pain medication every 4 hours as ordered and rates her pain as a 2 out of 10. Last night she was still awake at 0100. She states that she is comfortable but just can't fall asleep. Her vital signs are BP 124/76, P 78, R 12 and T 37.1 C (98.8 F)

2. "Dr, Smithson, this is Pam, the nurse caring for Mrs. Dodd. I'm calling because Mrs. Dodd is having difficulty sleeping." 1. Mrs. Dodd, 46 years old, was admitted 3 days ago following a motor vehicle accident. She is in balanced skeletal traction for a fractured left femur. She is having difficulty falling asleep. 4. Mrs. Dodd is taking her pain medication every 4 hours as ordered and rates her pain as a 2 out of 10. Last night she was still awake at 0100. She states that she is comfortable but just can't fall asleep. Her vital signs are BP 124/76, P 78, R 12 and T 37.1 C (98.8 F). 3. "I am calling to ask if you would order a hypnotic such as zolpidem (Ambien) to use on a prn basis."

Which statement made by a mother being discharged to home with her newborn infant indicates that she understands the discharge teaching related to best sleep practices? 1. "I'll give the baby a bottle to help her fall asleep." 2. "We'll put the baby on her back to sleep." 3. "We put the baby's stuffed animals in the crib to make her feel safe." 4. "I know the baby will not need to be fed until morning."

2. "We'll put the baby on her back to sleep."

From which type of isolation precaution is this mask (hard covers nose and mouth) designed to protect the nurse? 1. contact 2. Airborne 3. Standard 4. Protective

2. Airborne

A patient has been in the intensive care unit (ICU) for 3 days. For which common adaptation indicating ICU psychosis associated with sleep deprivation should the nurse assess the patient? 1. Hypoxia 2. Delirium 3. Lethargy 4. Dementia

2. Delirium Melatonin regulates the circadian phases of sleep. Environmental triggers called synchronizers adjust the sleep- wake cycle to a 24-hour solar day. Intensive care units have bright lights and increased sensory input that cause disorientation to day and night and interrupt sleep. Interrupted sleep results in lability of mood, irritability, excitability, suspiciousness, confusion, and delirium.

Which is the appropriate patient outcome for an adult who has disturbed sleep because of nocturia? 1. Report fewer early morning awakenings because of a wet bed. 2. Demonstrate a reduction in nighttime bathroom visits. 3. Resume sleeping immediately after voiding. 4. Use an incontinence device at night.

2. Demonstrate a reduction in nighttime bathroom visits. Demonstrating a reduction in night- time bathroom visits is an appropriate outcome for nocturia, which is voluntary urination during the night.

Which are most important for a nurse to consider when a patient reports the presence of pain? Select all that apply. 1. The extent of pain is directly related to the amount of tissue damage. 2. Fatigue increases the intensity of pain experienced by the patient. 3. Behavioral adaptations are congruent with statements about pain. 4. Giving opioids to a patient in pain will lead to an addiction. 5. The person feeling the pain is the authority on the pain.

2. Fatigue increases the intensity of pain experienced by the patient. Fatigue decreases a person's coping abilities which increases the intensity of pain. 5. The person feeling the pain is the authority on the pain. Pain is a personal experience. Margo McCaffery, a pain researcher, has indicated that pain is whatever the person in pain says it is and exists whenever the person in pain says it exists.

An emaciated patient is at risk for developing a pressure ulcer. In which position should the nurse avoid placing the patient? 1. Low-Fowler 2. Side-lying 3. Supine 4. Prone

2. In the side-lying position the majority of the body weight is borne by the greater trochanter. The bone is close to the surface of the skin, with minimal overlying protective tissue.

What is the last step in making an occupied bed that the nurse should teach a nursing assistant? 1.Raising both the side rails on the bed 2. Lowering the height of the bed toward the floor 3.Ensuring that the patient is in a comfortable position 4.Elevating the head of the bed to a semi-Fowler position

2. Lowering the height of the bed toward the floor

Which are examples of primary defenses that protect the body form infection? Select all that apply. 1. Antibiotic therapy 2. Lysozomes in saliva 3. The low pH of the skin 4. the alkaline environment of the vagina 5. Production of mucous by cells in the genitourinary tract.

2. Lysozomes in saliva 3. The low pH of the skin 5.Production of mucous by cells in the genitourinary tract.

A nurse is making an occupied bed. What should the nurse do to prevent plantar flexion? 1. Tuck in the top linens on just the sides of the bed 2. Place a toe pleat in the top linens over the feet 3. Let the top linens hang off the end of the bed 4. Use trochanter rolls to position the feet

2. Making a vertical or horizontal toe pleat at the foot of the bed over the patient's feet leaves room for the feet to move freely and avoids exerting pressure on the upper surface of the feet, thus minimizing plantar flexion.

A nurse is caring for a patient with Parkinson's disease who is experiencing difficulty swallowing. What potential problem associated with dysphagia has the greatest influence on the plan of care? 1.Anorexia 2.Aspiration 3.Self-care deficit 4. Inadequate intake

2.Aspiration

An older female adult explains to the nurse that she has insomnia. The nurse interviews the patient and her husband and reviews the patient's medication reconciliation form. Which factors does the nurse conclude are associated with the patient's insomnia? Select all that apply. 1. Metformin 2. Older adult 3. Female gender 4. Alcohol intake 5. Diphenhydramine 6. Catnaps during the day

2. Older adult Sleep patterns tend to change as one ages. Older people become sleepy earlier and wake up earlier (alteration in circadian rhythms), wake up more frequently (lower levels of growth hormone and melatonin), and experience less deep sleep (more rapid sleep cycles). 3. Female gender Hormonal shifts in women occur throughout life: monthly related to ovulation, during pregnancy, and during and after menopause. Hormonal changes can precipitate nausea, anxiety, weight gain, generalized discomfort, restless legs syndrome, acid reflux, and frequent urination. All of these physiological responses can precipitate insomnia. 4. Alcohol intake Alcohol is a sedative that can help one fall asleep but it prevents deeper stages of sleep and causes one to awaken frequently during the night and earlier in the morning.

A nurse working in A clinic is assessing patients of a variety of ages. People within which age group should the nurse particularly assessed for subtle signs and symptoms of sub clinical infections? 1. Children of school age 2. Older adults 3. Adolescents 4. Infants

2. Older adults

The nurse is caring for a patient with a nasogastric tube for gastric decompression. Which nursing action takes priority? 1. Discontinuing the wall suction when providing care 2. Positioning the patient in the semi-Fowler position 3.Instilling the tube with 30mL of air every 2 hours 4.Caring for the nares at least every 8 hrs.

2. Positioning the patient in the semi-Fowler position

A patient tells the nurse. "I think I have an ear infection." The nurse should assess this patient for which objective human responses to an ear infection. Select all that apply. 1. Throbbing pain 2. Purulent drainage 3. Elevated temperature 4. Dizziness when moving 5. Hearing a buzzing sound

2. Purulent drainage 3. Elevated temperature

A nurse is caring for a variety of patients, each experiencing one of the following problems. Which health problem places a patient at the greatest risk for complications associated with immobility? 1. Incontinence 2. Quadriplegia 3. Hemiparesis 4. Confusion

2. Quadriplegia, paralysis of all four extremities, places the patient at greatest risk for pressure ulcers because the patient has no ability to shift the body weight off of bony prominences or change position without total assistance.

A nurse is providing health teaching for a patient with the diagnosis of obstructive sleep apnea. Which aspect of sleep should the nurse explain is most often affected? 1. Amount 2. Quality 3. Depth 4. Onset

2. Quality Sleep apnea is the periodic cessation of breathing during sleep. Episodes occur during rapid-eye-movement (REM) sleep (interfering with dreaming) and non-rapid-eye- movement (NREM) sleep (interfering with restorative sleep), both of which reduce the quality of sleep.

A patient is experiencing discomfort associated with gastroesophageal reflux. In which position should the nurse teach the patient to sleep? 1. Right lateral 2. Semi-Fowler 3. Prone 4. Sims

2. Semi-Fowler Gastric secretions increase during rapid-eye-movement (REM) sleep. The semi-Fowler position limits gastroesophageal reflux because gravity allows the abdominal organs to drop, which reduces pressure on the stomach and results in less stomach contents flowing upward into the esophagus.

The nurse is providing health teaching for a patient using herbal compounds such as kava for sleep. Which points need to be included? (Select all that apply.) 1. Can cause urinary retention 2. Should not be used indefinitely 3. May have toxic effects on the liver 4. May cause diarrhea and anxiety 5. Are not regulated by the U.S. Food and Drug Administration (FDA)

2. Should not be used indefinitely 3. May have toxic effects on the liver 5. Are not regulated by the U.S. Food and Drug Administration (FDA)

Nursing staff members are sitting in the lounge taking their morning break. An unlicensed assistive personnel (UAP) tells the group that she thinks that the unit secretary has acquired immunodeficiency syndrome (AIDS) and proceeds to tell the nursing staff that the secretary probably contracted the disease from her husband, who is supposedly a drug addict. Which legal tort has the UAP violated? 1. Libel 2. Slander 3. Assault 4. Negligence

2. Slander Rationale: Defamation is a false communication or a careless disregard for the truth that causes damage to someone's reputation, either in writing (libel) or verbally (slander). An assault occurs when a person puts another person in fear of a harmful or an offensive contact. Negligence involves the actions pf professionals that fall below the standard of care for a specific professional group. Test-Taking Strategy: Note the SUBJECT, the legal tort violated. Focus on the data in the question and eliminate options 1 and 4 first because their definitions are unrelated to the data. Recalling that slander constitutes verbal defamation will direct you to the correct option from the remaining options.

A nurse is planning to help move a patient up in bed. What can the nurse do to reduce the risk of self-strain when performing this action? 1. Move the patient up against gravity 2. Use the large muscles of the legs 3. Bend the body from the waist 4. Keep the knees locked

2. To exert an upward lift the gluteal and leg muscles should be used, rather than the sacrospinal muscles of the back. These larger muscles fatigue less quickly, and their use protects the intervertebral disks.

A nurse must transfer a patient from a bed to a chair using a mechanical lift. What should the nurse do? 1. Ensure that there is a practitioner's order to move the patient using this device 2. Hook the longer straps on the end of the sling closest to the patient's feet 3. Place a sheepskin inside the sling so that it is under the patient 4. Lead with the patient's feet when exiting the bed

2. The longer straps/chains go in the holes for the seat support, which keep the legs and pelvis below the upper body. Appropriate placement of the upper and lower straps/chains creates a bucket seat in which a patient is moved safely.

A patient is experiencing lack of sleep because of pain. Which is the most appropriate goal for this patient? 1. The patient will be provided with a back massage every evening before bedtime. 2. The patient will report feeling rested after awakening in the morning. 3. The patient will request less pain medication during the night. 4. The patient will experience four hours of uninterrupted sleep.

2. The patient will report feeling rested after awakening in the morning. Sleep is a sensory experience that restores cerebral and physical functioning. Evaluations related to sleep are based on patient reports because effectiveness of sleep is a subjective assessment.

A school nurse is teaching children about fire safety procedures. What is the first thing they should be taught to do if their clothes catch fire? 1. Yell for help 2.Roll on the ground 3.Take their clothes off 4.Pour water on their clothes

2.Roll on the ground

An unconscious patient begins to vomiting. In what position should the nurse place the patient? 1. Supine 2.Side lying 3.Orthopneic 4. Low Fowler

2.Side lying

A nurse plans to use a trochanter roll when repositioning a patient. Where should the nurse place the trochanter roll? 1. Under the small of the back 2. Behind the knees when supine 3. Alongside the ilium to mid-thigh 4. In the palm of the hand with the fingers flexed

3

A patient who has a history of schizophrenia and is hallucinating is brought to the emergency department of a community hospital. The primary health care provider in charge refuses to admit the patient. The nurse attempts to educate the primary health care provider regarding the law involving patients who are gross disabled. The primary health care provider still refuses to admit the patient. What is the nurses next action? 1. Admit the patient against the primary health care providers wishes 2. Refuse to discharge the patient as per the primary health care providers orders 3. Describe the primary health care providers behavior to the nursing supervisor and ask for guidance 4. Call the primary health care providers supervisor and explain that the health care provider is acting inappropriately

3

A patient with impaired mobility is to be discharged within a week from the hospital. Which is the best example of a discharge goal for this patient? The patient will: 1. Understand range-of-motion exercises 2. Be taught range-of-motion exercises 3. Transfer independently to a chair 4. Be kept clean and dry

3

An immobilized bed-bound patient is placed on a 2-hour turning and positioning program. What should the nurse explain to the patient as to why this program is important? 1. Support comfort 2. Promote elimination 3. Maintain skin integrity 4. Facilitate respiratory function

3

Mrs. Field fell asleep while smoking in bed and drops a burning cigarette on her blanket. When she awakens, her bed is on fire, and she quickly calls the nurse. On observing the fire, the nurse should immediately: 1. Report the fire 2. Attempt to extinguish the fire 3. Assist Mrs. Field to a safe place 4. Close all windows and doors to contain the fire

3

Question 1. A patient has a wound that is healing by secondary intention. To best support healing of the wound, the nurse should expect the practitioner's order to state, "Clean wound with: 1. Betadine and apply a dry sterile dressing." 2. Normal saline and cover with a gauze dressing." 3. Normal saline and apply a wet-to-damp dressing." 4. Half peroxide and half normal saline and apply a wet-to-dry dressing."

3

Question 10. A bed-bound older adult has a stage I pressure ulcer in the sacral area. In which position should the nurse place the patient to relieve pressure and promote circulation to the sacral area? 1. Dorsal recumbent 2. Semi-Fowler 3. Lateral 4. Supine

3

Question 12. A nurse is assessing a patient who has a wound on the leg as the result of a bicycle accident. Which clinical manifestation indicates a localized inflammatory response? 1. Body temperature of 101.4°F 2. Heart rate of 102 beats/minute 3. The area around the wound is swollen 4. Exudate from the wound is greenish yellow in color

3

Question 13. The primary reason why the nurse should avoid glued-on artificial nails is because they: 1. Interfere with dexterity of the fingers 2. Could fall off in a patient's bed 3. Harbor microorganisms 4. Can scratch a patient

3

Question 16. When changing an abdominal dressing, the nurse identifies that the incision has separated and several loops of the patient's bowel are protruding from the wound. What should the nurse do first? 1. Take the patient's vital signs. 2. Notify the health-care provider immediately. 3. Cover the wound with a sterile saline dressing. 4. Place the patient in the Trendelenburg position.

3

Question 17. A primary health-care provider orders DuoDERM, a hydrocolloid dressing, for a patient with a wound that has a small amount of drainage. What should the nurse do when applying this dressing? 1. Stretch the dressing while placing it over the wound. 2. Apply tape around all 4 sides of the dressing after its application. 3. Warm the dressing between the hands before applying the dressing. 4. Ensure a ¼-inch margin of healthy skin around the wound when sizing the dressing.

3

Question 18. A nurse is caring for a group of patients experiencing various medical conditions. The patient with which condition is at the greatest risk for a wound infection? 1. Surgical creation of a colostomy 2. First-degree burn on the back 3. Puncture of the foot by a nail 4. Paper cut on the finger

3

Question 20. A nurse is caring for several patients. Which patient is at greatest risk for skin breakdown? 1. Patient who is dehydrated 2. Patient who has diaphoresis 3. Patient who is incontinent of feces 4. Patient who has difficulty moving up in bed

3

Question 4. A patient has a wound that is healing by secondary intention. The nurse is using sterile technique to irrigate the wound before applying a wet-to-damp dressing. What step of this procedure is required to maintain sterile technique? 1. Wipe the outside edges of the wound first. 2. Wash the hands again after setting up the sterile field. 3. Direct the flow of sterile irrigating solution to the inside top of the wound. 4. Place the irrigating solution on the side of the field farthest from the patient.

3

Question 8. A nurse is caring for a patient with a pressure ulcer. Which type of stressor is a pressure ulcer? 1. Microbiological 2. Developmental 3. Physiological 4. Physical

3

Sleep apnea is classified by how many types? Record your answer using a whole number. _________

3

Which is the most important nursing action when assisting a patient to move from a bed to a wheelchair? 1. Lowering the bed to 2 inches below the height of the patient's wheelchair 2. Applying pressure under the patient's axillae areas when standing up 3. Letting the patient help as much as possible when permitted 4. Keeping the patient's feet within 6 inches of each other

3

Which of the following would most immediately threaten an individual's safety? 1. 70% humidity 2. A sprained ankle 3. Lack of water 4. Unrefrigerated fresh vegetables

3

Which situation should the nursing staff refer to the hospital ethics committee? 1. The adult patient with renal insufficiency who refuses dialysis 2. The wife who wants her unconscious husband to have a drug that is available in a phase three trial 3. The parents of a developmentally disabled adult who disagree with the health care decisions of the court appointed guardian. 4. The adult patient recently diagnosed with metastatic cancer who wants hospice care but whose family want chemotherapy to be given immediately

3

A student nurse is gossiping with colleagues about a patient's many romantic relationships. The registered nurse warns the student nurse about talking about the patient's personal life in a public place. The nurse's warning is intended to prevent which error? 1 Assault 2 Libel 3 Invasion of privacy 4 Defamation of character

3 The registered nurse reprimands the student nurse for speaking in public about the patient's private life, because it is an invasion of the patient's privacy. Assault is intentional bodily harm caused by the perpetrator, but gossiping does not cause physical harm to the patient. The nurse is not committing libel, which is written defamation of character. Defamation of character is defined as making false public statements about a person, but there is no indication that the nurse is making false statements, just that the information is private.

9. A primary health-care provider writes the order Patient may shower. When preparing the patient for the shower, the nurse determines that the patient lacks the strength to tolerate standing for this procedure. Which should the nurse do? 1) Give a complete bed bath 2) Assist the patient into a bathtub 3) Use a commode chair in the shower 4) Place the patient in a chair at the sink

3 A commode chair or tub/shower chair is a water-resistant chair that allows the patient to sit safely during a shower. It permits a thorough cleansing and minimizes the physical demands on the patient, conserving the patient's energy.

28. A nurse educator is preparing a health curriculum for high school students. Which content should be included in the program that addresses the leading cause of physical injury during adolescence? 1. Driver education 2. Anger management 3. Alcohol and drug use 4. Irresponsible water safety Hint

3 Alcohol and drug use is the root cause of most physical injuries that occur in adolescence. Alcohol and drugs impair judgment, leading to reckless driving (leading cause of death in adolescence); fuels anger, which can lead to homicide (second leading cause of death in adolescence); and irresponsible behavior regarding water sports (drowning and paralysis).

26. A nurse just finished a complete bath for a patient with limited mobility. Which action is most important for the nurse to perform before leaving the bedside of the patient? 1. Raise all of the bedside rails. 2. Ensure that the water pitcher has fresh water. 3. Lower the height of the bed to the lowest position. 4. Position the bedpan in easy reach under the covers. Hint

3 All patients should have their beds maintained in the lowest position unless a member of the health-care team is providing direct care. This supports patient safety. It enables mobile patients to more easily enter and exit the bed. Also, if a patient should fall out of bed accidentally, the bed is closer to the floor, which may minimize patient injury.

A patient has just been diagnosed with hepatitis B. Which is the most appropriate nursing action to avoid invasion of privacy? 1 Inform the patient's spouse about the disease. 2 Do not inform the patient or the family members about the disease. 3 Obtain permission from the patient to inform the patient's spouse about the disease. 4 Obtain informed consent from the spouse to reveal the illness to the patient.

3 Hepatitis B is a sexually transmitted disease, and the patient may be unwilling to reveal the information about the disease to the spouse. Thus, the nurse should request consent from the patient to let the spouse know about the illness. The patient's spouse should not be informed about the illness without the patient's consent. The nurse should abide by the patient's right to be informed about the disease. The nurse must obtain the patient's consent to inform family members about the disease. Obtaining informed consent from the spouse to reveal the illness to the patient is an inappropriate nursing action.

17. A patient who abuses alcohol is admitted to the hospital for detoxification. The patient is intoxicated and agitated. The patient's agitation increases and the patient becomes verbally abusive to the nurse. What should the nurse do next? 1. Obtain an order for a physical restraint for the patient. 2. Offer emotional support while touching the patient's hand. 3. Stand between the patient and the exit to the room while talking with the patient. 4. Approach the patient's bedside and state that using bad language is unacceptable. Hint

3 Increasing agitation and verbal abuse may be the precursors to physical abuse. The nurse should stand in an area that will provide an escape if the patient attempts physical aggression.

24. Which nursing action is most important when ambulating a patient with a gait belt? 1. Position yourself slightly in front of and next to the patient when ambulating a patient with a gait belt. 2. Adjust the belt so that no fingers can be inserted between the belt and the patient's waist. 3. Assess for activity intolerance while ambulating a patient with a gait belt. 4. Hold the gait belt in the middle of the patient's back. Hint

3 Patients who need a gait belt when being ambulated generally have impaired balance or are weak and, therefore, must be assessed for clinical indicators of activity intolerance. It is not acceptable to just make this assessment after the completion of ambulation.

33. What safety instruction addresses a major cause of accidental death among people from all developmental levels? 1. Resist pressure to engage in high-risk activities. 2. Rise slowly to a standing position. 3. Wear a seat belt when in a car. 4. Cut all food into small pieces. Hint

3 The leading cause of accidental death in infants, toddlers, preschoolers, school-aged children, adolescents, and adults is motor vehicle accidents. Motor vehicle accidents are one of the leading causes of injuries in older adults after falls. Seat belts reduce the incidence of morbidity and mortality.

13. A nurse is teaching a group of older adults about safety precautions. What instruction should the nurse give them to prevent the leading cause of injuries in older adults? 1. Have your vision evaluated yearly. 2. Limit drinking alcohol to one glass a day. 3. Rise slowly when moving from a sitting to a standing position. 4. Wash your hands with an antimicrobial soap several times a day. Hint

3 The leading cause of injuries in older adults is falls that usually occur when moving from sitting to standing or when transferring from a bed to a chair.

52. An older adult who has left-sided weakness as the result of a brain attack (stroke, cerebral vascular accident) has the need to urinate several times during the night. What should the nurse do to ensure patient safety? 1. Encourage the use a bedpan at night. 2. Keep the light on in the bathroom at night. 3. Assist the patient to use a commode at night. 4. Encourage the patient to use a walker at night.

3 The nurse should assist patients who are unstable or have impaired mobility. A commode at the bedside limits the energy the patient needs to expend at night when the risk for falls escalates.

The nurse notes that the healthcare unit keeps a listing of the patient names at the front desk in easy view for healthcare providers to more efficiently locate the patient. Which does this action violate? 1 Mental Health Parity Act (MHPA) 2 Patient Self-Determination Act (PSDA) 3 Health Insurance Portability and Accountability Act (HIPAA) 4 Emergency Medical Treatment and Active Labor Act (EMTALA)

3 The privacy rule of the Health Insurance Portability and Accountability Act (HIPAA) requires patient information to be protected from unnecessary publication. The Mental Health Parity Act (MHPA) addresses the needs of the mental health patient, the Patient Self-Determination Act (PSDA) addresses a patient's right to refuse treatment and formulate advance directives, and the Emergency Medical Treatment and Active Labor Act (EMTALA) ensures appropriate screening and stabilization of the patient in an emergency setting.

After a central venous catheter is inserted into a patient's subclavian vein, the patient has severe dyspnea. Which should the nurse assess first? 1) Heart rate 2) Anxiety level 3) Breath sounds 4) Blood pressure

3 The subclavian vein is in proximity to the apex of the lung. If the central venous catheter inadvertently enters the lung, it will cause it to collapse (pneumothorax), resulting in severe shortness of breath (dyspnea), chest pain, cough, hypotension, tachycardia, and anxiety. Auscultation over the site of the collapsed lung will reveal an absence of breath sounds.

10. A nurse is teaching a class to parents of toddlers about how to reduce the risk of suffocation, asphyxiation, or drowning. What information is essential to include in this program? 1. Ensure that slats of cribs are more than 2⅜ inches wide. 2. Provide toys that are larger than 2 inches in diameter. 3. Keep venetian blind cords out of reach. 4. Use only light blankets when sleeping. Hint

3 This is essential to protect a toddler from accidentally causing self-strangulation.

44. A public health nurse is visiting a patient recently discharged from the hospital to the home. The patient has a history of violence. What is most important for the nurse to do when caring for this patient? 1. Apply restraints if the patient engages in threatening behavior. 2. Administer a prescribed sedative before providing care. 3. Identify when the level of anxiety is elevating. 4. Use gentle touch communicating acceptance.

3 This is the most important nursing action. Assessment is the first step of the nursing process. A patient with a history of violence may resort to violence when coping with anxiety. As soon as a nurse identifies that the patient's level of anxiety is elevating, the nurse should leave and return when the patient is less anxious.

A patient has been locked in a poorly ventilated room containing a furnace. Which condition is likely to be observed in the patient after a period of time? 1 Psychological injury 2 Orientation changes 3 Carbon monoxide poisoning 4 Decrease in core body temperature

3 A furnace, stove, or fireplace that is not properly vented introduces carbon monoxide into the environment and affects the person's oxygenation by binding with hemoglobin. A furnace in a poorly ventilated room would not lead to psychological injury. Sensory or communication impairments associated with delirium, dementia, and depression contribute to altered concentration and orientation changes. A decrease in core body temperature is observed in patients with hypothermia; the patient in this instance would be exposed to elevated temperatures, not low temperatures.

The nursing instructor is teaching a student nurse about using a patient-controlled analgesia (PCA) pump. Which intervention does the student nurse follow to establish the route of medication and rapid administration of the medication? 1 Administer the loading dose of analgesia as prescribed. 2 Attach the drug reservoir to the infusion device and prime the tubing. 3 Insert and secure a needleless adapter into the injection port nearest the patient. 4 Attach a needleless adapter to the tubing adapter of the patient-controlled module.

3 A needleless adapter is inserted into the injection port nearest the patient to establish the route of medication and facilitate continuous delivery of the medication. The nurse administers the loading dose of analgesia as prescribed by giving one-time doses manually or programming it into the PCA pump. Attaching the drug reservoir to the infusion device and prime tubing locks the system and prevents air from infusing into the intravenous (IV) tubing. Attaching a needleless adapter to the tubing adapter of a patient-controlled module is done to connect with the IV line. It does not facilitate continuous delivery of the medication.

A patient is in the first postoperative day following a nephrectomy. The patient is receiving morphine through a patient-controlled analgesia (PCA) device for management of pain. The patient complains of pain in the shoulders. The nurse understands that it is a referred pain. What explanation should the nurse give to the patient regarding the referred pain? 1 It is a pain that occurs sporadically over time. 2 It is a moderate pain that occurs for more than 6 months constantly. 3 It is a pain that is sensed at a site away from its actual origin or pathology. 4 It is neuropathic pain that is caused generally after cancer or a tumor.

3 A pain that is sensed at a site away from its actual origin or pathology is known as referred pain. A pain that occurs sporadically over time is known as chronic episodic pain. A moderate pain that occurs constantly for more than 6 months is known as chronic or persistent noncancerous pain. A cancer pain is neuropathic pain that is caused generally after cancer or a tumor.

A registered nurse is teaching a nursing student about various nonpharmacological pain management interventions. Which of the nursing student's statements indicates a need for further teaching? 1 "Biofeedback can help change a patient's perception of pain." 2 "Music therapy can be used in combination with pharmacological measures." 3 "Guided imagery provides effective pain relief for a patient who has acute appendicitis." 4 "Therapeutic touch is a complementary and alternative medicine pain relief method."

3 Acute pain cannot be effectively managed by nonpharmacological pain management interventions alone, so the nursing student requires further teaching to understand that guided imagery alone will be inadequate for a patient experiencing acute appendicitis. The remaining statements indicate understanding. Cognitive-behavioral interventions like biofeedback can change a patient's perception of pain. Any nonpharmacological intervention like music therapy can be used in combination with pharmacological interventions to provide pain relief. Therapeutic touch is a complementary and alternative pain relief method.

A nurse is teaching a class on home safety to a group of parents of children younger than 4 years of age. Which intervention is essential that the nurse should include in the class? 1. Cut up firm foods into half inch pieces. 2. Ensure that crib slats are greater than three inches apart. 3. Supervise balloon play and discard burst balloons immediately. 4. Monitor a young child eating marshmallows, popcorn, or grapes.

3 Balloon play should be supervised. Latex balloons and plastic bags are the main cause of suffocation deaths in young children.

A registered nurse (RN) is teaching a parent about safety measures for a 5-year-old child. Which statement made by the parent indicates the need for further learning? 1 "I should teach the child about bicycle safety." 2 "I should teach the child about safety during specific sports." 3 "I should teach the child how to operate electrical equipment." 4 "I should teach the child about safe use of equipment for play and work."

3 Children should be taught not to operate electrical equipment, because an electrical mishap may occur. Bicycle safety should be taught to children to reduce injuries from falling. Safety during sports should be taught to children to protect them from injuries. The children should be taught about safe and appropriate used of equipment to avoid injuries.

Which type of pain management is cold application? 1 Relaxation 2 Distraction 3 Cutaneous stimulation 4 Acupressure

3 Cold application stimulates the skin, which helps reduce pain perception, perhaps by releasing endorphins or activating large, fast-transmitting A-beta sensory nerve fibers. Relaxation techniques include meditation, yoga, guided imagery, and progressive relaxation exercises. Distraction works by diverting the patient's attention to something other than pain, thus reducing awareness of it. Acupressure involves the application of pressure, not cold.

A patient who is in the terminal stages of liver cancer reports continuous vomiting after taking oral opioid analgesics. The patient's weight is 85 pounds. The nurse applies a transdermal fentanyl patch to the patient. The next day, the patient informs the nurse that the pain is not alleviated. What could be the possible reason for this? 1 The dose of pain medication is not enough. 2 The number of patches used is not enough. 3 The route of administration of the analgesic is not correct. 4 The patient needs to wait longer for the medication to act.

3 Fentanyl is an opioid analgesic and is available for intravenous or transdermal administration. It is 100 times more potent than morphine. However, transdermal patches are not effective in patients weighing less than 100 pounds, because these patients have very little subcutaneous tissue for absorption. Therefore, the nurse should discuss a more appropriate analgesic drug with the primary health care provider. The dose and the number of patches for the therapeutic action are predetermined. The duration of drug action is about 48 to 72 hours.

Which recommendation would be appropriate to reduce the risk of falls in a patient with hemiparesis? 1 "You should perform range-of-motion exercises." 2 "You should make use of coping skills that you have previously used." 3 "You should consult an ophthalmologist for a visual assessment." 4 "You should touch one side of your body frequently with the other hand."

3 Hemiparesis is the condition in which there is weakness on one side of the body. Consulting an ophthalmologist for visual assessment will prevent the risk of fall in a hemiparesis patient. Performing range-of-motion exercises will be helpful for a patient with impaired physical mobility. Encouraging the patient to make use of coping skills that he or she has previously used is helpful for a patient with anxiety related to falls. A patient with unilateral neglect related to brain injury will benefit from frequently touching the left side of the body with the right hand.

The nurse is instructing the mother of an infant not to leave the mesh sides of playpens lowered. Which risk can be prevented by this intervention? 1 Falls 2 Choking 3 Asphyxiation 4 Strangulation

3 If mesh sides of playpens are lowered, the possibility exists for a child's head to become wedged in the lowered mesh side and may result in asphyxiation. Falls in infants and toddlers can be prevented by instructing the mother not to leave crib sides down or babies unattended on changing tables or in infant seats. Choking can be prevented by avoiding the use of pacifiers or ribbons attached to the string around the child's neck. Strangulation can be prevented by avoiding pillows, bumper pads, large stuffed toys, or comforters in the cribs.

The registered nurse (RN) is teaching a nursing student about interventions that should be performed in the event of an accidental poisoning. Which statement of the nursing student indicates a need for further teaching? 1 "I will identify the type and amount of the poisonous substance ingested by the victim." 2 "I will irrigate the skin or eye with copious amounts of cool tap water for 15 to 20 minutes." 3 "I will induce vomiting if the victim has ingested furniture polish, grease, or petroleum products." 4 "I will call 911 if the victim is collapsed and initiate CPR if indicated until emergency personnel arrive."

3 In cases in which the victim has ingested furniture polish, grease, or petroleum products, vomiting should never be induced. This may further complicate the victim's condition. In cases of poisoning in the skin or eye, irrigating the skin or eye with copious amounts of cool tap water for 15 to 20 minutes. Identifying the type and amount of substance ingested will help in determining the correct type and amount of antidote needed for treatment. In case the victim has collapsed, 911 should be called immediately. If cardiopulmonary resuscitation is indicated, it should be initiated and performed until emergency personnel arrive.

The registered staff nurse provides various instructions to the caregiver of an infant before discharge from the hospital. Which instruction promotes safety for the infant? 1 Provide pillows and a comforter in the crib. 2 Place the infant in a prone position to sleep. 3 Install deadbolts on exterior doors above the child's reach. 4 The spaces between crib slats need to be 10 cm.

3 Installing deadbolts on exterior doors above the child's reach prevents a crawling infant or toddler from leaving the house and wandering off. Pillows, comforters, or large stuffed toys should not be placed in a crib, because an infant can get twisted in them and suffocate. Infants should sleep on their back or side to help prevent sudden infant death syndrome (SIDS). The spaces between crib slats should be less than 6 cm apart because there is a possibility for a child's head to become wedged between them, and asphyxiation may occur.

The patient complains of intermittent back pain that travels down the left leg. What is this type of pain called? 1 Visceral 2 Referred 3 Radiating 4 Superficial

3 Intermittent or constant pain that travels down or along a body part is called radiating pain. Deep or visceral pain results from the stimulation of internal organs. It is diffuse and radiates in several directions. Referred pain is in a part of the body separate from the source of pain. Superficial pain is of short duration and is localized.

The registered nurse is teaching a group of student nurses about precautions to be taken when using oxygen tanks in a hospital. Which statement made by a student nurse indicates a need for further teaching? 1 "Check the oxygen tank's tubing for kinks." 2 "Post 'No Smoking' signs in patient rooms containing oxygen tanks." 3 "Place the oxygen tanks in an upright position on the floor when not in use." 4 "Take the primary healthcare provider's advice while changing the liter flow of oxygen."

3 Oxygen tanks should be secured so they don't fall over, possibly damaging the tank. The tanks should not be placed upright on the floor when not is use; they can instead be placed flat on the floor or placed in an upright position in stands. The nurse should check for kinks in the tubing to promote the effective flow of oxygen through the tube. The contact of oxygen with heat or a spark can cause combustion. Therefore, the nurse should post "No Smoking" signs in patient rooms containing oxygen tanks. The nurse should change the liter flow of oxygen only after obtaining permission from the primary healthcare provider.

Which event is classified as an environmental event on the National Quality Forum list? 1 Patient death due to physical assault in the health care facility 2 Patient death due to contaminated drugs in the health care facility 3 Patient death associated with a fall during care in the health care facility 4 Patient death due to spinal manipulative therapy in the health care facility

3 Patient death associated with a fall in the health care facility is classified as an environmental event on the National Quality Forum list. Patient death due to physical assault in a health care facility is reportable under criminal events. Patient death due to contaminated drugs in a health care facility is reportable as a product or device event. Patient death due to spinal manipulative therapy in health care facility is reportable under care management events.

The nurse is performing fall prevention measures for a patient. During which step of the nursing process does the nurse perform "Timed Get up and Go" (TUG) if a patient is able to ambulate? 1 Planning 2 Evaluation 3 Assessment 4 Implementation

3 Performing TUG when the patient is able to ambulate is included in the assessment step of the nursing process. Planning involves gathering equipment to promote organization and performing hand hygiene to reduce the transmissions of microorganisms. Evaluation is the basic step involved when the nurse is performing visual checks in a patient. Implementation involves adjusting the bed to a proper height and inspecting the area to prevent injuries during restraint application.

Which safety precaution performed by a parent lowers the risk of sudden infant death syndrome (SIDS) in a 1-year-old child? 1 Immunizing the infant 2 Filling crib with pillows 3 Having the infant sleep on his/her side or back 4 Attaching pacifier to string and placing it around the child's neck

3 Placing infants on their sides or backs confers the lowest risk of SIDS and it is the preferred position. Immunizing the child at an early age prevents the risk of several life-threatening complications. The parent should not fill the crib with pillows or bumper pads because these items increase the risk of suffocation or entrapment. String or ribbon around the neck increases the risk of choking.

5. A patient has a urinary retention catheter and an IV, and is receiving oxygen via nasal cannula. Which should the nurse do first when turning and positioning this patient? 1) Obtain the assistance of an additional caregiver 2) Enlist the assistance of the patient, if possible 3) Ensure the tubes are safely positioned 4) Document the patient care provided

3 Positioning the tubes safely is essential to prevent their accidental dislodgment or removal.

A student nurse is reading about the mode of action of nonsteroidal anti-inflammatory drugs (NSAIDS). The NSAID drug decreases the level of a chemical that is known to increase pain sensitivity. With which chemical does the NSAID react? 1 Renin 2 Serotonin 3 Prostaglandin 4 Diclofenac sodium

3 Prostaglandins are generated from the breakdown of phospholipids of the cell membrane and are known to increase pain sensitivity. NSAIDs act by decreasing the levels of such compounds in the blood. Renin is involved in balancing water and electrolytes in the body. Neurotransmitters such as serotonin inhibit the transmission of pain. Diclofenac sodium is a painkiller that reduces pain sensitivity.

A patient has been having seizures for more than 30 minutes. The nurse looks after the patient and implements the best efforts to keep the patient safe. Which nursing intervention may lead to complications in the patient? 1 Calling a rapid response team 2 Notifying a health care provider 3 Restraining the patient to prevent injuries 4 Maintaining the airway and administering oxygen

3 Seizures persisting beyond 30 minutes indicate status epilepticus, which is a medical emergency. Restraining the patient further aggravates the injuries during an active episode and should be avoided. Status epilepticus is managed by calling a rapid response team or code blue and notifying a health care provider. The nurse should also take the necessary steps to maintain the airway. If oxygen saturation has dropped to a critical level, then oxygen should be administered.

The nurse is teaching the parent of young child about car safety. Which statement by the parent indicates effective learning? 1 "I should secure my 6-month-old child in a forward-facing car seat." 2 "When my child is 1 year old, I can place his car safety seat in the front seat." 3 "I should place my 3-month old child in the back seat with a rear-facing convertible seat." 4 "I should not place my child in a front-facing seat before the age of 2."

3 The American Academy of Pediatrics (AAP) recommends that all infants and toddlers ride in the back seat with a rear facing only seat or rear facing convertible seat until the age of 2 or when the child reaches the highest weight or height allowed by the manufacturer of the rear-facing safety seat. A 1-year-old child should not be allowed to sit in the front seat. The child should be allowed to use a front-facing seat at the age of 2 or when the child reaches the highest weight or height allowed by the manufacturer of the rear-facing safety seat. The child can be placed in a front-facing seat before the age of 2 if the child has outgrown the rear-facing seat.

Which topical analgesic is effective for relieving postherpetic neuralgia in adults? 1 Capsaicin 2 ELA-Max/LMX 3 Lidoderm patch 4 Eutectic mixture of local anesthetics (EMLA)

3 The Lidoderm patch is effective for treating postherpetic neuralgia, a cutaneous neuropathic pain. Capsaicin is more appropriate for relieving minor aches and pains of the muscles and joints. ELA-Max/LMX and a eutectic mixture of local anesthetics (EMLA) are more often used to treat children.

Which is a serious reportable event included in the National Quality Forum list? 1 Discharging a patient with a cardiac condition 2 Severe bleeding while removing a foreign object during surgery 3 Disability associated with electric shock during care in a health care facility 4 Bruising associated with the use of bed rails during care in the health care facility

3 The National Quality Forum has compiled a list of serious reportable events that are a major focus of health care providers for patient safety initiatives. Events such as disability or death associated with electric shock while caring for a patient in a health care facility should be reported immediately. Discharging a cardiac patient is not a reportable event. Severe bleeding while performing a surgery is manageable and not a reportable event. Bruising is not a serious complication. Therefore, bruising associated with the use of bed rails during care in the health care facility is not considered a serious reportable event.

The nurse asks a patient to rate his pain from no pain to unbearable pain. Which pain rating scale is the nurse using for pain assessment? 1 Oucher scale t2 Numeric rating scale (NRS) 3 Visual analogue scale (VAS) 4 Verbal descriptor scale

3 The VAS assesses the pain level in the patient by rating the pain along a 10-centimeter line in 1-centimeter increments from no pain to unbearable pain. The Oucher scale requires the patient to look at six faces with different expressions and point at the face that best matches the pain he or she is experiencing. With the NRS, the nurse asks the patient to choose a number to rate the level of pain. With the verbal descriptor scale, the nurse asks the patient to describe his or her feelings about the intensity of pain.

The registered nurse (RN) is teaching a nursing student about safety for equipment-related accidents. Which statement by the nursing student indicates the need for further learning? 1 "I should use free flow protection devices." 2 "I should place a tag on faculty equipment." 3 "I will make regular safety checks of equipment." 4 "I should not operate the equipment without instructions."

3 The clinical engineering staff, not the nurse, should make regular safety checks of equipment. Free-flow protection devices should be used to avoid rapid infusion of intravenous fluids. When a piece of faculty equipment is found, a tag should be placed on it to prevent it from being used on another patient. Therapy equipment should not be monitored or operated without adequate instructions to avoid accidents. Equipment related accidents may result from malfunction, disrepair, or misuse of equipment or an electrical hazard.

The nurse is planning care for a ptinet with a wrist restraint. How often should a restraint be removed, the area massaged, and the joints moved through their full range? 1.Once a shift 2.Once an hour 3.Every 2 hours 4.Every 4 hours

3.Every 2 hours

The primary health care provider (PHP) prescribes a patient-controlled analgesia (PCA) pump to a postoperative patient for pain relief. The nurse reviews the PHP's order for the patient's name, the name of the medication, dose, frequency of medication, and lockout period. Why does the nurse perform this review? 1 To prevent medication errors 2 To ensure that the patient receives the correct medications 3 To ensure that the medication is administered safely 4 To avoid placing the patient at risk for allergic reactions

3 The nurse checks the computer printout with the PHP's order for the patient's name, the name of medication, dose, frequency of medication, and lockout period to ensure that the medication is administered safely. A second registered nurse confirms the PHP's order and correct setup of the PCA pump to prevent medication errors. To ensure that the patient receives the correct medications, the nurse checks the patient's prescription. The nurse checks the patient's history for drug allergies to avoid placing the patient at risk for allergic reactions.

The nurse is caring for an older adult in a health care setting and follows interventions to reduce the risk of tripping. Which intervention does the nurse implement in this situation? 1 Cleaning all spills promptly 2 Ensuring adequate glare-free lighting 3 Keeping the floor free of clutter and obstacles 4 Having assistive devices on the exit side of the bed

3 The nurse keeps the floor free of clutter and obstacles to reduce the risk of falling or tripping. The nurse cleans all spills promptly to reduce the risk of slipping on wet surfaces. The nurse ensures adequate glare-free lighting, becauseit may be a problem for older adults because of vision changes. The nurse keeps assistive devices on the exit side of the bed to provide added support when transferring out of bed.

A physician put a postoperative patient on a patient-controlled opioid analgesic pump to be used around the clock for a week. Which assessment should the nurse make at regular intervals? 1 Liver enzymes 2 Blood pressure 3 Respiratory rate 4 Body temperature

3 While there is little risk for overdose with patient-controlled analgesic pumps, respiratory depression is a side effect associated with opioids, so while the patient is on opioid pain management, the nurse should regularly check respiratory rate. A nurse may check liver enzymes in a patient who is taking acetaminophen, not opioids, because acetaminophen can adversely affect the liver. Whereas blood pressure and body temperature may be checked regularly, it is unlikely that the nurse is doing this to monitor for side effects of opioid pain management.

A patient who has alcoholism complains of disturbed sleep and fatigue. Which statements should the nurse include when teaching the patient? Select all that apply. 1 "Alcohol causes nightmares." 2 "Alcohol prevents the onset of sleep." 3 "Alcohol causes awakening from sleep." 4 "Alcohol causes difficulty returning to sleep." 5 "Alcohol reduces rapid eye movement (REM) sleep."

3 "Alcohol causes awakening from sleep." 4 "Alcohol causes difficulty returning to sleep." 5 "Alcohol reduces rapid eye movement (REM) sleep."

The nurse works on a medical-surgical unit. Which patients require further assessment to determine the risk of developing obstructive sleep apnea? Select all that apply. 1 The patient with arthritis 2 The patient with anemia 3 The patient with hypertension 4 The patient who is a chronic smoker 5 The patient who has type II diabetes

3 The patient with hypertension 4 The patient who is a chronic smoker 5 The patient who has type II diabetes

A nurse is caring for a patient who has an order for shortening a Penrose drain 1 inch daily. The nurse washes the hands, removes the soiled dressing, sets a sterile field, dons sterile gloves, and cleans around the drain with sterile saline solution as ordered. Place the following steps in the order in which they should be implemented by the nurse. 1. Complete dressing the wound. 2. Pull the drain out 1 inch gently and steadily. 3. Grip the Penrose drain with a pair of sterile forceps. 4. Remove the pin and replace it at the surface of the wound. 5. Cut off the excess drain using sterile scissors, ensuring that 2 inches remain outside the wound.

3 2 4 5 1

The nurse is sued for failure to monitor a patient appropriately after a procedure. Which statements are correct about this lawsuit? Select all that apply. 1 The nurse represents the plaintiff. 2 The defendant must prove injury, damage, or loss. 3 The person filing the lawsuit has the burden of proof. 4 The plaintiff must prove that a breach in the prevailing standard of care caused an injury. 5 The nurse is a witness

3 4 The patient as plaintiff must prove that the defendant nurse had a duty, breached the duty, and because of this breach caused the patient injury or damage. Thus, the plaintiff has the burden of proof. The nurse is the defendant, not a witness.

Which is a principle of proper body mechanics when lifting or carrying objects? 1 Keep the knees in a locked position. 2 Bend at the waist to maintain a center of gravity. 3 Maintain a wide base of support. 4 Hold objects away from the body for improved leverage

3 Maintain a wide base of support. Maintaining a wide base of support allows for proper body mechanics. Locking the knees or bending at the waist causes strain on the lower back. Holding objects close to the body helps use the center of gravity for leverage.

Which nonpharmacological techniques pose a risk of injury to the patient if the patient has a history of diabetic neuropathy? Select all that apply. 1 Yoga 2 Massage 3 Hot bath 4 Cold application 5 Relaxation exercises

3, 4 A patient who has diabetic neuropathy may not be able to adequately monitor temperature in areas affected by nerve damage, so application of any heat or cold may place this patient at a higher risk for injury. If done safely and properly, yoga, massage, and relaxation exercises should not place a patient with diabetic neuropathy at a higher risk for injury than any other patient.

The nurse is assessing the touch, pain, and temperature sensation of a patient who is diagnosed with diabetic neuropathy. Arrange in ascending order the parts of the central nervous system through which pain sensation is carried. 1. Cerebrum 2. Thalamus 3. Spinal cord 4. Medulla, pons, midbrain

3, 4, 2, 1 Spinal cord, Medulla, pons, midbrain, Thalamus, Cerebrum Pain sensation is transmitted from afferent fibers to the spinal cord. From the spinal cord, the pain sensation is carried to medulla, pons, and midbrain. From here it continues through the spinothalamic tract to the thalamus and then to the cerebrum.

Which nursing action would be appropriate to protect a patient during a seizure? Select all that apply. 1 Using supporting pillows for the patient who is on a bed 2 Guiding the patient from the floor to the bed during the seizure 3 Turning the patient to one side, having a slightly forwarded tilted head 4 Avoiding placing any objects into the mouth when the patient's teeth are clenched 5 Placing a pad under the patient's head after guiding them to the floor from a standing position

3, 4, 5 During a seizure, the nurse should turn the patient onto one side, and the patient's head should be slightly tilted forward. When the patient's teeth are clenched, objects should not be placed into the mouth. Before positioning the patient from standing to the floor, the nurse should place a pad under the patient's head. If the patient is in bed, the surrounding pillows should be removed. When the seizure is in progress, moving the patient from the floor to the bed would be inappropriate.

The risk management coordinator is preparing a program on the factors that contribute to falls in a hospital setting. Which factor that most often contributes to falls should be included in the program? 1. Wet floors 2.Frequent seizures 3.Advanced age of patients 4.Misuse of equipment by nurses

3.Advanced age of patients

The nurse attending to a postoperative patient finds that the patient's pain medications have been changed from morphine to ibuprofen. What are the possible reasons for the change in medication by the health care provider? Select all that apply. 1 The patient's pain has increased. 2 Morphine is known to cause seizures. 3 The patient experienced clinical respiratory depression. 4 Ibuprofen does not affect the central nervous system the way morphine does. 5 Ibuprofen does not interfere with bowel and bladder function.

3, 4, 5 Opioids (morphine) can cause respiratory depression in some patients who are not used to them. Secondly, unlike nonsteroidal anti-inflammatory drugs (NSAIDs) such as ibuprofen, opioids interfere with the activity of the central nervous system and affect the bowel and the bladder function. Opioids are generally prescribed when pain is severe. NSAIDs are useful for mild to moderate pain. Morphine is not known to cause seizures.

A professor is teaching a group of nursing students about patient-inherent accidents. Which are examples of patient-inherent accidents in the hospital setting? Select all that apply. 1 Medication administration error 2 Improper insertion of a urinary catheter 3 Ingestion of a foreign substance 4 Pinching fingers in drawers 5 Self-inflicted cuts

3, 4, 5 Patient-inherent accidents are accidents (other than falls) caused by the patient. Examples include self-inflicted cuts, injuries, and burns; ingestion or injection of foreign substances; self-mutilation or fire setting; and pinching fingers in drawers or doors. Medication administration error and improper insertion of urinary catheter are procedure-related accidents.

The nurse is admitting a 64-year-old patient who had a right hemisphere stroke and a recent fall. The spouse stated that the patient has a history of high blood pressure, which is controlled by an antihypertensive and a diuretic. Currently the patient exhibits left-sided neglect and problems with spatial and perceptual abilities and is impulsive. The patient has moderate left-sided weakness that requires the assistance of two nurses and the use of a gait belt to transfer the patient to a chair. The patient currently has an intravenous (IV) line and a urinary catheter in place. Which factors increase the patient's fall risk at this time? Select all that apply. 1 Smokes a pack a day 2 Used a cane to walk at home 3 Takes antihypertensive and diuretics 4 History of recent fall 5 Neglect, spatial and perceptual abilities, impulsive 6 Requires assistance with activity, unsteady gait 7 IV line, urinary catheter

3, 4, 5, 6, 7 Smoking is not a risk factor for falls. Using a cane at home is not a current risk factor for falls. Risk is determined by the patient's current status.

25. The nurse employed in a hospital is waiting to receive a report from the laboratory via the facsimile (fax) machine. The fax machine activates and the nurse expects the report, but instead receives a sexually oriented photograph. Which is the most appropriate nursing action? 1.Call the police. 2.Cut up the photograph and throw it away. 3.Call the nursing supervisor and report the incident. 4.Call the laboratory and ask for the individual's name who sent the photograph.

3. Ensuring a safe workplace is a responsibility of an employing institution. Sexual harassment in the workplace is prohibited by state and federal laws. Sexually suggestive jokes, touching, pressuring a co-worker for a date, and open displays of or transmitting sexually oriented photographs or posters are examples of conduct that could be considered sexual harassment by another worker. If the nurse believes that he or she is being subjected to unwelcome sexual conduct, these concerns should be reported to the nursing supervisor immediately. Option 1 is unnecessary at this time. Options 2 and 4 are inappropriate initial actions.

14. A client is brought to the emergency department by emergency medical services (EMS) after being hit by a car. The name of the client is unknown, and the client has sustained a severe head injury and multiple fractures and is unconscious. An emergency craniotomy is required. Regarding informed consent for the surgical procedure, which is the best action? 1.Obtain a court order for the surgical procedure. 2.Ask the EMS team to sign the informed consent. 3.Transport the victim to the operating room for surgery. 4.Call the police to identify the client and locate the family.

3. In general, there are two situations in which informed consent of an adult client is not needed. One is when an emergency is present and delaying treatment for the purpose of obtaining informed consent would result in injury or death to the client. The second is when the client waives the right to give informed consent. Option 1 will delay emergency treatment, and option 2 is inappropriate. Although option 4 may be pursued, it is not the best action.

17. The nurse who works on the night shift enters the medication room and finds a co-worker with a tourniquet wrapped around the upper arm. The co-worker is about to insert a needle, attached to a syringe containing a clear liquid, into the antecubital area. Which is the most appropriate action by the nurse? 1.Call security. 2.Call the police. 3.Call the nursing supervisor. 4.Lock the co-worker in the medication room until help is obtained.

3. Nurse practice acts require reporting impaired nurses. The board of nursing has jurisdiction over the practice of nursing and may develop plans for treatment and supervision of the impaired nurse. This incident needs to be reported to the nursing supervisor, who will then report to the board of nursing and other authorities, such as the police, as required. The nurse may call security if a disturbance occurs, but no information in the question supports this need, and so this is not the appropriate action. Option 4 is an inappropriate and unsafe action.

13. The nurse hears a client calling out for help, hurries down the hallway to the client's room, and finds the client lying on the floor. The nurse performs an assessment, assists the client back to bed, notifies the health care provider of the incident, and completes an incident report. Which statement should the nurse document on the incident report? 1.The client fell out of bed. 2.The client climbed over the side rails. 3.The client was found lying on the floor. 4.The client became restless and tried to get out of bed.

3. The incident report should contain the client's name, age, and diagnosis. The report should contain a factual description of the incident, any injuries experienced by those involved, and the outcome of the situation. The correct option is the only one that describes the facts as observed by the nurse. Options 1, 2, and 4 are interpretations of the situation and are not factual information as observed by the nurse.

Which statements by a patient indicate a precipitating factor associated with pain? Select all that apply. 1. "I usually feel a little dizzy and think I'm going to vomit when I have pain." 2. "My pain usually comes and goes throughout the night." 3. "I usually have pain after I get dressed in the morning." 4. "My pain feels like a knife cutting right through me." 5. "My incision hurts when I cough."

3. "I usually have pain after I get dressed in the morning." Anything that induces or aggravates pain is considered a precipitating factor of pain. For example, precipitating factors may be physical (e.g., exertion associated with activities of daily living, Valsalva maneuver), environmental (e.g., extremes in temperature, noise), or emotional (anxiety, fear). 5. "My incision hurts when I cough." Anything that induces or aggravates pain is considered a precipitating factor of pain. Coughing raises intra-abdominal pressure, which can aggravate the pain of a surgical incision. Patients are taught to support the operative site with the hands or a pillow when coughing to limit the extent of pain.

A primary health-care provider prescribes azithromycin "Z-Pak" for a patient with a diagnosis of chronic bronchitis. Which should the nurse teach the patient that is important to know about taking azithromycin? Select all that apply 1. "Take this medication with food" 2. "You can discontinue the medication as soon as you feel better" 3. "Take 500 mg on the first day and then 50 mg for 4 more days, for total of 1.5 g" 4. "The first dose should be taken after we notify you of the results of the culture and sensitivity" 5. "Avoid taking an antacid containing aluminum of magnesium within 2 hours of taking this medication"

3. "Take 500 mg on the first day and then 50 mg for 4 more days, total of 1.5 g" 5. "Avoid taking an antacid containing aluminum of magnesium with 2 hours of taking this medication."

A nurse is using the FLACC behavioral scale to assess an 8-month-old child's level of pain. The nurse identified that the patient's legs were drawn up to the abdomen and the patient was whimpering. The patient was squirming and shifting back and forth and had a constant frown and the chin was quivering. The infant was reassured when cuddled by the nurse. On a scale of 0 to 10, which is the child's level of pain? 1. 3 2. 5 3. 7 4. 9

3. 7 According to the FLACC behavioral scale to assess pain, the child's level of pain is 7. A constant frown with a quivering chin receives a score of 2. Legs drawn up to the abdomen receives a score of 2. Squirming and shifting back and forth receives a score of 1. Moaning and whimpering receives a score 1. Reassured by hugging receives a score of 1.

A nurse plans to teach a patient with hemiparesis to use a cane. What should the nurse teach the patient to do? 1. Move forward 1 step with the weak leg first followed by the strong leg and cane 2. Adjust the cane height 12 inches lower than the waist 3. Hold the cane in the strong hand when walking 4. Look at the feet when walking

3. A cane is a hand-gripped assistive device; therefore, the hand opposite the hemiparesis should hold the cane. Exercises can strengthen the flexor and extensor muscles of the arms and the muscles that dorsiflex the wrist.

What should the nurse do to best prevent a patient from falling? 1.Provide a cane 2.Keep walkways clear of obstacles 3.Assist the patient with ambulation 4.Encourage the patient to use the handrails in the hall

3.Assist the patient with ambulation

A nurse strains a back muscle when moving a patient up in bed. Which can the nurse do at home that utilizes the gate-control theory of pain relief to minimize the discomfort? 1. Use guided imagery. 2. Perform progressive muscle relaxation. 3. Apply a cold compress to the site for 20 minutes. 4. Take a nonsteroidal anti-inflammatory medication every 6 hours.

3. Apply a cold compress to the site for 20 minutes. Thermal therapy (e.g., application of heat or cold) stimulates the large A-delta fibers that close the gate that allows the transmission of pain impulses to the central nervous system.

The nurse employed in a hospital is waiting to receive a report from the laboratory via the facsimile (fax) machine. The fax machine activates and the nurse expects the report, but instead receives a sexually oriented photograph. Which is the MOST APPROPRIATE nursing action? 1. Call the police. 2. Cut up the photograph and throw it away. 3. Call the nursing supervisor and report the incident. 4. Call the laboratory and ask for the individual's name who sent the photograph.

3. Call the nursing supervisor and report the incident. Rationale: Ensuring a safe workplace is a responsibility of an employing institution. Sexual harassment in the workplace is prohibited by state and federal laws. Sexually suggestive jokes, touching, pressuring a co-worker for a date, and open displays of or transmitting sexually oriented photographs or posters are examples of conduct that could be considered sexual harassment by another worker. If the nurse believes that he or she is being subjected to unwelcome sexual conduct, these concerns should be reported to the nursing supervisor immediately. Option 1 is unnecessary at this time. Options 2 and 4 are inappropriate initial actions. Test-Taking Strategy: Note the strategic words MOST APPROPRIATE. Remember that using the organizational channels of communication is best. This will assist in directing you to the correct option.

The nurse who works on the night shift enters the medication room and finds a co-worker with a tourniquet wrapped around the upper arm. The co-worker is about to insert a needle, attached to a syringe containing a clear liquid, into the antecubital area. Which is the MOST APPROPRIATE action by the nurse? 1. Call security. 2. Call the police. 3. Call the nursing supervisor. 4. Lock the co-worker in the medication room until help is obtained.

3. Call the nursing supervisor. Rationale: Nurse practice acts require reporting impaired nurses. The board of nursing has jurisdiction over the practice of nursing and may develop plans for treatment and supervision of the impaired nurse. This incident needs to be reported to the nursing supervisor, who will then report to the board of nursing and other authorities, such as the police, as required. The nurse may call security if a disturbance occurs, but no information in the question supports this need, and so this is not the appropriate action. Option 4 is an inappropriate action and unsafe action. Test-Taking Strategy: Note the strategic words MOST APPROPRIATE. Eliminate option 4 first because this is an inappropriate and unsafe action. Recall the lines of organizational structure to assist in directing you to the correct option.

An immobilized bed-bound patient is placed on a 2-hour turning and positioning program. What should the nurse explain to the patient as to why this program is important? 1. Support comfort 2. Promote elimination 3. Maintain skin integrity 4. Facilitate respiratory function

3. Compression of soft tissue greater than 32 mm Hg prevents capillary circulation and compromises tissue oxygenation in the compressed area. Turning the patient relieves the compression of tissue in dependent areas, particularly those tissues overlying bony prominences.

Nurses should monitor for which systemic responses in immobilized patients? Select all that apply. 1. _____ Plantar flexion contracture 2. _____ Hypostatic pneumonia 3. _____ Dependent edema 4. _____ Muscle atrophy 5. _____ Pressure ulcer

3. Decreased calf muscle activity and pressure of the bed on the legs allow blood to accumulate in the distal veins. The resulting increased hydrostatic pressure moves fluid out of the intravascular compartment into the interstitial compartment, causing edema. 4. Atrophy is a decrease in the size of a tissue or an organ as a result of inactivity or decreased function. After 24 to 36 hours of inactivity, muscles begin to lose their contractile strength and begin the process of atrophy.

The nurse is developing a plan of care for a patient experiencing obstructive sleep apnea (OSA). Which intervention is appropriate to include on the plan? 1. Instruct the patient to sleep in a supine position. 2. Have patient limit fluid intake 2 hours before bedtime. 3. Elevate head of bed and assume a side or prone position. 4. Encourage the patient to take an over-the-counter sleep aid.

3. Elevate head of bed and assume a side or prone position.

Which is the most important nursing action when assisting a patient to move from a bed to a wheelchair? 1. Lowering the bed to 2 inches below the height of the patient's wheelchair 2. Applying pressure under the patient's axillae areas when standing up 3. Letting the patient help as much as possible when permitted 4. Keeping the patient's feet within 6 inches of each other

3. Encouraging the patient to be as self-sufficient as possible ensures that the transfer is conducted at the patient's pace, promotes self-esteem, and decreases the physical effort expended by the nurse.

Which concept associated with sleep should the nurse consider to plan nursing care for a hospitalized patient? 1. People require eight hours of uninterrupted sleep to meet energy needs. 2. Frequency of nighttime awakenings decreases with age. 3. Fear can contribute to the need to stay awake. 4. Bedrest decreases the need for sleep.

3. Fear can contribute to the need to stay awake. Fear of loss of control, the unknown, and potential death results in the struggle to stay awake, which interferes with the ability to relax sufficiently to fall asleep.

A nurse is transferring a patient from a bed to a wheelchair. What should the nurse do to quickly assess this patient's tolerance to the change in position? 1. Obtain a blood pressure 2. Monitor for bradycardia 3. Determine if the patient feels dizzy 4. Allow the patient time to adjust to the change in position

3. Feeling dizzy is a subjective response to orthostatic hypotension. Obtaining feed-back from the patient provides a quick evaluation of the patient's tolerance of the transfer.

A patient has a history of severe chronic pain. Which is the most important intervention associated with providing nursing care to this patient? 1. Asking what is an acceptable level of pain 2. Providing interventions that do not precipitate pain 3. Focusing on pain management intervention before pain is excessive 4. Determining the level of function that can be performed without pain

3. Focusing on pain management intervention before pain is excessive Administration of analgesics around the clock (ATC administration) at regularly scheduled intervals or by long-acting controlled-release transdermal patches maintains therapeutic blood levels of analgesics, which limit pain at levels of comfort acceptable to patients.

A nurse is caring for a patient who is having difficulty sleeping. Which patient responses indicate to the nurse that the patient is not obtaining adequate rapid-eye- movement (REM) sleep? Select all that apply. 1. Hyporesponsiveness 2. Immunosuppression 3. Irritability 4. Confusion 5. Vertigo

3. Irritability Rapid-eye-movement (REM) sleep is essential for maintaining mental and emotional equilibrium and, when interrupted, results in irritability and excitability. 4. Confusion REM sleep is essential for maintaining mental and emotional equilibrium and, when interrupted, results in confusion and suspiciousness.

A nurse is caring for a patient who is diagnosed with narcolepsy. Which is the most serious consequence of this disorder? 1. Inability to provide self-care 2. Impaired thought processes 3. Potential for injury 4. Excessive fatigue

3. Potential for injury Narcolepsy is excessive sleepiness in the daytime that can cause a person to fall asleep uncontrollably at inappropriate times (sleep attack) and result in physical harm to self or others.

Which concepts associated with rest and sleep must the nurse consider when planning nursing care? Select all that apply. 1. Energy demands increase with age. 2. Metabolic rate increases during rest. 3. Sleep requirements increase during stress. 4. Catabolic hormones increase during sleep. 5. Lack of awareness of the environment increases with sleep.

3. Sleep requirements increase during stress. Stress precipitates the sympathetic nervous system, increasing cortisone, norepinephrine, and epinephrine, which increase the metabolic rate. Physical and psychic energy expended is restored through rest and sleep. 5. Lack of awareness of the environment increases with sleep. Individuals experience varied levels of consciousness when asleep. There is a progressive lack of awareness of the environment as one passes from stages 1 through 4.

The nurse hears a client calling out for help, hurries down the hallway to the client's room, and finds the client lying on the floor. The nurse performs an assessment, assists the client back to bed, notifies the health care provider of the incident, and completes an incident report. Which statement should the nurse document on the incident report? 1. The client fell out of bed. 2. The client climbed over the side rails. 3. The client was found lying on the floor. 4. The client became restless and tried to get out of bed.

3. The client was found lying on the floor. Rationale: The incident report should contain the client's name, age, and diagnosis. The report should contain a factual description of the incident, any injuries experienced by those involved, and the outcome of the situation. The correct option is the only one that describes the facts as observed by the nurse. Options 1, 2, and 4 are interpretations of the situation that are not factual information as observed by the nurse. Test-Taking Strategy: Focus on the SUBJECT, documentation of events, and read the information in the question to select the correct option. Remember to focus on factual information when documenting, and avoid including interpretations. This will direct you to the correct option.

Which word is most closely associated with nursing care strategies to maintain functional alignment when patients are bed bound? 1. Endurance 2. Strength 3. Support 4. Balance

3. The line of gravity passes through the center of gravity when the body is correctly aligned; this results in the least amount of stress on the muscles, joints, and soft tissues. Bed-bound patients often need assistive devices such as pillows, sandbags, bed cradles, wedges, rolls, and splints to support and maintain the vertebral column and extremities in functional alignment.

Which is a primary reason why the nurse should avoid glued-on artifical nails? 1. They interfere with dexterity of the fingers 2. They could fall off in a patient's bed. 3. They harbor microorganisms. 4. They can scratch a patient.

3. They harbor microorganisms.

A patient with impaired mobility is to be discharged within a week from the hospital. Which is the best example of a discharge goal for this patient? The patient will: 1. Understand range-of-motion exercises 2. Be taught range-of-motion exercises 3. Transfer independently to a chair 4. Be kept clean and dry

3. This is a patient-centered goal and measurable.

A client is brought to the emergency department by emergency medical services (EMS) after being hit by a car. The name of the client is unknown, and the client has sustained a severe head injury and multiple fractures and is unconscious. An emergency craniotomy is required. Regarding informed consent for the surgical procedure, which is the BEST action? 1. Obtain a court order for the surgical procedure. 2. Ask the EMS team to sign the informed consent. 3. Transport the victim to the operating room for surgery. 4. Call the police to identify the client and locate the family.

3. Transport the victim to the operating room for surgery. Rationale: In general, there are two situations in which informed consent of an adult client is not needed. One is when an emergency is present and delaying treatment for the purpose of obtaining informed consent would result in injury or death to the client. The second is when the client waives the right to give informed consent. Option 1 will delay emergency treatment, and option 2 is inappropriate. Although option 4 may be pursued, it is not the best action. Test-Taking Strategy: Note the strategic word BEST. Recalling that when an emergency is present and a delay in treatment for the purpose of obtaining informed consent could result in injury or death will direct you to the correct option.

A 3 year old child is admitted to the pediatric unit. What is the best way for the nurse to maintain the safety of this preschool-age child? 1.Teaching the child how to use the call bell 2.Placing the child in a crib with high side rails 3.Keeping the child under constant supervision 4.Having the child stay in the playroom most of the day

3.Keeping the child under constant supervision

The nurse is planning care for a patient who requires bilateral arm restraints. Which information is important to consider when planning care for this patient? 1.Their use adequately prevents injuries 2.They require a practitioner's order to be applied 3.Reasons for their use must be clearly documented 4.Most patients recognize that they contribute to their safety

3.Reasons for their use must be clearly documented

Positioning to prevent pressure ulcers

30 degree lateral position; turn & rotate every 1-2 hours; head of bed elevated no more than 30 degrees; shift weight every 15 minutes while in wheelchair

A nurse is performing passive range-of-motion exercises for a patient who is in the supine position. Which motion occurs when the nurse bends the patient's ankle so that the toes are pointed toward the ceiling? 1. Supination 2. Adduction 3. Dorsal flexion 4. Plantar extension

4

A nurse is planning patient care based on moral and ethical principles. Which nursing statement demonstrates as effort to implement the principle of fidelity? 1."Lets talk about foods that are healthy and that you should include in your daily diet" 2."I know that you are out of work, so I have arranged for you to get follow-up care at our out-patient clinic" 3."You said that your son wants you to have this surgery. What is important is what you believe is best for you" 4."It's been half and hour and i am back as promised to ensure that the pain medication I gave you is providing relief"

4

A nurse is transferring a patient from the bed to a wheelchair using a mechanical lift. Which is a basic nursing intervention associated with this procedure? 1. Lock the base lever in the open position when moving the mechanical lift 2. Raise the mechanical lift so that the patient is six inches off the mattress 3. Keep the wheels of the mechanical lift locked throughout the procedure 4. Ensure the patient's feet are protected when on the mechanical lift

4

A patient with an order for bed rest has diaphoresis. What should the nurse use to best limit the negative effects of perspiration on dependent skin surfaces of this patient? 1. Ventilated heel protectors 2. Air-filled rings 3. Air mattress 4. Sheepskin

4

Question 14. A nurse assesses a patient and concludes that the patient is cachectic. For which skin integrity problem should the nurse determine that this patient is at the highest risk? 1. Altered tissue perfusion 2. Perineal excoriation 3. Reduced sensation 4. Pressure ulcers

4

Question 15. A nurse is caring for a patient who has an order for wound irrigation. What should the nurse do when implementing this procedure? 1. Use a piston syringe to forcefully cleanse debris from the wound. 2. Spray around the wound first, and then spray the inside of the wound from top to bottom. 3. Hold a piston syringe four inches from the wound surface when applying the solution. 4. Position the patient so that the irrigating solution will flow from the wound into an emesis basin.

4

Question 17. A patient has a wound infection. Which local human response should the nurse expect to identify? 1. Hyperthermia 2. Neutropenia 3. Malaise 4. Edema

4

Question 3. A nurse is caring for a postoperative patient who had abdominal surgery. What should the nurse do to help prevent postoperative wound dehiscence? 1. Keep the wound clean and dry. 2. Change the dressing every eight hours. 3. Medicate the patient for pain around the clock. 4. Encourage the patient to support the incision during activity.

4

Question 6. In what environment do bacteria rapidly multiply? 1. Hot 2. Cool 3. Cold 4. Warm

4

Question 9. What has the greatest impact on limiting the spread of microorganisms? 1. Disposable equipment 2. Double-bagging 3. Wearing gloves 4. Hand hygiene

4

The nurse is learning about various stages of nonrapid eye movement (NREM) sleep. One particular stage of NREM lasts for 15 to 30 minutes. It is the deepest stage of sleep, and it is very difficult to arouse the sleeper from this stage. The vital signs are lower than during normal waking hours. To which stage of NREM sleep is the nurse referring? Record your answer using a whole number. Stage __________

4

The nurse turns the palm of a patient's hand downward when performing range-of motion exercises. What word should the nurse use when documenting exactly what was done? 1. External rotation 2. Circumduction 3. Lateral flexion 4. Pronation

4

What do nurses sometimes do that increase their risk for injury when moving patients? 1. Use the longer, rather than the shorter, muscles when moving patients 2. Place their feet wide apart when transferring patients 3. Pull rather than push when turning patients 4. Misalign their backs when moving patients

4

Which nursing action is most effective in relation to the concept Immobility can lead to occlusion of blood vessels in areas where bony prominences rest on a mattress? 1. Encouraging the patient to breathe deeply 10 times per hour 2. Performing range-of-motion exercises twice a day 3. Placing a sheepskin pad under the sacrum 4. Repositioning the patient every 2 hours

4

Which stage pressure ulcer requires the nurse to measure the extent of undermining? 1. Stage 0 2. Stage I 3. Stage II 4. Stage III

4

11. A patient who is legally blind says to the nurse, "I once was able to see a little bit, but now I can't see anything." What should the nurse encourage the patient to do while hospitalized? 1. Wear dark-tinted eyeglasses. 2. Keep a light on in the room at all times. 3. Close the window blinds during the day. 4. Call for assistance when getting out of bed. Hint

4 A patient who is in a strange environment and who has a visual impairment is at an increased risk for falls. The patient should seek assistance with transfers and ambulating until the patient feels comfortable engaging in these activities and the nurse determines that the patient is safe.

Following an accident at a job site, a patient's employer contacts the hospital and asks the nurse about the patient's condition to determine the patient's likelihood of returning to work in the future. Which is the best response by the nurse to the employer? 1 "You need to speak to the primary healthcare provider." 2 "We can send the patient's medical records for your reference." 3 "The patient is doing well and will definitely be able to return to work." 4 "You will have to get the patient's permission to receive any related information."

4 According to the Health Insurance Portability and Accountability Act (HIPAA) of 1996, the patient's information should be kept confidential and the patient's privacy should be respected. Patient-related information should not be divulged to anybody without the patient's permission. Therefore, the nurse should ask the employer to seek the patient's permission to receive any patient-related medical information, including information about the feasibility of a patient returning to work. The primary healthcare provider is also not authorized to divulge patient information without the patient's consent. Providing the patient's medical records to others is a violation of HIPAA.

A woman who is a Jehovah's Witness has severe life-threatening injuries and is hemorrhaging following a car accident. The healthcare provider ordered two units of packed red blood cells to treat the woman's anemia. The woman's husband refuses to allow the nurse to give his wife the blood. What is the nurse's responsibility? 1 Obtain a court order to give the blood. 2 Coerce the husband into giving the blood. 3 Call security and have the husband removed from the hospital. 4 Abide by the husband's wishes and inform the healthcare provider.

4 Adult patients such as those who are Jehovah's Witnesses are able to refuse treatment for personal religious reasons.

45. A patient has bilateral wrist restraints because of repeated attempts to pull out tubes. How frequently should the nurse release the restraints, exercise the patient's hands and wrists, and document this activity? 1. Each hour 2. Each shift 3. Every 4 hours 4. Every 2 hours

4 Every 2 hours is often enough to perform ROM exercises to prevent contractures unless there are extenuating circumstances, such as when a patient has a neuromusculoskeletal or peripheral circulation problem.

12. A nurse is routinely monitoring a patient's pulse oximetry level. For which additional clinical indicator should the nurse monitor that is associated with the pulse oximetry level? 1) Heart rate 2) Blood pressure 3) Respiratory rate 4) Hemoglobin level

4 Pulse oximetry is a method of measuring the amount of oxygenated hemoglobin in the blood. Therefore, it is advisable to compare results routinely with the patient's hemoglobin level.

The healthcare provider has asked for a postmortem examination on a patient. Under which circumstances can a healthcare provider make the decision to perform an autopsy? 1 The patient was terminally ill. 2 The patient's autopsy reports are required for medical research. 3 The patient had an adverse drug effect that caused death. 4 The patient's death occurred under suspicious circumstances.

4 The health care provider can decide to conduct an autopsy only if the patient's death occurred under suspicious circumstances or if it is required by the law in the state in which the patient died, in order to determine the exact cause of death. Because the cause of death is known in a terminally ill patient, an autopsy need not be done for this patient. An autopsy cannot be done for research purposes, unless the patient or the caregivers have given their consent. In cases in which an adverse drug reaction has been identified as the cause of death in the patient, an autopsy is not required.

39. Which is an important step when transferring a patient using a mechanical lift? 1. Position the chair as close as possible to the patient's bed. 2. Remove the sling after the patient is transferred to the chair. 3. Position the sling at the middle of the patient's back to the ankles. 4. Attach the longer belts to the lower grommets on each side of the sling.

4 This is correct. When the longer belts/chains are attached to the bottom of the sling and the shorter belts/chains are attached to the top of the sling, the patient will be raised to a sitting position when the lift raises the sling and the patient up and off the bed.

37. A nurse is caring for a patient who fell by the side of the bed when attempting to use the commode. What should the nurse do first when completing responsibilities associated with this incident? 1. Initiate an incident report. 2. Notify the nursing supervisor of the patient's fall. 3. Document the incident in the patient's clinical record. 4. Have a primary health-care provider examine the patient immediately.

4 This is the first action that the nurse should implement. The patient needs to be examined by a primary health-care provider to ensure that the patient has not sustained an injury.

16. A patient who is cognitively impaired is admitted to the hospital for pneumonia. The patient has a history of wandering at night. What should the nurse do to ensure the safety of this patient? 1. Encourage a family member to remain with the patient every night. 2. Obtain a sedative to be administered to the patient at bedtime. 3. Apply a vest restraint when the patient plans to go to sleep. 4. Activate the bed alarm on the patient's bed. Hint

4 A bed-exiting device (e.g., position sensor on the leg, weight sensor under the mattress of a bed) should be used to alert caregivers when the patient is attempting to exit the bed. A health team member should immediately assist the patient and maintain safety when the alarm sounds.

A 65-year-old patient is experiencing mild musculoskeletal pain. Which drug is the primary health care provider most likely to prescribe? 1 Aspirin 2 Naproxen 3 Ibuprofen 4 Acetaminophen

4 A physician may first recommend acetaminophen to this patient because the pain is mild, and acetaminophen is relatively safe and widely available over the counter for musculoskeletal pain. The physician may prescribe aspirin, naproxen, or ibuprofen, but these may be second-choice drugs because they are nonsteroidal antiinflammatory drugs which carry a risk for bleeding, especially in older adults, and may not be necessary if the pain is mild.

54. A nurse is teaching a group of high school students on the golf team how to provide for safety during an electrical storm. Which suggestion should be included in the program? 1. Do not hold golf clubs. 2. Take off shoes with cleats. 3. Move toward water hazards. 4. Seek shelter under the nearest tree. Hint

4 A toaster is an electrical appliance; fire involving live electrical wires or equipment is extinguished with a type C or type ABC fire extinguisher.

47. A nurse is teaching a class for a community group about how to prevent the most common cause of fatal accidents in the home. Which information is most important to include that addresses this concern? 1. Remove scatter rugs from the home. 2. Check for the presence of lead paint. 3. Use gates at the bottom and top of stairs. 4. Store cleaning products in a locked cabinet. Hint

4 Accidental poisoning and unintentional overdose are the leading causes of death in the home and are the second leading cause of death after motor vehicle accidents overall. Household cleaners are followed by medicine, houseplants, cosmetics, pesticides, kerosene, gasoline, furniture polish, lighter fluid, and other chemicals. The five leading causes of accidental death are motor vehicle accidents, poisoning (accidental or unintentional), falls, choking/suffocation, drowning, and fire/flames/smoke.

Which patients are at higher risk of motor vehicle accidents according to the Centers for Disease Control and Prevention (CDC)? 1 2-year-old patient 2 30-year-old patient 3 55-year-old patient 4 16-year-old patient

4 According to the CDC, the risk of motor vehicle accidents is higher among 16- to 19-year-old drivers than any other age group, because teens are more likely to underestimate dangerous situations. A 2-year-old patient is not at elevated risk of motor vehicle accidents, because 2-year-olds do not use motor vehicles. A 30- or 55-year-old adult is not at elevated risk of motor vehicle accidents according to the CDC.

Which pain management drug is considered the best tolerated and safest analgesic? 1 Fentanyl 2 Tramadol 3 Acetylcysteine 4 Acetaminophen

4 Acetaminophen is considered the best tolerated and safest analgesic used in pain management. Fentanyl and tramadol are opioids, which have the potential for significant side effects and often result in patients building a tolerance to them. Acetylcysteine is not an analgesic; rather, it is used to treat acetaminophen overdose.

23. Which intervention is most effective in reducing the major cause of injury in the hospital setting for patients who are older adults? 1. Assist all older adults with toileting activities. 2. Elevate all bedside rails of older adults at night. 3. Place a fall precautions sign on the door to a room with an older adult. 4. Identify medications taken by an older adult that may increase the risk of falls. Hint

4 Assessment is the first step in the nursing process. Assessments should be completed before making a nursing diagnosis and planning and implementing care. Older adults often experience one or more chronic illness and receive a variety of medications. Polypharmacy is a risk factor for adverse reactions and drug interactions that may cause an older adult to fall.

The nurse is assessing a patient who complains of back pain. After taking the patient's history, the nurse does not expect the physician to recommend acetaminophen. Which of the patient's statements led the nurse to this conclusion? 1 "I drink alcohol occasionally." 2 "I have been constipated for 3 days." 3 "I am allergic to penicillin." 4 "Two months ago, I was diagnosed with hepatitis B."

4 Because acetaminophen can cause hepatotoxicity, and a patient who has hepatitis B has a compromised liver, the nurse may expect the physician to avoid recommending acetaminophen to this patient. Acetaminophen is safe for a patient who consumes alcohol occasionally, but it should be used with caution for a patient who frequently drinks alcohol. Occasional alcohol intake might not affect the administration of acetaminophen. Nonsteroidal antiinflammatory drugs, not acetaminophen, may aggravate constipation. An allergy to penicillin will not necessarily predispose a patient to an allergy to acetaminophen.

Which drug is unsafe for the central nervous system as a supplement to epidural anesthesia? 1 Aspirin 2 Naproxen 3 Ibuprofen 4 Oxycodone

4 Because opioid analgesics like oxycodone depress the central nervous system, they are not safe in combination with epidural anesthesia because of possible additive central nervous system adverse effects. Aspirin, naproxen, and ibuprofen are nonsteroidal antiinflammatory drugs (NSAIDs) which do not affect the central nervous system.

The registered nurse is teaching a patient about the use of cold therapy in acute pain management. Which of the patient's statements indicates a need for further teaching? 1 "I will apply ice two to five times a day." 2 "I will apply ice with a lightweight cover, with firm pressure to my skin." 3 "I will apply ice within a 6-inch circular area near where I have pain." 4 "I will place ice between my thumb and index finger if I have shoulder pain."

4 Cold is effective for tooth or mouth pain, not shoulder pain, when the ice is placed on the web of the hand between the thumb and index finger. This is an acupressure point that influences nerve pathways to the face and head. The remaining statements indicate understanding: Ice can be applied two to five times a day with firm pressure to the skin, covered with a lightweight cloth. Ice should also be applied within a 6-inch circular area near the site of pain.

A new medical resident writes an order for oxycodone SR 10 mg PO q12 hours prn. Which part of the order does the nurse question? 1 The drug 2 The time interval 3 The dose 4 Prn

4 Controlled- or extended-release opioid formulations such as oxycodone are available for administration every 8 to 12 hours around the clock (ATC). Health care providers should not order these long-acting formulations prn.

Why would a primary health care provider prescribe acetylcysteine to a patient who is on pharmacological pain management therapy? 1 Overdose of aspirin 2 Overdose of fentanyl 3 Overdose of morphine 4 Overdose of acetaminophen

4 Dangerous hepatotoxic overdoses of acetaminophen are treated with acetylcysteine. Antiulcer drugs may be prescribed to treat gastric bleeding caused by overdose of aspirin. Overdoses of fentanyl and morphine may be treated with naloxone.

Which skill is implemented by the nurse when planning to prevent falls in patients? 1 Reviewing the patient's medication 2 Making the patient's environment safe 3 Determining if the patient has a history of recent falls 4 Gathering the equipment and performing hand hygiene

4 During the planning phase, the nurse gathers the equipment and performs hand hygiene. During assessment, the nurse reviews the patient's medication. During implementation, the nurse makes the patient's environment safe. During assessment, the nurse determines if the patient has a history of recent falls.

After having received 0.2 mg of naloxone intravenous push (IVP), a patient's respiratory rate and depth are within normal limits. The nurse now plans to implement which actions? 1 Discontinue all ordered opioids. 2 Close the room door to allow the patient to recover. 3 Administer the remaining naloxone over 4 minutes. 4 Assess patient's vital signs every 15 minutes for 2 hours.

4 Every 15 minutes for 2 hours following drug administration reassess patients who receive naloxone because the duration of the opioid may be longer than the duration of the naloxone, and respiratory depression may return.

Which suggestion would be appropriate to prevent unilateral neglect in a patient with hemiparesis? 1 "You should perform range-of-motion exercises." 2 "You should use a walker and cane around the home." 3 "You should consult with an ophthalmologist for visual assessment." 4 "You should touch one side of the body frequently with the other hand."

4 Hemiparesis is a condition in which there is weakness on one side of the body. A patient with unilateral neglect related to brain injury will benefit from touching the left side of the body frequently with the right hand. Performing range-of-motion exercises is helpful for patients with impaired physical mobility. Using a walker or cane around the home is helpful for patients with impaired physical mobility. Consulting an ophthalmologist for visual assessment will help prevent the risk of falls in a patient with hemiparesis.

A patient took more than the prescribed amount of acetaminophen and is experiencing hepatotoxicity. Which drug might the nurse anticipate the health care provider to use to treat this patient? 1 Naloxone 2 Tramadol 3 Oxycodone 4 Acetylcysteine

4 Overdose of acetaminophen may lead to hepatotoxicity, which is treated with acetylcysteine. Naloxone is used to reverse the adverse effects of opioids, not acetaminophen. Tramadol and oxycodone are used to manage pain, not to treat acetaminophen overdose.

After assessing pain in a 9-year-old child using a numeric rating scale (NRS), the nurse documents the score as 5. What does the nurse interpret from this score? 1 The child has no pain. 2 The child has mild pain. 3 The child has severe pain. 4 The child has moderate pain.

4 The score range of 4 to 6 indicates moderate pain. A score of 0 indicates that the child is relaxed and comfortable without any pain. The score range of 1 to 3 indicates that the child has mild pain. The score range of 7 to 10 indicates that the child has severe pain.

A nurse accidentally administers an opioid to a patient. What should the nurse do first? 1. Call the nursing supervisor. 2. Administer the antidote immediately. 3. Instruct the patient to drink a glass of water. 4. Contact the patient's primary health-care provider.

4 Informing the nursing supervisor should be done after another more important intervention.

A patient returning to the nursing unit after knee surgery is verbalizing pain at the surgical site. What is the nurse's first action? 1 Call the patient's health care provider. 2 Administer pain medication as ordered. 3 Check the patient's vital signs. 4 Assess the characteristics of the pain.

4 It is necessary to monitor pain on a regular basis along with other vital signs. It is important for the nurse to understand that pain assessment is not simply a number.

Which pain characteristics might the nurse suspect in a patient with kidney stones? 1 The pain is of short duration and localized. 2 The pain is diffuse and radiates in several directions. 3 The pain radiates from the site of the injury to another body part. 4 The pain is in a part of the body separate from the source of pain.

4 Kidney stones cause groin pain, an example of referred pain, which occurs in a part of the body separate from the source of pain. Superficial or cutaneous pain, for example from a needle stick or small cuts, is of short duration and is localized. Deep or visceral pain, for example from angina pectoris, is diffuse and radiates in several directions. Radiating pain travels down from the site of injury to another body part, for example in sciatica.

A patient has had arthritic pain for 8 years and has surgery to remove a buildup of septic fluid. Postoperative, the patient received morphine through a patient-controlled analgesia (PCA) device for the management of pain. After a while, the patient starts getting drowsy and symptoms of respiratory depression begin to appear. The nurse is ordered to administer naloxone. What is the rate of administering naloxone? 1 Intravenous push at the rate of 1 mL every 1 minute 2 Intravenous push at the rate of 1 mL every 2 minutes 3 Intravenous push at the rate of 0.5 mL every 1 minute 4 Intravenous push at the rate of 0.5 mL every 2 minutes

4 Naloxone is an antidote for respiratory depression caused by opioids. The dosage of naloxone is 0.4 mg diluted by 9 mL saline. This is administered by intravenous push at the rate of 0.5 mL every 2 minutes. This dosage is optimal for reversal of respiratory depression. Doses larger than this can cause severe pain and other serious complications.

A child in the hospital starts to have a grand mal seizure while playing in the playroom. Which is the most important nursing intervention during this situation? 1 Begin cardiopulmonary respiration. 2 Restrain the child to prevent injury. 3 Place a tongue blade over the tongue to prevent aspiration. 4 Clear the area around the child to protect the child from injury.

4 Once a seizure begins, the nurse needs to monitor the patient and provide a safe environment. A seizure is not an indication for cardiopulmonary resuscitation. A person having a seizure should not be restrained, but the environment should be made safe. Objects should not be forced into the mouth.

A patient with chronic low back pain who took an opioid around-the-clock (ATC) for the past year decided to abruptly stop the medication for fear of addiction. He is now experiencing shaking, chills, abdominal cramps, and joint pain. What does the nurse recognize as these symptoms? 1 Addiction 2 Tolerance 3 Pseudoaddiction 4 Physical dependence

4 Physical dependence is a state of adaptation that is manifested by a drug-class specific withdrawal syndrome produced by abrupt cessation of the drug, rapid dose reduction, decreasing blood levels of the drug, and/or the administration of an antagonist.

The nurse is advising the mother of a 4-month-old infant to remove plastic bags from the home. Which risk is the nurse addressing? 1 Choking 2 Poisoning 3 Head injury 4 Suffocation

4 Plastic bags from the cleaners or grocery store may cause an infant to suffocate. Choking can be prevented by avoiding the use of toys with small parts like buttons. Poisoning may occur due to toxic or poisonous substances, including plants. Head injury may be caused by falls.

White, shiny, flexible bands of fibrous tissue binding joints together and connecting various bones and cartilage types are known as: 1. Joints 2. Muscles 3. Tendons 4. Ligaments

4 Rationale: Ligaments are white, shiny, flexible bands of fibrous tissue that bind joints and connect bones and cartilage.

What is the immediate intervention if a patient on oxycodone 10 mg/mL infusion therapy experiences respiratory depression? 1 Administering acetylcysteine 2 Reducing the dose of oxycodone to 5 mg/mL 3 Decreasing the rate of infusion 4 Administering 0.4 mg of naloxone

4 Respiratory depression is a serious side effect of opioid administration. Naloxone counters the effects of opioids, so this drug would be used to treat respiratory depression resulting from oxycodone administration. Acetylcysteine is used to counter acetaminophen, not opioid, overdose. Decreasing the dose or rate of infusion may be done to decrease less serious side effects like drowsiness or nausea.

Which statement is true regarding restraints? 1 Restraints are ordered prn. 2 The use of restraints involves a psychological adjustment for the family. 3 Informed consent from family members is required before using restraints. 4 Restraints are a part of the patient's prescribed medical treatment and plan of care.

4 Restraints may be a part of a patient's prescribed medical treatment and plan of care. Restraints are not ordered prn. The use of restraints involves a psychological adjustment for the patient and the family. Informed consent from family members is required before using restraints in long-term care facilities

A nurse is preparing a safety program for parents about accidental poisoning. What information should the nurse include concerning what to do in the event a child ingests a poison? 1. Have the child drink salt water. 2. Administer Ipecac immediately. 3. Take the child to an emergency department. 4. Seek guidance from a poison control center.

4 Salt water induces vomiting. The ingested poison may cause additional damage to the esophagus and mouth if vomiting is induced. Ipecac is an emetic that induces vomiting. The ingested poison may cause additional damage to the esophagus and mouth if vomiting is induced. This action could delay an immediate response that may help minimize a negative consequence. Poison control centers provide immediate instructions regarding what to do for the specific poison ingested. Never employ a home remedy without directions from a poison control center.

The nurse is caring for a patient with muscle weakness. The patient is provided with skid-proof footwear. Which safety measure is implemented in this situation? 1 Enabling to remain alert 2 Promoting cooperation 3 Allowing for safe exit from bed 4 Preventing falls from slipping on floor

4 Skid-proof footwear can prevent the patient from slipping on the floor. Muscle weakness may lead to unexpected falls that can cause injuries. Skid-proof footwear can help prevent falls that are caused by slipping on the floor. Hearing aids and glasses enable patients to remain alert within their environments. Clear explanations of safety measures help elicit cooperation from the patient and family members. Providing side rails on the bed helps in the safe exit of the patient from the bed.

The nurse is teaching a group of nursing students about pain sensations in infants. Which information should be included in the teaching about pain sensation in infants? 1 Infants do not perceive pain sensation immediately after birth. 2 Nurses cannot accurately assess pain in infants. 3 Infants cannot express pain sensation in the first month of life. 4 Infants learn to perceive pain by experiencing the first unpleasant stimulus.

4 Some common misconceptions about pain sensation in infants exist, of which the nurse should be aware. Infants immediately respond to pain on experiencing the first noxious stimulus. Infants feel pain from birth; a functional processing of pain is developed by mid to late gestation. Nurses can use behavioral changes and alterations in vital signs to assess pain in infants. Although infants cannot verbalize pain, they can express pain by crying.

According to the National Quality Forum, which event is considered a care management event? 1 Abduction of a patient at any age 2 An immediate postoperative death 3 Serious disability associated with patient elopement 4 Stage III pressure ulcers acquired after admission to a health care facility

4 Stage III or IV pressure ulcers acquired after admission to a health care facility is a serious reportable care management event. The abduction of a patient at any age is a criminal event. Intraoperative or immediate postoperative death is a serious surgical event. Patient death or serious disability associated with patient elopement is a serious, reportable patient protection event.

In a pediatric ward, one of the newborns died of sudden infant death syndrome (SIDS). Which nursing measure lowers the risk of death due to SIDS? 1 Massaging the baby's heels 2 Attaching pacifiers with a string around the baby's neck 3 Gently rubbing the baby's back 4 Having the baby sleep on his or her back

4 Sudden infant death syndrome (SIDS) is a condition in which the infant dies due to an unexplained cause. The American Academy of Pediatrics recommends having the baby sleep on his or her back to reduce the risk of sudden infant death syndrome (SIDS). Massaging the heels helps in managing an apneic episode. Pacifiers should not be attached with a string around the neck, because this increases the risk of choking. Rubbing the baby's back is helpful in stimulating respiration in newborns.

The nurse is assessing a patient who has sustained severe injuries in a motor vehicle accident. The patient is in severe pain and is diaphoretic. On assessment, the patient's heart rate is increased, pupils are dilated, and blood pressure is decreased. Which finding is caused by the stimulation of the parasympathetic nervous system? 1 Diaphoresis 2 Dilation of pupils 3 Increased heart rate 4 Decrease in blood pressure

4 Superficial pain or mild-to-moderate pain stimulates the sympathetic nervous system. The parasympathetic nervous system is stimulated by continuous, deep, or severe pain involving visceral organs. Stimulation of the parasympathetic nervous system has an inhibitory effect on the body systems and causes a decrease in blood pressure. The sympathetic nervous system prepares the body for a fight-or-flight response. Diaphoresis, dilation of pupils, and increased heart rate are caused by the stimulation of the sympathetic system.

When teaching a patient about transcutaneous electrical nerve stimulation (TENS), which information should the nurse include? 1 TENS works by causing distraction. 2 TENS therapy does not require a health care provider's order. 3 A TENS unit must remain plugged in at all times 4 TENS electrodes are applied near or directly on the site of pain.

4 TENS involves stimulation of the skin with a mild electrical current passed through external electrodes. The therapy requires a health care provider order. The TENS unit consists of a battery-powered transmitter, lead wires, and electrodes. Place the electrodes directly over or near the site of pain.

The registered nurse is evaluating the performance of a student nurse who is performing a back massage for a patient with back pain. Which action by the student nurse needs correction? 1 Using long, gliding strokes along the muscles of the spine 2 Beginning at the sacral area and massaging in a circular motion 3 Kneading the skin by gently grasping tissue between the thumb and fingers 4 Kneading downward along one side of the spine from the shoulders to the buttocks

4 The nurse should knead upward along one side of the spine from buttocks to shoulders, not downward from the shoulders to the buttocks. The nurse should massage each body part for at least 10 minutes and use long, gliding strokes along the muscles of the spine. The massage should begin at the sacral area and progress in a circular motion while moving upward from the buttocks to the shoulders. The nurse should knead the skin by gently grasping tissue between the thumb and fingers.

A nurse administers epidural anesthesia to a patient in the terminal stages of cancer for pain relief. Which nursing intervention is then necessary? 1 Administering supplemental doses of opioid 2 Assessing vitals once every hour after administering the first dose 3 Administering anticoagulant medications with the epidural 4 Notifying the health care provider if the patient develops pain at the epidural insertion site

4 The nurse should notify the health care provider if the patient develops pain at the epidural insertion site, because it may indicate development of an epidural hematoma. Administering supplemental opioids could lead to dangerous additive central nervous system adverse effects. The nurse should monitor vital signs and respiratory rate once every 15 minutes after the administration of epidural anesthesia to ensure stable vitals; once every hour is not enough. Anticoagulants and antiplatelet drugs should not be administered to the patient, because they may increase the risk of hematoma formation.

A patient with diabetes who is on metformin has been taking morphine and nortriptyline for the past week to treat neuropathic pain. The patient is diagnosed with an upper respiratory tract infection and is prescribed antibiotics. Which drug taken by the patient is an adjuvant pain medication? 1 Antibiotic 2 Morphine 3 Metformin 4 Nortriptyline

4 The primary purpose of an adjuvant drug such as an antiepileptic, muscle relaxant, sedative, or anxiolytic is to treat conditions other than pain. Some of these drugs have analgesic properties and reduce pain effectively when used with or without pain medications. The diabetic patient takes nortriptyline, which is an antidepressant. It is also used as an adjuvant analgesic to morphine, which is an opioid analgesic in pain management. Morphine is the primary drug that provides pain relief. Antibiotics and metformin do not have an analgesic affect. Antibiotics are used to treat infections and do not have an analgesic effect. Metformin is an oral hypoglycemic drug and does not have any effect on pain relief.

Which stage pressure ulcer requires the nurse to measure the extent of undermining? 1. Stage 0 2. Stage I 3. Stage II 4. Stage III

4. In a stage III pressure ulcer there is full- thickness skin loss involving damage to subcutaneous tissue that may extend to the fascia and there may or may not be undermining, which is tissue destruction underneath intact skin along wound margins.

Which pain management method is considered a nonpharmacological complementary and alternative intervention? 1 Distraction 2 Biofeedback 3 Guided imagery 4 Therapeutic touch

4 Therapeutic touch is a nonpharmacological complementary and alternate pain management intervention. Distraction, biofeedback, and guided imagery, music are nonpharmacological, but considered cognitive-behavioral, not alternative and complementary, interventions.

48. A nurse is caring for a patient who tends to become confused. Which information communicated to the patient is the best intervention to prevent falls in a hospital setting? 1. "I am your nurse for today and I will be here for you whenever you call me for help." 2. "When you finish in the bathroom, activate the call light and I will help you back to your bed." 3. "When you decide you want to walk around the unit, I would appreciate it if you would use the walker." 4. "I am going to transfer you to your wheelchair now and move you to an area near the nurses' station." Hint

4 This is the best intervention of the options presented to help keep a patient safe from falls. By positioning the patient's wheelchair next to the nurses' station, staff members can continuously supervise a confused patient.

The nurse has conducted an informative session on discouraging pseudoaddiction to a group of people in a community. Which group of patients should be the main target for the nurse's teachings? 1 Patients with a history of taking over-the-counter medicines 2 Patients with a history of drinking coffee for more than 5 years 3 Patients who say that heroin increases concentration 4 Patients who repeatedly seek multiple medical opinions for chronic pain relief

4 When a patient with chronic pain seeks pain medication from multiple primary health care providers, the patient is called a drug seeker but not an illicit drug abuser. This kind of addiction is called pseudoaddiction. Such drug seekers should be referred to pain specialists. Pseudoaddiction is not related to a history of taking over-the-counter medicines or the history of drinking coffee for more than 5 years. The patients who say that heroin increases concentration do not have pseudoaddiction.

7. A nurse is caring for an older adult who is cognitively impaired and has a history of pulling out tubes and falling. List the following safety devices in the order of least restrictive to most restrictive that may be employed to ensure the safety of this patient. 1. Cloth vest 2. Two wrist straps 3. Four side rails up 4. Bed exiting alarm device 5. Four-point restraint tied to the bed frame

4 3 1 2 5 Answer 4 is first because a bed exiting alarm device will signal caregivers when the patient attempts to exit the bed. These devices do not curtail the patient's movement but will alert staff members that the patient needs supervision. Answer 3 is second because although four side rails will curtail the patient to the bed, the patient is still able to turn and sit up with ease. Answer 1 is third because a cloth vest permits turning from side to side and sitting up but physically restricts the patient to the bed by the use of straps tied to the bed frame. Answer 2 is fourth because two wrist restraints curtail the movement of the upper extremities and prevent turning from side to side; also, they curtail the patient to the bed because the straps are tied to the bed frame. Answer 5 is fifth because this is the most restrictive physical restraint because the extremities are for all practical purposes immobilized; all four extremities are tied to the bed frame.

The total sleep time of a patient is 200 minutes. If 80% of sleep time is spent in nonrapid eye movement (NREM) sleep, how much time is spent in rapid eye movement (REM) sleep? Record your answer using a whole number. _________ minutes

40

The nurse is teaching a group of student nurses about the stages of sleep. Which event does the nurse discuss while explaining rapid eye movement (REM) sleep? 1 Bedwetting 2 Daydreaming 3 Sleepwalking 4 Skeletal muscle paralysis

4 Skeletal muscle paralysis

While monitoring a patient's vital signs, the nurse finds that the patient's pulse and respiration during sleep are significantly lower than during waking hours. The nurse also recalls that during this stage of sleep the body releases human growth hormone. The patient is in which nonrapid eye movement (NREM) stage of sleep? 1 Stage 1 2 Stage 2 3 Stage 3 4 Stage 4

4 Stage 4

A patient has hypothyroidism. Which stage of the sleep cycle is decreased in the patient? 1 Stage 1 of nonrapid eye movement 2 Stage 2 of nonrapid eye movement 3 Stage 3 of nonrapid eye movement 4 Stage 4 of nonrapid eye movement

4 Stage 4 of nonrapid eye movement

How is stage 1 of nonrapid eye movement (NREM) sleep similar to stage 2 of NREM sleep? 1 Both stages last for 10 to 20 minutes. 2 Both stages include a light level of sleep. 3 The muscles are completely relaxed in both stages 4 The sleeper can be easily awakened in both stages.

4 The sleeper can be easily awakened in both stages.

What caution should be given to older adults regarding the long-term use of sedatives and hypnotics? 1 They cause headaches and nausea. 2 They are expensive and difficult to obtain. 3 They cause severe depression and anxiety. 4 They lead to sleep disruption.

4 They lead to sleep disruption.

A patient has paraplegia and wears weight-supporting braces. Which crutch gait pattern should the patient be trained to use during ambulation? 1 A two-point gait 2 A four-point gait 3 A three-point gait 4 A swing-through gait

4 A swing-through gait While using the swing-through gait pattern, the patient places the crutches one stride in front and then swings to or through them while they support his or her weight. This gait pattern does not require weight bearing on the legs; therefore, it is the correct gait pattern for the patient. In a two-point gait pattern, the patient moves a crutch at the same time as the opposing leg that resembles arm motion during normal walking. It requires weight bearing on one leg, and may not be possible in paraplegia. A four-point crutch gait requires weight bearing on both legs. Each leg is moved alternately with each opposing crutch. Therefore, it is not appropriate for this patient. A three-point gait requires the patient to bear all of the weight on one foot. The affected foot does not touch the ground. This gait pattern may not be suitable for this patient.

The nurse is positioning a patient who had a syncope episode. The nurse supports the patient's body at the hip level, stands with the feet apart, extends both legs of the patient to let them slide against the floor, and bends the knees to lower the body of the patient. Which action by the nurse may lead to musculoskeletal injury? 1 Supporting the patient's body at hip level 2 Bending the knees of the patient to lower the body 3 Standing with feet apart while holding the patient 4 Extending both legs of the patient to let them slide against the floor

4 Extending both legs of the patient to let them slide against the floor If the patient has syncope, the nurse immediately holds the patient to prevent a fall. The nurse stands with the feet apart while holding the patient, which provides a wide base for support. The nurse should extend one leg and let the patient slide against the leg, and gently lower the patient onto the floor, while protecting the patient's head. Do not keep the patient's legs extended as that could lead to musculoskeletal injury. Bending the patient's knees provides flexibility to lower the patient's body, and prevents musculoskeletal injury. The nurse should gently lower the patient's body to the floor. The nurse provides support to the patient's body at hip or waist level in order to maintain the patient's center of gravity at midline.

How does exercise affect the pulmonary system? 1 Increased production of body heat 2 Improved muscle tolerance to physical exercise 3 Improved myocardial contraction, which strengthens the cardiac muscle 4 Increased respiratory rate and depth followed by a quicker return to the resting state

4 Increased respiratory rate and depth followed by a quicker return to the resting state The effect of exercise on the pulmonary system is an increased respiratory rate and depth followed by a quicker return to the resting state. The effect of exercise on the metabolic system includes an increased production of body heat. The effect of exercise on the musculoskeletal system includes improved muscle tolerance to physical exercise. The effect of exercise on the cardiovascular system includes improved myocardial contraction, which strengthens the cardiac muscle.

How is the isotonic form of exercise different from the isometric form? 1 Isotonic exercises promote osteoblastic activity. 2 Isotonic exercises enhance circulatory functioning. 3 Isotonic exercises increase muscle mass, tone, and strength. 4 Isotonic exercises cause muscle contraction and changes in muscle length

4 Isotonic exercises cause muscle contraction and changes in muscle length Isotonic exercises cause muscle contraction and changes in muscle length, whereas isometric exercise involves tightening or tensing muscles without moving the body parts. Both isotonic and isometric forms of exercise promote osteoblastic activity. Both forms of exercise enhance circulatory functioning. Both forms of exercise increase muscle mass, tone, and strength.

How is it beneficial to the patient to keep his or her back, neck, pelvis, and feet in proper alignment while the health care team lifts him or her? 1 This provides a broad base of support. 2 This places stress on the larger muscles. 3 This reduces the patient's muscular strain. 4 This reduces the risk of injury to the patient.

4 This reduces the risk of injury to the patient. When lifting a patient, the health care team should be careful to keep the patient's back, neck, pelvis, and feet in proper alignment to prevent unwanted twisting. This reduces the risk of injury to the lumbar vertebrae and related muscle groups. Flexing the knees and keeping the feet wide apart helps to maintain a broad base of support. Using the arms and legs more during lifting places stress on larger muscles and reduces back injury. Patient-handling equipment such as height-adjustable beds and air-assisted devices should be used to reduce the caregivers' muscle strain during the procedure if required.

The nurse is teaching pain management to a group of caregivers. Which information should be included? Select all that apply. 1 Chronic pain is often psychological. 2 Only hospitalized patients experience severe pain. 3 Psychogenic pain is not real. 4 Regular administration of analgesics will not lead to addiction. 5 Patients with minor illnesses may also experience severe pain.

4, 5 Misconceptions about pain often lead to poor nursing care. Therefore, it is important to know these misconceptions in order to promote appropriate pain management in patients. Regular administration of analgesics does not lead to addiction. Therefore, analgesics should be administered whenever the need arises. Although a patient may suffer from minor illness, he or she may experience severe pain that should not be ignored. A common misconception is that chronic pain is often psychological. However, chronic pain may have a pathological origin. Another misconception is that only hospitalized patients experience pain. Patients who are not hospitalized may also experience pain that needs to be addressed. Another misconception is that psychogenic pain is not real.

The registered nurse (RN) is teaching a patient's family members about environmental assessments for substance abuse. Which statements should the nurse include in the teaching? Select all that apply. 1 "You should observe for increased aggressiveness." 2 "You should check for the changes in style of dress." 3 "You should observe for changes in interpersonal relationships." 4 "You should check for the presence of drug-oriented magazines." 5 "You should observe for the presence of blood spots on clothing.

4, 5 The environmental clues that indicate substance abuse are the presence of drug-oriented magazines and the presence of blood spots on the patient's clothing, which could be caused by the injection of drugs into the body. Increased aggressiveness, changes in style of dress, and changes in interpersonal relationships are psychological clues that indicate substance abuse.

Which nursing activities are performed during safety planning for a patient? Select all that apply. 1 Identify the patient's perceptions of safety needs and risks. 2 Determine impact of the underlying illness on the patient's safety. 3 Determine effect of environmental influence on the patient's safety. 4 Consult with occupational and physical therapists for assistive devices. 5 Select interventions that will improve the safety of the patient's home environment.

4, 5 The nursing activities during the safety planning phase for a patient include consulting with occupational and physical therapists for assistive devices and selecting interventions that will improve the safety of the patient's home environment. When assessing the patient's safety, the nurse should identify the patient's perceptions of safety needs and risks. According to the critical thinking model for safety, determining the impact of the underlying illness on the patient's safety, and the effect of environmental influence on the patient's safety are the nursing activities related to the assessing phase.

16. The nurse arrives at work and is told to report (float) to the intensive care unit (ICU) for the day because the ICU is understaffed and needs additional nurses to care for the clients. The nurse has never worked in the ICU. The nurse should take which action first? 1.Call the hospital lawyer. 2.Refuse to float to the ICU. 3.Call the nursing supervisor. 4.Identify tasks that can be performed safely in the ICU.

4. Floating is an acceptable legal practice used by hospitals to solve understaffing problems. Legally, the nurse cannot refuse to float unless a union contract guarantees that nurses can work only in a specified area or the nurse can prove the lack of knowledge for the performance of assigned tasks. When encountering this situation, the nurse should set priorities and identify potential areas of harm to the client. The nursing supervisor is called if the nurse is expected to perform tasks that he or she cannot safely perform. Calling the hospital lawyer is a premature action.

9. The nurse has made an error in a narrative documentation of an assessment finding on a client and obtains the client's record to correct the error. The nurse should take which action to correct the error? 1.Documenting a late entry into the client's record 2.Trying to erase the error for space to write in the correct data 3.Using whiteout to delete the error to write in the correct data 4.Drawing one line through the error, initialing and dating, and then documenting the correct information

4. If the nurse makes an error in narrative documentation in the client's record, the nurse should follow agency policies to correct the error. This includes drawing one line through the error, initialing and dating the line, and then documenting the correct information. A late entry is used to document additional information not remembered at the initial time of documentation. Erasing data from the client's record and the use of whiteout are prohibited.

A nursing instructor delivers a lecture to nursing students regarding the issue of client's rights and asks a nursing student to identify a situation that represents an example of invasion of client privacy. Which situation, if identified by the student, indicates an understanding of a violation of this client right? 1.Performing a procedure without consent 2.Threatening to give a client a medication 3.Telling the client that he or she cannot leave the hospital 4.Observing care provided to the client without the client's permission

4. Invasion of privacy occurs with unreasonable intrusion into an individual's private affairs. Performing a procedure without consent is an example of battery. Threatening to give a client a medication constitutes assault. Telling the client that the client cannot leave the hospital constitutes false imprisonment.

A nurse is performing an admitting interview. Which patient statement about pain should cause the most concern for the nurse? 1. "I try to pretend that it is not part of me, but it takes a lot of effort." 2. "My pain medication works, but I'm afraid of becoming addicted." 3. "At home I take something for the pain before it gets too bad." 4. "They say my pain may get worse, and I can't stand it now."

4. "They say my pain may get worse, and I can't stand it now." The level of pain tolerance is exceeded. The present pain must be relieved and the patient assured that future pain also will be controlled.

18. A hospitalized client tells the nurse that a living will is being prepared and that the lawyer will be bringing the will to the hospital today for witness signatures. The client asks the nurse for assistance in obtaining a witness to the will. Which is the most appropriate response to the client? 1."I will sign as a witness to your signature." 2."You will need to find a witness on your own." 3."Whoever is available at the time will sign as a witness for you." 4."I will call the nursing supervisor to seek assistance regarding your request."

4. Living wills, also known as natural death acts in some states, are required to be in writing and signed by the client. The client's signature must be witnessed by specified individuals or notarized. Laws and guidelines regarding living wills vary from state to state, and it is the responsibility of the nurse to know the laws. Many states prohibit any employee, including the nurse of a facility where the client is receiving care, from being a witness. Option 2 is nontherapeutic and not a helpful response. The nurse should seek the assistance of the nursing supervisor.

An 87-year-old woman is brought to the emergency department for treatment of a fractured arm. On physical assessment, the nurse notes old and new ecchymotic areas on the client's chest and legs and asks the client how the bruises were sustained. The client, although reluctant, tells the nurse in confidence that her son frequently hits her if supper is not prepared on time when he arrives home from work. Which is the most appropriate nursing response? 1."Oh, really. I will discuss this situation with your son." 2."Let's talk about the ways you can manage your time to prevent this from happening." 3."Do you have any friends that can help you out until you resolve these important issues with your son?" 4."As a nurse, I am legally bound to report abuse. I will stay with you while you give the report and help find a safe place for you to stay."

4. The nurse must report situations related to child or elder abuse, gunshot wounds and other criminal acts, and certain infectious diseases. Confidential issues are not to be discussed with nonmedical personnel or the client's family or friends without the client's permission. Clients should be assured that information is kept confidential, unless it places the nurse under a legal obligation. Options 1, 2, and 3 do not address the legal implications of the situation and do not ensure a safe environment for the client.

An 87-year-old woman is brought to the emergency department for treatment of a fractured arm. On physical assessment, the nurse notes old and new ecchymotic areas on the client's chest and legs and asks the client how the bruises were sustained. The client, although reluctant, tells the nurse in confidence that her son frequently hits her if supper is not prepared on time when he arrives home from work. Which is the MOST APPROPRIATE nursing response? 1. "Oh, really. I will discuss the situation with your son." 2. "Let's talk about the ways you can manage your time to prevent this from happening." 3. "Do you have any friends that can help you out until you resolve these important issues with your son?" 4. "As a nurse, I am legally bound to report abuse. I will stay with you while you give the report and help find a safe place for you to stay."

4. "As a nurse, I am legally bound to report abuse. I will stay with you while you give the report and help find a safe place for you to stay." Rationale: The nurse must report situations related to child or elder abuse, gunshot wounds and other criminal acts, and certain infectious diseases. Confidential issues are not to be discussed with nonmedical personnel or the client's family or friends without the client's permission. Clients should be assured that information is kept confidential, unless it places the nurse under a legal obligation. Options 1, 2, and 3 do not address the legal implications of the situation and do not ensure a safe environment for the client. Test-Taking Strategy: Note the strategic words MOST APPROPRIATE. Focus on the data in the question and note that an 87-year-old woman is receiving physical abuse by her son. Recall the nursing responsibilities related to client safety and reporting obligations. Options 1, 2, and 3 should be eliminated because they are COMPARABLE OR ALIKE in that they do not protect the client from injury.

Which patient statement indicates that the patient is experiencing bruxism? 1. "I walk around in my sleep almost every night, but I don't remember it." 2. "I annoy the whole family with the loud snoring noises I make at night." 3. "I occasionally urinate in bed when I am sleeping, and it's embarrassing." 4. "I am told by my wife that I make a lot of noise grinding my teeth when I sleep."

4. "I am told by my wife that I make a lot of noise grinding my teeth when I sleep." Bruxism, clenching and grinding of the teeth, is a parasomnia that occurs during stage II NREM sleep. Usually, it does not interfere with sleep for the affected individual but rather the sleeper's partner.

Which statement made by the patient indicates a need for further teaching on sleep hygiene? 1. "I'm going to do my exercises before I eat dinner." 2. "I'm going to bed every night at about the same time." 3. 'I set my alarm to get up at the same time every morning." 4. "I moved my computer to the bedroom so I could work before I go to sleep."

4. "I moved my computer to the bedroom so I could work before I go to sleep."

A hospitalized client tells the nurse that a living will is being prepared and that the lawyer will be bringing the will to the hospital today for witness signatures. The client asks the nurse for assistance in obtaining a witness to the will. Which is the MOST APPROPRIATE response to the client? 1. "I will sign as a witness to your signature." 2. "You will need to find a witness on your own." 3. "Whoever is available at the time will sign as a witness for you." 4. "I will call the nursing supervisor to seek assistance regarding your request."

4. "I will call the nursing supervisor to seek assistance regarding your request." Rationale: Living wills, also known as natural death acts in some states, are required to be in writing and signed by the client. The client's signature must be witnessed by specified individuals or notarized. Laws and guidelines regarding living wills vary from state to state, and it is the responsibility of the nurse to know the laws. Many states prohibit any employee, including the nurse of a facility where the client is receiving care, from being a witness. Option 2 is nontherapeutic and not a helpful response. The nurse should seek assistance of the nursing supervisor. Test-Taking Strategy: Note the strategic words MOST APPROPRIATE. Options 1 and 3 are COMPARABLE OR ALIKE and should be eliminated first. Option 2 is eliminated because it is a nontherapeutic response.

A patient who had a total abdominal hysterectomy two days ago reports abdominal pain at level 5 on a 0-to-10 pain scale. After assessing the pain further, which should the nurse do first? 1. Reposition the patient. 2. Offer a relaxing back rub. 3. Use distraction techniques. 4. Administer the prescribed analgesic.

4. Administer the prescribed analgesic. Major abdominal surgery involves extensive manipulation of internal organs and a large abdominal incision that require adequate pharmacological intervention to provide relief from pain.

A patient requests pain medication for severe pain. Which should the nurse do first when responding to this patient's request? 1. Use distraction to minimize the patient's perception of pain. 2. Place the patient in the most comfortable position possible. 3. Administer pain medication to the patient quickly. 4. Assess the various aspects of the patient's pain.

4. Assess the various aspects of the patient's pain. All the factors that affect the pain experience should be assessed, including location, intensity, quality, duration, pattern, aggravating and alleviating factors, and physical, behavioral, and attitudinal responses. Assessment must precede intervention.

A nurse is teaching a community health education class about rest and sleep. Which concept related to sleep should the nurse include? 1. Total time in bed gradually decreases as one ages. 2. Sleep needs remain consistent throughout the life span. 3. Alcohol intake interferes with one's ability to fall asleep. 4. Bedtime routines are associated with an expectation of sleep.

4. Bedtime routines are associated with an expectation of sleep. An expectation of an outcome of behavior usually becomes a self- fulfilling prophecy. Bedtime rituals include activities that promote comfort and relaxation (e.g., music, reading, and praying) and hygienic practices that meet basic physiological needs (e.g., bathing, brushing the teeth, and toileting).

At which time does a nurse medicate a patient for pain for it to be considered preemptive analgesia? 1. Before a patient goes to sleep 2. At equally distant times around the clock 3. As soon as a patient reports the occurrence of pain 4. Before doing a dressing change that has been painful in the past

4. Before doing a dressing change that has been painful in the past The word preemptive means preventive, anticipatory, and defensive. Therefore, preemptive analgesia is administered before an activity or intervention that may precipitate pain in an attempt to limit the anticipated pain.

Which patient condition identified by a nurse is unrelated to infection? 1 Catabolism 2. Hyperglycemia 3. Ketones in the urine 4. Decreased metabolic activity

4. Decreased metabolic activity

The nurse has made an error in a narrative documentation of an assessment finding on a client and obtains the client's record to correct the error. The nurse should take which action to correct the error? 1. Documenting a late entry into the client's record. 2. Trying to erase the error for space to write in the correct data. 3. Using whiteout to delete the error to write in the correct data. 4. Drawing one line through the error, initialing and dating, and then documenting the correct information.

4. Drawing one line through the error, initialing and dating, and then documenting the correct information. Rationale: If the nurse makes an error in narrative documentation in the client's record, the nurse should follow agency policies to correct the error. This includes drawing one line through the error, initialing and dating the line, and then documenting the correct information. A late entry is used to document additional information not remembered at the initial time of documentation. Erasing data from the client's record and the use of whiteout are prohibited. Test-Taking Strategy: Focus on the SUBJECT, correcting a documentation error, and use principles related to documentation. Recalling that alterations to a client's record are to be avoided will assist in eliminating options 2 and 3. From the remaining options, focusing on the SUBJECT of the question and using knowledge regarding the principles related to documentation will direct you to the correct option.

The nurse moves a patient's leg through range of motion demonstrated in the figure. What word should the nurse use when documenting exactly what was done during range-of-motion exercises? (picture of a leg with an arrow pointing outwards and back) 1. Eversion 2. Circumduction 3. Plantar flexion 4. External rotation

4. External rotation occurs when the entire leg is rolled outward from the body so that the toes point away from the opposite leg.

The nurse arrives at work and is told to report (float) to the intensive care unit (ICU) for the day because the ICU is understaffed and needs additional nurses to care for the clients. The nurse has never worked in the ICU. The nurse should take which action FIRST? 1. Call the hospital lawyer. 2. Refuse to float to the ICU. 3. Call the nursing supervisor. 4. Identify tasks that can be performed safely in the ICU.

4. Identify tasks that can be performed safely in the ICU. Rationale: Floating is an acceptable legal practice used by hospitals to solve understaffing problems. Legally, the nurse cannot refuse to float unless a union contract guarantees that nurses can work only in a specified area or the nurse can prove the lack of knowledge for the performance of assigned tasks. When encountering this situation, the nurse should set priorities and identify potential areas of harm to the client. The nursing supervisor is called if the nurse is expected to perform tasks that he or she cannot safely perform. Calling the hospital lawyer is a premature action. Test-Taking Strategy: Note the strategic word FIRST. Eliminate option 2 first because of the word REFUSE. Next, eliminate options 1 and 3 because they are premature actions.

Which does the nurse determine is a secondary line of defense against infection? 1. Mucous membrane of the respiratory tract 2. Urinary tract environment 3. Integumentary system 4. Immune response

4. Immune response

What do nurses sometimes do that increase their risk for injury when moving patients? 1. Use the longer, rather than the shorter, muscles when moving patients 2. Place their feet wide apart when transferring patients 3. Pull rather than push when turning patients 4. Misalign their backs when moving patients

4. Misaligning the back when moving patients occurs most often when not facing the direction of the move. Twisting (rotation) of the thoracolum-bar spine and flexion of the back place the line of gravity outside the base of support, which can cause muscle strain and disabling injuries.

A nurse in a community center is conversing with a group of older adults who voiced fears about falling. What is the most common consequence associated with older adults' fear of falling that the nurse should discuss with them? 1. Impaired skin integrity 2. Occurrence of panic attacks 3. Self-imposed social isolation 4. Decreased physical conditioning

4. Most falls occur when ambulating. Fear of falling results in the conscious choice not to place oneself in a position where a fall can occur. Disuse and muscle wasting cause a reduction of muscle strength at the rate of 5% to 10% per week so that within 2 months of immobility more than 50% of a muscle's strength can be lost. In addition, there is a decreased cardiac reserve. These responses result in decreased physical conditioning.

A nursing instructor delivers a lecture to nursing students regarding the issue of client's rights and asks a nursing student to identify a situation that represents an example of INVASION OF CLIENT PRIVACY. Which situation, if identified by the student, indicates an understanding of a violation of this client right? 1. Performing a procedure without consent. 2. Threatening to give a client medication. 3. Telling the client that he or she cannot leave the hospital. 4. Observing care provided to the client without the client's permission.

4. Observing care provided to the client without the client's permission. Rationale: Invasion of privacy occurs with unreasonable intrusion into an individual's private affairs. Performing a procedure without consent is an example of battery. Threatening to give a client a medication constitutes assault. Telling the client that the client cannot leave the hospital constitutes false imprisonment. Test-Taking Strategy: Focus on the SUBJECT, invasion of privacy. Noting the words WITHOUT THE CLIENT'S PERMISSION in option 4 will direct you to this option.

The nurse is administering a benzodiazepine sleep aid to an older adult. What should be the priority assessment for the patient? 1. Incontinence 2. Nausea and vomiting 3. Bradycardia 4. Respiratory depression

4. Respiratory depression

A nurse is planning a teaching program for a patient with a diagnosis of obstructive sleep apnea. Which should the nurse plan to discuss with this patient? 1. Using the ordered device that supports airway patency 2. Placing two pillows under the head when sleeping 3. Requesting a sedative to promote sleep 4. Sleeping in the supine position

4. Sleeping in the supine position A continuous positive airway pressure (CPAP) device worn when sleeping keeps the upper airway patent by maintaining an open pathway that facilitates gas exchange.

A patient with an order for bed rest has diaphoresis. What should the nurse use to best limit the negative effects of perspiration on dependent skin surfaces of this patient? 1. Ventilated heel protectors 2. Air-filled rings 3. Air mattress 4. Sheepskin

4. The soft tuffs of sheepskin allow air to circulate, thereby promoting the evaporation of moisture that can precipitate skin breakdown.

Which nursing action is most effective in relation to the concept Immobility can lead to occlusion of blood vessels in areas where bony prominences rest on a mattress? 1. Encouraging the patient to breathe deeply 10 times per hour 2. Performing range-of-motion exercises twice a day 3. Placing a sheepskin pad under the sacrum 4. Repositioning the patient every 2 hours

4. Turning a patient relieves pressure on the capillary beds of the dependent areas of the body, particularly the skin overlying bony prominences, which reestablishes blood flow to the area.

A nurse is caring for the following group of patients with infections. Which infection is classified as a hospital-acquired infection? 1. Respiratory infection contacted from a visitor 2. Vaginal canal infection in a postmenopausal woman 3. Urinary tract infection in a patient who is sedentary 4. Wound infection caused by unwashed hands of caregiver

4. Wound infection caused by unwashed hands of caregiver

A patient has dysphagia. Which nursing action takes priority when feeding this patient? 1. Ensuring that dentures are in place 2.Medicating for pain before providing meals 3.Providing verbal cueing to swallow each bite 4.Checking the mouth for emptying between every bite

4.Checking the mouth for emptying between every bite

A nurse in the nursing education department of a community hospital is planning an in-service education class about injury prevention.Which factor that most commonly causes injuries in hospitalized patients should be included in he teaching plan? 1.Malfunctioning equipment 2.Failure to use restraints 3.Visitors 4.Falls

4.Falls

What is an appropriately worded goal for a patient who is at risk for falling? The patient will be: 1.Taught how to call for help to ambulate." 2.Kept on bed rest when dizzy." 3.Restrained when agitated." 4.Free from trauma."

4.Free from trauma."

The nurse is caring for a confused patient. What should the nurse do to prevent this patient from falling? 1.Encourage the patient to use the corridor handrails 2. Place the patient in a room near the nurses' station 3.Reinforce how to use the call bell 4.Maintain close supervision

4.Maintain close supervision

Which time of day is of most concern for the nurse when trying to protect a patient with dementia from injury? 1.Afternoon 2.Morning 3.Evening 4.Night

4.Night

What is the correct order of steps for using belt restraints? 1. Place the restraint at the waist. 2. Bring the ties through the slots in the belt. 3. Make the patient roll to the side. 4. Remove the wrinkles in clothing. 5. Apply the belt over the clothes. 6. Place the patient in a sitting position. 7. Help the patient lie down in bed.

6, 5, 1, 4, 2, 7, 3 The first step for using belt restraints is placing the patient in a sitting position in the bed. The second step is applying the belt over the patient's clothes, gown, or pajamas. The next step is making sure to place the restraint at the waist, and not in the chest or abdomen. This step is followed by removing wrinkles or creases in the clothing. Then, the patient should be helped to lie down in bed. The last step is helping the patient roll to the side.

A patient complains of chest pain. When assessing the pain, you decide that its origin is cardiac - rather than respiratory or gastrointestinal - when it? A) Does not occur with respiratory variations. B) Is peripheral and may radiate to the scapular region. C) Is aggravated by inspiratory movements. D) Is non radiating and occurs during inspiration.

A

The nurse goes to assess a new patient and finds him lying supine in bed. The patient tells the nurse that he feels short of breath. Which nursing action should the nurse perform first? A) Raise the head of the bed to 45 degrees. B) Take his oxygen saturation with a pulse oximeter. C) Take his blood pressure and respiratory rate. D) Notify the health care provider of his shortness of breath.

A

The nurse needs to apply oxygen to a patient who has a precise oxygen level prescribed. Which of the following oxygen-delivery systems should the nurse select to administer the oxygen to the patient? A) Nasal cannula B) Venturi mask C) Simple face mask without inflated reservoir bag D) Plastic face mask with inflated reservoir bag

A

Two hours after surgery the nurse assesses a patient who had a chest tube inserted during surgery. There is 200 mL of dark-red drainage in the chest tube at this time. What is the appropriate action for the nurse to perform? A) Record the amount and continue to monitor drainage B) Notify the health care provider C) Strip the chest tube starting at the chest D) Increase the suction by 10 mm Hg

A

What noncritical item used requires a surface disinfection? 1 Endoscopes 2 Stethoscope 3 Intravascular catheter 4 Anesthesia equipment

A stethoscope is a noncritical item that requires disinfection. An endoscope is a semi-critical item that requires high-level disinfection. An intravascular catheter is a critical item that requires sterilization. Anesthesia equipment is a semi-critical item that requires high-level disinfection.

What are some interventions and the rationalizations to use that intervention to reduce the likelihood of a shear injury to a patient. keep bed at at 30 degree angle?

A transfer device can pick up a patient and prevent his or her skin from sticking to the bed sheet as he is repositioned. A second intervention would be to position the patient with the head of the bed to be elevated at 30 degrees, which prevents him or her from sliding. A third intervention would be to educate the patient and his or her caregiver on the importance of not sliding on the sheets when repositioning.

Which positioning aid prevents external rotation of the hips when the patient is in the supine position? 1 Thin pillow 2 Thick pillow 3 Trapeze bar 4 Trochanter roll

A trochanter roll prevents external rotation of the hips when a patient is in the supine position. A thin pillow or thick pillow would not be helpful for preventing the external rotation of the hips, but may lead to increased flexion when the appropriate pillow size is not taken. The trapeze bar allows the patient to raise the upper extremities to raise the trunk off the bed, which helps in decreasing the shearing action from sliding across or up and down the bed.

The nurse is caring for a postoperative patient. Which nursing action should be avoided if deep vein thrombosis (DVT) is suspected in an immobilized patient? 1 Assessing the feet for temperature 2 Measuring the calf and thigh circumference 3 Assessing for calf pain on dorsiflexion of the foot 4 Observing for loss of skin integrity in the lower extremities

A unilateral increase in calf circumference is an early indicator for deep vein thrombosis. Therefore, circumferences of the thigh and calf should be measured to assess for DVT. The temperature of the feet is not a reliable assessment in determining a DVT, but is not contraindicated for this patient. Assessing for calf pain on dorsiflexion of the foot is contraindicated for a patient who is suspected to have developed DVT. Loss of skin integrity is a sign of increased risk of DVT; therefore, skin assessment in the lower extremities is a correct action.

A nursing student has been written up several times for being late with providing patient care and for omitting aspects of patient care and not knowing basic procedures that were taught in the skills course one term earlier. The nursing student says, "I don't understand what the big deal is. As my instructor, you are there to protect me and make sure I don't make mistakes." What is the best response from the nursing instructor? 1. "You are expected to perform at the level of a professional nurse." 2. "You are expected to perform at the level of a nursing student." 3. "You are practicing under the license of the nurse assigned to the patient." 4. "You are expected to perform at the level of a skilled nursing assistant."

ANS: 1 Although nursing students are not employees of the health care agency where they are having their clinical experience, they are expected to perform as professional nurses would in providing safe patient care. Different levels of standards do not apply. Nursing students, just as nurses, provide safe, complete patient care, or they don't. No standard is used for nursing students other than that they must meet the standards of a professional nurse. The nursing instructor, not the nurse assigned to the patient, is responsible for the actions of the nursing student.

A nurse performs cardiopulmonary resuscitation (CPR) on a 92-year-old with brittle bones and breaks a rib during the procedure, which then punctures a lung. The patient recovers completely without any residual problems and sues the nurse for pain and suffering, and for malpractice. What key point will the prosecution attempt to prove? 1. The CPR procedure was done incorrectly. 2. The patient would have died if nothing was done. 3. The patient was resuscitated according to policy. 4. Patients with brittle bones might sustain fractures when chest compressions are done.

ANS: 1 Certain criteria are necessary to establish nursing malpractice. In this situation, although harm was caused, it was not because of failure of the nurse to perform a duty according to standards the way other nurses would have performed in the same situation. The nurse would have had to have done the procedure correctly, or the patient most likely would not have survived without any residual problems such as brain damage. The fact that the patient sustained injury as a result of age and physical status does not mean the nurse breached any duty to the patient. The nurse would need to make sure the defense attorney knew that the cardiopulmonary resuscitation (CPR) was done correctly. Without intervention, the patient most likely would not have survived. The prosecution would try to prove that a breach of duty had occurred, which had caused injury, not that cardiopulmonary resuscitation was done correctly. The defense team, not the prosecution, would explain the correlation between brittle bones and rib fractures during CPR.

The nurse calculates the medication dose for an infant on the pediatric unit and determines that the dose is twice what it should be. The pediatrician is contacted and says to administer the medication as ordered. What is the next action that the nurse should take? (Select all that apply.) 1. Notify the nursing supervisor. 2. Check the chain of command policy for such situations. 3. Give the medication as ordered. 4. Give the amount calculated to be correct. 5. Contact the pharmacy for clarification.

ANS: 1, 2 Nurses follow health care providers' orders unless they believe the orders are in error or may harm patients. Therefore, the nurse needs to assess all orders. If an order seems to be erroneous or harmful, further clarification from the health care provider is necessary. If the health care provider confirms an order and the nurse still believe that it is inappropriate, the nurse should inform the supervising nurse or follow the established chain of command. The supervising nurse should be able to help resolve the questionable order, but only the health care provider who wrote the order or a health care provider covering for the one who wrote the order can change the order. Harm to the infant could occur if the medication dosage was too high. The nurse cannot change an order. Giving the amount calculated to be correct would not be what another nurse would do in the same situation. Although the pharmacy is an excellent resource, only the health care provider can change the order.

A patient with sepsis as a result of long-term leukemia dies 25 hours after admission to the hospital. A full code was conducted without success. The patient had a urinary catheter, an intravenous line, an oxygen cannula, and a nasogastric tube. What question is priority for the nurse to ask the family before beginning postmortem care? 1. "Do you want to assist in bathing your loved one?" 2. "Is an autopsy going to be done?" 3. "To which funeral home do you want your loved one transported?" 4. "Do you want me to remove the lines and tubes before you see your loved one?"

ANS: 2 An autopsy or postmortem examination may be requested by the patient or the patient's family, as part of an institutional policy, or if required by law. Because the patient's death occurred as a result of long-term illness, not under suspicious circumstances, and more than 24 hours after admission to the hospital, whether to conduct a postmortem examination would be decided by the family, and consent would have to be obtained from the family. The nurse needs to know the policy to follow regarding removal of lines when an autopsy is to be done. Asking about bathing the deceased patient is a valid question but is not priority, because the nurse needs to know the protocol to follow if an autopsy is to be done. Finding out which funeral home the deceased patient is to be transported to is valid but is not priority, because other actions must be taken before the deceased patient is transported from the hospital. Removal of lines and tubes is not a decision made by the family if an autopsy is to be done. The nurse must first check the protocol to be followed.

A patient has just been told that he has approximately six months to live and asks about advance directives. Which statements by the nurse give the patient correct information? (Select all that apply.) 1. "You have the right to refuse treatment at any time." 2. "If you want certain procedures or actions taken or not taken, and you might not be able to tell anyone at the time, you need to complete documents ahead of time that give your health care provider this information." 3. "You will be resuscitated at any time to allow you the longest length of survival." 4. "You might want to think about choosing someone who will make medical decisions for you in the event that you are unable to make your desires known." 5. "We will get someone who knows the state's guidelines to assist you in setting up your living will." 6. "If you travel to another state, your living will should cover your wishes."

ANS: 1, 2, 4, 5 The ethical doctrine of autonomy ensures the patient the right to refuse medical treatment. Living wills are written documents that direct treatment in accordance with a patient's wishes in the event of a terminal illness or condition. With this legal document, the patient is able to declare which medical procedures he or she wants or does not want when terminally ill or in a persistent vegetative state. Each state providing for living wills has its own requirements for executing the health care proxy or durable power of attorney for health care (DPAHC). This is a legal document that designates a person or persons of one's choosing to make health care decisions when the patient is no longer able to make decisions on his or her own behalf. This agent makes health care treatment decisions based on the patient's wishes. Cardiopulmonary resuscitation (CPR) is an emergency treatment provided without patient consent. Health care providers perform CPR on an appropriate patient unless a do not resuscitate (DNR) order has been placed in the patient's chart. The statutes assume that all patients will be resuscitated unless a written DNR order is found in the chart. Legally competent adult patients can consent to a DNR order verbally or in writing after receiving appropriate information from the health care provider. Differences among the states have been noted regarding advance directives, so the patient should check state laws to see if a state will honor an advance directive that was originated in another state.

The nurse hears a physician say to the charge nurse that he doesn't want that same nurse caring for his patients because she is stupid and won't follow his orders. The physician also writes on his patient's medical records that the same nurse, by name, is not to care for any of his patients because of her incompetence. What component(s) of defamation has the physician committed? (Select all that apply.) 1. Slander 2. Invasion of privacy 3. Libel 4. Assault 5. Battery

ANS: 1, 3 Slander occurred when the physician spoke falsely about the nurse, and libel occurred when the physician wrote false information in the chart. Both of these situations could cause problems for the nurse's reputation. Invasion of privacy is the release of a patient's medical information to an unauthorized person such as a member of the press, the patient's employer, or the patient's family. Assault is any action that places a person in apprehension of a harmful or offensive contact without consent. No actual contact is necessary. Battery is any intentional touching without consent.

A nurse gives an incorrect medication to a patient without doing all of the mandatory checks, but the patient has no ill effects from the medication. What actions should the nurse take after reassessing the patient? (Select all that apply.) 1. Notify the health care provider of the situation. 2. Document in the patient's medical record that an occurrence report was filed. 3. Document in the patient's medical record why the omission occurred. 4. Discuss what happened with all of the other nurses and staff on the unit. 5. Continue to monitor the patient for any untoward effects from the medication. 6. Send an occurrence report to risk management after completing it.

ANS: 1, 5, 6 Examples of an occurrence include an error in technique or procedure such as failing to properly identify a patient. Institutions generally have specific guidelines to direct health care providers how to complete the occurrence report. The report is confidential and separate from the medical record. The nurse is responsible for providing information in the medical record about the occurrence. It is also best for the nurse to discuss the occurrence with nursing management only. The risk management department of the institution also requires complete documentation. The fact that an occurrence report was completed is not documented in the patient's medical record. No discussion of why the omission in procedure occurred should be documented in the patient's medical record. Errors should be discussed only with those who need to know such as the health care provider, appropriate administrative personnel, and risk management.

A 17-year-old patient, dying of heart failure, wants to have his organs removed for transplantation after his death. What action by the nurse is correct? 1. Prepare the organ donation form for the patient to sign while he is still oriented. 2. Instruct the patient to talk with his parents about his desire to donate his organs. 3. Notify the physician about the patient's desire to donate his organs. 4. Contact the United Network for Organ Sharing after talking with the patient.

ANS: 2 An individual over age 18 may sign the form allowing organ donation upon death. In this situation, the parents would need to sign the form because the teenager is under age 18. The nurse cannot allow the patient to sign the organ donation document because he is younger than age 18. The physician will be notified about the patient's wishes after the parents agree to donate the organs. The nurse caring for the patient does not contact the United Network for Organ Sharing. A transplant coordinator will be the liaison for this organization.

Conjoined twins are in the neonatal department of the community hospital until transfer to the closest medical center. A photographer from the local newspaper gets off the elevator on the neonatal floor and wants to take pictures of the infants. What initial action should the nurse take? 1. Escort the cameraman to the neonatal unit while a few pictures are taken quietly. 2. Tell the cameraman where the hospital's public relations department is located. 2. Ask the cameraman to wait while permission is obtained from the physician. 4. Ask the cameraman how the pictures are to be used in the local newspaper.

ANS: 2 In some cases, information about a scientific discovery or a major medical breakthrough or an unusual situation is newsworthy. In this case, anyone seeking information needs to contact the hospital's public relations department to ensure that invasion of privacy does not occur. It is not the nurse's responsibility to decide independently the legality of disclosing information. The nurse does not have the right to allow the cameraman access to the neonatal unit. This would constitute invasion of privacy. The physician has no responsibility regarding this situation and cannot allow the cameraman on the unit. It is not the nurse's responsibility to find out how the pictures are to be used. This is a task for the public relations department.

A nurse works full-time on the oncology unit at the hospital and works part-time on weekends giving immunizations at the local pharmacy. While giving an injection on a weekend, the nurse caused injury to the patient's arm and is now being sued. How will the hospital's malpractice insurance provide coverage for this nurse? 1. It will provide coverage as long as the nurse followed all procedures, protocols, and policies correctly. 2. The hospital's malpractice insurance covers this nurse only during the time the nurse is working at the hospital. 3. As long as the nurse has never been sued before this incident, the hospital's malpractice insurance will cover the nurse. 4. The hospital's malpractice insurance will provide approximately 50% of the coverage the nurse will need.

ANS: 2 Malpractice insurance provided by the employing institution covers nurses only while they are working within the scope of their employment at that institution. It is always wise to find out if malpractice insurance is provided by a secondary place of employment, in this case, the pharmacy, or the nurse should carry an individual malpractice policy to cover situations such as this.

A pediatric oncology nurse floats to an orthopedic trauma unit. What actions should the nurse manager of the orthopedic unit take to enable safe care to be given by this nurse? 1. Provide a complete orientation to the functioning of the entire unit. 2. Determine patient acuity and care the nurse can safely provide. 3. Allow the nurse to choose which meal time she would like. 4. Assign nursing assistive personnel to assist her with care.

ANS: 2 Nurses who float need to inform the supervisor of any lack of experience in caring for the type of patients on the nursing unit. They also need to request and receive an orientation to the unit. Supervisors are liable if they give a staff nurse an assignment that he or she cannot safely handle. Before accepting employment, learn the policies of the institution regarding floating, and have an understanding as to what is expected. A basic orientation is needed, whereas a complete orientation of the functioning of the entire unit would take a period of time that would exceed what the nurse has to spend on orientation. Allowing the nurse to choose which meal time she would like is a nice gesture of thanks for the nurse, but it does not enable safe care. Having nursing assistive personnel may help the nurse complete basic tasks such as hygiene and turning, but it does not enable safe nursing care that she is ultimately responsible for.

A newly hired experienced nurse is preparing to change a patient's abdominal dressing and hasn't done it before at this hospital. Which action by the nurse is best? 1. Ask another nurse to do it so the correct method can be viewed. 2. Check the policy and procedure manual for the agency's method. 3. Change the dressing using the method taught in nursing school. 4. Ask the patient how the dressing change has been recently done.

ANS: 2 The Joint Commission requires accredited hospitals to have written nursing policies and procedures. These internal standards of care are specific and need to be accessible on all nursing units. For example, a policy/procedure outlining the steps to follow when changing a dressing or administering medication provides specific information about how nurses are to perform. The nurse being observed may not be doing the procedure according to the agency's policy or procedure. The procedure taught in nursing school may not be consistent with the policy or procedure for this agency. The patient is not responsible for maintaining the standards of practice. Patient input is important, but it's not what directs nursing practice.

A nurse notices that his neighbor's preschool children are often playing on the sidewalk and in the street unsupervised and repeatedly takes them back to their home and talks with the parent available, but the situation continues. What immediate action by the nurse is mandated by law? 1. Talk with both parents about safety needs of their children. 2. Contact the appropriate community child protection agency. 3. Tell the parents that the authorities will be contacted shortly. 4. Take pictures of the children to support the overt child abuse.

ANS: 2 The nurse has a duty to report this situation to protect the children. Any health care professional who does not report suspected child abuse or neglect may be liable for civil or criminal legal action. The person making the report has legal immunity if the report is made in good faith. Talking with the parents is not mandated by law. There is no obligation to tell the parents that they will be reported to authorities. There is no obligation for the nurse to take pictures of the children.

A confused patient with a urinary catheter, nasogastric tube, and intravenous line keeps touching these needed items for care. The nurse has tried to explain to the patient that he should not touch these lines, but the patient continues. What is the best action by the nurse at this time? 1. Apply restraints loosely on the patient's dominant wrist. 2. Try other approaches to prevent the patient from touching these care items. 3. Notify the health care provider that restraints are needed immediately to maintain the patient's safety. 4. Allow the patient to pull out lines to prove that the patient needs to be restrained.

ANS: 2 The risks associated with the use of restraints are serious. A restraint-free environment is the first goal of care for all patients. Many alternatives to the use of restraints are available, and the nurse should try all of them before notifying the patient's health care provider. The situation states that the patient is touching the items, not trying to pull them out. At this time, the patient's well-being is not at risk. The nurse will have to check on the patient frequently and then will determine if the health care provider needs to be informed of the situation. Restraints can be used (1) only to ensure the physical safety of the resident or other residents, (2) when less restrictive interventions are not successful, and (3) only on the written order of a health care provider. The health care provider needs to know the situation but also needs to know that all approaches possible have been used before writing an order for restraints. Allowing the patient to pull out any of these items could cause harm to the patient.

A patient's condition is slowly deteriorating. What actions should the nurse take to provide the best care possible? (Select all that apply.) 1. Allow the nursing student to receive verbal orders from the physician in the room while the nurse is in the medication area down the hall. 2. Document the patient's status changes in the medical record in a timely manner. 3. Document that the health care provider has been notified of the specific patient status, including date and time that messages were left. 4. Check the chart for frequent orders. 5. Omit charting what the health provider's response is to notification of the patient's status change.

ANS: 2, 3 Clear, concise, and timely communication is essential whenever charting in the patient's medical record occurs. Nursing students are not permitted to receive verbal orders. Documentation regarding communication with the health care provider must contain what was communicated by the nurse and the health care provider, orders if given, date, time, and identification of who is documenting the situation.

An obstetrical nurse comes across an automobile accident. The patient seems to have a crushed upper airway, and while waiting for emergency medical services to arrive, the nurse makes a cut in the trachea and inserts a straw from her purse to provide an airway. The patient survives and has a permanent problem with his vocal cords, making it difficult to talk. Which statement is true regarding the nurse's performance? 1. The nurse acted appropriately and saved the patient's life. 2. The nurse acted within the guidelines of the Good Samaritan Law. 3. The nurse took actions beyond those that are standard and appropriate. 4. The nurse should have just stayed with the patient and waited for help.

ANS: 3 An obstetric nurse would not have been trained in performing a tracheostomy or a cricotomy, and doing so would be beyond what she has been trained or educated to do. The nurse did not do what another nurse would have done in the same situation. The nurse is not protected by the Good Samaritan Law because she acted outside of her scope of practice and training. The nurse should have acted within what she was trained and educated to do in this circumstance, not just stay with the patient.

A recent immigrant who does not speak English is alert and requires hospitalization. What is the initial action that the nurse must take to enable informed consent to be obtained? 1. Ask a family member to translate what the nurse is saying. 2. Notify the health care provider that the patient doesn't speak English. 3. Request an official interpreter to explain the terms of consent. 4. Use hand gestures and medical equipment while explaining in English.

ANS: 3 An official interpreter must be present to explain the terms of consent if a patient speaks only a foreign language. A family member or acquaintance who speaks a patient's language should not interpret health information. Family members can tell those caring for the patient what the patient is saying, but privacy regarding the patient's condition, assessment, etc., must be protected. There is no way that the nurse can know that the family member is translating exactly what the nurse is saying. Privacy must be ensured and accurate information must be provided to the patient. After consent is obtained for treatment, the health care provider would be notified because little can be done without consent. The health care provider needs to have the translator available during the history and physical, as well as at other times, but the first step is to get a translator to obtain informed consent because this is not an emergency situation. Using hand gestures and medical equipment is inappropriate when communicating with a patient who does not understand the language spoken. Certain hand gestures may be acceptable in one culture and not appropriate in another. The medical equipment may be unknown and frightening to the patient, and the patient still doesn't understand what is being said.

A new nurse notes that the health care unit keeps a listing of patient names in a closed book behind the front desk of the nursing station so patients can be located easily. What action is most appropriate for the nurse to take? 1. Move the book to the upper ledge of the nursing station for easier access. 2. Talk with the nurse manager about the listing being a violation of the Health Insurance Portability and Accountability Act (HIPAA). 3. Use the book as needed while keeping it away from individuals not involved in patient care. 4. Ask the nurse manager to move the book to a more secluded area.

ANS: 3 The privacy section of the HIPAA provides standards regarding accountability in the health care setting. These rules include patient rights to consent to the use and disclosure of their protected health information, to inspect and copy their medical record, and to amend mistaken or incomplete information. This document limits who is able to access a patient's record. It establishes the basis for privacy and confidentiality about patients in any manner. The book is located where only staff would have access. It is not the responsibility of the new nurse to move items used by others on the patient unit. The listing is protected as long as it is used appropriately as needed to provide care. There is no need to move the book to a more secluded area.

While recovering from a severe illness, a hospitalized patient states that he wants to change his living will, which he signed nine months ago. Which response by the nurse is most appropriate? 1. "Check with your admitting health care provider whether a copy is on your chart." 2. "Have you talked with your attorney recently about a living will?" 3. "Your living will can be changed only once each calendar year." 4. "Let me check with someone here in the hospital who can assist you."

ANS: 4 Each health care facility has personnel who are familiar with the state laws and can assist the patient in revising a living will. They may be in the admissions or risk management department. Checking with the health care provider about the presence of a living will on the chart has nothing to do with the patient's desire to change the living will. The question states that the patient wants to change his living will. Asking whether he has talked to his lawyer recently is a closed-ended question that passes the responsibility to someone else, that is, the attorney, and does not address the patient's current desire to change the living will. It is the nurse's responsibility to find an appropriate person in the facility to assist the patient. A living will can be changed whenever the patient decides to change it, as long as the patient is competent.

A nursing student in the final term of nursing school is overheard by a nursing faculty member telling another student that she got to insert a nasogastric tube in the emergency department while she was working as a nursing assistant. What advice is best for the nursing faculty member to give to the nursing student? 1. "Just be careful when you are doing new procedures and make sure you are following directions by the nurse." 2. "Review your procedures before you go to work, so you will be prepared to do them if you have a chance." 3. "The nurse should not have allowed you to insert the nasogastric tube because something bad could have happened." 4. "You are not allowed to perform any procedures other than those in your job description even with the nurse's permission."

ANS: 4 When nursing students work as nursing assistants or nurse's aides when not attending classes, they should not perform tasks that do not appear in a job description for a nurse's aide or assistant. The nursing student should always follow the directions of the nurse, unless doing so violates the institution's guidelines or job description under which the nursing student was hired. The nursing student should be able to safely complete the procedures delegated as a nursing assistant, and reviewing those not done recently is a good idea, but it has nothing to do with the situation. This option does not address the situation that the nursing student acted outside the job description for the nursing assistant position. The focus of the discussion between the nursing faculty member and the nursing student should be on following the job description under which the nursing student is working.

The nurse is caring for an elderly patient admitted with nausea, vomiting, and diarrhea. Upon completing the health history, which priority concern would require collaboration with social services to address the patient's health care needs? a. The electricity was turned off 2 days ago. b. The water comes from the county water supply. c. A son and family recently moved into the home. d. The home is not furnished with a microwave oven.

ANS: A Electricity is needed for refrigeration of food, and lack of electricity could have contributed to the nausea, vomiting, and diarrhea—potential food poisoning. This discussion about the patient's electrical needs can be referred to social services. The water supply, the increased number of individuals in the home, and not having a microwave may or may not be concerns but do not pertain to the current health care needs of this patient.

The patient has been diagnosed with a respiratory illness and complains of shortness of breath. The nurse adjusts the temperature to facilitate the comfort of the patient. What is the usual comfort range for most patients? a. 65° F to 75° F b. 60° F to 75° F c. 15° C to 17° C d. 25° C to 28° C

ANS: A The comfort zone for most individuals is the range between 65° F and 75° F (18.3° C to 23.9° C). The other ranges do not reflect the average person's comfort zone.

The nurse is preparing a patient for surgery. The nurse explains that the reason for writing in indelible ink on the surgical site the word "correct" is to a. Distinguish the correct surgical site. b. Label the correct patient. c. Comply with the surgeon's preference. d. Adhere to the correct regulatory standard.

ANS: A The purpose of writing on the surgical site as part of the Universal Protocol from the Joint Commission is to distinguish the correct site on the correct patient and match with the correct surgeon for patient safety and prevention of wrong site surgery. All patients who are having an invasive procedure should receive labeling in many different ways, including the record and patient armbands. Writing in indelible ink may comply with the surgeon's preference, but safety is the driving factor. Although labeling of the site helps to meet regulatory standards, this is not the reason to do this activity—the reason is to keep the patient safe

A nursing student has been written up several times for being late with providing patient care and for omitting aspects of patient care and not knowing basic procedures that were taught in the skills course one term earlier. The nursing student says, "I don't understand what the big deal is. As my instructor, you are there to protect me and make sure I don't make mistakes." What is the best response from the nursing instructor? a. "You are expected to perform at the level of a professional nurse." b. "You are expected to perform at the level of a nursing student." c. "You are practicing under the license of the nurse assigned to the patient." d. "You are expected to perform at the level of a skilled nursing assistant."

ANS: A Although nursing students are not employees of the health care agency where they are having their clinical experience, they are expected to perform as professional nurses would in providing safe patient care. Different levels of standards do not apply. Nursing students, just as nurses, provide safe, complete patient care, or they don't. No standard is used for nursing students other than that they must meet the standards of a professional nurse. The nursing instructor, not the nurse assigned to the patient, is responsible for the actions of the nursing student.

A nurse performs cardiopulmonary resuscitation (CPR) on a 92-year-old with brittle bones and breaks a rib during the procedure, which then punctures a lung. The patient recovers completely without any residual problems and sues the nurse for pain and suffering, and for malpractice. What key point will the prosecution attempt to prove? a. The CPR procedure was done incorrectly. b. The patient would have died if nothing was done. c. The patient was resuscitated according to policy. d. Patients with brittle bones might sustain fractures when chest compressions are done.

ANS: A Certain criteria are necessary to establish nursing malpractice. In this situation, although harm was caused, it was not because of failure of the nurse to perform a duty according to standards the way other nurses would have performed in the same situation. The nurse would have had to have done the procedure correctly, or the patient most likely would not have survived without any residual problems such as brain damage. The fact that the patient sustained injury as a result of age and physical status does not mean the nurse breached any duty to the patient. The nurse would need to make sure the defense attorney knew that the cardiopulmonary resuscitation (CPR) was done correctly. Without intervention, the patient most likely would not have survived. The prosecution would try to prove that a breach of duty had occurred, which had caused injury, not that cardiopulmonary resuscitation was done correctly. The defense team, not the prosecution, would explain the correlation between brittle bones and rib fractures during CPR.

28. The nurse is assessing a patient for opioid tolerance. Which finding supports the nurse's assessment? a. Increasingly higher doses of opioid are needed to control pain. b. The patient needed a substantial dose of naloxone (Narcan). c. The patient asks for pain medication close to the time it is due around the clock. d. The patient no longer experiences sedation from the usual dose of opioid.

ANS: A Opioid tolerance occurs when a patient needs higher doses of an opioid to control pain. Naloxone (Narcan) is an opioid antagonist that is given to reverse the effects of opioid overdose. Taking pain medications regularly around the clock is an effective way to control pain. The pain medication for this patient is most likely effectively managing the patient's pain because the patient is not asking for the medication before it is due. A patient no longer experiencing a side effect of an opioid does not indicate opioid tolerance.

8. A 24-year-old Asian woman is in labor and refuses to receive any sort of anesthesia medication. Which alternative treatment is best for this patient? a. Relaxation and guided imagery b. Transcutaneous electrical nerve stimulation (TENS) c. Herbal supplements with analgesic effects d. Pudendal block

ANS: A Some cultures prefer nonpharmacological measures for pain control. In the case of a patient in labor, relaxation with guided imagery is often an effective supplement for pain management because it provides women with a sense of control over their pain. Relaxation and guided imagery can be used during any phase of health or illness. TENS units are typically used to manage postsurgical and procedural pain. Herbal supplements need to be evaluated for safety during pregnancy. Additionally, some patients consider herbal supplements to be another form of medication, and they are not typically used to control acute pain. A pudendal block is a type of regional anesthesia; use of it does not respect the patient's wishes for nonpharmacological pain control.

3. Which of the following statements made by a patient reflects that the patient understands the relationship between the gate control theory of pain and the use of meditation to relieve pain? a. "Meditation controls pain by blocking pain impulses from coming through the gate." b. "Meditation will help me sleep through the pain because it opens the gate." c. "Meditation stops the occurrence of pain stimuli." d. "Meditation alters the chemical composition of pain neuroregulators, which closes the gate."

ANS: A The gate theory states that pain impulses cause pain when they get through gates that are open. Pain is blocked when the gates are closed. Nonpharmacologic pain relief measures, such as meditation, work by closing the gates, which keeps pain impulses from coming through. Meditation does not open pain gates or stop pain from occurring. Meditation also does not have an effect on pain neuroregulators.

29. A nurse is caring for a patient with rheumatoid arthritis who is now going to be taking 2 acetaminophen (Tylenol) tablets every 6 hours to control pain. Which part of the patient's social history is the nurse most concerned about? a. Patient drinks 1 to 2 glasses of wine every night. b. Patient smokes 2 packs of cigarettes a day. c. Patient occasionally smokes marijuana. d. Patient takes antianxiety medications.

ANS: A The major adverse effect of acetaminophen is hepatotoxicity. Because both alcohol and acetaminophen are metabolized by the liver, when taken together, they can cause liver damage. Smoking cigarettes and smoking marijuana are not healthy behaviors, but their effects on health are not affected by acetaminophen. Antianxiety medications can be taken with acetaminophen.

18. The nurse is evaluating the effectiveness of guided imagery for pain management as used for a patient who has second- and third-degree burns and needs extensive dressing changes. Which statement best describes that guided imagery is effectively controlling the patient's pain during dressing changes? a. The patient's need for analgesic medication decreases during the dressing changes. b. The patient rates pain during the dressing change as a 6 on a scale of 0 to 10. c. The patient's facial expressions are stoic during the procedure. d. The patient can tolerate more pain, so dressing changes can be performed more frequently.

ANS: A The purpose of guided imagery is to allow the patient to alter the perception of pain. Guided imagery works in conjunction with analgesic medications, potentiating their effects. If the patient needs less pain medication during dressing changes, then guided imagery is helping to manage the patient's pain. A rating of 6 on a 0 to 10 scale indicates that the patient is having moderate pain and shows that this patient is not experiencing pain relief at this time. A person who is stoic is not showing feelings, which makes it difficult to know whether or not the patient is experiencing pain. The ability to change dressings more frequently is not a way to evaluate the effectiveness of guided imagery.

The nurse calculates the medication dose for an infant on the pediatric unit and determines that the dose is twice what it should be. The pediatrician is contacted and says to administer the medication as ordered. What is the next action that the nurse should take? (Select all that apply.) a. Notify the nursing supervisor. b. Check the chain of command policy for such situations. c. Give the medication as ordered. d. Give the amount calculated to be correct. e. Contact the pharmacy for clarification.

ANS: A, B Nurses follow health care providers' orders unless they believe the orders are in error or may harm patients. Therefore, the nurse needs to assess all orders. If an order seems to be erroneous or harmful, further clarification from the health care provider is necessary. If the health care provider confirms an order and the nurse still believe that it is inappropriate, the nurse should inform the supervising nurse or follow the established chain of command. The supervising nurse should be able to help resolve the questionable order, but only the health care provider who wrote the order or a health care provider covering for the one who wrote the order can change the order. Harm to the infant could occur if the medication dosage was too high. The nurse cannot change an order. Giving the amount calculated to be correct would not be what another nurse would do in the same situation. Although the pharmacy is an excellent resource, only the health care provider can change the order.

A patient has just been told that he has approximately six months to live and asks about advance directives. Which statements by the nurse give the patient correct information? (Select all that apply.) a. "You have the right to refuse treatment at any time." b. "If you want certain procedures or actions taken or not taken, and you might not be able to tell anyone at the time, you need to complete documents ahead of time that give your health care provider this information." c. "You will be resuscitated at any time to allow you the longest length of survival." d. "You might want to think about choosing someone who will make medical decisions for you in the event that you are unable to make your desires known." e. "We will get someone who knows the state's guidelines to assist you in setting up your living will." f. "If you travel to another state, your living will should cover your wishes."

ANS: A, B, D, E The ethical doctrine of autonomy ensures the patient the right to refuse medical treatment. Living wills are written documents that direct treatment in accordance with a patient's wishes in the event of a terminal illness or condition. With this legal document, the patient is able to declare which medical procedures he or she wants or does not want when terminally ill or in a persistent vegetative state. Each state providing for living wills has its own requirements for executing the health care proxy or durable power of attorney for health care (DPAHC). This is a legal document that designates a person or persons of one's choosing to make health care decisions when the patient is no longer able to make decisions on his or her own behalf. This agent makes health care treatment decisions based on the patient's wishes. Cardiopulmonary resuscitation (CPR) is an emergency treatment provided without patient consent. Health care providers perform CPR on an appropriate patient unless a do not resuscitate (DNR) order has been placed in the patient's chart. The statutes assume that all patients will be resuscitated unless a written DNR order is found in the chart. Legally competent adult patients can consent to a DNR order verbally or in writing after receiving appropriate information from the health care provider. Differences among the states have been noted regarding advance directives, so the patient should check state laws to see if a state will honor an advance directive that was originated in another state.

1. The nurse is administering ibuprofen (Advil) to an older patient. Which of the following assessment data causes the nurse to hold the medication? (Select all that apply.) a. Past medical history of gastric ulcer b. Patient states last bowel movement was 4 days ago c. Stated allergy to aspirin d. Patient states has 2/10 intermittent joint pain e. Patient experienced respiratory depression after administration of an opioid medication

ANS: A, C NSAIDs can cause bleeding, especially in the gastrointestinal (GI) tract; therefore, NSAIDs are most likely contraindicated in this patient. Patients with an allergy to aspirin are sometimes also allergic to other NSAIDs. The nurse needs to verify that the health care provider is aware of the history of GI bleeding and of allergy to aspirin before administering ibuprofen. NSAIDs do not interfere with bowel function and are used for the treatment of mild to moderate acute intermittent pain. NSAIDs also do not suppress the central nervous system.

A nurse gives an incorrect medication to a patient without doing all of the mandatory checks, but the patient has no ill effects from the medication. What actions should the nurse take after reassessing the patient? (Select all that apply.) a. Notify the health care provider of the situation. b. Document in the patient's medical record that an occurrence report was filed. c. Document in the patient's medical record why the omission occurred. d. Discuss what happened with all of the other nurses and staff on the unit. e. Continue to monitor the patient for any untoward effects from the medication. f. Send an occurrence report to risk management after completing it.

ANS: A, E, F Examples of an occurrence include an error in technique or procedure such as failing to properly identify a patient. Institutions generally have specific guidelines to direct health care providers how to complete the occurrence report. The report is confidential and separate from the medical record. The nurse is responsible for providing information in the medical record about the occurrence. It is also best for the nurse to discuss the occurrence with nursing management only. The risk management department of the institution also requires complete documentation. The fact that an occurrence report was completed is not documented in the patient's medical record. No discussion of why the omission in procedure occurred should be documented in the patient's medical record. Errors should be discussed only with those who need to know such as the health care provider, appropriate administrative personnel, and risk management.

A homeless adult patient presents to the emergency department. The nurse obtains the following vital signs: temperature 94.8° F, blood pressure 100/56, apical pulse 56, respiratory rate 12. Which of the vital signs should be addressed immediately? a. Respiratory rate b. Temperature c. Apical pulse d. Blood pressure

ANS: B Hypothermia is defined as a core body temperature of 95° F or below. Homeless individuals are more at risk for hypothermia owing to exposure to the elements.

11. A nurse is caring for a patient who recently had spinal surgery. The nurse knows that patients usually experience acute pain following this type of surgery. The patient refuses to get up and walk and is not moving around in the bed. However, the patient is stoic and denies experiencing pain at this time. What most likely explains this patient's behavior and response to surgery? a. The surgery successfully cured the patient's pain. b. The patient's culture is possibly influencing the patient's experience of pain. c. The patient is experiencing urinary retention because of manipulation of the spine during surgery; this is preventing the patient from experiencing pain. d. The nurse is allowing personal beliefs about pain to influence pain management at this time.

ANS: B A patient's culture often influences the patient's expression of pain. In this case, the patient has just had surgery, and the nurse knows that this surgical procedure usually causes patients to experience pain. It is important at this time for the nurse to examine cultural and ethnic factors that are possibly affecting the patient's lack of expression of pain at this time. Even if surgery corrects neurological factors that create chronic pain, surgery causes pain in the acute period. Urinary retention usually creates pain and does not mask surgical pain. The nurse is not allowing personal beliefs to influence pain management because the nurse is attempting to determine the reason why the patient is not verbalizing the experience of pain.

4. A nursing student is planning care for an elderly patient who is experiencing pain. Which of the following statements made by the nursing student indicates the need for the nursing professor to clarify the nursing student's knowledge? a. "Older patients often have difficulty determining what is causing their pain." b. "It is safe to administer opioids to older adults as long as you start with small doses and frequently assess the patient's response to the medication." c. "As adults age, their ability to perceive pain decreases." d. "Patients who have dementia probably experience pain, and their pain is not always well controlled."

ANS: B Aging does not affect the ability to perceive pain. Sometimes older adults have difficulty interpreting their pain and determining its cause because multiple diseases and vague symptoms affect similar parts of the body. Opioids are safe to use in older adults as long as they are slowly titrated and the nurse frequently monitors the patient. Current evidence shows that patients with dementia most likely experience unrelieved pain because their pain is difficult to assess.

A patient with sepsis as a result of long-term leukemia dies 25 hours after admission to the hospital. A full code was conducted without success. The patient had a urinary catheter, an intravenous line, an oxygen cannula, and a nasogastric tube. What question is priority for the nurse to ask the family before beginning postmortem care? a. "Do you want to assist in bathing your loved one?" b. "Is an autopsy going to be done?" c. "To which funeral home do you want your loved one transported?" d. "Do you want me to remove the lines and tubes before you see your loved one?"

ANS: B An autopsy or postmortem examination may be requested by the patient or the patient's family, as part of an institutional policy, or if required by law. Because the patient's death occurred as a result of long-term illness, not under suspicious circumstances, and more than 24 hours after admission to the hospital, whether to conduct a postmortem examination would be decided by the family, and consent would have to be obtained from the family. The nurse needs to know the policy to follow regarding removal of lines when an autopsy is to be done. Asking about bathing the deceased patient is a valid question but is not priority, because the nurse needs to know the protocol to follow if an autopsy is to be done. Finding out which funeral home the deceased patient is to be transported to is valid but is not priority, because other actions must be taken before the deceased patient is transported from the hospital. Removal of lines and tubes is not a decision made by the family if an autopsy is to be done. The nurse must first check the protocol to be followed.

A 17-year-old patient, dying of heart failure, wants to have his organs removed for transplantation after his death. What action by the nurse is correct? a. Prepare the organ donation form for the patient to sign while he is still oriented. b. Instruct the patient to talk with his parents about his desire to donate his organs. c. Notify the physician about the patient's desire to donate his organs. d. Contact the United Network for Organ Sharing after talking with the patient.

ANS: B An individual over age 18 may sign the form allowing organ donation upon death. In this situation, the parents would need to sign the form because the teenager is under age 18. The nurse cannot allow the patient to sign the organ donation document because he is younger than age 18. The physician will be notified about the patient's wishes after the parents agree to donate the organs. The nurse caring for the patient does not contact the United Network for Organ Sharing. A transplant coordinator will be the liaison for this organization.

22. The nurse knows that which technique is best for assessing pain in a child who is 4 years of age? a. Ask the parents if they think their child is in pain. b. Use the FACES scale. c. Ask the child to rate the level of pain on a 0 to 10 pain scale. d. Check to see what previous nurses have charted.

ANS: B Assessing pain intensity in children requires special techniques. Young children often have difficulty expressing their pain. The FACES scale assesses pain in children who are verbal. Because a 4-year-old is verbal, this is an appropriate scale to use with this child. Parents' statement of pain is not an effective way to assess pain in children because children's statements are the most important. The 0 to 10 pain scale is too difficult for a 4-year-old child to understand. Previous documentation by nurses will tell you what the child's pain has been but will not tell you the child's current pain intensity.

25. The nurse is assessing how a patient's pain is affecting mobility. Which assessment question is most appropriate? a. "Have you considered working with a physical therapist?" b. "What activities, if any, has your pain prevented you from doing?" c. "Would you please rate your pain on a scale from 1 to 10 for me?" d. "What effect does your pain medication typically have on your pain?"

ANS: B Because the nurse is interested in knowing whether the patient's pain is affecting mobility, the priority assessment question is to ask the patient how the pain affects his or her ability to participate in normal activities of daily living. Although a physical therapist is a good resource to have, especially if pain is severely affecting mobility, considering working with a physical therapist does not describe the effect of pain on the patient's mobility. Assessing quality of pain and effectiveness of pain medication does not help the nurse to understand how it is affecting the patient's mobility

23. Which statement made by a nursing educator best explains why it is important for nurses to determine a patient's medical history and recent drug use? a. "Health care providers have a responsibility to prevent drug seekers from gaining access to drugs." b. "This information is useful in determining what type of pain interventions will most likely be effective in providing pain relief." c. "Some recreational drugs have pharmaceutical counterparts that may be more effective in managing pain." d. "Getting this information gives the nurse an opportunity to provide patient teaching about drug abstinence."

ANS: B In providing effective pain management, it is important to understand the patient's history, what drugs the patient has already tried, and what interventions work best or have negative actions. It is not the nurse's responsibility to judge or question a patient's pain or label her as a drug seeker. Nurses need to avoid labeling patients as drug seekers because this term is poorly defined and creates bias and prejudice among other health care providers. Although certain recreational drugs do have pharmaceutical counterparts, this is not the sole purpose of assessing drug use. The nurse needs more information beyond a patient's medical and medication history to determine whether a patient needs teaching about drug abstinence.

Conjoined twins are in the neonatal department of the community hospital until transfer to the closest medical center. A photographer from the local newspaper gets off the elevator on the neonatal floor and wants to take pictures of the infants. What initial action should the nurse take? a. Escort the cameraman to the neonatal unit while a few pictures are taken quietly. b. Tell the cameraman where the hospital's public relations department is located. c. Ask the cameraman to wait while permission is obtained from the physician. d. Ask the cameraman how the pictures are to be used in the local newspaper.

ANS: B In some cases, information about a scientific discovery or a major medical breakthrough or an unusual situation is newsworthy. In this case, anyone seeking information needs to contact the hospital's public relations department to ensure that invasion of privacy does not occur. It is not the nurse's responsibility to decide independently the legality of disclosing information. The nurse does not have the right to allow the cameraman access to the neonatal unit. This would constitute invasion of privacy. The physician has no responsibility regarding this situation and cannot allow the cameraman on the unit. It is not the nurse's responsibility to find out how the pictures are to be used. This is a task for the public relations department.

A nurse works full-time on the oncology unit at the hospital and works part-time on weekends giving immunizations at the local pharmacy. While giving an injection on a weekend, the nurse caused injury to the patient's arm and is now being sued. How will the hospital's malpractice insurance provide coverage for this nurse? a. It will provide coverage as long as the nurse followed all procedures, protocols, and policies correctly. b. The hospital's malpractice insurance covers this nurse only during the time the nurse is working at the hospital. c. As long as the nurse has never been sued before this incident, the hospital's malpractice insurance will cover the nurse. d. The hospital's malpractice insurance will provide approximately 50% of the coverage the nurse will need.

ANS: B Malpractice insurance provided by the employing institution covers nurses only while they are working within the scope of their employment at that institution. It is always wise to find out if malpractice insurance is provided by a secondary place of employment, in this case, the pharmacy, or the nurse should carry an individual malpractice policy to cover situations such as this.

A pediatric oncology nurse floats to an orthopedic trauma unit. What actions should the nurse manager of the orthopedic unit take to enable safe care to be given by this nurse? a. Provide a complete orientation to the functioning of the entire unit. b. Determine patient acuity and care the nurse can safely provide. c. Allow the nurse to choose which meal time she would like. d. Assign nursing assistive personnel to assist her with care.

ANS: B Nurses who float need to inform the supervisor of any lack of experience in caring for the type of patients on the nursing unit. They also need to request and receive an orientation to the unit. Supervisors are liable if they give a staff nurse an assignment that he or she cannot safely handle. Before accepting employment, learn the policies of the institution regarding floating, and have an understanding as to what is expected. A basic orientation is needed, whereas a complete orientation of the functioning of the entire unit would take a period of time that would exceed what the nurse has to spend on orientation. Allowing the nurse to choose which meal time she would like is a nice gesture of thanks for the nurse, but it does not enable safe care. Having nursing assistive personnel may help the nurse complete basic tasks such as hygiene and turning, but it does not enable safe nursing care that she is ultimately responsible for.

33. A nurse is caring for a patient who fell on the ice and has connective tissue damage in the wrist and hand. What does type of pain does the nurse document that the patient has? a. Visceral pain b. Somatic pain c. Peripherally generated pain d. Centrally generated pain

ANS: B Somatic pain comes from bone, joint, or muscle. Visceral pain arises from the visceral organs such as the GI tract and pancreas. Peripherally generated pain can be caused by polyneuropathies or mononeuropathies. Centrally generated pain results from injury to the central or peripheral nervous system.

A newly hired experienced nurse is preparing to change a patient's abdominal dressing and hasn't done it before at this hospital. Which action by the nurse is best? a. Ask another nurse to do it so the correct method can be viewed. b. Check the policy and procedure manual for the agency's method. c. Change the dressing using the method taught in nursing school. d. Ask the patient how the dressing change has been recently done.

ANS: B The Joint Commission requires accredited hospitals to have written nursing policies and procedures. These internal standards of care are specific and need to be accessible on all nursing units. For example, a policy/procedure outlining the steps to follow when changing a dressing or administering medication provides specific information about how nurses are to perform. The nurse being observed may not be doing the procedure according to the agency's policy or procedure. The procedure taught in nursing school may not be consistent with the policy or procedure for this agency. The patient is not responsible for maintaining the standards of practice. Patient input is important, but it's not what directs nursing practice.

16. A patient who had a motor vehicle accident 2 days ago is experiencing pain and is receiving patient-controlled analgesia (PCA). How does the nurse know that the patient is experiencing effective pain management with the PCA? a. The patient is sleeping and is difficult to arouse. b. The patient rates pain at an acceptable level of 3 on a 0 to 10 scale. c. Sufficient medication is left in the PCA syringe. d. The patient presses the control button to deliver pain medication.

ANS: B The effectiveness of pain relief measures is determined by the patient. If the patient is satisfied with the amount of pain relief, then pain measures are effective. A patient who is sleeping and is difficult to arouse is possibly oversedated; the nurse needs to assess this patient further. The amount of medication left in the PCA syringe does not indicate whether pain management is effective. Pressing the button shows that the patient knows how to use the PCA but does not evaluate pain management.

21. A nurse receives an order from a health care provider to administer Vicodin ES, which contains 750 mg acetaminophen and 7.5 mg hydrocodone, to a patient who is experiencing 8/10 postsurgical pain. The order is to give 2 tablets every 6 hours by mouth as needed for pain. What is the nurse's best next action? a. Give the Vicodin ES to the patient immediately because the patient is experiencing severe pain. b. Ask the health care provider to verify the dosage and frequency of the medication. c. Ask the health care provider for an order for a nonsteroidal antiinflammatory drug (NSAID). d. Ask the health care provider for an order to play music for the patient, in addition to providing the pain medication.

ANS: B The maximum 24-hour dosage for acetaminophen is 4 grams. If the patient took 2 tablets of Vicodin ES every 6 hours, the patient would take in 6 grams of acetaminophen in 24 hours. This exceeds the safe dosage of acetaminophen, so the best action is to question this order. Giving the medication as ordered would possibly result in the patient taking more acetaminophen than what is considered a safe dose. Acetaminophen overdose can result in liver failure. NSAIDs are used to treat mild to moderate pain. At this moment, the patient is experiencing severe pain. Implementing music therapy is a nursing intervention and is an independent nursing action. Thus, an order to start music therapy is not needed

17. The nurse recognizes that which of the following is a modifiable contributor to a patient's perception of pain? a. Age and gender b. Anxiety and fear c. Culture d. Previous pain experience

ANS: B The nurse can take measures to ease the patient's anxiety and fear related to pain. Age, gender, culture, and previous pain experience are all nonmodifiable factors that the nurse can help the patient to understand, but the nurse cannot alter them.

A nurse notices that his neighbor's preschool children are often playing on the sidewalk and in the street unsupervised and repeatedly takes them back to their home and talks with the parent available, but the situation continues. What immediate action by the nurse is mandated by law? a. Talk with both parents about safety needs of their children. b. Contact the appropriate community child protection agency. c. Tell the parents that the authorities will be contacted shortly. d. Take pictures of the children to support the overt child abuse.

ANS: B The nurse has a duty to report this situation to protect the children. Any health care professional who does not report suspected child abuse or neglect may be liable for civil or criminal legal action. The person making the report has legal immunity if the report is made in good faith. Talking with the parents is not mandated by law. There is no obligation to tell the parents that they will be reported to authorities. There is no obligation for the nurse to take pictures of the children.

30. The nurse is caring for a patient who suddenly experiences chest pain. What is the nurse's first priority? a. Call the rapid response team. b. Ask the patient to rate and describe the pain. c. Raise the head of the bed. d. Administer pain relief medicati

ANS: B The nurse's ability to establish a nursing diagnosis, plan and implement care, and evaluate the effectiveness of care depends on an accurate and timely assessment. The other responses are all interventions; the nurse cannot know which intervention is appropriate until the nurse completes the assessment, makes a nursing diagnosis, and plans care.

A confused patient with a urinary catheter, nasogastric tube, and intravenous line keeps touching these needed items for care. The nurse has tried to explain to the patient that he should not touch these lines, but the patient continues. What is the best action by the nurse at this time? a. Apply restraints loosely on the patient's dominant wrist. b. Try other approaches to prevent the patient from touching these care items. c. Notify the health care provider that restraints are needed immediately to maintain the patient's safety. d. Allow the patient to pull out lines to prove that the patient needs to be restrained.

ANS: B The risks associated with the use of restraints are serious. A restraint-free environment is the first goal of care for all patients. Many alternatives to the use of restraints are available, and the nurse should try all of them before notifying the patient's health care provider. The situation states that the patient is touching the items, not trying to pull them out. At this time, the patient's well-being is not at risk. The nurse will have to check on the patient frequently and then will determine if the health care provider needs to be informed of the situation. Restraints can be used (1) only to ensure the physical safety of the resident or other residents, (2) when less restrictive interventions are not successful, and (3) only on the written order of a health care provider. The health care provider needs to know the situation but also needs to know that all approaches possible have been used before writing an order for restraints. Allowing the patient to pull out any of these items could cause harm to the patient.

A patient's condition is slowly deteriorating. What actions should the nurse take to provide the best care possible? (Select all that apply.) a. Allow the nursing student to receive verbal orders from the physician in the room while the nurse is in the medication area down the hall. b. Document the patient's status changes in the medical record in a timely manner. c. Document that the health care provider has been notified of the specific patient status, including date and time that messages were left. d. Check the chart for frequent orders. e. Omit charting what the health provider's response is to notification of the patient's status change.

ANS: B, C Clear, concise, and timely communication is essential whenever charting in the patient's medical record occurs. Nursing students are not permitted to receive verbal orders. Documentation regarding communication with the health care provider must contain what was communicated by the nurse and the health care provider, orders if given, date, time, and identification of who is documenting the situation.

9. Which of the following statements made by the patient indicates to the nurse that teaching on a patient-controlled analgesia (PCA) device has been effective? a. "This is the only pain medication I will need to be on." b. "I can administer the pain medication as frequently as I need to" c. "I feel less anxiety about the possibility of overdosing." d. "I will need the nurse to notify me when it is time for another dose."

ANS: C A PCA is a device that allows the patient to determine the level of pain relief delivered, reducing the risk of oversedation. Its use often eases anxiety because the patient is not reliant on the nurse for pain relief. Other medications, such as oral analgesics, can be given in addition to the PCA machine. The PCA does have a time limit to prevent overdose, but the patient can lengthen the amount of time between doses. One benefit of PCA is that the patient does not need to rely on the nurse to administer pain medication; the patient determines when to take the medication.

12. A nurse is providing discharge teaching for a patient with a fractured humerus. The patient is going home with Vicodin (5/325). What important patient education does the nurse provide? a. "Be sure to eat a meal high in fat before taking the medication, to avoid a stomach ulcer." b. "Narcotics can be addictive, so do not take them unless you are in severe pain." c. "You need to drink plenty of fluids and eat a diet high in fiber." d. "As your pain severity lessens, you will begin to give yourself once-daily intramuscular injections."

ANS: C A common side effect of opioid analgesics is constipation. Therefore, the nurse encourages the patient to drink fluids and eat fiber to prevent constipation. Although medications can be irritating to the stomach, eating a diet high in fat does not prevent gastric ulcers. To best manage pain, the patient needs to take pain medication before painful procedures or activities or before pain becomes severe. As the patient's pain gets better, the strength of the medications will decrease. IM, IV, and topical analgesics are used for more severe and chronic pain.

6. The nurse anticipates administering an opioid fentanyl patch to which patient? a. A 15-year-old adolescent with a broken femur b. A 30-year-old adult with cellulitis c. A 50-year-old patient with prostate cancer d. An 80-year-old patient with a broken hip

ANS: C A fentanyl patch is an extended-relief opioid that provides pain relief for 24 hours a day. This is ideal for patients who have chronic severe pain, such as those who have cancer. The other patients are expected to experience acute pain. Therefore, they will most likely benefit from oral or IV opioids for short-term pain relief.

An obstetrical nurse comes across an automobile accident. The patient seems to have a crushed upper airway, and while waiting for emergency medical services to arrive, the nurse makes a cut in the trachea and inserts a straw from her purse to provide an airway. The patient survives and has a permanent problem with his vocal cords, making it difficult to talk. Which statement is true regarding the nurse's performance? a. The nurse acted appropriately and saved the patient's life. b. The nurse acted within the guidelines of the Good Samaritan Law. c. The nurse took actions beyond those that are standard and appropriate. d. The nurse should have just stayed with the patient and waited for help.

ANS: C An obstetric nurse would not have been trained in performing a tracheostomy or a cricotomy, and doing so would be beyond what she has been trained or educated to do. The nurse did not do what another nurse would have done in the same situation. The nurse is not protected by the Good Samaritan Law because she acted outside of her scope of practice and training. The nurse should have acted within what she was trained and educated to do in this circumstance, not just stay with the patient.

A recent immigrant who does not speak English is alert and requires hospitalization. What is the initial action that the nurse must take to enable informed consent to be obtained? a. Ask a family member to translate what the nurse is saying. b. Notify the health care provider that the patient doesn't speak English. c. Request an official interpreter to explain the terms of consent. d. Use hand gestures and medical equipment while explaining in English.

ANS: C An official interpreter must be present to explain the terms of consent if a patient speaks only a foreign language. A family member or acquaintance who speaks a patient's language should not interpret health information. Family members can tell those caring for the patient what the patient is saying, but privacy regarding the patient's condition, assessment, etc., must be protected. There is no way that the nurse can know that the family member is translating exactly what the nurse is saying. Privacy must be ensured and accurate information must be provided to the patient. After consent is obtained for treatment, the health care provider would be notified because little can be done without consent. The health care provider needs to have the translator available during the history and physical, as well as at other times, but the first step is to get a translator to obtain informed consent because this is not an emergency situation. Using hand gestures and medical equipment is inappropriate when communicating with a patient who does not understand the language spoken. Certain hand gestures may be acceptable in one culture and not appropriate in another. The medical equipment may be unknown and frightening to the patient, and the patient still doesn't understand what is being said.

24. A nurse is supervising a student who is caring for a patient with chronic pain. Which statement by the student indicates an understanding of pain management? a. "This patient says her pain is a 5, but she is not acting like it. I am not going to give her any pain medication." b. "The patient is sleeping, so I pushed her PCA button for her." c. "I need to reassess the patient's pain 1 hour after administering oral pain medication." d. "It wasn't time for the patient's medication, so when she requested it, I gave her a placebo."

ANS: C Because oral medications usually peak in about an hour, you need to reassess the patient's pain within an hour of administration. Nurses must believe any patient report of pain, even if nonverbal communication is not consistent with pain ratings. The patient is the only person who should push the PCA button. Pushing the PCA when a patient is sleeping is dangerous and may lead to narcotic overdose or respiratory depression. Giving the patient a placebo and telling her it is her medication is unethical.

19. A nurse is providing medication education to a patient who just started taking ibuprofen, a nonselective nonsteroidal antiinflammatory drug (NSAID). Which statement made by the nurse best indicates how ibuprofen works? a. "Ibuprofen helps to remove factors that cause or stimulate pain." b. "Ibuprofen reduces anxiety, which will help you better cope with your pain." c. "Ibuprofen helps to decrease the production of prostaglandins." d. "Ibuprofen binds with opiate receptors to reduce your pain."

ANS: C NSAIDs like ibuprofen most likely work by decreasing the synthesis of prostaglandins to inhibit cellular responses to inflammation. Ibuprofen does not remove factors that cause pain, nor does it enhance coping with pain. Opioids bind with opiate receptors to modify perceptions of pain.

1. What is the most appropriate way to assess the pain of a patient who is oriented and has recently had surgery? a. Assess the patient's body language. b. Observe cardiac monitor for increased heart rate. c. Ask the patient to rate the level of pain. d. Ask the patient to describe the effect of pain on the ability to cope.

ANS: C Pain is a subjective measure. Therefore, the best way to assess a patient's pain is to ask the patient to rate the pain. Nonverbal communication, such as body language, is not as effective in assessing pain, especially when the patient is oriented. Heart rate sometimes increases when a patient is in pain, but this is not a symptom that is specific to pain. Pain sometimes affects a patient's ability to cope, but assessing the effect of pain on coping assesses the patient's ability to cope; it does not assess the patient's pain.

20. A nurse has brought the patient his scheduled pain medication. The patient asks the nurse to wait to give pain medication until the time for the dressing change, which is 2 hours away. Which response by the nurse is most therapeutic? a. "This medication will still be providing you relief at the time of your dressing change." b. "OK, swallow this pain pill, and I will return in a minute to fill your wound." c. "Would you like medication to be given for dressing changes on top of your regularly scheduled medication?" d. "Your medication is scheduled for this time, and I can't adjust the time for you. I'm sorry, but you must take your pill right now."

ANS: C STAT doses of medication can be given to patients in certain circumstances, as with an extensive dressing change. By asking to hold off on the dose, the patient is indicating that the dressing changes are extremely painful. The regularly scheduled dose might not be as effective for the patient. Oral medications take 30 to 60 minutes to take effect. If the nurse began the dressing change right then, the medication would not have been absorbed yet. The patient has the right to refuse to take a medication. It is the nurse's responsibility to communicate with the provider and with the patient about a pain control plan that works for both.

13. A patient arrives at the emergency department experiencing a headache and rates the pain as 7 on a 0 to 10 pain scale. What nonpharmacological intervention does the nurse implement for this patient while awaiting orders for pain medication from the health care provider? a. Frequently reassesses the patient's pain scores b. Reassures the patient that the provider will come to the emergency department soon c. Softly plays music that the patient finds relaxing d. Teaches the patient how to do yoga

ANS: C The appropriate nonpharmacological pain management intervention is to quietly play music that the patient finds relaxing. Music diverts a person's attention away from pain and creates relaxation. Reassessing the patient's pain scores is done during evaluation. Building the patient's expectation of the provider's arrival does not address the patient's pain. Although yoga promotes relaxation, nurses teach relaxation techniques only when a patient is not experiencing acute pain. Because the patient is having acute pain, this is not an appropriate time to provide patient teaching.

10. A nurse is caring for a patient who is experiencing pain following abdominal surgery. What information is important for the nurse to tell the patient when providing patient education about effective pain management? a. "To prevent overdose, you need to wait to ask for pain medication until you begin to experience pain." b. "You should take your medication after you walk to make sure you do not fall while you are walking." c. "We should work together to create a regular schedule of medications that does not allow for breakthrough pain." d. "You need to take oral pain medications when you experience severe pain."

ANS: C The best way to manage pain is to develop a schedule of medications that are given around the clock to prevent breakthrough pain. The nurse should not wait until pain is experienced because it takes medications 10-30 minutes to begin to relieve pain. The nurse administers pain medications before painful activities, such as walking, and administers intravenous medications when a patient is having severe pain.

5. The nurse is caring for two patients; both are having a hysterectomy. The first patient is having the hysterectomy after a complicated birth. The second patient has uterine cancer. What will most likely influence the experience of pain for these two patients? a. Neurological factors b. Competency of the surgeon c. Meaning of pain d. Postoperative support personnel

ANS: C The patient's perception of pain is influenced by psychological factors, such as anxiety and coping, which in turn influence the patient's experience of pain. Each patient's experience is different. The degree and quality of pain perceived by a patient are related to the meaning of the pain. Neurological factors can interrupt or influence pain perception, but neither of these patients is experiencing alterations in neurological function. The knowledge, attitudes, and beliefs of nurses, physicians, and other health care personnel about pain affect pain management but do not necessarily influence a patient's pain perceptions.

A new nurse notes that the health care unit keeps a listing of patient names in a closed book behind the front desk of the nursing station so patients can be located easily. What action is most appropriate for the nurse to take? a. Move the book to the upper ledge of the nursing station for easier access. b. Talk with the nurse manager about the listing being a violation of the Health Insurance Portability and Accountability Act (HIPAA). c. Use the book as needed while keeping it away from individuals not involved in patient care. d. Ask the nurse manager to move the book to a more secluded area.

ANS: C The privacy section of the HIPAA provides standards regarding accountability in the health care setting. These rules include patient rights to consent to the use and disclosure of their protected health information, to inspect and copy their medical record, and to amend mistaken or incomplete information. This document limits who is able to access a patient's record. It establishes the basis for privacy and confidentiality about patients in any manner. The book is located where only staff would have access. It is not the responsibility of the new nurse to move items used by others on the patient unit. The listing is protected as long as it is used appropriately as needed to provide care. There is no need to move the book to a more secluded area.

26. The nurse is teaching a student nurse about pain assessment scales. Which statement by the student indicates correct understanding? a. "You cannot use a pain scale to compare the pain of my patient with the pain of your patient." b. "When patients say they don't need pain medication, they aren't in pain." c. "Pain assessment scales determine the quality of a patient's pain." d. "A patient's behavior is more reliable than the patient's report of pain."

ANS: C To gain a better understanding of a patient's current pain status and to determine what interventions are needed, the nurse should assess both current and previous pain scores. A patient who rates pain at 4 might find the pain manageable if over the previous 24 hours, he had rated his pain at 10. Some patients do not express their pain or do not wish to take medications to relieve the pain. This does not mean they aren't in pain; the nurse can try nonpharmacological therapies for this patient.

7. What nursing intervention is most effective in preventing injury to a patient following administration of epidural anesthesia? a. Keeping the reversal agent in a syringe in the patient's bedside table b. Applying a gauze dressing to the epidural catheter insertion site c. Labeling the tubing that leads to the epidural catheter d. Asking the nursing assistive personnel to check on the patient at least once every 2 hours

ANS: C To reduce the accidental administration of IV medications into the epidural catheter, the tubing that leads to the epidural catheter needs to be labeled clearly. Medications used to reverse the action of the anesthetic medication need to be kept in a secured location, not in the patient's room in an unsecured location. The epidural insertion site needs to be covered by a clear occlusive dressing to prevent infection and allow the nurse to assess the site. Patients receiving epidural anesthesia need to be monitored every 15 minutes until stabilized and then at least hourly.

32. Which of the following is the best way for the nurse to manage pain for a patient with chronic pain from arthritis? a. Administer pain medication before any activity. b. Provide intravascular bolus as needed for breakthrough pain. c. Give medications around-the-clock. d. Administer pain medication only when nonpharmacological measures have failed.

ANS: C When a patient with arthritis has chronic pain, the best way to manage pain is to take medication regularly throughout the day to maintain constant pain relief. "Before activity" is nonspecific, and the medication may not have time to work before activity. If the patient waits until having pain to take the medication, pain relief takes longer. Nonpharmacological measures are used in conjunction with medications unless requested otherwise by the patient.

The nurse is caring for a patient with a urinary catheter. After the nurse empties the collection bag and disposes of the urine, the next step is to a. Use alcohol-based gel on hands. b. Wash hands with soap and water. c. Remove eye protection and dispose of in garbage. d. Remove gloves and dispose of in garbage.

ANS: D After disposing of the urine, the first step in removing personal protective equipment is removing gloves and disposing of them properly. In this scenario, the next step would be to remove eye protection followed by hand hygiene. Wash hands if the hands are visibly soiled; otherwise the use of alcohol-based gel is indicated for routine decontamination of hands.

A home health nurse is performing a home assessment for safety. Which of the following comments by the patient would indicate a need for further education? a. "I will schedule an appointment with a chimney inspector next week." b. "Daylight savings is the time to change batteries on the carbon monoxide detector." c. "If I feel dizzy when using the heater, I need to have it inspected." d. "When it is cold outside in the winter, I can warm my car up in the garage."

ANS: D Allowing a car to run in the garage introduces carbon monoxide into the environment and decreases the available oxygen for human consumption. Garages should be opened and not just cracked to allow fresh air into the space and allay this concern. Checking the chimney and heater, changing the batteries on the detector, and following up on symptoms such as dizziness, nausea, and fatigue are all statements that would indicate that the individual has understood the education.

While recovering from a severe illness, a hospitalized patient states that he wants to change his living will, which he signed nine months ago. Which response by the nurse is most appropriate? a. "Check with your admitting health care provider whether a copy is on your chart." b. "Have you talked with your attorney recently about a living will?" c. "Your living will can be changed only once each calendar year." d. "Let me check with someone here in the hospital who can assist you."

ANS: D Each health care facility has personnel who are familiar with the state laws and can assist the patient in revising a living will. They may be in the admissions or risk management department. Checking with the health care provider about the presence of a living will on the chart has nothing to do with the patient's desire to change the living will. The question states that the patient wants to change his living will. Asking whether he has talked to his lawyer recently is a closed-ended question that passes the responsibility to someone else, that is, the attorney, and does not address the patient's current desire to change the living will. It is the nurse's responsibility to find an appropriate person in the facility to assist the patient. A living will can be changed whenever the patient decides to change it, as long as the patient is competent.

34. The nurse is caring for an infant in the intensive care unit. Which of the following is the most accurate description of factors that will influence the perception and management of pain for this patient? a. Infants cannot tolerate analgesics owing to an underdeveloped metabolism. b. Infants have an increased sensitivity to pain when compared with older children. c. Pain cannot be accurately assessed in infants. d. Infants respond behaviorally and physiologically to painful stimuli.

ANS: D Infants cannot verbally express their pain, but they do express pain with behavioral cues and physiological indicators. Infants can tolerate analgesics, but proper dosing and close monitoring are essential. Infants and older children have the same sensitivity to pain. Pain can be assessed even though the neonate cannot verbalize; the nurse can observe behavioral clues. Nurses use behavioral cues and physiological responses to assess pain in infants.

27. The nurse is administering pain medication for several patients. Which patient does the nurse administer medication to first? a. The patient who needs to take a scheduled dose of maintenance pain medication b. The patient who needs to be premedicated before walking c. The patient with a PCA running who needs to have the syringe replaced d. The patient who is experiencing 8/10 pain and has a STAT order for pain medication

ANS: D STAT medications need to be given as soon as possible. In addition, this patient is the priority because of the report of severe pain. The other patients need pain medication, but their situations are not as high a priority as that of the patient with the STAT medication order.

2. A nurse is caring for a patient who recently had an abdominal hysterectomy and states that she is experiencing severe pain. The patient's blood pressure is 110/60, and her heart rate is 60. Additionally, the patient does not appear to be in any distress. Which response by the nurse is most therapeutic? a. "Your vitals do not show that you are having pain; can you describe your pain?" b. "You do not look like you are in pain." c. "OK, I will go get you some narcotic pain relievers immediately." d. "What would you like to try to alleviate your pain?"

ANS: D The nurse must believe that a patient is in pain whenever the patient reports that he or she is in pain, even if the patient does not appear to be in pain. Whenever the patient reports pain, the nurse needs to collaborate with the patient to determine the best method of pain relief, whether it be medication, meditation, or repositioning. The nurse must be careful to not judge the patient based on vital signs or nonverbal communication and must not assume that the patient is seeking narcotics. The patient is a partner in pain management, so going to get narcotics to treat the pain without consulting with the patient first is not appropriate

31. The nurse is caring for a patient who recently had surgery to repair a hernia. The patient's pain was 7 out of 10 before receiving pain medication. One hour after receiving an oral opioid, the patient ranks his pain at 3 out of 10. The patient asks the nurse why he isn't receiving more pain medication. Which is the nurse's best response? a. "This medication can be given only every 4 hours. It is not time for you to have any other pain medication right now." b. "I will notify the health care provider to come perform an assessment if your pain doesn't improve in 30 minutes." c. "If the pain becomes severe, we may need to transfer you to an intensive care unit." d. "It can take 2 hours for oral pain medication to work, and your pain is going down. Let's try boosting you up in bed and putting an ice pack on the incision to see if that helps."

ANS: D The patient is responding well to the oral pain medication and it can take up to 2 hours for oral medications to relieve pain. Trying nonpharmacological interventions as an addition to opioid medications is appropriate at this time. If nonpharmacological interventions combined with the oral opioid are ineffective, the nurse needs to notify the health care provider and ask for a change in the medication or for additional pain medication. Saying that the patient has to wait 4 hours for additional pain medication is inaccurate because the nurse needs to provide further nursing interventions if pain is not relieved at an acceptable level for the patient. Admission to an intensive care unit is not typically necessary to manage pain following surgery for a hernia.

14. A patient who has had type 2 diabetes for 26 years is beginning to experience peripheral neuropathy in the feet and lower legs, which is causing the patient to have a decreased ability to feel pain in the lower extremities. The nurse is providing education to the patient to prevent injury to the feet. The nurse tells the patient to always wear shoes or slippers when walking. Which of the following statements made by the nurse best explains the rationale for this instruction? a. "Wearing shoes blocks pain perception and helps you adapt to pain, which ends up protecting your feet." b. "Shoes provide nonpharmacological pain relief to people with diabetes and peripheral neuropathy." c. "Since you cannot feel pain as much in your feet, you need to open your neurological gates to allow pain sensations to come through. Wearing shoes helps to open those gates, which protects your feet." d. "You have lost the ability to withdraw from pain because of your peripheral neuropathy. If you step on something and are not wearing shoes, you will not feel it; this could possibly cause injury to your foot."

ANS: D This patient is losing the ability to feel pain owing to peripheral neuropathy. The patient will no longer have protective reflexes to prevent injury to the feet. Wearing shoes prevents the patient from injuring the foot because they protect the feet. Shoes do not block pain perception, nor do they help people adapt to pain. Shoes are not a form of nonpharmacological pain relief. Wearing shoes will not have an effect on opening or closing the pain gates.

A nursing student in the final term of nursing school is overheard by a nursing faculty member telling another student that she got to insert a nasogastric tube in the emergency department while she was working as a nursing assistant. What advice is best for the nursing faculty member to give to the nursing student? a. "Just be careful when you are doing new procedures and make sure you are following directions by the nurse." b. "Review your procedures before you go to work, so you will be prepared to do them if you have a chance." c. "The nurse should not have allowed you to insert the nasogastric tube because something bad could have happened." d. "You are not allowed to perform any procedures other than those in your job description even with the nurse's permission."

ANS: D When nursing students work as nursing assistants or nurse's aides when not attending classes, they should not perform tasks that do not appear in a job description for a nurse's aide or assistant. The nursing student should always follow the directions of the nurse, unless doing so violates the institution's guidelines or job description under which the nursing student was hired. The nursing student should be able to safely complete the procedures delegated as a nursing assistant, and reviewing those not done recently is a good idea, but it has nothing to do with the situation. This option does not address the situation that the nursing student acted outside the job description for the nursing assistant position. The focus of the discussion between the nursing faculty member and the nursing student should be on following the job description under which the nursing student is working.

Identify the descriptive characteristics of body alignment and mobility related to the following developmental stage. (Adolescents)

Adolescents experience a tremendous growth spurt. In girls, the hips widen, and fat is deposited in upper arm, thighs, and buttocks. In boys, long bone growth and muscle mass are increased.

A nursing student uses a surgical mask to assist in a sterile surgical procedure. Which action made by the nursing student indicates a need for correction? 1 Dropping the used mask in a trash receptacle 2 Removing the mask when leaving the surgical room 3 Removing the mask by touching the outer surface of the mask 4 Tying the two lower ties snugly around the neck

After using a surgical mask, the mask should be removed by untying the bottom mask strings, followed by the top strings. The outer surface of the mask should never be touched while removing it. Used masks should be dropped in a trash receptacle. Surgical masks should be removed before leaving the surgical room to prevent infection. While using a surgical mask, the two lower ties of the mask should be tied around the neck.

A nurse administers a back rub to a patient after first providing for privacy and maintaining standard precautions. Place the following steps in the order in which they should be implemented. 1. Apply warmed lotion to your hands. 2. Position the patient in the side-lying position. 3. Assess the skin for color, turgor, and skin breakdown. 4. Arrange the gown and top linens so that the patient's back is exposed. 5. Use a variety of strokes to massage the muscles of the back and sacral area.

Answer: 2 4 3 1 5 The first step is to position the patient in the side-lying position because this provides for a comfortable, supported position during the procedure. The second step is to arrange the gown and linens so that the patient's back is exposed because this provides access to the patient's back. The third step is to assess the skin to ensure that there are no indications of a problem that is a contraindication for having a back rub. The fourth step is to warm the lotion in your hands because warm lotion is more comfortable and supports muscle relaxation. A variety of strokes (e.g., effleurage, pétrissage, tamponage, small circular movements, and feathering) relieves muscle tension, promotes physical and emotional relaxation, and increases circulation to the area.

The nurse is teaching a patient diagnosed with arthritis of the knees about physical activity and lifestyle changes. Which statement by the patient indicates correct understanding of the teaching? Correct1 "I will perform exercises in a pool." 2 "I will go for a walk every morning." 3 "I will increase my intake of foods rich in vitamin D." 4 "I will frequently perform weight-bearing exercises."

Aquatic therapy involves performing exercises in a pool, which is an appropriate therapy used to regain joint mobility in severe arthritis of knees. Aquatic therapy would be more beneficial for this patient than walking. Increased intake of foods rich in vitamin D is beneficial to strengthen the bones, but it would not help a patient regain joint mobility. Weight-bearing exercises strengthen the bones and the joints, but there is a risk for fractures in this patient while performing these exercises. Therefore, aquatic therapy is preferred for severe arthritis.

Which normal flora of the human colon can cause an infection when it enters the bloodstream? 1 Escherichia coli 2 Candida albicans 3 Bacteroides fragilis 4 Plasmodium falciparum

Bacteroides fragilis is a part of the normal flora of the human colon. This microorganism can cause infections if it enters the blood stream or tissue during injury or surgery. Escherichia coli causes gastroenteritis in the colon. Candida albicans causes candidiasis, pneumonia, and sepsis. Plasmodium falciparum causes malaria.

In which order should an exercise program be initiated? 1. Choosing appropriate athletic shoes 2. Recording the baseline fitness scores 3. Building up a routine gradually and dividing the exercise time throughout the day 4. Developing a balanced routine of aerobic activity and strength training

Before initiating an exercise program, the baseline fitness scores are measured. This may include recording the patient's pulse rate before and after walking a mile, the time taken to walk a mile, the number of push-ups that can be done at a time, the patient's waist circumference, and the patient's body mass index. Then, a fitness program is designed to include a balanced daily routine of aerobic activity and strength training. Athletic shoes designed for the specific exercise are chosen. Finally, a routine is built gradually and the exercise time is divided throughout the day.

The nurse understands the increased risk of musculoskeletal injuries due to activities such as lifting objects, pushing beds, and bathing and feeding patients. Which measures should the nurse adopt to prevent injury? Select all that apply. Correct1 Bend at the knees. 2 Relax the abdominal muscles. 3 Maintain an erect trunk and bent knees. 4 Get assistance when moving patients. 5 Keep the weight to be lifted as close to the body as possible.

Bending at the knees helps to maintain the center of gravity and thus keeps the body aligned. Bending at the knees uses the stronger leg muscles to do the lifting, preventing exhaustion of other small muscles. Maintaining the trunk erect with knees bent helps multiple muscle groups work together and prevents overworking a single muscle. Getting assistance when shifting patients promotes patient safety and prevents musculoskeletal injury. The risk of injury is less if the object is in the same plane as the person lifting it and is close to the center of gravity for balance. Therefore, the object should be kept as close to the body as possible. The abdominal muscles should be tightened and the pelvis should be tucked. This position provides balance and prevents the back from injury.

Infection

CDC states that a wound that has a purulent drainage is infected regardless of whether a culture has been obtained or not; or even if the culture is negative

The nurse is teaching a patient about the use of herbal medicines. Which statements by the patient indicate effective learning? Select all that apply. 1 "Herbal medicines cause insomnia." 2 "Herbal medicines cause sleep apnea." 3 "Herbal medicines always cause allergic reactions." 4 "Herbal medicines may interact with prescribed medication." 5 "Herbal medicines are not regulated by the U.S. Food and Drug Administration."

Correct 4 "Herbal medicines may interact with prescribed medication." Correct 5 "Herbal medicines are not regulated by the U.S. Food and Drug Administration."

The practitioner orders OOB for a patient. How is the nurse functioning when moving this patient out of bed to a chair? 1. Dependently 2. Independently 3. Collaboratively 4. Interdependently

Correct Answer: 1 (Determining the extent of activity desirable for a patient is within the practitioner's, not a nurse's, scope of practice. Following activity orders is a dependent function of the nurse.) 2. The responsibility to determine a patient's activity level is not within the legal scope of nursing practice. 3. A practitioner works independently when determining a patient's desired activity level. 4. The nurse is following the practitioner's order to get the patient OOB. There are no restrictions or parameters in relation to the order. However, the nurse must use judgment before, during, and after a transfer if a patient's condition changes.

A nurse expert is called to testify in a lawsuit regarding professional nursing malpractice primarily to testify: 1. About standards of nursing care as they apply to the facts in the case 2. With regard to laws governing the practice of nursing 3. For the prosecution 4. For the defense

Correct Answer: 1 (The American Nurses Association Standards of Clinical Nursing Practice are authoritative statements by which the national organization for nursing describes the responsibilities for which its practitioners are accountable. An expert nurse is capable of explaining these standards as they apply to the situation under litigation. These professional standards are one criterion that helps a judge or jury determine if a nurse committed malpractice or negligence.) 2. An expert nurse is not an expert in the law. The expert nurse's role is not to make judgments about the laws as they apply to the practice of nursing. 3. A nurse expert can testify for either the prosecution or the defense. 4. A nurse expert can testify for either the defense or the prosecution.

When the nurse attempts to administer a medication to a patient, the patient refuses to take the medication because it causes diarrhea. The nurse provides teaching about the medication, but the patient continues to adamantly refuse the medication. What should the nurse do first? 1) Document the patient's refusal to take the medication. 2) Notify the practitioner of the patient's refusal to take the medication. 3) Discuss with a family member the need for the patient to take the medication. 4) Explain again to the patient the consequences of refusing to take the medication.

Correct Answer: 1 (The patient has the right to refuse) 2. Notifying the practitioner eventually should be done, but it is not the priority at this time. 3. Discussing the situation with a family member without the patient's consent is a violation of confidentiality. 4. The patient has been taught about the medication and adamantly refuses the medication. Further teaching at this time may be viewed by the patient as badgering.

When caring for a terminally ill patient, a family member says, "I need your help to hasten my mother's death so that she is no longer suffering." What should the nurse do based on the position of the American Nurse association in relation to assisted suicide? 1) Not participate in active euthanasia. 2) Participate based on personal values and beliefs. 3) Participate when the patient is experiencing severe pain. 4) Not participate unless two practitioners are consulted and the patient has had counseling.

Correct Answer: 1 (Withholding the medication and docu- menting the patient's refusal are the appropriate interventions. Patient's have a right to refuse care.) 2. Notifying the practitioner eventually should be done, but it is not the priority at this time. 3. Discussing the situation with a family member without the patient's consent is a violation of confidentiality. 4. The patient has been taught about the medication and adamantly refuses the medication. Further teaching at this time may be viewed by the patient as badgering.

A student nurse is about to graduate from an accredited nursing program. The student nurse understands that an action unrelated to a state Nurse Practice Act is: 1. Setting guidelines for nurses' salaries in the state 2. Establishing reciprocity for licensure between states 3. Determining minimum requirements for nursing education 4. Maintaining a list of nurses who can legally practice in the state

Correct Answer: 1. State Nurse Practice Acts define and regulate the practice of nursing within the state. The salary of nurses is determined through negotiations between nurses or their representatives, such as a union or a professional nursing organization, and the representatives of the agency for which they work. 2. A state's Nurse Practice Act determines the criteria for reciprocity for licensure. 3. A state's Nurse Practice Act stipulates minimum requirements for nursing education. 4. A state's Nurse Practice Act defines the criteria for licensure within the state. The actual functions, such as maintaining a list of nurses who can legally practice in the state, may be delegated to another official body such as a State Board of Nursing or State Education Department.

What is the primary purpose of the American Nurses Association Standards of Clinical Nursing Practice? 1. Establish criteria for quality practice 2. Define the philosophy of nursing practice 3. Identify the legal definition of nursing practice 4. Determine educational standards for nursing practice

Correct Answer: 1. The ANA Standards of Clinical Nursing Practice describe the nature and scope of nursing practice and the responsibilities for which nurses are accountable. 2. A philosophy incorporates the values and beliefs about the phenomena of concern to a discipline. The ANA Standards of Clinical Nursing Practice reflect, not define, a philosophy of nursing. Each nurse and nursing organization should define its own philosophy of nursing. 3. The laws of each state define the practice of nursing within the state. 4. Educational standards are established by accrediting bodies, such as the National League for Nursing Accrediting Commission, the Commission on Collegiate Nursing Education, and state education departments.

What is the main purpose of the American Nurses Association? 1. Establish standards of nursing practice 2. Recognize academic achievement in nursing 3. Monitor educational institutions granting degrees in nursing 4. Prepare nurses to become members of the nursing profession

Correct Answer: 1. The American Nurses Association has established Standards of Care and Standards of Professional Performance. These standards reflect the values of the nursing profession, provide expectations for nursing practice, facilitate the evaluation of nursing practice, and define the profession's accountability to the public. 2. Sigma Theta Tau, the international honor society of nursing, recognizes academic achievement. 3. The National League for Nursing Accrediting Commission, the Commission on Collegiate Nursing Education, and state education departments monitor educational institutions granting degrees in nursing. 4. Schools of nursing (diploma, associate de- gree, and baccalaureate) educate individuals for entry into the practice of nursing.

The nurse is informed that a credentialing team has arrived and is in the process of assessing quality of care delivered at the hospital. What is the organization associated with the credentialing of hospitals? 1. Joint Commission 2. National League for Nursing 3. American Nurses Association 4. National Council Licensure Examination

Correct Answer: 1. The Joint Commission (formerly the Joint Commission on Accreditation of Healthcare Organizations) evaluates health-care organizations' compliance with Joint Commission standards. Accreditation indicates that the organization has the capabilities to provide quality care. In addition, federal and state regulatory agencies and insurance companies require Joint Commission accreditation. 2. The National League for Nursing (NLN) fosters the development and improvement of nursing education and nursing service. 3. The American Nurses Association (ANA) is the national professional organization for nursing in the United States. Its purposes are to promote high standards of nursing practice and to support the educational and professional advancement of nurses. 4. In the United States, graduates of educa- tional programs that prepare students to be- come Licensed Practical Nurses or Registered Professional Nurses must successfully complete the National Council Licensure Examination-PN (NCLEX-PN) and the National Council Licensure Examination-RN (NCLEX-RN), respectively, as part of the criteria for licensure.

When choosing a nursing school in the United States that awards an associate degree, a future student nurse should consider schools that have met the standards of nursing education established by which organization? 1. National League for Nursing Accrediting Commission 2. North American Nursing Diagnosis Association 3. American Nurses Association 4. Sigma Theta Tau

Correct Answer: 1. The National League for Nursing Accrediting Commission (NLNAC) is an organization that appraises and grants accreditation status to nursing programs that meet predetermined structure, process, and outcome criteria. 2. The North American Nursing Diagnosis Association (NANDA) developed a constantly evolving taxonomy of nursing diagnoses to provide a standardized language that focuses on the patient and related nursing care. 3. The American Nurses Association (ANA) is the national professional organization for nursing in the United States. It does not accredit schools of nursing. 4. Sigma Theta Tau, the international honor society of nursing, recognizes academic achievement. It does not accredit schools of nursing.

The nurse is implementing an ordered bowel preparation for a patient who is scheduled for a colonoscopy. Which is the most serious consequence that is prevented by an effective bowel preparation? 1. Discomfort 2. Misdiagnosis 3. Wasted expense 4. Psychological stress

Correct Answer: 2 (Fecal material in the intestines can interfere with the visualization, collection, and analysis of data obtained through a colonoscopy, resulting in diagnostic errors.) 1. Although this may occur, it is not the most serious outcome of an inappropriate preparation for a colonoscopy. 3. A test may have to be cancelled or per- formed a second time if the patient has an ineffective bowel preparation. Although this is a serious consequence, it is not life threatening. 4. Although this is a serious consequence, it is not life threatening.

A patient is asked to participate in a medical research study. The nurse describes to the patient and family members how the patient is protected by the: 1. Code of Ethics 2. Informed Consent 3. Nurse Practice Act 4. Constitution of the United States

Correct Answer: 2 (Informed consent is an agreement by a client to accept a course of treatment or a procedure after receiving complete information necessary to make a knowledgeable decision.) 1. A code of ethics is the official statement of a group's ideals and values. It includes broad statements that provide a basis for professional actions. 3. Nurse Practice Acts define the scope of nursing practice; they are unrelated to informed consent. 4. The Constitution of the United States addresses broad individual rights and responsibilities. The rights related to nursing practice and patients include therights of privacy, freedom of speech, and due process.

The nurse initiates a visit from a member of the clergy for a patient. How is the nurse functioning when initiating this visit? 1. Interdependently 2. Independently 3. Dependently 4. Collegially

Correct Answer: 2 (The nurse is initiating the referral to the member of the clergy and is therefore working independently. Nurses are legally permitted to diagnose and treat human responses to actual or potential health problems.) 1. The nurse does not need a practitioner's order to make a referral to a member of the clergy. An interdependent intervention requires a practitioner's order associated with a parameter. 3. This action is within the scope of nursing practice. The nurse does not need a practitioner's order to make a referral to a member of the clergy. 4. The nurse can make a referral to a mem- ber of the clergy without collaborating with another professional health-care team member.

Identify the actions that are examples of slander. Select all that apply. 1. _____ Volunteer telling another volunteer a patient's age 2. _____ Nurse explaining to a patient that another nurse is incompetent 3. _____ Personal care assistant sharing information about a patient with another patient 4. _____ Unit manager documenting a nurse's medication error in a performance appraisal 5. _____ Housekeeper who is angry at a nurse erroneously telling another staff member that the nurse uses cocaine

Correct Answer: 2, 5 2. This is an example of slander. Slander is a false spoken statement resulting in damage to a person's character or reputation. 5. This is an example of slander. It is a malicious, false statement that may damage the nurse's reputation. 1. This is a violation of the patient's right to confidentiality, not slander. 3. This is a violation of the patient's right to confidentiality, not slander. 4. This is not slander because it is a written, not spoken, statement and it documents true, not false, information.

Which organization is responsible for ensuring that Registered Nurses are minimally qualified to practice nursing? 1) Sigma Theta Tau 2) State Boards of Nursing 3) American Nurses Association 4) Constituent leagues of the National League for Nursing.

Correct Answer: 2. (The National Council of State Boards of Nursing is responsible for the NCLEX examinations; however, the licensing authority in the jurisdiction in which the graduate takes the examination verifies the acceptable score on the examination.) 1. Sigma Theta Tau, the international honor society of nursing, recognizes academic achievement and leadership qualities, encourages high professional standards, fosters creative endeavors, and supports excellence in the profession of nursing. This organization does not grant licensure. 3. The American Nurses Association (ANA) is the national professional organization for nursing in the United States. It fosters high standards of nursing practice; it does not grant licensure. 4. The National League for Nursing (NLN) is committed to promoting and improving nursing service and nursing education; it does not grant licensure

The client who requires a co-signature for a valid consent for surgery is a: 1. 15-year-old mother whose infant requires exploratory surgery 2. 40-year-old resident in a home for developmentally disabled adults 3. 90-year-old adult who wants more information about the risks of surgery 4. 50-year-old unconscious trauma victim who needs insertion of a chest tube

Correct Answer: 2. A client living in a protected environment such as a home for developmen- tally disabled adults may not have the mental capacity to make medical decisions and requires the signature of a court-appointed legal representative. This person could be a parent, sibling, relative, or unrelated individual. 1. A mother may legally make medical deci- sions for her children even if the mother is younger than 18 years of age. 3. Older adults can make decisions for them- selves as long as they understand the risks and benefits of the surgery and are not receiving medication that may interfere with cognitive ability. 4. The insertion of a chest tube to inflate a lung is an emergency intervention to facilitate respiration and oxygenation. This emergency procedure is implemented to sustain life and does not require a signed consent if the client is incapacitated.

Nursing practice is influenced by the doctrine of respondent superior. What is the basic concept related to this theory of liability? 1. Nurses must respond to the Supreme Court when they commit acts of malpractice 2. Health-care facilities are responsible for the negligent actions of the nurses whom they employ 3. Nurses are responsible for their actions when they have contractual relationships with patients 4. The laws absolve nurses from being sued for negligence if they provide inappropriate care at the scene of an accident

Correct Answer: 2. Every state has its own Nurse Practice Act that describes and defines the legal boundaries of nursing practice within the state. 1. Nursing team members or an interdisci- plinary team of health-care providers write standardized care plans. 3. The National League for Nursing Accrediting Commission, the Commission on Collegiate Nursing Education, and state education departments are the major or- ganizations accrediting nursing education programs in the United States. 4. The American Nurses Association and other specialty organizations offer certification in specialty areas in nursing practice.

Licensure of Registered Professional Nurses is required primarily to protect: 1. Nurses 2. Patients 3. Common law 4. Health-care agencies

Correct Answer: 2. Licensure indicates that a person has met minimal standards of competency, thus protecting the public's safety. 1. Licensure does not protect the nurse. Licensure grants an individual the legal right to practice as a Registered Nurse. 3. Licensure does not protect common law. Common law comprises standards and rules based on the principles established in prior judicial decisions. 4. Licensure does not protect health-care agencies. The Joint Commission deter- mines if agencies meet minimal standards of health-care delivery, thus protecting the public.

Nursing practice is influenced by the doctrine of respondeat superior. What is the basic concept related to this theory of liability? 1. Nurses must respond to the Supreme Court when they commit acts of malpractice 2. Health-care facilities are responsible for the negligent actions of the nurses whom they employ 3. Nurses are responsible for their actions when they have contractual relationships with patients 4. The laws absolve nurses from being sued for negligence if they provide inappropriate care at the scene of an accident

Correct Answer: 2. The ancient legal doctrine respondeat superior means "let the master answer." By virtue of the employer-employee relationship, the employer is responsible for the conduct of its employees. 1. This is unrelated to respondeat superior. Negligence and malpractice, which are unintentional torts, are litigated in local courts by civil actions between individuals. 3. Individual responsibility is unrelated to respondeat superior. A nurse can have an independent contractual relationship with a patient. When a nurse works for an agency, the contract between the nurse and patient is implied. In both instances the nurse is responsible for the care provided. 4. This is unrelated to respondeat superior. Good Samaritan laws do not provide absolute immunity.

When attempting to administer a 10:00 PM sleeping medication, the nurse assesses that the patient appears to be asleep. What should the nurse do? 1. Withhold the drug 2. Notify the practitioner 3. Awaken the patient to administer the drug 4. Administer it later if the patient awakens during the night

Correct Answer: 3. Administering a medication is a dependent function of the nurse. The prescription should be followed as written if the prescription is reasonable and prudent. This medication was not a PRN medication but rather a standing order. 1. This is a violation of the practitioner's or- der. Drug administration is a dependent nursing function. 2. This is unnecessary. 4. The drug should be administered as prescribed not at a later time.

The nurse completes an Incident Report after a patient falls while getting out of bed unassisted. What is the main purpose of this report? 1. Ensure that all parties have an opportunity to document what happened 2. Help establish who is responsible for the incident 3. Make data available for quality-control analysis 4. Document the incident on the patient's chart

Correct Answer: 3. Incident Reports help to identify patterns of risk so that corrective action plans can take place. 1. The nurse who identified or created the potential or actual harm completes the Incident Report. The report identifies the people involved in the incident, describes the incident, and records the date, time, location, actions taken, and other relevant information. 2. Documentation should be as factual as possible and avoid accusations. Questions of liability are the responsibility of the courts. 4. The report is not part of the patient's medical record, and reference to the report should not be made in the patient's medical record.

The nurse must administer a medication. What should the nurse do first? 1. Check the patient's identification armband 2. Ensure the medication is in the medication cart 3. Verify the practitioner's prescription for accuracy 4. Determine the appropriateness of the prescribed medication

Correct Answer: 3. The administration of medications is a dependent function of the nurse. The practitioner's prescription should be verified for accuracy. The prescription must include the name of the patient, the name of the drug, the size of the dose, the route of administration, the number of times per day to be administered, and any related parameters. 1. Although this action is essential for the safe administration of a medication to a patient, it is not the first step of this procedure. 2. Although this may be done as a time- management practice, it is not the first step when preparing to administer a medication to a patient. 4. A nurse is legally responsible for the safe administration of medications; therefore, the nurse should assess if a medication prescription is reasonable. However, this is not the first step when preparing to administer a medication to a patient.

An anxious patient repeatedly uses the call bell to get the nurse to come to the room. Finally the nurse says to the patient, "If you keep ringing, there will come a time I won't answer your bell." What legal term is related to this statement? 1. Slander 2. Battery 3. Assault 4. Libel

Correct Answer: 3. This is an example of assault. Assault is a verbal attack or unlawful threat causing a fear of harm. No actual contact is necessary for a threat to be an assault. 1. This is not an example of slander, which is a false spoken statement resulting in damage to a person's character or reputation. 2. This is not an example of battery, which is the unlawful touching of a person's body without consent. 4. This is not an example of libel, which is a false printed statement resulting in damage to a person's character or reputation.

A Registered Nurse witnesses an accident and assists the victim who has a life-threatening injury. What should the nurse do to meet the most important standard when acting as a Good Samaritan at the scene of an accident? 1. Seek consent from the injured party before rendering assistance 2. Implement every critical-care intervention necessary to sustain life 3. Stay at the scene until another qualified person takes over responsibility 4. Insist on helping because a nurse is the best-qualified person to provide care

Correct Answer: 3. When a nurse renders emergency care, the nurse has an ethical responsibility not to abandon the injured person. The nurse should not leave the scene until the injured person leaves or another qualified person assumes responsibility. 1. Depending on the injured person's physi- cal and emotional status, the person may or may not be able to consent to care. 2. When a nurse helps in an emergency, the nurse is required to render care that is consistent with care that any reasonably prudent nurse would provide under simi- lar circumstances. The nurse should not attempt interventions that are beyond the scope of nursing practice. 4. A nurse should offer assistance, not insist on assisting, at the scene of an emergency.

A practitioner asks the nurse to witness an informed consent. Which patient does the nurse identify is unable to give an informed consent for surgery? 1) 16 year old boy who is married 2) 35 year old woman who is depressed 3) 50 year old woman who does not speak English 4) 65 year old man who has received a narcotic for pain.

Correct Answer: 4 (Narcotics depress the CNS, including decision-making abilities. 1 - Because legally anyone under 18 who is married can make the decision. 2 - wrong because a depressed person can make these decisions until proven mentally incompetent. 3 - Wrong because this person can provide informed consent after translations.

A practitioner writes a prescription for a medication that is larger than the standard dose. What should the nurse do? 1) Inform the supervisor 2) Give the drug as prescribed. 3) Give the average dose of the medication. 4) Discuss the prescription with the practitioner.

Correct Answer: 4 (Nurses have a prof. resonsibility to know/investigate the standard dose for medications being administered. In addition, nurses are responsible for their own actions regardless of whether there is a written prescription) 1 - It is unnecessary to call the supervisor. 2 - This is unsafe for the patient and may result in malpractice. 3 - Changing a medication prescription is not within the scope of nursing practice.

A patient is scheduled to have surgery, and informed consent is to be obtained. Place these steps in the order in which they should performed. 1. The patient is willing to sign the consent voluntarily 2. The patient signs the consent in the presence of the nurse 3. The nurse determines that the patient is alert and competent to give consent 4. The practitioner informs the patient of the risks and benefits of the procedure

Correct Answer: 4, 3, 1, 2 4. It is the responsibility of the practitioner to provide all the information necessary to make a knowledgeable decision. Patients have a legal right to have adequate and accurate information to make informed decisions. 3. Patients must be competent to sign a consent form. The patient must be alert, competent, and in touch with reality. Confused, sedated, unconscious, or minor patients may not give con- sent. Minor patients who are married, parents, emancipated, or serving in the United States military can provide a legal consent. 1. Patients must give their consent voluntarily and without coercion. 2. The health-care provider witnessing the signing of the consent must ensure that the signature is genuine.

The nurse changes a patient's dry sterile dressing. How is the nurse functioning when performing this task? 1. Interdependently 2. Collaboratively 3. Independently 4. Dependently

Correct Answer: 4. A nurse is not permitted legally to prescribe wound care. The nurse needs a practitioner's order to provide wound care. 1. The changing of a dry sterile dressing is an interdependent action by the nurse when the practitioner's order for wound care states: Dry Sterile Dressing PRN. 2. In this situation, the nurse is not working with other health-care professionals to implement a practitioner's order. 3. This intervention is not within the scope of nursing practice without a practitioner's order.

When considering legal issues the word contract is to liable as standard is to: 1. Rights 2. Negligence 3. Malpractice 4. Accountability

Correct Answer: 4. Liable means a person is responsible (accountable) for fulfilling a contract that is enforceable by law. Accountable means a person is responsible (liable) for meeting standards, which are expectations established for making judgments or comparisons. 1. Although patients have a right to receive care that meets appropriate standards, the word right does not have the same relationship to the word standard as the relationship between the words contract and liable. 2. The words standards and negligence do not have the same relationship as contract and liable. Negligence involves an act of commission or omission that a reasonably prudent person would not do. 3. The words standards and malpractice do not have the same relationship as contract and liable. Malpractice is negligence by a professional person.

Which factor is unique to malpractice when comparing negligence and malpractice? 1. The action did not meet standards of care 2. The inappropriate care is an act of commission 3. There is harm to the patient as a result of the care 4. There is a contractual relationship between the nurse and patient

Correct Answer: 4. Only malpractice is misconduct performed in professional practice, where there is a contractual relationship between the patient and nurse, which results in harm to the patient. 1. There is a violation of standards of care with both negligence and malpractice. 2. Negligence and malpractice both involve acts of either commission or omission. 3. The patient must have sustained injury, damage, or harm with both negligence and malpractice.

A faculty member of a nursing program is conducting an informational session for potential nursing students. The faculty member includes the information that at the completion of the program licensure to practice is: 1. A responsibility of the American Nurses Association 2. Granted on graduation from a nursing program 3. Approved by the National League for Nursing 4. Required by state law

Correct Answer: 4. The Nurse Practice Act in a state stipulates the requirements for licensure within the state. 1. The ANA Standards of Clinical Nursing Practice do not address licensure. 2. When a person graduates from a school of nursing, the individual receives a diploma that indicates completion of a course of study; the diploma is not a license to practice nursing. 3. The National League for Nursing (NLN) promotes nursing service and nursing education; it is not involved with licensure.

The patient's diet order is "clear liquids to regular as tolerated." How is the nurse functioning when progressing the patient's diet to full liquid? 1. Dependently 2. Independently 3. Collaboratively 4. Interdependently

Correct Answer: 4. The practitioner's order implies a progression in the diet as tolerated. The nurse uses judgment to determine the time of this progression, which is an interdependent action. 1. This dietary order has parameters that exceed a simple dependent function of the nurse. 2. Prescribing a dietary order for a patient is outside the scope of nursing practice. 3. Collaborative or collegial interventions are actions the nurse carries out in conjunction with other health-care team members.

The nurse administers an incorrect dose of a medication to a patient. What is the primary purpose of documenting this event in an Incident Report? 1) Record the event for future litigation. 2) Provide a basis for designing new policies. 3) Prevent similar situations from happening again. 4) Ensure accountability for the cause of the accident.

Correct answer: 3 (Risk-management committees use stats about accidents & incidents to identify patterns of risk and prevent future accidents/incidents.) 1 - Wrong because although documentation of an incident may be used in a court of law, it is not the primary reason for an incident report. 2- This is not the primary reason. New policies may or may not have to be written and implemented. 4 - Although nurses are always accountable for their actions, accountability for the cause of an incidence is the role of the courts.

When the nurse is administering a medication to a confused patient, the patient says, "this pill looks different from the one I had before." What should the nurse do? 1) Ask what the other pill looked like. 2) Explain the purpose of the medication. 3) Check the original medication prescription. 4) Encourage the patient to take the medication.

Correct answer: 3 (This is the safest intervention because it goes to the original source of the prescription.) 1 - Wrong because This action by itself is unsafe because the patient is confused and the information obtained may be innacurate. 2 - This intervention ignores the patient's concern. 4 - This action ignores the patient's statement and is unsafe without obtaining additional information.

A patient informs a nurse about sleep disturbances caused by jet lag. What sleep disorder does the nurse suspect? 1 Inadequate sleep hygiene 2 Adjustment sleep disorder 3 Rapid eye movement (REM) sleep-behavior disorder 4 Behaviorally induced circadian rhythm sleep disorder

Correct4 Behaviorally induced circadian rhythm sleep disorder

A patient with iron deficiency anemia reports rhythmical movements of the feet and legs and an itching sensation in the muscles before sleep. Which condition is likely to be found in the patient? 1 Insomnia 2 Cataplexy 3 Narcolepsy 4 Restless legs syndrome

Correct4 Restless legs syndrome

What nursing intervention should the nurse provide to a patient who is diagnosed with fatigue? 1 Encourage short naps if possible 2 Instruct the patient to keep an exercise log 3 Have the patient keep a sleep log for a week 4 Use active listening when talking to patient

Correct1 Encourage short naps if possible

What nursing intervention should the nurse provide to promote sleep in a patient who has a sedentary lifestyle? 1 Encourage short naps if possible 2 Instruct the patient to keep an exercise log 3 Have the patient keep a sleep log for a week 4 Use active listening when talking to the patient

Correct2 Instruct the patient to keep an exercise log

The nurse is taking a sleep history from a patient. Which statement made by the patient needs further follow-up? 1 "I always feel tired when I wake up in the morning." 2 "I go to bed at the same time each night." 3 "It takes me about 15 minutes to fall asleep." 4 "Sometimes, I have to get up during the night to urinate."

Correct1 "I always feel tired when I wake up in the morning."

A nurse is teaching a patient about the major risk factors related to obstructive sleep apnea. What statement about risk factors if made by the nurse would be correct? 1 "Obesity and hypertension are major risk factors of obstructive sleep apnea (OSA)." 2 "Smoking and heart failure are major risk factors of obstructive sleep apnea." 3 "Alcohol and type II diabetes are major risk factors of obstructive sleep apnea." 4 "Smoking and a family history of obstructive sleep apnea are major risk factors of obstructive sleep apnea."

Correct1 "Obesity and hypertension are major risk factors of obstructive sleep apnea (OSA)."

The patient reports vivid dreaming to the nurse. Through understanding of the sleep cycle, the nurse recognizes that vivid dreaming occurs during which sleep phase? 1 REM sleep 2 Stage 1 NREM sleep 3 Stage 4 NREM sleep 4 Transition period from NREM to REM sleep

Correct1 REM sleep

Which statement made by the patient indicates a need for further teaching on sleep hygiene? 1 "I'm going to do my exercises before I eat dinner." 2 "I'll have a glass of wine at bedtime to relax." 3 "I set my alarm to get up at the same time every morning." 4 "I moved my computer to the den to do my work."

Correct2 "I'll have a glass of wine at bedtime to relax."

Which priority nursing intervention does the nurse incorporate into a plan of care to promote sleep for a hospitalized patient? 1 Have patient follow hospital routines. 2 Avoid awakening patient for nonessential tasks. 3 Give prescribed sleeping medications at dinner. 4 Turn television on low to late-night programming.

Correct2 Avoid awakening patient for nonessential tasks.

Which statement is true regarding the use of benzodiazepines? 1 Benzodiazepines can be safely prescribed to infants. 2 Benzodiazepines are initially taken in small dosages. 3 Benzodiazepines can be safely used by nursing mothers. 4 Benzodiazepines can be safely used by pregnant women.

Correct2 Benzodiazepines are initially taken in small dosages.

The nurse finds that an obese patient snores loudly. The patient stops breathing for a few seconds and then resumes breathing. This happens several times during sleep. What could be the diagnosis? 1 Narcolepsy 2 Sleep apnea 3 Sleep deprivation 4 Paroxysmal nocturnal dyspnea

Correct2 Sleep apnea

Which statement made by an older adult best demonstrates understanding regarding taking a sleep medication? 1 "I'll take the sleep medicine for 4 or 5 weeks until my sleep problems disappear." 2 "Sleep medicines won't cause any sleep problems once I stop taking them." 3 "I'll talk to my health care provider before I use an over-the-counter sleep medication." 4 "I'll contact my health care provider if I feel extremely sleepy in the mornings."

Correct3 "I'll talk to my health care provider before I use an over-the-counter sleep medication."

Which statement made by a mother being discharged to home with her newborn infant indicates a need for further teaching? 1 "I won't put the baby to bed with a bottle." 2 "For the first few weeks we're putting the cradle in our room." 3 "My grandmother told me that babies sleep better on their stomachs." 4 "I know I'll have to get up during the night to feed the baby when he wakes up."

Correct3 "My grandmother told me that babies sleep better on their stomachs."

Which aspect of care should be included when teaching a patient with narcolepsy? 1 The need to perform strenuous exercise 2 The importance of avoiding caffeine 3 A recommendation to take regular naps of no more than 20 minutes 4 The proper method of using continuous positive airway pressure (CPAP)

Correct3 A recommendation to take regular naps of no more than 20 minutes

What nursing intervention should the nurse provide to a hospitalized individual in order to promote sleep? 1 Avoid allowing extended rest periods 2 Give prescribed analgesics at bedtime 3 Have the patient follow hospital routines 4 Avoid waking the patient for nonessential tasks

Correct4 Avoid waking the patient for nonessential tasks

The nurse is developing a plan of care for a patient experiencing narcolepsy. Which intervention is appropriate to include on the plan? 1 Instruct the patient to increase carbohydrates in the diet. 2 Have patient limit fluid intake 2 hours before bedtime. 3 Preserve energy by limiting exercise to morning hours. 4 Encourage patient to take one or two 20-minute naps during the day

Correct4 Encourage patient to take one or two 20-minute naps during the day.

Which sleep disorder is classified as a sleep-related breathing disorder? 1 Narcolepsy 2 Nightmare disorder 3 Sleep-related bruxism 4 Primary central sleep apnea

Correct4 Primary central sleep apnea

Which nursing measure best promotes sleep in a school-age child? 1 Encourage evening exercise. 2 Offer a glass of hot chocolate before bedtime. 3 Make sure that the room is dark and quiet. 4 Use quiet activities consistently before bedtime.

Correct4 Use quiet activities consistently before bedtime.

After surgery the patient with a closed abdominal wound reports a sudden "pop" after coughing. When the nurse examines the surgical wound site, the sutures are open, and pieces of small bowel are noted at the bottom of the now-opened wound. Which corrective intervention should the nurse do first? A) Allow the area to be exposed to air until all drainage has stopped B) Place several cold packs over the area, protecting the skin around the wound C) Cover the area with sterile, saline-soaked towels and immediately notify the surgical team; this is likely to indicate a wound evisceration D) Cover the area with sterile gauze, place a tight binder over it, and ask the patient to remain in bed for 30 minutes because this is a minor opening in the surgical wound and should reseal quickly

Cover the area with sterile, saline-soaked towels and immediately notify the surgical team; this is likely to indicate a wound evisceration

A patient has been newly diagnosed with emphysema. In discussing his condition with the nurse, which of his statements would indicate a need for further education? A) "I'll make sure that I rest between activities so I don't get so short of breath." B) "I'll rest for 30 minutes before I eat my meal." C) "If I have trouble breathing at night, I'll use two to three pillows to prop up." D) "If I get short of breath, I'll turn up my oxygen level to 6 L/min."

D

A patient who started smoking in adolescence and continues to smoke 40 years later comes to the clinic. The nurse understands that this patient has an increased risk for being diagnosed with which disorder: A) Alcoholism and hypertension B) Obesity and diabetes C) Stress-related illnesses D) Cardiopulmonary disease and lung cancer

D

Which of the following statements made by a student nurse indicates the need for further teaching about suctioning a patient with an endotracheal tube? A) "Suctioning the patient requires sterile technique." B) "I'll apply suction while rotating and withdrawing the suction catheter." C) "I'll suction the mouth after I suction the endotracheal tube." D) "I'll instill 5 mL of normal saline into the tube before hyperoxygenating the patient."

D

Which finding may indicate metabolic change as a result of immobility? 1 Muscle atrophy 2 Peripheral edema 3 Delayed wound healing 4 Orthostatic hypotension

Delayed wound healing is a metabolic change, because the rate of healing is affected by nutritional intake and nutrient absorption. Muscle atrophy is a musculoskeletal change due to immobility. Peripheral edema and orthostatic hypotension are cardiovascular changes due to immobility.

Negative outcome of immobility to Infants, Toddlers, and Preschoolers:

Delays the child's gross motor skills, intellectual development, or musculoskeletal development

The nurse is caring for a child with clubfoot. Which should the nurse advise the caregiver to apply on the child? 1 Denis Browne splint 2 Knee braces 3 Abduction splints 4 Ankle-foot orthotic

Denis Browne splints are used for children with clubfoot to align the foot in the correct position. Knee braces are used for patients with knock-knee (genu valgum). Abduction splints are used for children with congenital hip dysplasia. An ankle-foot orthotic is used to maintain the position of the foot in patients with footdrop.

Assessment of Body alignment

Determines normal physiological changes, identifies deviations, learning needs, trauma, and risk factors

The nurse is assessing four patients by asking them to perform range-of-motion exercises. Which patient does the nurse identify as having limited mobility? 1 Patient 1 2 Patient 2 3 Patient 3 4 Patient 4

For Patient 3, the flexion of the neck that brings the chin to rest on the chest should be 45 degrees. Flexion of only 10 degrees indicates disability in the pivotal joint. In Patient 1, the extension of the elbow to 90 degrees indicates normal function. In Patient 2, the wrist with the hand in slight dorsiflexion is a normal finding. In Patient 4, a slight flexion of the thumb in opposition to the fingers is a normal finding.

Pressure Ulcer Stage 3

Full thickness tissue loss, deep crater; subcutaneous fat may be visible; bone, tendon, and muscle are not exposed; slough may be present; may include undermining and tunneling

The nurse has redressed a patient's wound and now plans to administer a medication to the patient. Which is the correct infection control procedure? 1 Leave the gloves on to administer the medication. 2 Remove gloves and administer the medication. 3 Remove gloves and perform hand hygiene before administering the medication. 4 Leave the medication on the bedside table to avoid having to remove gloves before leaving the patient's room.

Gloves need to be changed and hand hygiene performed to prevent transfer of microorganisms from one source (the wound) to another (the nurse's hands). Gloves are not a foolproof method of preventing contamination of the nurse's hands so it is necessary to perform hand hygiene according to approved protocol. Medications should never be left at the bedside. It is the nurse's responsibility to ensure that patients take their medications.

The nurse works in a medical-surgical unit. Which patient should the nurse evaluate as the highest risk for health care-associated infections (HAIs)? 1 A 20-year-old patient admitted with gastroenteritis 2 A 24-year-old patient admitted with a fracture of the leg 3 A 34-year-old patient admitted for appendectomy Correct4 A 53-year-old diabetic patient admitted for herniorraphy

Health care-associated infections (HAIs) are those that are acquired by patients in the hospital during their stays. People whose immunity is compromised are at risk of these infections. Those who are at greater risk include the elderly, the malnourished, or those who have some underlying conditions that compromise their immunity, such as diabetes or malignancies. Therefore, the 53-year-old diabetic patient is at increased risk of an HAI. Gastroenteritis, fracture, and appendectomy do not increase the risk of HAIs.

4 processes to full thickness wound repair

Hemostasis, inflammatory repsonse, proliferation, and remodeling

Postural drainage position for adult bilateral lung segment

High-fowler's

Which is a mode of transmission for the human immunodeficiency virus (HIV) infection? Incorrect1 Vectors 2 Droplet 3 Vehicles 4 Airborne

Human immunodeficiency virus (HIV) infections are transmitted through vehicles such as blood and body fluids. Vectors such as mosquitoes transmit malaria. Infections such as Ebola and tuberculosis are transmitted by droplet nuclei. Infections such as influenza, Ebola, and tuberculosis are airborne.

For a patient who has a muscle sprain, localized hemorrhage, or hematoma, which wound care product helps prevent edema formation, control bleeding, and anesthetize the body part? A) Binder B) Ice bag C) Elastic bandage D) Absorptive diaper

Ice bag

The nurse is caring for a patient who is immobile. Which cardiovascular changes does the nurse expect to observe in the patient? Select all that apply. 1 Thrombus formation 2 Orthostatic hypotension 3 Increased cardiac output 4 Increased cardiac workload 5 Increased circulating fluid volume

Immobilization affects the cardiovascular system, frequently causing thrombus formation, increased cardiac workload, and orthostatic hypotension. As immobilization increases, cardiac output falls, further decreasing cardiac efficiency and increasing workload. In the immobilized patient, decreased circulating fluid volume and pooling of blood in the lower extremities occurs.

Describe how Postural abnormalities affect mobility:

Impaired body alignment, balance and mobility

A 60-year-old female patient sustained a femur fracture due to a fall in the bathroom. The patient complains of severe pain and expresses that she did not expect a fracture as the fall was not very severe. The nurse tells the patient that fracture is common for people her age. Which condition should the nurse cite as the most common cause of fracture in postmenopausal patients? 1 Osteosarcoma 2 Osteoclastoma 3 Osteomyelitis 4 Osteoporosis

In a postmenopausal patient, osteoporotic changes in the bone increase the risk of fractures. Impaired calcium metabolism causes bone resorption, making it less dense. Bones affected by resorption are fragile and more susceptible to fractures. Osteosarcoma and osteoclastoma are bone tumors. Osteomyelitis is an infection of the bone.

A 60-year-old female patient sustained a femur fracture due to a fall in the bathroom. The patient complains of severe pain. Which mineral supplement is likely to be added to the patient's prescription to manage osteoporosis? 1 Zinc 2 Calcium 3 Sodium 4 Iron

In osteoporosis the bones lack calcium due to demineralization. Therefore, calcium supplements are added to the prescription to improve bone strength. Zinc is useful in cellular metabolism but has no role in bone health. Sodium is important for cells to function but has no role in promoting bone health. Iron is needed for hemoglobin production. It does not help in managing osteoporosis related to bone changes.

Which areas of the patient would bear weight when placed in the side-lying position? Select all that apply. Correct1 Hip 2 Heel 3 Ileum 4 Shoulder 5 Humerus

In the side-lying position, most of the patient's body weight rests on the hip and shoulder. In supported Fowler's position, the pressure would be on the heels. In Sims' position, the major portion of the body weight rests on the ileum, humerus, and clavicle.

The nurse is teaching a community group about ways to minimize the risk of developing osteoporosis. Which statements made by a woman in the audience reflect a need for further education? 1 "I usually go swimming with my family at the YMCA three times a week." 2 "I need to ask my doctor if I should have a bone mineral density check this year." 3 "If I don't drink milk at dinner, I'll eat broccoli or cabbage to get the calcium that I need in my diet." 4 "I'll check the label of my multivitamin. If it has calcium, I can save money by not taking another pill."

Just because a multivitamin has calcium in it does not mean that the woman is receiving enough to meet her needs. She must know her requirement and make the decision based on that rather than on the value for calcium on the label. Any type of activity can help with calcium metabolism and is beneficial for helping to reduce the risk of osteoporosis. Patients should have a bone density scan yearly or according to their primary care physician's recommendations. To decrease the risk of osteoporosis, patients should be eating foods high in calcium and vitamin D.

The nurse provides care to four patients with different medical conditions in four units. In which medical unit should the nurse use an N95 respirator? Correct1 Medical unit I 2 Medical unit II 3 Medical unit III 4 Medical unit IV

Laryngeal tuberculosis is an airborne infection that spreads through small droplets. Therefore, the nurse should use an N95 respirator to prevent infection in medical unit I. Shigella infection spreads through contact; contact precautions should be followed in medical unit II. Scabies spreads through contact; contact precautions should be followed in medical unit III. Disseminated varicella zoster spreads through contact; contact precautions should be followed in medical unit IV.

Postural drainage position for adult right lower lobe lung segment

Left side-lying in Trendelenburg's position

A registered nurse (RN) teaches a nursing student about applying a sterile gown. The registered nurse instructs the student to lift the folded gown directly upward and step back from the table. What is the reason behind this instruction? 1 To provide proper positioning 2 To ensure dexterity 3 To provide a wide margin of safety 4 To ensure that the hair is protected from contamination

Lifting the folded gown and stepping back from the table provides a wide margin of safety and avoids contamination of the gown. This action does not provide proper positioning. The nurse's dexterity Is ensured while wearing gloves. Wearing a clean cap protects the hair from contamination.

The registered nurse is teaching a nursing student about the interventions performed to prevent deep vein thrombosis in an immobilized patient. Which statement made by the nursing student indicates a need for correction? 1 "I will massage the legs." 2 "I will instruct the patient to avoid crossing the legs." 3 "I will assist the patient in performing range-of-motion exercises." 4 "I will position the patient without applying pressure on the posterior of knee."

Massaging the leg should be avoided in cases of deep vein thrombosis, because it may lead to dislodgement of the thrombus and result in severe complications. Crossing of the legs should be avoided to increase the blood circulation. Performing range-of-motion exercises reduce the risk of contractures and aid in preventing thrombi. Proper positioning without applying pressure on the posterior of knee reduces a patient's risk of thrombus formation, because compression of the leg veins is minimized.

Identify the descriptive characteristics of body alignment and mobility related to the following developmental stage. (Older adults)

Older adults experience a progressive loss of total bone mass because of physical inactivity, hormonal changes, and increased osteoclastic activity.

A nurse is caring for four patients in an intensive care unit. Which patient should be bathed twice a week? a. An older adult patient with dry skin b. An older adult patient with bowel incontinence c. An older adult patient with urinary incontinence d. An older adult patient with bariatric abnormalities

Older adults with dry skin and no other health conditions should be bathed twice a week to prevent further dryness. Patients with bowel incontinence and urinary incontinence should be given a bath each time their clothes are soiled to prevent the spread of infections. Bariatric patients are also instructed to take strict hygiene measures to prevent the occurrence of infections due to sweating and an inability to administer self-care. p. 835

A patient is isolated for pulmonary tuberculosis. The nurse notes that the patient seems to be angry, but he knows that this is a normal response to isolation. Which is the best intervention? 1 Provide a dark, quiet room to calm the patient. 2 Reduce the level of precautions to keep the patient from becoming angry. 3 Explain the reasons for isolation procedures and provide meaningful stimulation. 4 Limit family and other caregiver visits to reduce the risk of spreading the infection.

Patients on isolation precautions [1] [2] [3] may interpret the needed restrictions as a sign of rejection by the healthcare worker. Explaining the reasons for isolation and providing meaningful stimulation can help the patient accept and adapt to the isolation precautions. Keeping a patient in a dark, quiet room can increase the patient's perception of social isolation and decrease sensory stimulation. Reducing the level of precautions can lead to cross contamination and hospital-acquired infections. Family and caregiver visits do not need to be limited unless the family members or caregivers are immunocompromised.

The nurse is caring for patients on a medical-surgical unit. The nurse plans the patients' care and instructs the nursing assistant to assist in repositioning patients every 2 hours. Which patients are at the greatest risk for complications if not repositioned properly? Select all that apply. 1 A 20-year-old unconscious patient 2 A 90-year-old frail patient 3 A 65-year-old patient who is visually impaired 4 A 40-year-old patient who has paraplegia 5 A 30-year old patient who has cholecystitis

Patients who are at the greatest risk for complications if not properly repositioned are those who are unconscious, frail, or paralyzed. The 20-year-old unconscious patient is at risk for pulmonary and cardiac complications, and at risk for deteriorated skin condition. The 90-year-old frail patient would be unable to change position independently. The 40-year-old paraplegia patient would be unable to move independently and is at risk of complications. The 65-year-old patient with visual impairment and the 30-year-old patient with cholecystitis can move by themselves. These patients are not at risk for developing complications.

A patient is diagnosed with pulmonary tuberculosis. Which personal protection equipment (PPE) is most important to be worn whenever entering the patient's room? 1 Gown 2 Gloves 3 Head cap 4 Respirator

Patients with pulmonary tuberculosis require airborne precautions because the droplets are smaller than 5 microns and remain for longer periods in the air. Therefore, a respirator is the most appropriate personal protection equipment (PPE) that the nurse should use. Gowns and gloves are most important when a nurse performs a physical examination to avoid a contact infection. A head cap is applied when the nurse is in a surgical room.

A family member is providing care to a loved one who has an infected leg wound. What would you instruct the family member to do after providing care and handling contaminated equipment or organic material? 1 Wear gloves before eating or handling food. 2 Place any soiled materials into a bag and double bag them. 3 Have the family member check with the doctor about the need for immunization. 4 Perform hand hygiene before and after care and/or handling contaminated equipment or material.

Performing hand hygiene before and after care interrupts the transmission of microorganisms from family members. Hands should be washed before eating or handling food, but gloves are not necessary. In the home care setting, soiled materials should be placed in an impervious plastic or brown paper bag. Immunizations are important, but in this situation, they would not protect against a leg wound infection.

What is the most effective technique in preventing and controlling the transmission of an infection? 1 Performing hand hygiene 2 Using isolation precautions 3 Performing sterilization procedures 4 Wearing personal protective equipment (PPE)

Performing hand hygiene is the most effective and basic technique in preventing and controlling the transmission of infection. Isolation precautions cannot control the transmission of microorganisms that cause infections unless the nurses and other health care workers follow proper control measures to prevent infections. Performing sterilization procedures help to control the transmission of infections through surgical instruments or other medical materials. Wearing personal protective equipment (PPE) is used to perform procedures that carry the risk of direct contact with contaminated material. This intervention is a more complicated step, however.

The nurse cares for a patient who is scheduled for surgery. Which objects would require high-level disinfection with phenolics? 1 Stethoscopes 2 Urinary catheters 3 Surgical instruments Correct4 Anesthesia equipment

Phenolics are used for high-level disinfection. Semi-critical items such as anesthesia equipment, endoscopes, and endotracheal tubes require high-level disinfection or sterilization. Noncritical items such as stethoscopes require a disinfection of surfaces. Critical items such as urinary catheters and surgical instruments require sterilization.

Which nursing action prevents tension on the spinal column and adduction of the hips while logrolling the patient? 1 Crossing patient's arms on the chest 2 Placing the patient in the supine position 3 Rolling a drawsheet alongside of the patient 4 Placing small pillows between patient's knees

Placing small pillows between the patient's knees helps prevent tension in the spinal column and adduction of the hips. Crossing the patient's arms across the chest prevents injury to the arms. Placing the patient in the supine position on the side of the bed prepares the patient for turning onto the side. Fanfolding, or rolling the drawsheet alongside the patient, provides strong handles to grip the drawsheet.

Complications of Immobility in relation to the integumentary system

Pressure ulcers: Impairment of the skin as a result of prolonged ischemia in tissues

Which can cause contracture of a joint? 1 The adductors muscles are weakened as a result of immobility. 2 The muscle fibers become shortened because of disuse. 3 The calcium-to-phosphorus ratio becomes disrupted. 4 There is a deficiency in vitamin D.

The adductor muscles are stronger than the abductor muscles; when patients are immobile and the joint is not exercised through its range of motion (ROM), the adductor muscle fibers shorten, resulting in the contracture of that joint, which is usually permanent.

Which part of a sterile gown is actually considered sterile? 1 Collar of the gown 2 Area below the waist 3 Underside of the sleeves 4 Anterior surface of the sleeves

The anterior surface of the sleeves is considered sterile. The collar of the gown, the area below the waist, and the underside of the sleeves are not considered as sterile.

A nurse reviews the laboratory reports of a patient with infection. Which laboratory parameter would be normal during infection? Correct1 Basophil count 2 Monocyte count 3 Neutrophil count 4 Lymphocyte count

The basophil count would be normal during infection. The monocyte count may increase if the patient has a protozoan infection. The neutrophil count may increase if the patient has an acute suppurative infection. The lymphocyte count may increase if the patient has a chronic bacterial or viral infection.

A patient is being transferred from bed to stretcher. Which precautions should the nurse take to ensure patient safety during transfer? Select all that apply. 1 Release the brakes of the bed to allow movement. 2 Raise the bed to the level of the stretcher. 3 Cross the patient's arms on chest while transferring. 4 Involve multiple caregivers for safe transfer. 5 Unlock the stretcher's wheels once it is in place alongside the bed.

The bed should be raised to the level of the stretcher to allow the patient to slide from the bed to the stretcher. Keep the patient's arms crossed when transferring to prevent any injury to the arm. Three caregivers are needed to transfer a patient safely and are positioned specifically to minimize caregivers stretching. The bed brakes should be locked to prevent it from moving. Once the stretcher is placed alongside the bed, the wheels should be locked to prevent further movement.

Which statement regarding health care-associated infections requires correction? 1 The costs of health care-associated infections are reimbursed. 2 Health care-associated infections can significantly increase the cost of health care. 3 The length of hospitalization influences the risk of health care-associated infections. 4 Health care-associated infections result from the delivery of health services in a healthcare facility.

The costs of health care-associated infections (HAIs) are not reimbursed; therefore, the prevention of HAIs plays an important role in the managed care of health care systems. HAIs can increase the cost of health care because they increase infections in patients. The invasive procedure involved, the therapies received, and the length of hospitalization can influence the risk of HAIs in patients. HAIs can be caused by invasive procedures performed during the delivery of health services in a health care facility.

The nurse is caring for a patient whose calcium intake must increase because of high risk factors for osteoporosis. Which menus should the nurse recommend? 1 Cream of broccoli soup with whole wheat crackers and tapioca for dessert 2 Hamburger on soft roll with a side salad and an apple for dessert 3 Low-fat turkey chili with sour cream and fresh pears for dessert 4 Chicken salad on toast with tomato and lettuce and honey bun for dessert

The dairy and broccoli in the soup, the whole grain crackers, plus the tapioca are all great sources of calcium.

The nurse is caring for a patient diagnosed with a spinal cord injury at T7 level. For which areas of care should the nurse seek out a physical therapist for assistance? Select all that apply. 1 Assisting with management of bed sores 2 Managing the controls of the ventilator 3 Preventing the development of any postural abnormalities 4 Assisting the patient in ambulation 5 Prescribing exercises to strengthen trunk musculature

The duty of a physical therapist is to assist patients in activities that enhance mobility such as ambulation, prescribing exercises to improve strength, and addressing any postural abnormalities that could further hamper mobility. A person with T7-level spinal cord injury does not require ventilator support. Physical therapists could assist in bed mobility to prevent bed sores, and they are knowledgeable about wound care, but managing bed sores is the duty of the nurse.

What are the functions of the skeletal system? Select all that apply. 1 Provide support 2 Regulate calcium 3 Regulate posture 4 Contribute balance 5 Protect vital organs

The functions of the skeletal system include providing joint flexibility and support, regulating calcium, and protecting vital organs. The nervous system regulates movement and posture. Body alignment contributes to balance of the body.

When does the nurse wear a gown? 1 The patient's hygiene is poor. 2 The nurse is assisting with medication administration. 3 The patient has acquired immunodeficiency syndrome (AIDS) or hepatitis. 4 Blood or body fluids may get on the nurse's clothing from a task that he or she plans to perform.

The gown serves as a barrier between the patient's blood and/or body fluids and potential contact with the caregiver's skin. In some cases, if the patient's hygiene is unacceptable, it may require the nurse to wear a gown, but it is not the best answer. It is not necessary to wear a gown during medication administration or if the patient has AIDS or hepatitis.

A patient sustained a femur fracture due to a fall in the bathroom. The attending nurse understands that the femur bone forms a hip joint with the pelvic bone. Which type of joint is the hip joint? 1 Hinge joint 2 Pivot joint 3 Cartilaginous joint 4 Ball-and-socket joint

The head of the femur acts as a ball and the pelvic bone as a socket, and together they form a ball-and-socket type of joint , which is a freely movable synovial joint. The elbow and knee joints are examples of hinge joints. The joints in the neck are pivot joints. Cartilaginous joints are found in the sternum where cartilage unites with the bone.

Which trouble areas should be assessed for correction when the patient is placed in the position depicted in the image? 1 Ears 2 Knees 3 Sacrum

The image depicts the supported Fowler's position. The trouble areas in this position include the sacrum and heels. Unprotected pressure points at the ears are the trouble areas in side-lying position. Unprotected pressure points at the knees and humerus are the trouble points in Sims' position.

What is the significance of the position depicted in the image? 1 Reduces the risk of foot drop 2 Reduces the risk of pressure ulcers 3 Reduces the risk of deep vein thrombosis 4 Reduces the risk of musculoskeletal injury

The image indicates the 30-degree lateral position. This position would be beneficial for reducing the risk of pressure ulcers. The risk of foot drop is reduced by placing the patient in the supine or prone positions. The risk of deep vein thrombosis is reduced by placing the patient in a supported Fowler's position. Placing the patient in the supine position reduces the risk of musculoskeletal injury.

A patient who has hemiplegia is unable to dorsiflex and invert the feet. Which condition does the patient likely have? 1 Lordosis 2 Footdrop 3 Genu varum 4 Genu valgum

The inability to dorsiflex and invert the feet indicates that the patient has footdrop which is usually found in patients who are bedridden and immobile. The foot becomes permanently fixed in plantar flexion, making ambulation difficult. The patient is unable to lift the toes off the ground. Exaggeration of the anterior convex curve of the lumbar spine is called lordosis. One or both legs bent outward at the knee indicates genu varum. Legs curved inward so that the knees come together while walking is a sign of genu valgum.

Identify the descriptive characteristics of body alignment and mobility related to the following developmental stage. (Infants)

The infant's spine is flexed and lacks the anteroposterior curves; as growth and stability increase, the thoracic spine straightens, and the lumbar spinal curve appears, allowing for sitting and standing.

Which indicates that additional assistance is needed to transfer a patient from the bed to the stretcher? 1 The patient is 5 feet 6 inches and weighs 120 lbs. 2 The patient speaks and understands English. 3 The patient received an injection of morphine 30 minutes ago for pain. 4 The nurse feels comfortable handling a patient of this size and with this level of cooperation.

The morphine injection would change the patient's ability to safely follow directions and participate in the transfer; therefore, additional help would be needed to safely transfer the patient from the bed to the stretcher. The patient's height and weight, ability to speak English, and the nurse's comfort level in handling the patient do not change the patient's ability to participate in the transfer.

Which action is performed by the nurse in the given image? 1 Opening of the last and innermost flap 2 Opening of the first side flap and pulling aside 3 Opening of the second side flap and pulling it aside 4 Opening of the outermost flap of the sterile kit away from body

The nurse is opening the outermost flap of the sterile kit away from the body to avoid microbial contamination. This is the first step of opening a sterile kit. The first side flap is opened and pulled aside in the second step. The second side flap is opened and pulled aside in the third step. The last and the innermost flap are opened in the last step.

Identify the measures with which the nurse must be familiar to reduce exposure to radiation:

The nurse limits the time spent near the source, makes the distance from the source as great as possible, and uses shielding devices

The nurse is assessing a patient for the movement of the hip ball and socket joint. Which range-of-motion (ROM) exercise should the nurse ask the patient to perform to assess adduction? 1 Move the leg forward and up. 2 Move the leg behind the other leg. 3 Move the leg laterally away from body. 4 Move the leg back toward the medial position.

The nurse should ask the patient to move the leg back toward the medial position and beyond if possible to assess adduction in the hip joint. To assess flexion in the hip joint, the leg should be moved forward and up. To assess extension in the hip joint, the leg should be moved behind the other leg. To assess abduction in the hip joint, the leg should be moved laterally away from the body.

A registered nurse teaches a group of nursing students about home care considerations for patients with infections. Which statement made by the nursing student indicates the need for further learning? 1 "I should determine potential sources of contamination." 2 "I should evaluate hand washing facilities in the patient's home." 3 "I should anticipate the need for alternative hand washing products." 4 "I should see if cold running water faucets are available."

The nurse should check if warm running water faucets at the patient's home. The nurse should determine potential sources of contamination and possible preventive measures. The nurse should evaluate all hand washing facilities in the patient's home. The nurse should anticipate the need for alternative hand washing products for use in emergency and immediate situations.

A patient is admitted to a hospital for surgery to correct urinary incontinence. About what should the nurse instruct the patient in order to avoid infections while cleaning the perineal region? 1 Wipe from the urinary meatus toward the rectum. 2 Wipe from the rectum toward the urinary meatus. 3 Clean the perineal region once a day. 4 Cleaning the perineal area is more important for young women then for older women past menopause.

The nurse should instruct the patient about cleaning the perineal region correctly to reduce the incidence of genitourinary infections. The rectum is a highly contaminated area compared to the urinary meatus. It is necessary to wipe the perineal area from the urinary meatus toward the rectum, as cleaning should be done from least to most contaminated regions to reduce infections. Wiping from the rectum to the urinary meatus would increase the risk for contracting infection. Cleaning the perineal area only once a day increases the risk for contracting infection; the perineal region should be cleaned after every voiding and bowel movement. Keeping the perineal region meticulously clean is particularly important for older women who may wear incontinence pads.

Which statement is true regarding donning a sterile gown? 1 The circulatory nurse should also wear sterile gown. 2 Nurses should wear sterile gowns before applying masks. 3 The anterior surface of the sleeves of a gown is also considered sterile. 4 The nurse should wear sterile gowns while assisting a health care provider during surgery.

The nurse should wear a sterile gown while caring for a patient with large open wounds and while assisting the healthcare provider during invasive procedures, such as inserting an arterial catheter. A circulatory nurse generally does not wear a sterile gown. Nurses should not apply a sterile gown until after applying a mask and surgical cap. Only certain areas of the gown are considered sterile; the collar and the anterior surface of the sleeves may not be considered sterile.

The student nurse performs a Braden Scale to predict a patient's risk for pressure ulcer development. The student accurately determines the Braden Scale is 22. To reduce pressure ulcer development in this patient, the nurse should implement which of the following patient needs? A) The patient needs a bed with a low-air loss mattress system B) The patient needs standard nursing care and precautions C) The patient need a foam overlay on the mattress of the bed D) The patient needs a bed with an air-fluidized system

The patient needs standard nursing care and precautions

The nurse is caring for a retired patient who has had a stroke. The nurse needs to assist the family in adapting to the condition and also help in rehabilitating the patient. Which interdisciplinary team members should the nurse collaborate with? Select all that apply. 1 Social worker 2 Speech therapist 3 Occupational therapist 4 Physical therapist 5 Prosthetist

The physical therapist may collaborate to help the patient rehabilitate by giving the patient gait training. The physical therapist is the head of the rehabilitation team and should be able to guide the family members for rehabilitation. The occupational therapist may help the patient with activities of daily living (ADLs), such as dressing, bathing and toileting, or household chores. The prosthetist's role is to make prosthesis for amputees. The patient has not had an amputation and does not require prosthesis. The social worker focuses on assessment and evaluation of the patient's social situation. The patient's needs are related to the physical mobility; a social worker may not be of help.

The registered nurse is teaching a nursing student about the safety guidelines for nursing skills. Which statement by the nursing student indicates the need for further learning? 1 "I should raise the side rail on the opposite side of the bed from where I'm standing." 2 "I should evaluate the patient for correct body alignment." 3 "I should determine the type of assistance required for safe positioning." 4 "I should arrange the positioning equipment as close as possible to the patient's bed."

The positioning equipment should be arranged in such a way that it does not interfere with the positioning process; therefore, the equipment should not always be placed next to or too far away from the bed, but should be placed appropriately. The side rails on the side of the bed should be raised on the opposite side where the nurse stands to prevent the patient from falling out of the bed. The nurse should evaluate the patient for correct body alignment and pressure risks after repositioning. The nurse should determine the amount and type of assistance required for safe positioning, including any transfer equipment and the number of personnel to safely transfer the patient.

A patient has an indwelling urinary catheter. Why does an indwelling urinary catheter present a risk for urinary tract infection? 1 It keeps an incontinent patient's skin dry. 2 It can get caught in the linens or equipment. 3 It obstructs the normal flushing action of urine flow. 4 It allows the patient to remain hydrated without having to urinate.

The presence of a catheter in the urethra breaches the natural defenses of the body . The reflux of microorganisms up the catheter lumen from the drainage bag or backflow of urine in the tubing increases the risk of infection. A catheter can help in keeping an incontinent patient's skin dry, but that normally does not lead to a urinary tract infection. The catheter can become caught up in the linens or with other equipment, but that does not cause a urinary tract infection. A patient with a catheter is producing urine and urinating; thus the patient is staying hydrated and still urinating.

The nurse is planning discharge instructions for a patient diagnosed with human immunodeficiency virus (HIV). Which statement made by the patient would indicate effective teaching? 1 "The virus cannot spread through sexual contact." 2 "The virus can spread through feces only when I have symptoms of the disease." 3 "The virus can be spread to another person by contact with body fluids." 4 "The virus can cause Rocky Mountain spotted fever."

The primary routes of HIV infection are associated with contact of HIV-infected body fluids such as blood or semen, blood transfusions, sharing of infected needles, and needle-stick injuries. The virus spreads through sexual contact and does not spread through feces. The virus does not cause Rocky Mountain spotted fever, which is caused by Rickettsia rickettsii.

Which trouble points would the nurse anticipate while positioning the patient in the Sims' position? Select all that apply. Correct1 Lack of foot support 2 Increased cervical flexion 3 Lateral flexion of the neck 4 Lack of protection for pressure points 5 Pressure on posterior aspects of knee

The trouble points that are common in the Sims' position are lack of foot support, lateral flexion of the neck, and lack of protection for pressure points. Increased cervical flexion and pressure on posterior aspects of the knee are the common trouble areas in the supported Fowler's position.

Which urinary elimination changes are often observed as the period of immobility continues for a patient? Select all that apply. 1 Increased fluid intake 2 Increased urinary output 3 Increased risk of renal calculi 4 Increased urinary concentration Correct5 Increased risk for urinary tract infections

The urinary elimination changes that are often observed as the period of immobility continues are increased renal calculi, as immobilized patients may have hypercalcemia that causes calcium stones. As the immobility continues, the fluid intake decreases and the concentration of urine increases. As the concentration of urine increases, urinary tract infections also increase. As the period of immobility continues, fluid intake often diminishes and, therefore, urinary output also decreases.

"No-touch" technique

This technique is a method of changing surface dressings without touching the wound or the surface of any dressing that might be in contact with the wound. Adherent dressings should be grasped by the corner and removed slowly, whereas gauze dressings can be pinched in the center and lifted off.

While caring for a postoperative patient, the nurse spreads trochanter rolls on the bed before positioning the patient. In which position is the nurse preparing to place the patient? 1 Sims' position 2 Prone position 3 Supine position 4 Supported Fowler's Position

Trochanter rolls are used to increase comfort for the patient and to reduce the risk of injury to the skin and musculoskeletal system when the patient is placed in the supine position. In the Sims' position, the patient places the weight on the anterior ileum, humerus, and clavicle. Trochanter rolls would not be used in this position. In the prone position, the nurse uses pillows under the lower extremities to reduce the risk of foot drop. In the supported Fowler's position, the knees are supported to reduce the risk of deep vein thrombosis.

On assessing your patient's sacral pressure ulcer, you note that the tissue over the sacrum is dark, hard, and adherent to the wound edge. What is the correct stage for this patient's pressure ulcer? A) Stage II B) Stage IV C) Unstageable D) Suspected deep tissue damage

Unstageable

A patient had a left-sided cerebrovascular accident (CVA) 3 days ago and is receiving 5000 units of heparin subcutaneously every 12 hours to prevent thrombophlebitis. The patient is receiving enteral feedings through a small-bore nasogastric (NG) tube because of dysphagia. Which symptom requires the nurse to call the healthcare provider immediately? 1 Pale yellow urine 2 Unilateral neglect 3 Slight movement noted on the right side 4 Coffee ground-like aspirate from the feeding tube

When patients are receiving medications such as heparin or enoxaparin, the nurse must assess for signs of bleeding. These include overt signs, such as bleeding from the gums, or covert signs, which can be detected by testing the stool or observing the patient's aspirate from nasogastric (NG) tubes for coffee ground-like matter. These are signs of bleeding in the gastrointestinal tract. Pale yellow urine is not cause for concern, because it may be diluted and pale due to the extra fluids the patient may be given. Unilateral neglect in a cerebrovascular accident (CVA) is common. Slight movement that was not there during the previous neurological check is important and should be documented, but it is not necessary to call the healthcare provider.

Briefly explain how assessment of body alignment and posture is carried out. (Recumbent)

When recumbent, the vertebrae are in straight alignment without observable curves; the head and neck should be aligned without excessive flexion or extension.

What is the desired outcome when the head of the bed is elevated to 60 degrees in supported Fowler's position? 1 Improves ventilation 2 Prevents shoulder dislocation 3 Relieves the pressure on heels 4 Prevents hyperextension of knee

When the head of the bed is elevated to 60 degrees in supported Fowler's position, it improves ventilation. Using a pillow to support the arms would help in preventing shoulder dislocation. Use of heel pressure relief devices would reduce pressure on the heels. Placing a small pillow under the thigh can prevent hyperextension of the knee.

Which device should the nurse use to prevent external rotation of the hips when the patient is in a supine position? 1 Trochanter roll 2 Positioning boots 3 Trapeze bars 4 Pillows

When the patient is in the supine position, the nurse should use a trochanter roll to prevent external rotation of the hips. When the hips are correctly aligned, the patella faces directly upward. The positioning boots help in preventing footdrop. Patients use trapeze bars to lift themselves during repositioning. Pillows may not be helpful in preventing the external rotation of the hips.

Which action made by the nurse may contaminate the surface of the sterile item? 1 Disposing of the outer wrapper 2 Holding the arm over the sterile field 3 Peeling the wrapper onto the nondominant hand 4 Allowing the drape to unfold on a surface above waist level

While adding sterile items, the nurse should not hold the arm over the sterile field because it may contaminate the surface of the sterile item. Disposing of the outer wrapper prevents accidental contamination of the sterile field. The nurse should carefully peel the wrapper onto the nondominant hand. The nurse should allow the drape to unfold on a surface above the waist and work surface to prevent contamination.

Which action increases the risk of contamination while applying a sterile gown? 1 Lifting the gown upward and stepping forward near the table 2 Grasping the inside front of the gown with both hands just below neckband 3 Asking the circulating nurse to tie the back of the gown at the neck and waist 4 Slipping both arms into the armholes with the hands at shoulder level

While applying a sterile gown, the nurse should lift the gown directly upwards and step backwards (not forward) away from the table to provide a wide margin of safety. The nurse can ask the circulating nurse to tie the back of the gown at the neck and waist; this action can reduce the risk of contamination. Clean hands can touch the inside of the gown without contaminating the outer surface. With the hands at shoulder level, the nurse should slip both arms into the armholes simultaneously to prevent contamination.

While using an antiseptic hand rub to perform hand hygiene, the nurse feels dryness in his or her hands after rubbing them together for 10 to 15 seconds. What is the reason for this dryness? 1 Occurrence of an allergic reaction 2 Insufficient time taken to rub hands Correct3 Insufficient antiseptic solution applied 4 Complete antimicrobial action maintained

While maintaining hand hygiene, if the nurse's hands are dry after rubbing them together for 10 to 15 seconds, an insufficient volume of product was likely applied. Dryness of the hands does not indicate allergic reaction. Ten to 15 seconds is a sufficient amount to time to rub the hands the together. Complete antimicrobial action is not achieved if an insufficient antiseptic solution is applied.

A nurse performs hand hygiene before providing direct patient care. Which action made by the nurse may cause an infection? 1 Wearing rings on both hands 2 Maintaining nail tips less than a quarter-inch long 3 Rubbing both hands together after applying the antiseptic 4 Pushing the wristwatch and long uniform sleeves above the wrists

While performing hand hygiene, finger rings should be removed. Wearing rings increases the risk for infection because the skin underneath the rings carries a high bacterial load. The nail tips should be less than a quarter-inch long. Antiseptic should be applied on the palm and both hands should be rubbed together. Any wristwatches and long uniform sleeves should be pushed above the wrist to provide complete access to the fingers, hands, and wrists.

A licensed practical nurse is preparing to assist in a sterile procedure. Which nursing action is appropriate in surgical hand asepsis? 1 Scrubbing the hands for 5 minutes 2 Washing over the rings and watch 3 Keeping the hands and arms below the elbows 4 Allowing the water to flow from the elbows to the hands

While performing surgical hand asepsis, the nurse should scrub the hands for 5 minutes to eliminate transient microorganisms and reduce resident hand flora. During a sterilizing procedure, the nurse should remove all jewelry and accessories, such watches and rings. The hands should be above the elbows while performing a surgical scrub.

While positioning a hemiplegic patient in the supported Fowler's position, the nurse positions the head of the patient against a small pillow with the chin slightly forward. What is the rationale behind this nursing action? 1 Promote circulation 2 Support lumbar vertebrae 3 Prevent shoulder dislocation 4 Prevent flexion contractures

While positioning a hemiplegic patient who is unable to control head movement in the supported Fowler's position, the nurse should position the head on a small pillow with the chin slightly forward to prevent hyperextension of the neck. Using pillows to support the arms and hands of a patient who does not have voluntary control promotes circulation by preventing venous pooling. Positioning a small pillow at the lower back of a hemiplegic patient helps support the lumbar vertebrae and decreases flexion of the vertebrae. Use of pillows to support arms and hands may help prevent shoulder dislocation from the effect of a downward pull of unsupported arms.

The nurse pours a sterile liquid into a container. Which action made by the nurse is appropriate? 1 Holding the bottle with its label pointed outside the palm of the hand 2 Placing the cap with the inner surface facing down on the table 3 Keeping the edge of the bottle close to the edge of the container 4 Pouring a small amount in a disposable cap before pouring in the container

While pouring a sterile liquid into a container, a small amount of liquid should be poured in a disposable cap before it is poured into the container because the discarded solution cleans the lip of the bottle. The cap should be placed with its inner surface facing upwards on the table because the inner surface should not be contaminated. The nurse should hold the bottle with its label in the palm of the hand to prevent the possibility of the solution wetting and fading the label. The edge of the bottle should be kept away from the container.

A registered nurse evaluates the nursing assistive personnel who is wearing a mask. Which action made by the nursing assistive personnel indicates a need for correction? Correct1 Having a casual conversation while wearing a mask 2 Changing an unused mask due to a moist feeling 3 Tying the two top ties at the back of the head and above the ears 4 Wearing the mask such that the top of the mask fits below the glasses

While wearing a mask, talking should be kept to a minimum to reduce respiratory airflow. A mask that has become moist does not provide a barrier to microorganisms and should be discarded. While wearing a mask, the two top ties should be tied at the back of the head and above the ears. The top of the mask should fit below the glasses.

Postural drainage

a component of pulmonary hygiene; it consists of drainage, positioning, and turning and is sometimes accompanied by chest percussion and vibration

Syndesmosis is a fibrous joint between the tibia and fibula.

a fibrous joint between the tibia and fibula.

Identify the two types of canes that are available and their use.

a. A single straight-legged cane is used to support and balance a patient with decreased leg strength. b. A quad cane provides more support and is used for partial or complete leg paralysis or some hemiplegia.

A patient who is cognitively impaired and has dementia requires hygiene care. The patient often displays aggressive behavior, such as screaming and hitting during the bath. Which techniques would make the bathing experience less stressful for both the nurse and the patient? a. Allow the patient to perform as much of the care as possible. b. Start by washing the face. c. Try an alternative to traditional bathing, such as the bag bath. d. Use restraints to prevent the patient from injuring self or the nurse. e. Be quick to keep the bath as short as possible.

a. Allow the patient to perform as much of the care as possible. c. Try an alternative to traditional bathing, such as the bag bath. Patients with cognitive impairment may respond to bathing by acting out aggressively. Studies have indicated a number of triggering events, including washing the face first. The bag bath has been shown to result in a lower incidence of aggressive behavior than traditional bathing. Use of restraints is not warranted and can actually lead to injury, because the patient often fights against the restraints. Cognitively impaired patients respond better when the nurse uses a gentle approach and avoids rushing. p. 836

Identify the psychosocial effects tat occur with immobilization

a. Emotional and behavioral responses b. Sensory alterations c. Changes in coping

The nurse is caring for an older adult who has had diabetes for 10 years. For which signs of peripheral neuropathy should the nurse observe? a. Foot deformities b. Abnormal gait c. Shiny appearance of the skin d. Absent or decreased pulses e. Muscle wasting of the lower extremities

a. Foot deformities b. Abnormal gait e. Muscle wasting of the lower extremities A patient with diabetes showing signs of foot deformities, abnormal gait, and muscle wasting of lower extremities may have peripheral neuropathy. These signs occur due to damage to the nerves controlling the function of the muscles. Absent or decreased pulses or shiny appearance of the skin are signs of vascular insufficiency. p. 838

Explain the four standard crutch gaits: a. Four-point gait b. Three-point gait c. Two-point gait d. Swing-through gait

a. Four-point gait: Each leg is moved alternatively with each opposing crutch so three points are on the floor at all times. b. Three-point gait: Bears weight on both crutches and then on the uninvolved leg, repeating the sequence. c. Two-point gait: There is at least partial weight bearing on each foot. d. Swing-through gait: Weight is placed on supportive legs; crutches are one stride in front and then swings through with the crutches, supporting the patient's weight.

While caring for a patient, the nurse instructs the patient to perform oral hygiene at least twice a day. Which conditions is the nurse preventing the patient from developing? a. Gingivitis b. Xerostomia c. Ulcerations d. Dental caries e. Periodontal disease

a. Gingivitis d. Dental caries e. Periodontal disease Rationale Performing oral care at least twice a day and flossing reduces the risk of gingivitis, dental caries, and periodontal disease. Xerostomia might not be reduced by performing regular oral care. Ulcerations are not reduced by performing regular oral care. Test-Taking Tip: Identify option components as correct or incorrect. This may help you identify a wrong answer. p. 823

Actual or potential Nursing Diagnoses related to an immobilized or partially immobilized patient

a. Ineffective Airway Clearance b. Ineffective Coping c. Impaired Physical Mobility d. Impaired Urinary Elimination e. Risk for Impaired Skin Integrity f. Risk For Disuse Syndrome g. Social Isolation

List three outcomes for a patient with deficits in activity and exercise.

a. Participates in prescribed physical activity while maintaining appropriate heart rate, blood pressure, and breathing rate. b. Verbalizes an understanding of the need to gradually increase activity based on tolerance and symptoms c. Expresses understanding of balancing rest and activity

Nursing intervention to reduce impact of immobility on the Musculoskeletal System

a. Perform active and passive ROM exercises b. CPM machines

Nursing intervention to reduce impact of immobility on the Cardiovascular System

a. Reduce orthostatic hypotension; early mobilization b. Reduce cardiac workload; avoid Valsalva movements c. Prevent thrombus formation; prophylaxis (heparin, SDCs, and TEDs)

The nurse is performing a routine health checkup of a patient who is addicted to drugs and has had multiple sexual partners. The nurse finds lice in the patient's scalp hair and pubic hair. What is the next best option for lice treatment if chemicals fail? a. Remove the lice manually. b. Shave the scalp hair. c. Shave the pubic hair. d. Apply normal shampoo.

a. Remove the lice manually. There are many chemical treatments for lice. When these treatments fail, the next best option is to manually remove the lice. Shaving the hair of the scalp or pubic area is not necessary for permanent removal of lice. While removing the lice, the nurse should apply medicated shampoo. A normal shampoo would not be useful in removing the lice. p. 831

While caring for a patient, the nurse suspects improper turning. Which condition does the nurse suspect? a. Scraping of the skin b. Maceration of the skin c. Accidental injuring of the skin d. Mechanical irritation of the skin

a. Scraping of the skin Improper turning or positioning techniques can lead to scraping or stripping of the skin surface. Maceration occurs from continuous exposure to moisture. Accidental injury can occur if finger rings are not removed. Mechanical irritation of the skin can occur if smooth linen cloths are not used. Test-Taking Tip: Make certain that the answer you select is reasonable and obtainable under ordinary circumstances and that the action can be carried out in the given situation. p. 822

The nurse observes an altered gait while assisting a patient to the bathroom. Which condition is most likely associated with this assessment finding? a. Stroke b. Heart failure c. Renal disease d. Diabetes mellitus

a. Stroke A stroke frequently causes leg weakness or paralysis. This commonly results in altered walking patterns. Heart failure and renal diseases increase the risk of tissue edema, particularly in dependent areas such as the feet, but do not typically result in a significantly altered gait. Neuropathic changes associated with diabetes mellitus may cause altered sensation in the feet, but the gait is most likely unaffected until the late stage of the disease. p. 863

The nurse and nursing assistive person (NAP) are performing the nail and foot care of three patients. The patients include a 30-year-old man with peripheral vascular disease, a woman with diabetes, and a 10-year-old child with otitis media. Which interventions performed by the nurse and NAP are appropriate? a. The nurse clips the nails of the child. b. The NAP soaks the foot of the man. c. The NAP clips the toenails of the man. d. The nurse soaks the nails of the child in warm water. e. The nurse soaks the nails of the woman in hot water.

a. The nurse clips the nails of the child. d. The nurse soaks the nails of the child in warm water. Appropriate measures and delegations should be performed during foot and nail care. It is the nurse's responsibility to soak and clip the nails of the child. Care of the child should not be delegated to the nursing assistive person (NAP). The nails should be soaked in warm water to make them soft and help in clipping. The feet of the patients with diabetes or circulatory problems should not be soaked in water as doing so can cause maceration of the skin. These patients may have impaired peripheral sensation (diabetes) or circulation, so any injury to the foot while clipping the nails should be avoided. Clipping of the nails for these patients should not be delegated to the NAP. Soaking of the nails should be avoided in a patient with diabetes, because it can cause maceration of the skin. p. 864

A physical restraint is:

any manual method, physical or mechanical device, material or equipment that immobilizes or reduces the ability of a patient to move his or her arms, legs, body, or head freely - use only as a last resort

A nurse cares for a patient who has arthritis. Which questions should the nurse ask to determine the patient's religious and cultural practices? a. "Does bathing cause shortness of breath?" b. "How can I best help you with hair care?" c. "Do you have any preferences for how you bathe?" d. "Which bathing activities may cause discomfort?" e. "How comfortable are you with my care?"

b. "How can I best help you with hair care?" c. "Do you have any preferences for how you bathe?" e. "How comfortable are you with my care?" Asking questions such as which is the best way to provide effective hair care,and what is the patient's bathing preferences are helpful for determining the patient's cultural or religious practices. Some patients may have their limits or preferences regarding hygiene, so the nurse should ask about the patient's comfort zone. Asking questions such as, "Does bathing cause shortness of breath?" and "Which bathing activities may cause discomfort?" helps the nurse determine the patient's tolerance of hygiene activities. p. 826

Which instruction should the registered nurse give to the nursing assistive personnel who is providing a complete bed bath to a patient? a. Wear gloves if necessary b. Avoid massaging the reddened area under the skin folds c. Wash the patient's face with chlorhexidine gluconate solution d. Disconnect the intravenous pump tubing while removing the patient's gown

b. Avoid massaging the reddened area under the skin folds Special care should be taken to clean the skin folds. Massaging the areas under the reddened skin folds may increase skin inflammation and redness. The nurse should always use gloves to prevent soiling of hands and transmission of infections while providing patient hygiene. Chlorhexidine gluconate solution contains antimicrobials, so it should not be used to wash the patient's face. The nursing assistive personnel should use clear water or mild soap and water instead. Intravenous pumps should not be disconnected while removing the patient's gown because it may worsen the patient's health condition. p. 854

A patient undergoing radiation therapy for head and neck cancer reports tooth decay. What is a possible reason for the condition? a. Infection in the gums b. Decreased salivary pH c. Inflammation of the mucosa d. Accumulation of secretions on the gums

b. Decreased salivary pH Radiation therapy for the head and neck may reduce salivary flow and decrease the pH of the saliva, which leads to tooth decay. Radiation therapy may not be a reason for gum infection. Inflammation may be caused by radiation therapy or immunosuppression, but it does not cause tooth decay. Accumulation of secretions on the gums is a result of dehydration that is unassociated with radiation therapy. p. 839

The nurse evaluates the knowledge of the nursing assistive person after teaching the process of washing a patient's abdomen. Which statements of the nursing assistive person need correction? a. A towel maintains warmth and privacy. b. Draping causes the exposure of body parts. c. Keeping the skin folds clean and dry prevents skin irritations. d. Dressing the affected side causes a decreased range of motion. e. Moisture that collects in the skin folds predisposes the patient to skin maceration.

b. Draping causes the exposure of body parts. d. Dressing the affected side causes a decreased range of motion. Draping involves covering or wrapping loosely; this action prevents the unnecessary exposure of body parts. Dressing the affected side first allows for easier manipulation of the gown above the body part with reduced range of motion (ROM). A towel maintains warmth and privacy, in addition to preventing the soiling of bed linen. Keeping the skin folds clean and dry prevents skin irritation or allergic reactions. Moisture and sediment that collect in the skin folds predispose the patient to skin maceration. p. 857

A registered nurse teaches a patient about skin protection. Which statement made by the patient indicates the need for further teaching? a. "I should use emollients." b. "I should use alkaline soaps." c. "I should keep my bed linens and clothing wet." d. "I should minimize the exposure of skin to moisture."

c. "I should keep my bed linens and clothing wet." The patient's bed linens and clothing should be kept dry to protect the skin. The patient should use emollients to soften the skin and prevent moisture loss. The patient should use alkaline soaps and minimize skin exposure to moisture to prevent ulcer formation. p. 822

The nurse is teaching a group of nursing students about bathing and perineal care. What instructions should the nurse include in the teaching? a. Wash the eye from outer to inner canthus. b. Bathe the arm using long, firm strokes from axilla to fingers. c. Raise and support the arm above the head to wash, rinse, and dry axilla thoroughly. d. Soak any crusts on eyelids for 2 to 3 minutes with a damp cloth before attempting removal. e. Wash, rinse, and dry the forehead, cheeks, nose, neck, and ears without soap, if the patient prefers.

c. Raise and support the arm above the head to wash, rinse, and dry axilla thoroughly. d. Soak any crusts on eyelids for 2 to 3 minutes with a damp cloth before attempting removal. e. Wash, rinse, and dry the forehead, cheeks, nose, neck, and ears without soap, if the patient prefers. Raising the arm exposes the axilla and facilitates washing and cleaning. Soaking any crusts on the eye helps to ease removal of the crust from the eye. Soaps tend to dry the skin. Therefore, if the patient prefers not to use soap, the face should be washed with water. The eye should be washed from inner to outer canthus to prevent secretions form entering the nasolacrimal duct. The arm should be bathed from fingers to axilla to promote venous return. p. 854

Which interventions should the nurse perform to provide effective foot care for an elderly patient who has diabetes? a. Use oval corn pads. b. Cut the calluses using scissors. c. Refrain from the application of adhesive tape. d. Refrain from soaking the patient's foot while caring. e. Use over-the-counter liquid preparation to remove corns.

c. Refrain from the application of adhesive tape. d. Refrain from soaking the patient's foot while caring. The elderly patient has thin and delicate skin that has an increased risk of tearing while removing adhesive tape. Therefore, the nurse should refrain from applying adhesive tape to prevent tearing the patient's skin. The nurse should refrain from soaking a diabetic patient's feet, because soaking may result in infection or tissue breakdown. The nurse should not use oval corn pads, because the pads can exert pressure on toes and decrease circulation to the surrounding tissues. The nurse should not cut calluses with scissors, because it disturbs skin integrity and leads to infection. Over-the-counter liquid preparations such as salicylic acid should not be used to remove corns, because these can cause ulcerations and burns in the patient's feet. p. 838

hemorrhage

can be internal or external, pulse gets faster, and blood pressure drops; massive bleeding; this is an emergency situation

Ligament rupture----?????? check this info,

can be partial (just some of the fibres that make it up are torn) or complete (it is torn through completely). Knee injuries can cause pain, swelling, tenderness, bruising and reduced movement of your knee. Your knee joint may feel unstable and you may walk with a limp.

Shearing

combination of pressure and friction, where the skin and subcutaneous layers will adhere to the surface of the bed and the bones & muscles will slide in direction of the body, causes damage in the deeper tissues

Avulsion

complete or partial tearing away of skin or body part

Dressings for Pressure Ulcer Stage 2

composite film, hydrocolloid, hydrogel

Jackson-Pratt Drain

constant low pressure bulb; if drainage exceeds ability of bulb or bladder notify MD; empty when 1/2 full; release suction prior to removing

A patient who has had diabetes and hypertension for the past 15 years is given broad-spectrum antibiotics for an infection. The patient also takes antipyretics for fever. Which drug may lead to an opportunistic infection in the patient's oral cavity? a. Antipyretic drugs b. Hypoglycemic drugs c. Antihypertensive drugs d. Broad-spectrum antibiotics

d. Broad-spectrum antibiotics In a patient who is taking broad-spectrum antibiotics, the normal flora of the mouth are disrupted. Therefore, the patient becomes prone to opportunistic infections. The nurse should do a thorough examination of the oral regions in such patients. Antipyretics, hypoglycemic drugs, and antihypertensive drugs do not lead to opportunistic infections. These drugs are not known to disrupt the normal flora of the mouth. These drugs are also not known to affect the immune system and increase susceptibility to infections. p. 831

Which structure helps to hold a tooth firmly in place? a. Neck b. Enamel c. Mandible d. Periodontal membrane

d. Periodontal membrane The periodontal membrane lies below the gum margins, surrounds the teeth, and holds the teeth firmly in place. The neck is the constricted part of the tooth that connects the crown and the root. The enamel covers the crown that surrounds the tooth like a tight collar. The mandible and maxilla tear and grind the ingested food so that it can be mixed with saliva and swallowed for digestion. p. 823

A patient complains of heartburn and a bitter-tasting fluid regurgitating into his mouth during sleep. In which position will the patient feel comfortable? a. Prone b. Supine c. Lateral recumbent d. Reverse Trendelenburg's

d. Reverse Trendelenburg's The patient has symptoms of gastric esophageal reflux disorder. In reverse Trendelenburg's position, the entire bed frame is tilted with the foot of the bed down. This promotes gastric emptying and prevents esophageal reflux. Therefore, the patient will be comfortable in this position. The prone position is a very uncomfortable position, because it exerts more pressure on the abdomen. Supine and lateral recumbent positions should be avoided, because they promote gastric reflux. p. 848

Which type of bath involves bathing from a bath basin with a patient sitting in a chair? a. Sitz bath b. Tub bath c. Partial bed bath d. Sponge bath

d. Sponge bath A sponge bath involves bathing from a bath basin with the patient sitting in a chair. A sitz bath is used for a patient with a swollen and inflamed rectum, perineum, and genital area. A tub bath is given to patients in long-term care. A partial bed bath involves washing selected areas of the patient such as the face or perineal area. This type of bath may be done when the patient is in pain or nauseated. p. 836

While performing a partial bed bath for an elderly patient, the practical nurse offers a bedpan. Why would the nurse offer a bedpan? a. To decrease the chance of drafts b. To promote the patient's safety c. To promote proper body alignment d. To decrease the chances of interruption

d. To decrease the chances of interruption Providing a bedpan or urinal to an elderly patient during a bath will help to provide comfort by decreasing the chance of interruption during the bath. The nurse decreases the chance of drafts by closing the doors and windows of the bathing area. Keeping the water warm will provide comfort and safety. Raising the side rails and level of the bed to a comfortable height will help to promote proper body alignment. p. 854

The nurse advises a patient to avoid the use of dry razors. Which condition is the nurse trying to prevent? a. Chemical irritation b. Mechanical irritation c. Loss of stratum corneum d. Weakening of the epidermis

d. Weakening of the epidermis The use of dry razors can scrape or strip the skin's surface. Weakening of the epidermis occurs by scraping or stripping the skin's surface. Chemical irritation may occur due to the misuse of soap, detergents, cosmetics, deodorant, and depilatories. Mechanical irritation may occur when smooth fiber cloths are not used. Loss of stratum corneum may occur due to friction. p. 822

Suspected deep tissue injury

depth unknown; localized area of purple or maroon colored intact skin and or blood filled blister with damage to underlying tissue; area with be painful, firm, mushy, boggy, warmer, or cooler than adjacent tissue

Wound

disruption of normal anatomical structure and function resulting from pathological processes beginning internally or externally.

airborne transmission

droplets that suspend in the air

Cascade cough

patient takes a slow, deep breath and holds it for 2 seconds while contracting expiratory muscles. Then he or she opens the mouth and performs a series of coughs throughout exhalation, thereby coughing at progressively lowered lung volumes. This technique promotes airway clearance and a patent airway in patients with large volumes of sputum.

Partial rebreather mask

simple mask with a reservoir bag that should be at least one third to one half full on inspiration and delivers from 40% to 70% FIO2 with a flow rate of 6 to 10 L/min. The flow rate should be a minimum of 10 L/min and deliver FIO2 of 60% to 80%. Frequently inspect the reservoir bag to make sure that it is inflated. If it is deflated, the patient is breathing large amounts of exhaled carbon dioxide. High-flow oxygen systems should be humidified.

Hyperemic changes

skin redness after a period of tissue ischemia due to pressure, when pressure is relieved skin turns red due to vasodilation

Debridement

the removal of necrotic tissue so that healthy tissue can regenerate

Eschar

the scab or dry crust that results form death of the skin

Chemical debridement

the use of a topical enzyme preparation, Dakin's solution, or sterile maggots. Topical enzymes induce changes in the substrate resulting in the breakdown of necrotic tissue. Depending on the type of enzyme used, the preparation either digests or dissolves the tissue. These preparations require a health care provider's order. Dakin's solution breaks down and loosens dead tissue in a wound. Apply the solution to gauze and apply to the wound. Sterile maggots are used in a wound because it is thought that they ingest the dead tissue.`

Mechanical debridement

the use of wet-to-dry saline gauze dressings. Place moistened gauze into the wound and allow the dressing to dry thoroughly before "pulling" the gauze that has adhered to the tissue out of the pressure ulcer. This is a nonselective method of debridement because devitalized and viable tissues are both removed; thus it is not used routinely. Never use this method in a clean, granulating wound.

Abnormal reactive hyperemia (non-blanching hyperemia

there is no blanching and deep tissue damage is probable


Kaugnay na mga set ng pag-aaral

Sustantivos Femeninos: -Sión, -Ción, -Dad, -Tad, -Tud, -Umbre

View Set

Is Kopi Luwak on your bucket list?

View Set

Chapter 2: Financial Statements, Taxes, and Cash Flow

View Set

The Middle East - By James and Vamsi 2022

View Set

Treating Parent and Child Relationships Chapter 1

View Set

STA15_03_Lesson9_10_U32_Place an order_Check an order 2

View Set

Chapters 13: Psychosocial Problems

View Set